Sei sulla pagina 1di 168

ENGLISH lifted from what particular novel?

a. The Prince and the Pauper


1. Robert Frost wrote the poem Acquainted with the b. The Little Prince
Night from which the stanza is taken:
c. The Right One
I have been one acquainted with the night. d. The Pearl
I have walked out in rain-back in rain. Answer: B
I have out walked the farthest city light
The poet in the stanza talks of? 7. He was the American President who said, “Ask
not what America will do for you, but what together
a. isolation and loneliness
we can do for the freedom of man.”
b. joy getting out of the house a. Gerald Ford
c. youthful delight playing in the rain b. Franklin Roosevelt
d. happiness in having been acquainted with the c. Henry Truman
night d. John F. Kennedy
Answer: A Answer: D

8. History is the witness that ______ passing of time.


2. Which is the BEST WAY to write the underlined a. testifies
portion of this sentence? Researchers also speculate b. will testify
that some teachers might have given boys more c. testifies for
computer time because parents and d. testifies for the
teachers expected boys to need computers for future Answer: D
careers.
9. When I met Liza yesterday, it was the first time I
a. expected
_____ her since Christmas.
b. expecting a. saw
c. will expect b. had seen
d. will have expected c. have seen
Answer: D d. have been seen
Answer: D
3. The wounded soldiers were visited by the
10. The commander ordered his men to hold on the
president who honoured them with ____ for their
fort. What was the message?
_____. a. Surrender in arms
a. medals – valor b. Keep on with the fight
b. gun salute – bravery c. Disregard the peace negotiation
c. appointments – dedication d. Rescue the hostage victims fast and early
d. money – sacrifice Answer: B
Answer: A
11. “She is a vision of feminine pulchritude.” This
stands for the following EXCEPT
4. Every June, Manila has its festival of outstanding a. Loveliness
Filipino films. This was a project initiated by b. Comeliness
a. Arsenio Lacson c. Homeliness
b. German Moreno d. Physical beauty
c. Antonio Villegas Answer: C
d. Ramon Bagatsing
12. The Nibelungenlied is a
Answer: C a. Latin Myth
b. Chinese legend
5. Which among the sounds below is voiceless? c. Russian folk song
a. /b/ d. Medieval German epic
b. /z/ Answer: D
c. /g/
13. An association wherein the name of something is
d. /p/
substituted by something that represents it.
Answer: D a. Metonymy
b. Comparison
6. “Only the heart can see rightly.” This statement is c. Euphemism

1
d. Personification
Answer: A 21. The copyreader found the news story boring. He
found it full of _____.
14. Because the moon rotates on its axis at the same a. Adjectives
time as it ______ around the earth, we see the same b. Verbs
side c. Pronouns
a. Revolve d. Adverbs
b. Revolves Answer: C
c. Is revolving
d. Has been revolving 22. There were three guests on the stage. They were
Answer: B made up of a parent, the governor and the principal.
Who should be acknowledged first by the
15. In English verse, a poetic foot having 1 stressed valedictorian?
syllable followed by 1 unstressed syllable is ______. a. The classmates
a. Trochaic b. The principal
b. Iambic c. The governor
c. Dactylic d. The parent
d. Anapaestic Answer: C
Answer: A
23. What is suggested in the opening line? June 13,
16. Senators were accused by activists of washing 1986 - they came from all over America - 200,000
their hands with the perfumes of Arabia. This state is heroes strong, with their families.
commonly known as a. The writer holds great admiration for the veterans
a. Guilt b. The writer is opposed to the Vietnam War
b. Triumph c. The writer was a veteran of the war
c. Indecision d. The writer is a flag-waving patriot
d. Aggression Answer: A
Answer: A
24. A readability mismatch happens when the
17. It is a collection of religious poetry written by reading levels of books exceed the reading levels of
Rabindranath Tagore. the students. In this situation, the students experience
a. Mahabharata frustration and they fall short of the expected or
b. Gitanjali desired output. A student who finds himself/ herself
c. The Ramayana in such a mismatch will likely do which of the
d. Bhagavad Gita following?
Answer: B a. Give an intelligent critique of the selection or
story read
18. I suggest that he _____ in the room for one week. b. Write a comprehensive reaction paper regarding
a. Stay the selection or story read
b. Stayed c. Present an argument that the selection or story
c. Staying read was not properly written
d. Stays d. Manifest an expected and commensurate
Answer: A emotional reaction to the selection or story read
Answer: C
19. The manager told his workers, “We have to
reduce our workforce.” What did he mean? 25. What does this mean: Excuses are for losers…
Workers are free to leave those who take responsibility for their actions are
Workers are warned of possible lay off real winners in life. This tells of?
Workers have to double time on their jobs a. losers often fail because they find reasons for
Workers should submit themselves to a reducing losing
gym b. a winner can also be a quitter
Answer: B c. accountability of one’s action tells of bravery
d. excuses are needed to justify any failure
20. The parent remarked, “__________ I come late, Answer: C
just lock the door.”
a. In the absence 26. A couple accepted a wedding invitation. They
b. In the process showed pleasure in these remarks
c. In the event a. All guests congratulated the organizers and the
d. In the case couple
Answer: C b. Guests came in and out of the ceremonies

2
c. The ceremonies were very impressive B. Gravity
d. The couple felt uneasy with the priest C. Deprivations
Answer: C D. Viciousness
3. INDULGENT parents spoil their children by giving in to
27. What correction should be made to this sentence? their every demands and whims.
One of the theories is that the first child receives A. Loving
more of the parents' attention than other children so B. Beneficial
C. Generous
first-borns tend to be more intellectual.
D. Yielding
a. Change is to are
4. Open:Secretive::Forthright: __________
b. Insert a comma after children
A. Spiteful
c. Change parents' to parent's B. Mask
d. Change theories to theory's C. Outspoken
Answer: B D. Honest
5. Negligent; Requirement:: Remiss :__________
28. Carl Sundburg wrote "Jazz Fantasia" which has A. Task
for its first stanza: B. Duty
C. Injury
Drum on your drums, batter on your banjos, sob on D. Problem
the long cool winding saxophones. Go to it, O 6. It is _________ to try to destroy pest completely with
jazzmen. pesticides, for as each new chemical pesticide is
Which words illustrate alliteration? introduced, the pests gradually become _________ to it.
a. Batter and banjos A. useless, drawn
b. Sob and winding B. futile, resistant
c. Long and cool C. worthwhile, immune
d. To and it D. pointless, vulnerable
Answer: A 7. The famous Dr. C. Drew’s technique for preserving and
storing blood plasma for emergency use proved so
29. What is meant by AT SIXES AND SEVENS in _________ that it became the _______ for the present
this sentence? blood bank system by the Red Cross.
A. irrelevant, inspiration
B. effective, model
We moved into the house last week, but I'm afraid
C. complex, blueprint
everything, is still at sixes and sevens.
D. urgent, pattern
a. The things have not been shipped.
8. An A-rating represents the ULTIMATE honor a film will
b. In a state of confusion ever have. Which of the following is the opposite of the
c. In an orderly manner capitalized word?
d. The boxes are still intact A. Greatest
Answer: B B. Pinnacle
C. Least
30. What is the mood of these lines? D. Supreme
9. The VINDICTIVE politician spread rumors about his
Daylight, I must wait for the sunrise opponent. Which of the following is an antonym of the
I must think of a new life capitalized word?
And I mustn't give in. A. Revengeful
When the dawn comes B. Forgiving
tonight will be a memory, too C. Spiteful
And a new day will begin. D. Malicious
a. Afraid 10. Vegetarians have a variety of reasons for their beliefs.
b. Sarcastic Some refuse to eat meat because they believe that the
c. Depressed killing of animals is unnecessary or barbaric. They feel
d. Hopeful that processes such as butchering are degrading to the
Answer: D people who practice them and cruel to the animals
slaughtered. Others believe that meat is harmful to the
1. He is more concerned with MUNDANE matters like the human body and that a purely vegetable diet is more
movement of stock market. The capitalized word means: nutritious. Still others choose vegetarianism because they
A. Simple believe that raising animals for meat is an inefficient use
B. Financial of land. They say that farmland could be put to better use
C. Worldly growing crops for human consumption. Which of the
D. Investments following statements is best supported by the above
2. The DEPRAVITY of the criminal’s behavior shocked us passage?
all. The capitalized word means: A. Some vegetarians choose not to eat meat mainly for
A. Harshness health reasons.

3
B. A purely vegetable diet is more nutritious than a diet C. vain
that includes meat. D. vanity
C. Vegetarians are opposed to any use of animals for the 17. Because they did not accept his basic _______ they
benefit of humans. were ______ by his argument.
D. Vegetarians favor laws to reduce the consumption of A. assumption, convinced
meat. B. bias, impressed
C. supposition, justified
11. The amount of your retirement benefits from Social D. motivation, confused
Security depends, in part, on when you retire. The earliest 18. The failed July 27, 2003 mutiny was believed by many
that you can begin to receive benefits is age 62. The as a plan by the Magdalo group as an attempt to stage a
longer you wait to begin receiving benefits (up to age 70), takeover of government. It was viewed as a desperate
the higher your benefits will be. If you were born between attempt by the mutineers who were supported by
1938 and 1960, you cannot receive “full retirement” influential and politically motivated men who financed the
benefits until age 66. If you were born in 1960 or later, mutiny, according to the Feliciano Commission. The
your full retirement age will be 67. If you wait to receive action was found to be unconstitutional as it violates both
benefits past your full retirement age, the amount you the Articles of War and the Revised Penal Code. Such act
receive will continue to increase until you reach age 70. can lead to either Court Martial or Civilian proceedings or
There is no advantage to waiting past age 70 to begin even both. The best title for the above article is:
receiving benefits. You can continue to work while you A. Failed Mutiny
receive retirement benefits. If you do, the amount of your B. Illegal Soldiers Uprising
benefits will be reduced by one dollar for every two C. Coup d’etat
dollars you earn above a certain limit, until you reach your D. Articles of War and Punishment
full retirement age. Once you reach your full retirement 19. Determine which may be the best order in which the
age, this limit no longer applies. The amount you earn will sentences should be arranged to result in a well-
have no effect on your benefits. According to this organized paragraph.
passage, which of the following statements is true? A. Now we shall do everything we can to enforce this law.
A. You cannot receive retirement benefits from Social B. In the struggle against crime, government and people
Security while you are still working. should join hands
B. If you were born in 1950, your full retirement age is 66. C. Republic Act 7659, the death penalty law is
C. If you continue to work past your full retirement age, government’s response in law against cruelty and
your benefits will be reduced by one dollar for every two heartlessness.
dollars you earn. D. The law’s main objective is not retribution but reform
D. There is no advantage to waiting past your full and rehabilitation of offenders and would be offenders.
retirement age to begin receiving benefits. E. All countries in the world are presently siege by violent
12. The volunteers from the fire department ________ crime.
quickly and extinguished a fire on North Country Road. A. CEBAD
A. will respond B. EBCDA
B. responded C. CEDAB
C. will have responded D. ACEBD
D. have responded 20. Determine which may be the best order in which the
13. In Tuesday’s paper, the owner of the supermarket sentences should be arranged to result in a well-
was recognized for helping a customer who ________ on organized paragraph.
the icy sidewalk. A. Tagaytay is the destination for us who love nature.
A. falls B. It provides an ideal picnic ground for outdoor lovers.
B. had fallen C. Unfortunately, the place ends up abused by outdoor
C. would fall lovers who litter the place.
D. has fallen D. It is a place which can be easily reached by a two-hour
14. We arranged the flowers and placed ________ in the drive from Makati City.
center of the table. A. CDAB
A. them B. ADBC
B. it C. CBDA
C. this D. CABD
D. that
15. The person ________ made this delicious Answers: 1A 2D 3C 4A 5B 6C 7C 8C 9B 10A 11B 12B
cheesecake has my vote. 13B 14A 15B 16C 17D 18B 19B 20B
A. that MATH
B. who 1. Two buses leave the same station at 8:00 pm. One
C. which
bus travels north at the rate of 30 kph and the other
D. whose
16. Being too _______ will undoubtedly make other men travels east at 40 kph. How many kilometers apart
hate you. are the buses at 10 pm?
A. vane a. 140 km
B. vein

4
b. 100 km 8. What is the missing terms in the series 5, 20, 80,
c. 70 km ___,1280, ___, 20, 480?
d. 50 km a. 50;210
Answer: B b. 40;160
c. 35;135
2. Calculate the mean absolute deviation of the d. 320;5120
following numbers: 60, 80, 100, 75 and 95 Answer: D
a. 12.4
b. 14.2 9. At what rate per annum should P2400 be invested
c. 16.1 so that it will earn an interest of P800 in 8 years?
d. 18.9 a. 6 ½ %
Answer: A b. 5 ½ %
c. 4.17 %
3. Which of the following is the factorization of the d. 6 %
binomial x2 - 42? Answer: C
a. (x + 4)(x + 2)
b. (x – 4)2 10. The area of a rectangle is (x2 + 2x - 8). If its
c. x(x + 2x + 2) length is x + 4, what is its width?
d. (x – 4)(x + 4) a. x + 2
Answer: D b. x - 2
c. x + 1
4. What value of x will satisfy the equation: 0.4(5x - d. x + 6
1470) = x? Answer: B
a. 490
b. 2,130 11. What is the value of 12⅙ - 3 ⅜ - 5 ⅔ + 20 ¾?
c. 1470 a. 21 1/8
Answer: D b. 22
c. 23 7/8
5. Which of the following has the greatest value: d. 21
a. 3 + 32 + (3 + 3)2 Answer: C
b. 33
c. [(3 + 3)2]2 12. The vertex angle of an isosceles triangle is 20°.
d. (3 + 3 + 3)2 What is the measure of one of the base angles?
Answer: C a. 150°
6. The average of 5 different counting numbers is 20. b. 60°
What is the highest possible value that one of the c. 75°
numbers can have? d. 80°
a. 20 Answer: D
b. 40
c. 30 13. Ana and Beth do a job together in three hours.
d. 90 Working alone, Ana does the job in 5 hours. How
Answer: D long will it take Beth to do the job alone?
a. 3 and 1/3 hours
7. Three brothers inherited a cash amount of P62,000 b. 2 and 1/3 hours
and they divided it among themselves in the ratio of c. 3 hours
5:4:1. How much more is the largest share than the d. 7 and 1/2 hours
smallest share? Answer: D
a. P75,000
b. P30,000 14. How much greater is the sum of the first 50
c. P24,800 counting numbers greater than the sum of the first
Answer: C 100 counting numbers?
a. 110

5
b. 3,775 owner pays them P50 per hour. How much would
c. 3,155 Danny receive for that day?
d. 1200 a. P 230
Answer: N b. P500
c. P160
15. Which of the following has the largest value? d. P480
a. 85 Answer: B
b. 39
c. 65 21. A retailer buys candies for P90.25. The pack has
d. 94 35 pieces of candies. If she sells each candy for
Answer: A P2.25, how much profit does she make?
a. P11.50
16. A water tank contains 18 liters when it is 20% b. P56.25
full. How many liters does it contain when 50% full? c. P37.50
a. 60 d. P18.75
b. 30
c. 58 22. An online shop sells a certain calculator for P950
d. 45 and charges P150 for shipping within Manila,
Answer: D regardless of the number of calculators ordered.
Which of the following equations shows the total
17. The edges of a rectangular solid have these cost (y) of an order as a function of the number
measures: 1.5 feet by 1½ feet by 3 inches. What is its of calculators ordered (x)?
volume in cubic inches? a. y = (950 + 150)x
a. 324 b. y = 150x +950
b. 225 c. x = 950y + 150
c. 972 d. y = 950x + 150
d. 27 Answer: D
Answer: C
23. One side of a 45° - 45° - 90° triangle measures x
18. In a certain school, the ratio of boys to girls is 5 cm. What is the length of its hypotenuse?
is to 7. If there are 180 boys and girls in the school, a. X √3 cm
how many boys are there? b. X cm
a. 105 c. (X √3)/2 cm
b. 90 d. X √2 cm
c. 45 Answer: D
d. 75
Answer: D 24. The legs of one right triangle are 9 and 12, while
those of another right triangle are 12 and 16. How
19. Ruben’s grades in 6 subjects are 88, 90, 97, 90, much longer is the perimeter of the larger triangle
91 and 86? What is the least grade that he should aim than the perimeter of the smaller triangle?
for in the 7th subject if he has to have an average of a. 84
88? b. 7
a. 92 c. 12
b. 74 d. 14
c. 88 Answer: C
d. 85
Answer: B 25. Determine the midpoint of the line segment
joining the points (7, -3) and (-1, 6).
20. On a certain day, three computer technicians a. (2, 3/2)
took turns in manning a 24-hour internet shop. The b. (2, -3/2)
number of hours Cesar, Bert, and Danny were on c. (3, 3/2)
duty was in the ratio 3:4:5, respectively. The shop d. (1, 5/2)

6
Answer: C 2. Alin sa mga sumusunod ang pinakatamang
pangungusap?
26. Which of these has the longest perimeter? a. Ang mga kabataan sa lansangan ay naglalaro at
nagtatakbuhan sa lansangan.
a. A square 21 cm on a side
b. Ang kabataan ay naglalaro kung maliwanag ang
b. A rectangle 19 cm long and 24 cm wide buwan at nagtatakbuhan sa lansangan.
c. An equilateral triangle whose side is 28 cm c. Ang mga kabataan ay naglalaro at nagtatakbuhan
d. A right triangle whose two legs are 24 and 32 cm sa lansangan kung maliwanag ang gabi.
Answer: D d. Ang mga kabataan kung maliwanag ang buwan ay
nagtatakbuhan sa lansangan at naglalaro.
27. How many square inches are in 2 square yard? Answer: C
a. 900
3. Piliin ang gawi ng pagsasalita: Kasiyahan ko nang
b. 144 makitang kayo’y nagmamahalan.
c. 1296 a. Pangarap
d. 2,592 b. Pagkontrol ng kilos
Answer: D c. Pagkuha ng impormasyon
d. Pagbabahagi ng damdamin
28. In a playground for Kindergarten kids, 18 Answer: D
children are riding tricycles or bicycles. If there are
4. Ang kaugnayan ng pagkakapatay kina Burgos,
43 wheels in all, how many tricycles are there? Gomez at Zamora sa panitikang Pilipino ay
a. 8 a. Nanatiling masigla ang diwang Pilipino
b. 9 b. Nakagising sa damdaming makabayan ng mga
c. 7 Pilipino
d. 11 c. Natutong lumabag sa batas at lumaban sa may
kapangyarihan ang mga Pilipino
Answer: C
d. Naimpluwensyahan ang diwang alipin ng mga
Pilipino
29. Nelia takes ¾ hour to dress and get ready for Answer: B
school. It takes 4/5 hour to reach the school. If her
class starts promptly at 8:00 am; what is the latest 5. Laging UMUUKILKIL sa isipan ng ama ang
time she can jump out of bed in order not to be late nasirang pangako ng anak.
for school? a. Sumasagi
b. Gumugulo
a. 6:42 am
c. Bumubuhay
b. 6:27 am d. Sumasapi
c. 6:57 am Answer: B
d. 7:02 am
Answer: B 6. Ang Kagawaran ng Ugnayang Panlabas ng
Pilipinas ay gulong-gulo kapag may giyera sa ibang
30. Which common fraction is equivalent to 0.215? bansa. Ano ang unang ginagampanan ng ambassador
ng bansa?
a. 43/200
a. Bilangin ang mga nasugatan at nasawi
b. 27/125 b. Alamin ang mga tirahan ng mga Pilipino sa
c. 21/50 bayang iyon.
d. 108/375 c. Ipunin ang mga maykaya at ipalipad pauwi.
Answer: A d. Bayaang magsipag-uwian sa sariling sikap ang
bawat isa
FILIPINO Answer: B

1. Sinabi ni Carlos P. Romulo sa isa niyang akda, 7. Ano ang pokus ng pandiwa sa pahayag na “Bumili
“Ang Pilipino ay may dugong maharlika.” Ano ang ng bagong sasakyan si Angelo”?
kahulugan nito? a. Pokus sa direksyon
a. Ang Pilipino ay nanggaling sa malayang lahi b. Pokus sa kagamitan
b. Ang Pilipino ay sadyang mabuti ang budhi. c. Pokus sa sanhi
c. Ang Pilipino ay galing sa mayamang lahi d. Pokus sa aktor
d. Ang Pilipino ay madaling maipagbili Answer: D
Answer: A

7
8. Kabaliwan at paglulustay ang inyong ginagawa b. nang – nang
taon-taon. Higit na marami ang maralitang c. ng – nang
nangangailangan ng salapi at dunong. Ang d. nang – kapag
nagsasalita ay Answer: C
a. Kuripot
b. Matipid 15. Ang butong tinangay ng aso, walang pagsalang
c. Maramot nalawayan ito. Ang kaisipang ito ay tumutukoy sa
d. Praktikal katotohanan ng ______.
Answer: D a. Pagnanakaw
b. Pagtatanan
9. Nasa anong kaganapan ng pandiwa ang c. Pagpapakasal
pangungusap? d. Pakikipagkaibigan
Naglaro ng basketball sa Rizal Stadium ang Answer: B
koponan ng aming pamantasan.
16. Anong uri ng pagbigkas ang salitang
a. Sanhi
“dambuhala”?
b. Tagaganap
a. Malumi
c. Kagamitan
b. Mabilis
d. Ganapan
c. Maragsa
Answer: D
d. Malumay
Answer: A
10, Sa aling salita magkakaroon ng saglit na
paghinto kung pinagpipilitang si Rose ang nakabasag
17. Ang katawagan sa pangngalan, pang-abay, pang-
ng pinggan?
uri at pandiwa ay?
Hindi si Rose ang nakabasag ng pinggan.
a. Palabuuan
a. Rose b. Pangkayarian
b. Hindi c. Pangnilalaman
c. Nakabasag d. Palaugnayan
d. Pinggan Answer: C
Answer: B
18. Ang panukalang inihain niya ay lubhang malalim
11. Anong tayutay ang tinutukoy sa pahayag. at mahirap arukin.
Durog ang katawang bumagsak sa semento si a. Abutin
Miguel. b. Unawain
c. Sukatin
a. Pagtutulad d. Tanggalin
b. Pagbibigay katauhan Answer: B
c. Pagmamalabis
d. Pagwawangis 19. Ang wikang Filipino ay hawig sa mga wika sa
Answer: C Asya. Alin dito ang pinagmulan ng wikang Filipino?
a. Bahasa
12. Sino ang pinagkalooban ng karangalan bilang
c. Nihonggo
“Unang Tunay na Makata” noong 1708?
d. Mandarin
a. Jose dela Cruz
d. Malayo-Polinesyo
b. Felipe de Jesus
Answer: D
c. Francisco Balagtas
d. Jose Corazon de Jesus
20. Ano ang katumbas ng “Dekalogo” ni Apolinario
Answer: B
Mabini na nagsasaad ng aral sa Filipino?
a. Mosaic Law
13. “Magtatrabaho ako at ikaw ay mag-aaral upang
d. Code of Ethics ni Kalantiaw
makatapos ka ng pag-aaral.” Anong uri ng
c. New Society ni Pres. Marcos
pangungusap ito?
d. Code of Citizenship ni Pres. Quezon
a. Payak
Answer: A
b. Tambalan
c. Hugnayan
21. Siya ay hinirang na taga-sensus ng bahay-bahay.
d. Langkapan
Ano ang kanyang nalikom?
Answer: D
a. Ang bilang ng tao sa bahay
b. Ang kayamanan ng may-bahay
14. Mag-aalas-singko na _____ umaga _____
c. Ang datos tungkol sa mga bata sa bawat bahay
magising siya.
a. ng – ng
8
d. Ang datos tungkol sa mga naninirahan sa bawat c. Nilimbag sa Filipino ang diploma ngunit may
bahay Ingles
Answer: D d. Nalimbag sa Filipino ang diploma
Answer: D
22. Kami ang kabataang siyang magiging pag-asa ng
bayan. Paano ginamit ang salitang may salungguhit? 29. Ang gintong panahon ng mga manunulat noong
a. Pagtukoy panahon ng Amerikano ay batid sa uring
b. Pagpuri a. Sanaysay
c. Panghalip b. Nobela
d. Pagmamalaki c. Panulaan
Answer: A d. Maikling kwento
Answer: D
23. Walang tubig kahapon. Ito ay pangungusap na:
a. May paksa 30. Alin sa mga sumusunod ang may wastong gamit
b. Walang pandiwa ng tinig ng pandiwa?
c. May panaguri a. Ang hinog na papaya na kinuha sa puno ni Marie.
d. Walang paksa b. Kinuha ni Marie ang hinog na papaya sa puno.
Answer: D c. Kinuha sa puno ang hinog na papaya ni Marie.
d. Papayang hinog ang kinuha sa puno ni Marie.
24. Ipinagmamalaki mo siya, BAHAG naman pala Answer: B
ang kanyang BUNTOT. Ang ibig sabihin ng salitang
may malaking titik ay: SCIENCE
a. Kuripot
b. Traydor 1. Which of the following statements best describes a
c. Duwag hypothetical element with an electron configuration
d. Mahiyain of 1s22s22p63s23p5?
Answer: C a. The hypothetical element has an atomic number of
11
25. Sabihin ang gawi ng pananalitang ito: “Bawal b. The hypothetical element is a member of Group
tumawid, may namatay na dito!” V, otherwise called the Nitrogen Group.
a. Pananakot c. The hypothetical element is in the fifth position in
b. Pagtukoy the p-block, along the third period of the periodic
c. Babala table.
d. Paalala d. The hypothetical element is located at the third
Answer: C position of the p-block, along the fifth period of the
periodic table.
26. Pinakamahalagang nobelang Pilipino sa Answer: C
maraming taon na nalimbag noong 1906 at
tumalakay nang masinsinan sa paksang puhunan 2. Despite the observed diversity among organisms,
laban paggawa at sa sosyalismo ang _____. they are all made from the same set of biomolecules
a. Luha ng Buwaya composed of monomeric units except:
b. Banaag at Sikat a. Proteins
c. Ibong Mandaragit b. Carbohydrates
d. Pangginggera c. Nucleic Acids
Answer: B d. Lipids and Fats
Answer: D
27. Ayon kay Balagtas, “ang laki sa layaw,
3. What will be formed when radium isotope, with
karaniwa’y hubad” kaya ang mga bata ay
88 protons and 138 neutrons undergoes alpha decay?
a. jeproks
a. Radon Atom (Rn222) with 86 Protons
b. nag-aartista
b. Francium Atom (Fr222) with 87 Protons
c. nakapagtatapos sa pag-aaral
c. Actinium Atom (Ac222) with 89 Protons
d. hindi sumusunod sa magulang
d. Thorium Atom (Th232) with 90 Protons
Answer: D
Answer: A
28. Noong taong 1962, ano ang pagbabago sa 4. Which of the following examples best illustrates
paglimbag ng diploma at sertipiko ng pagtatapos? application of Boyle’s Law?
a. Pinahihintutan ang pribadong paaralan na a. A tire becomes harder as more air is pumped into
maglimbag sa wikang Ingles it.
b. Nilimbag sa Tagalog ang diploma sa di-Tagalog b. A sealed aerosol can explodes when thrown into a
na bayan fire.
9
c. A balloon expands and bursts when exposed to nitrogen
direct sunlight. Answer: D
d. A scuba divers stops at certain depths as he
ascends to the ocean’s surface. 10. Which of the following sentences about
Answer: D greenhouse effect is INCORRECT?
a. Greenhouse gases trap heat in the atmosphere
5. Three liquids A, B, C were studied in a laboratory. which are returned to the earth’s surface.
Liquid A was found to float over B and C. It was b. Greenhouse effect is important in maintaining the
also found that liquid A flows fastest among the temperature of the earth.
three. What can be said about liquid A? c. Greenhouse effect is due to gases that absorb the
a. Densest and most viscous green region of light from the sun.
b. Densest and least viscous d. Greenhouse effect increases the overall surface
c. Least dense and most viscous temperature of the earth.
d. Least dense and least viscous Answer: C
Answer: D
11. When a gardener propagates a plant by taking
6. Which of the following statements represents a cuttings, he plants his cutting in a well-watered soil
physical change? in a plant pot. What is the most likely reason why he
a. An antacid tablet forms bubbles when dissolved in may then cover the plant and pot with a lightly
water. perforated polythene bag?
b. A flashlight beam slowly gets dimmer and finally a. To reduce the water demand of the cutting.
dies out over time. b. To decrease the rate of gaseous exchange by the
c. The lawn grows thicker every day because plant.
fertilizers were added into the soil. c. To reduce the chance of attack by pests.
d. Frozen mango juice melted when left standing at d. To protect the plant from cold weather.
room temperature for 30 minutes. Answer: A
Answer: D
12. It is a common observation that mushrooms
7. Which of the following best explains why farmers thrive few days after lightning strikes. Which among
burn rice straw and hull during seasons of harvest? the following biogeochemical cycles is involved in
a. Burning rice hulls and straws produce compounds this process?
that act as repellant for pests which may damage a. Nitrogen cycle
plantation. b. Carbon-Oxygen cycle
b. The smoke produced by burning rice hulls and c. Phosphorus cycle
straws stimulate growth and fruit bearing of trees. d. Sulfur cycle
c. Rice hulls and straws are burned so that more Answer: A
spaces will be available for planting next set of
crops. 13. Which of the following factors contribute to an
d. Ash from burnt rice hulls and straws are rich in increase in human population?
compounds that could neutralize acidic soil so that I. Immigration II. Emigration
more crops will grow III. Natality IV. Mortality
Answer: D a. III and IV
b. II and III
8. Acid rain occurs when _________. c. I only
a. carbon dioxide combines with water in the d. I and III
atmosphere. Answer: D
b. phosphorus-rich water in lakes evaporates to form
phosphoric acid. 14. Shown below is a simple food web in a grassy
c. sulfur released in burning fossil fuels combines community. The arrow symbol means eaten by.
with water in the atmosphere. What would happen if all snakes are killed?
d. excess hydrogen is released into the atmosphere to Grass>Grasshopper>Frog>Snake
produce acids. Bird>Snake
Answer: C a. Grass population would increase.
b. Grasshopper population would increase.
9. Why is it difficult to integrate nitrogen gas from c. Bird and frog populations would increase.
the atmosphere into the nitrogen cycle of the d. Grasshopper and bird populations would increase.
biosphere? Answer: C
a. Nitrogen is very abundant in the atmosphere
b. Living organisms quickly absorb nitrogen gas 15. Which of the following shows mechanical
c. Oceans quickly absorb nitrogen gas weathering of rocks?
d. Few organisms can directly utilize atmospheric a. formation of caverns

10
b. acids dissolves rocks d. gases, solids
c. freezing water between rock particles Answer: D
d. iron in rocks combine with oxygen
Answer: C 22. Which has a greater density, a lake full of water
or a cupful of water?
16. PAGASA announces the approach of the a. The cup full of water
seasonal winds. The familiar names used are Amihan b. The lake full of water
and Habagat, internationally known as ________ and c. Not enough information
________ respectively. d. They have the same density
a. Northeast and southwest Answer: D
b. Trade wind and easterlies
c. Southwest and northeast 23. A stainless steel spoon feels colder than a plastic
d. Westerlies and easterlies spoon because stainless steel
Answer: A a. absorbs less heat from the hand than plastic does
b. is really colder than plastic
17. Fog is a cloud with its base at or very near the c. has a lower temperature than plastic
ground. The formation of fog generally occurs after d. conducts heat away from the hand faster than
the ground has lost heat by: plastic does
a. Evaporation Answer: D
b. Convection
c. Conduction 24. Why is it NOT advisable to repeatedly open the
d. Radiation door of a refrigerator?
Answer: D a. It will loosen the hinges of the refrigerator's door
b. Leads to wastage in electrical energy.
18. Why do we see the sun rise in the east? c. Repeated opening introduces bacteria in to the
a. The earth revolves eastward. refrigerator.
b. The earth rotates from west to east. d. The warm air outside lowers the temperature
c. We are located in the 20th meridian. inside thus making the refrigeration less-efficient.
d. On the globe, we are located in the east. Answer: B
Answer: B
25. What kind of mirror is used in cars to give the
19. If a voltage of 100 volts produces a current of 5 driver a wider area and smaller image of the traffic
amperes in an electrical device, what is the behind him/her?
resistance? a. Double concave
a. 95 Ohms b. Convex
b. 20 Ohms c. Plane
c. 105 Ohms d. Concave
d. 500 Ohms Answer: B
Answer: B
26. Why do we hear thunder some seconds after
20. Which of the following best differentiates an seeing lightning?
earthquake's intensity from its magnitude? a. Light appears brighter in the sky.
a. Intensity describes 'the depth from which the b. Light travels faster than sound.
earthquake originated’ while magnitude refers to ‘the c. Sound travels 1.331 m/s
energy of the earthquake’. d. Sound is released later actually
b. Intensity cannot be measured while magnitude can Answer: B
be measured using a seismograph.
c. Intensity refers to the strength of the quake while 27. Water has a higher specific heat than iron. What
magnitude refers to the degree of destruction it does this mean?
caused at the epicenter. a. Water is hotter than iron
d. Intensity is a measure of how much damage an b. Water heats more rapidly than iron
earthquake cause at the surface while magnitude is c. Water is more dense than iron
the strength of the quake. d. Water heats more slowly than iron
Answer: D Answer: D

21. Comparing the speed of sound in liquids, gases, 28. If a colorblind man marries a woman who has
and solids, the speed of sound is usually lowest in normal vision and no history of the disease, it is
____ and highest in ____. most probable that all of their :
a. solids, gases a. daughters will be carriers
b. gases, liquids b. daughters will be colorblind
c. liquids, solids c. sons will be carriers

11
d. sons will be colorblind school even if they themselves did not go to school
Answer: A d. A married man who flirts with someone else other
than his wife seems acceptable but a married woman
29. Albino corn seedlings may grow several inches who flirts with another man is condemned.
tall. However, they will eventually die, primarily Answer: D
because
a. direct sunlight will destroy their cells 5. Which part of Asia does the Arabian peninsula
b. they lack adequate root system occupy?
c. they cannot produce their own food a. Northwest
d. they cannot obtain carbon dioxide b. Southeast
Answer: C c. Southwest
d. Northeast
30. Mimosa pudica, locally known as Makahiya is Answer: C
called as such because its leaflets tend to close when
touched. This organismal response to a given stimuli 6. The important factors which have contributed to
is called: the weakness in the internalization of desirable
a. Hydrotropism values is the?
b.Thigmotropism a. Lack of models among the very people expected
c. Geotropism to exemplify these values
d. Phototropism b. Use of approaches which are mainly cognitive
Answer: B rather than effective
c. Lack of follow up systems from one grade level to
another
d. Minimum recognition and appreciation given to
Below is the LET Reviewer for General Education
teachers
GENED: Social Science Part 1. We encourage
Answer: A
readers/ reviewees to use the comment boxes after
the article for discussion.
7. To govern is to rule and the government rules by
1. What factor was a major cause of both World War laws. Whose main duty is the enforcement of laws?
I and World War II? a. Police department
a. The spread of Marxian ideas into Europe b. Judiciary department
b. The dropping of atomic bombs c. Legislative department
c. Nationalism and national borders d. Executive department
d. The rise of totalitarian fascist states Answer: D
Answer: D
8. The term that refers to the class of Filipinos who
2. In which organization is the Philippines a member were free and independent
to fight communist aggression? a. Timawa
a. APEC b. Maharlika
b. ASA c. Aliping namamahay
c. UN d. Aliping saguiguilid
d. SEATO Answer: A
Answer: D
9. The Spanish expedition responsible for naming
3. The theory that population increases by the archipelago Filipinas
geometrical ratio while the means of subsistence a. Magellan’s expedition
increases by arithmetical ratio is attributed to? b. Loarca expedition
a. Karl Marx c. Legaspi expedition
b. Robert Malthus d. Villalobos expedition
c. Emile Durkheim Answer: D
d. Aristotle
Answer: B 10. The third and last military governor of the
Philippines was
4. In what instance is the Filipino double-standard a. Gen. Wesley Merritt
morality shown? b. Gen. Elwell Otis
a. A couple brings their sick child to the doctor then c. Gen. Arthur MacArthur
later to the espiritista d. Gen. Douglas MacArthur
b. Young parents bring up their children in a manner Answer: C
different from how they were brought up
c. Illiterate parents are eager to send their children to
12
11. Which Katipunan member commuted from a. Promoted a society ruled by religious leaders
Cavite to Manila to buy materials used to make b. Rejected violence as a way to political power
ammunitions? c. Led a successful nationalistic movement in their
a. Teresa Magbanua respective countries
b. Agueda Esteban d. Supported Marxist philosophy to change existing
c. Teodora Alonso governments
d. Trinidad Tecson Answer: B
Answer: B
19. Nebuchadnezzar was to the Babylonian Empire
12.Which economic system is based on free as Asoka was to the ______ Empire.
enterprise? a. Roman
a. Globalism b. Gupta
b. Mixed economies c. Greek
c. Capitalism d. Maurya
d. Communism Answer: D
Answer: C
20. With the opposition of the parity rights in mind,
13. How is the so-called colonial mentality who does NOT belong to the group?
manifested? a. Claro M. Recto
a. Cultural relativism b. Jose Laurel
b. Cultural diversity c. Manuel Roxas
c. Xenocentrism d. Pedro Taruc
d. Ethnocentrism Answer: C
Answer: C
21. When the Filipino reformists asked for the
14. Which is a safeguard against unfair trade assimilation of the Philippines by Spain, what did
practices like short-weighing? they ask for? For the Philippines to ____
a. Total Quality Movement a. Become independent from Spain
b. Consumerism b. Become a province of Spain
c. Consumer vigilance c. Be independent from Spain with certain conditions
d. Substandardization d. Be represented in the Spanish Cortes
Answer: C Answer: B

15. If the seven continents were arranged from 22. Who among the Presidents changed the date of
largest to smallest, in which order does Australia our celebration of Independence day from July 4 to
fall? June 12?
a. 4th a. Ramon Magsaysay
b. 5th b. Diosdado Macapagal
c. 6th c. Carlos Garcia
8. 7th d. Ferdinand Marcos
Answer: D Answer: B

16. In which continent can we find stormy Cape 23. In which country did the Philippines participate
Horn which is known as the graveyard of ships and in the world’s peacekeeping operations by sending
sailors? doctors, nurses, soldiers and police?
a. Africa a. Israel
b. Australia b. East Timor
c. South America c. Iraq
d. Asia d. Iran
Answer: C Answer: C

17. Which led to the creation of Pakistan as a nation 24. In which poem did Rizal write about offering
in 1947? Religious differences between one’s life for one’s country?
a. Hindus and Christians a. A La Juventud Filipina
b. Christians and Muslims b. Song of Maria Clara
c. Hindus and Buddhists c. Sa Aking mga Kabata
d. Hindus and Muslims d. Mi Ultimo Adios
Answer: D Answer: D

18. Which is common to Sun Yat Sen and Mahatma 25. As an effect of our geography, in which of the
Gandhi? following island/s do people travel mostly by water?

13
a. Luzon technology in education?
b. Mindanao a. Improves critical-thinking abilities.
c. Visayas b. Unlimited access to games and other internet sites.
d. Visayas and Mindanao
c. Allows cooperative learning.
Answer: C
d. Increases self-expression.
26. As an insular country, to which principle does Answer: B
the Philippines adhere when it comes to territorial
boundary? 2. Which situation shows that technology can be
a. Two hundred nautical miles of the country’s coast used to motivate students to learn?
b. Three hundred fifty nautical miles from shore
a. Teacher Eric employs educational games related
c. Three-mile territorial limit
d. Archipelagic doctrine to the lesson they are taking.
Answer: D b. Miss Cleo designs lessons which use cooperative
learning with technology integration.
27. Which part/s of the Visayas has/have c. Mr. Jimmy utilizes computer assisted instruction
comparatively more excessively moist climate and programs so that students will learn at their own
limited arable lands?
pace.
a. Western
b. Eastern d. Teacher Sarah assigns projects to her class where
c. Eastern and Central self-expression and creativity is acknowledged.
d. Central and Western Answer: A
Answer: B
3. He is considered as the “Father of Modern Media
28. Which follows Pres. Garcia’s “Filipino First
in Education”.
Policy”? Filipinos
a. Should buy and consume Filipino products only a. Jean Piaget
b. Should not contribute to the brain drain problem b. Robert Gagne
c. Should be selective in the entry in the entry of c. B.F. Skinner
foreign professionals in the country d. Edgar Dale
d. Were to be given first preference in all matters Answer: D
related to the economic development of the country
Answer: D
4. It is a term to denote a whole range of
29. Aside from the Philippines, which countries technologies associated with processing information
claim part of the Spratly islands in the South China and with sending and receiving messages.
Sea? a. Educational Technology
a. China, Thailand, Brunei, Malaysia b. Information and Communication Technology
b. China, Taiwan, Vietnam, Malaysia
c. Media Technology
c. Indonesia, China, Taiwan, Malaysia
d. China, Thailand, Brunei, Malaysia d. Instructional Systems Design
Answer: B Answer: B

30. The following are our constitutional rights 5. A methodology widely used for developing
EXCEPT to new training programs.
a. Free access to legal assistance which shall not be
a. Microsoft
denied due to poverty
b. Have access to all records of the government b. World Wide Web
c. Be presumed innocent until proven guilty c. Instructional Systems Design
d. Form association and labor unions d. Computer Technology
Answer: B Answer: C

6. This is considered to be the first manual data


Below is the LET Reviewer for General Education
processing device developed in China in the 12th
GENED: Information and Communication
century A.D.
Technology (ICT) Part 1. We encourage readers/
a. Hieroglyphics
reviewees to use the comment boxes after the article
b. Papyrus
for discussion.
c. Printing Press
d. Abacus
1. Which of the following is not a benefit of

14
Answer: D b. rules when you decided
c. rules you are deciding
d. rules when deciding
7. He is considered to be the “Father of Computing” 3. What correction should be made to this sentence?
because of his contributions to the basic design of Most State tourism departments and some travel
agencies have bed and breakfast listings
computer.
a. John Napier a. Insert a comma after agencies
b. William Oughtred b. Change have to has
c. Change tourism to Tourism
c. Blaise Pascal d. Change State to state
d. Charles Babbage 4. What correction should be made to this sentence?
Answer: D Recently, educators examined the effectiveness of
computer instruction in schools.
8. Which is the standard input device that accepts a. Replace educators with educator's
letters, numbers and commands from the user? b. Change the spelling of examined to examine
a. Trackpad c. Change schools to School
d. Replace computer with computer's
b. Lightpen 5. Which is the BEST WAY to write the underlined
c. Mouse portion of this sentence?
d. Keyboard However, if a fire extinguisher is handy, a quick-
thinking person often can use them to put out a small
Answer: D fire.

9. Mr. Rico carefully studies the materials he a. They b. Him c. Them d. lt


6. What correction should be made to this sentence?
acquired from the internet. He always examines if First-borns often pattern their behavior after they're
the author is qualified to present the material. This parents and other adults.
practice is?
a. Replace their to they're
a. necessary to ensure that the materials are reliable. b. Change pattern to patterned
b. unethical because he does not trust the author. c. Replace they're to their
c. unnecessary because all resources from the d. No correction is necessary
7. What correction should be made to this sentence?
internet are dependable. Most fire-related death's result from household fires;
d. just a waste of time. yet many people do not have fire extinguishers in
Answer: A their homes.

a. Remove the comma after fires


10. When a teacher asks the consistency of a b. Change result to results
material taken from the internet with other available c. Change have to has
d. Replace death's with deaths
materials, he/she is concerned with its? 8. What correction should be made to this sentence?
a. relevance Most fire-related death's result from household fires;
b. accuracy yet many people do not have fire extinguishers in
their homes.
c. coverage
d. currency a. Remove the comma after fires
Answer: B b. Change result to results
c. Change have to has
d. Replace death's with deaths
GE NER AL ED UC ATI ON 9. Which is the BEST WAY to write the underlined
portion of this sentence?
The hosts also benefit from running such a business
1. Which is the BEST WAY to write the underlined because they can stay at home, make money, and
portion of this sentence? meeting a variety of people
However, their VCR kept them from missing their
a. Get to meet c. And meet
favorite primetime shows. b. To meet d. And be meeting
10. Which is the BEST WAY to write the underlined
a. keepes c. hard kept portion of this sentence?
b. keeps d. keeped The studies revealing that, for various reasons, girls
2. Which is the BEST WAY to write the underlined spent less time working with computers' than boys.
portion of this sentence?
A person should keep in mind some basic safety
a. Revealing studies c. Studies' revelations
rules when you are deciding whether or not to use a b. Studies revealed d. Studies will reveal
fire extinguisher 11. Which is the BEST WAY to write the underlined'
portion of this sentence?
a. rules you decide Recently psychologists have been researching birth

15
order, their research suggests that personality and a. A second course of action
intelligence are based partly on where a child ranks in b. Without much persuading
the family. c. The second chance
d. Have a second alternative choice
a. Order. Their c. Order, or their 22. What is meant by AT SIXES AND SEVENS in this
b. Order and their d. Order their sentence?
12. Which is the BEST WAY to write the underlined We moved into the house last week, but I'm afraid
portion of this sentence? everything, is still at sixes and sevens.
There is smoke detectors in many homes to warn
residents of a fire, but fire extinguishers can actually a. The things have not been shipped.
b. In a state of confusion
help people fight fires.
c. In an orderly manner
d. The boxes are still intact
a. There is c. There are
23. Which word ends with [S] pronounced [Z]?
b. There is d. There are
a. Maps c. Laughs
13. Which is the BEST WAY to write the underlined
b. Jakes d. Buys
portion of this sentence?
24. What is meant by SOFT OPTION in these
Researchers also speculate that some teachers
sentences?
might have ‘given boys more computer time because Rebecca realized that if she stayed in her present job
parents and teachers expected, boys to need it would mean competing with an envious rival.
computers for future careers. Leaving the company would probably be a soft
option.
a. Will expect c. Will have expected
b. Expected d. Expecting
a. An action that is difficult to take
14. What correction should be made to this sentence?
b. An action that is not agreeable
Buying a fire extinguisher knowing how to use it, and
c. An action that is easier.
placing it in a location familiar to all family members d. An action that is weakly funded
can help protect families against fire. 25. Which word contains the voiced Th?
a. Thankb. These c. Think d. Thing
a. Insert a comma after extinguisher 26. To gain the attention of the audience, the trick is
b. Change the spelling of families to family __________?
c. Change placing to place a. start low, speak hurriedly
d. Insert a comma after help b. start high, speak loudly
15. Which is the BEST WAY to write the underlined c. start high, speak rapidly
portion of this sentence? d. start low, speak slowly
Many viewers taped shows to watch later. 27. Which word contains the [ae] sound?
a. Carriage b. Cabin c. Caste d. Can
a. Tapped c. Tape 28. Which of the following words DOES NOT contain the
b. Had taped d. Had tapped [voiceless th]?
16. What is meant by LAST DITCH in this sentence? a. Mouth b. Breath c. Teeth d. Health
The aged bishop was prepared to fight to the last 29. Which pitch is used for the word STRANGE?
ditch to defend his good name. What a strange story!

a. One's last courage c. One's last hope a. 3 b. 2 c. 4 d. 1


b. One's last strategy d. One's last defense 30. What do the following lines CONVEY?
17. Which ward in the passage does NOT require a Midnight, not a sound from the pavement.
change in pitch to show confidence?
a. Am b. Captain c. Master d. Fate Has the moon lost her memory?
18. Which word is read on a high note to describe the
kind of day? She is smiling alone.
This is a cold day.
In the lamp light the withered leaves
a. Is b. CoId c. Day d. This
19. Which verb in the sentence is pronounced with the Collect at my feet
ending as [d]?
They laughed and joked as they walked and played. And the wind begins to moan

a. Joked c. Walked a. Confusion c. Loneliness


b. Laughed d. Played b. Optimism d. Eagerness
20. What is meant by the expression TO GET BLOOD 31. What is the mood of these lines?
OUT OF A STONE in these sentences? Daylight, I must wait for the sunrise
Geraldo has owed me fifty thousand pesos for over a
year now. I’ve asked him for it on several occasions, I must think of a new life
but it’s like trying to get blood out of a stone
And I mustn't give in.
a. Something is impossible.
b. Someone refuses to cooperate. When the dawn comes
c. Someone is willing to give what is asked.
d. Someone wants revenge. tonight will be a memory, too
21. What is meant by TWO PINS in this sentence?
For two pins I could have hit him on the nose And a new day will begin.

16
a. Afraid c. Depressed 38. Anong uri ng panghalip ang salitang may salungguhit
b. Sarcastic d. Hopeful sa pangungusap? May padalang tulong ang
32. What is suggested in the opening line? pamahalaan para sa kanila.
June 13, 1986 - they came from all over America a. Pambalana c. Paari
- 200,000 heroes strong, with their families. b. Palagyo d. Palayon
39. Nasa anong kaganapan ng pandiwa ang
a. The writer holds great admiration for the veterans pangungusap?, Naglaro ng basketball sa Rizal
b. The writer was a veteran of the war Stadium ang koponan ng aming pamantasan.
c. The writer is opposed to the Vietnam War a. Sanhi c. Kagamitan
d. The writer is a flag-waving patriot b. Tagaganap d. Ganapan
33. Carl Sundburg wrote "Jazz Fantasia" which has for its 40. Alin uri ng parirala ang may salungguhit sa
first stanza: pangungusap? Utang sa kanyang sipag at sikap sa
Drum on your drums, batter on your banjos, sob paggawa ang kanilang maalwang pamumuhay.
on the long cool winding saxophones. Go to it, O a. Pangngalan c. Pawatas
b. Pangngalang-diwa d. Pang-ukol
jazzmen. Which words illustrate alliteration?
41. Si Dr. Jose Rizal ay sumulat ng aklat ng itinampok sa
ibat' ibang bansa. Ang pangungusap ay nagagamit
a. Batter and banjos c. Sob and winding
bilang __________?
b. Long and cool d. To and it
a. panuring c. tuwirang layon
34. Which of the following lines is a simile?
b. pamuno d. paksa
a. "Holding wonder like a cup"
42. Kilalanin ang uri ng pariralang may salungguhit. Ang
b. "Life has loneliness to sell"
pangangalaga sa mga likas na yaman ay tungkulin
c. "Eyes that love you, arms that hold”
nating lahat.
d. "Buy it and never count the cost"
a. Pangngalan c. Pang-ukol
35. The stanza below is taken from "Barter" by
b. Pangngalang-diwa d. Pawatas
SaraTeasdale.
43. Sabihin ang aspekto ng pandiwa sa pangungusap na
Life has loneliness to sell,
ito. "Mag-aral sa bahay ng mga araling ukol sa
halaman."
Music like a curve of gold,
a. Pawatas c. lmperpektibo
b. Kontemplatibo d. Perpektibo
Scent of pine trees in the rain,
44. Dadalaw sa mga paaralan si Dr. Filemon S. Salas,
ang tagapamanihala ng mga paaralang lungsod, sa
Eyes that love you, arms that hold,
lungsod ng Pasay. Ang pangungusap ay nagagamit
bilang
And for your spirit's still delight, a. panuring c. tuwirang layon
b. paksa d. pamuno
Holy thoughts that star the night. 45. Alin ang di karaniwang anyo ng pandiwang HINTAY
KA?
To what does Teasdale compare music? a. Tay b. Tayka c. Intay d. Teka
46. Ano ang ipinahihiwatig ng salitang may salungguhit?
a. The scent of pine trees c. Eyes that love Matayog anq lipad ni Pepe kaya't bata pa siya'y
b. A curve of gold d. The rain nagsisikap na siya.
36. Robert Frost wrote the poem "Acquainted with the a. May kayabangan si Pepe.
Night" from which the stanza is taken: b. Mataas ang pangarap ni Pepe.
c. Marunong si Pepe.
I have been one acquainted with the night. d. Ibig ni Pepeng maabot ang langit
47. Sa "Espiritu ni Bathala ang nangangalaga ng
I have walked out in rain-back in rain. kanilang kalusugan" ang ipinahihiwatig na katangian
ay __________.
I have out walked the farthest city light a. malinis c. maliksi
b. mabisa d. makapangyarihan
The poet in the stanza talks of 48. Alin sa mga salita ang kasingkahulugan ng salitang
may salungguhit?
a. isolation and loneliness. Ang ama ni Anita ay kilalang bulanggugo sa
b. happiness in having been acquainted with the kanilang lalawigan.
night.
c. joy getting out of the house.
a. Laging ibinubulong
d. youthful delight playing in the rain.
b. Laging handang gumasta
37. Lines 11 and 12 are taken from the poem "maggie
c. Laging handang makipag-away
and milly and molly and may."
d. Laging handang makipagtalo
For whatever we lose (like a you or a me)
49. Anong tayutay ang tinutukoy nito?
Durog ang katawang bumagsak sa semento si Miguel.
it's always ourselves we find in the sea
a. Pagtutulad c. Pagmamalabis
Which of the following ideas is the author expressing? b. Pagbibigay katauhan d. Pagwawangis
50. Sa alin salita magkakaroon ng saglit na paghinto
a. The sea is a source of life and death. kung pinapilitang si Rose ang nakabasag ng
b. The sea represents all of our moods. pinggan? Hindi si Rose ang nakabasag ng pinggan.
c. The sea is the best place for a person to reflect a. Rose c. Nakabasag
about life. b. Hindi d. Pinggan
d. The sea and its surroundings can give people a 51. Alin ang kahulugan ng KAHIRAMANG SUKLAY?
fresh view on life. a. Kakilala b. Kaibigan c. Karibal d. Kalahi

17
52. Alin antas ng tono ang lumilitaw sa bahaging may a. Ang pagbabalik ay dapat ipaghanda nang malaki.
salungguhit ng pangungusap na b. Ang pagbabago ng kapatid ay dapat
nagsasalaysay?Magbabasa ng mga gawain ang pahalagahan.
guro. c. Dapat silang magsaya sa muli nilang
a. 1 b. 3 c. 2 d. 4 pagsasama-sama
53. Alin antas ng tona ang lumilitaw sa bahaging may d. Ang pagsasama nila ay dahil sa muling
salungguhit ng pangungusap na nagdududa? pagbabalik ng kapatid
Nagpuputol ng puno ang lalaki. 62. Si Mariano Ponce ay propagandistang may sagisag
a. 1 b. 2 c. 4 d3 sa panulat na
54. Alin ang kahulugan ng AGAW-BUHAY? a. Tamaraw c. Kapre
a. Masiglang-masigla b. Tikbalang d. Kalapate
b. Malapit nang mamatay 63. Anong aral ang ibinibigay ng sumusunod na
c. Pagpapatuloy ng buhay salawikain?
d. Mahirap na buhay "Ang taong napapailalim, ay naipapaibabaw rin."
55. Alin ang naayong pamagat sa tanagang sinulat ni
Jose Villa Panganiban? a. Maaring ngayon ay hirap pagdating ng bukas ay
Ano man sa daigdig, may ginhawa rin.
b. Tiyak ang pag-unlad kapag nauna ang hirap
Maaring magamit, c. Kung ano ang ibig natin ay mangyayari
d. Magtiis kung dumarating ang hirap
Ano mang masaisip; 64. Alin ang di karaniwang anyo ng pandiwang WINIKA
KO?
Di sukat maiipit. a. Ikako c. Kako
b. Wikako d. Wika ko
a. Pagkainip c. Pag-asa 65. A politician wants to get his message to 2/3 of the
b. Paraya d. Pagbibigay population of 48,000 in Bulacan. However, his
56. Ano ang kahulugan ng taludtod na ito? advertising campaign reaches only 3/4 of the number
"Ang anak man ay alagaan sa marubdob na he intended. How many people does he actually
pagsuyo sikapin ma sa sarili'y huwag siyang maging luko reach?
talipandas sa paglaki na sa sama marahuyo sa lahi mo't a. 16,000 c. 24,000
b. 10,000 d. 36,000 e. none of these
sa Bayan may isang tinik sa balaho."
66. Mr. Garcia owns a 10½ hectare tract of land. He
plans to subdivide this tract into ¼ hectare lots. He
a. Mahalin ang anak ng walang hangganan.
must first set aside 1/16 of the total land for roads.
b. Tamang pagpapalaki sa anak ang dapat.
How many lots will this tract yield?
c. Suyuin ang anak at ibigay lahat ng hilig.
a. 30 b. 35 c. 42 d. 45
d. Paligayahin ang tahanan.
b. none of these
57. Kaninong tula hango ang sumusunod?
67. A meter was cut at the 35-cm mark. What is the ratio
"Ang hindi magmahal sa sariling wika mahigit sa
of the smaller piece to the larger piece?
hayop at malansang isda." a. 7:13 c. 35:100
b. 65:35 d. 65:100 e. none of these
a. Jose Rizal c. Apolinario Mabini 68. The carat is a unit of measure used to weigh precious
b. Emilio Jacinta d. Graciano Lopez Jaena stones. It equals 3.086 grains. How many grains does
58. Alin sa mga sumusunod ang aral na ibinibigay ng a 2.8 carat diamond weigh?
epikong Muslim na INDARAPATRA ATSULAYMAN? a. 864.08 c. 8.6408
a. Pagmamahal c. Katapatan b. 86.408 d. 8640.8 e. none of these
b. Katapangan d. Pagtanaw ng utang-na-Ioob 69. A carpenter wanted three pieces of wood each 1⅝
59. Alin sa mga sumusunod ang aral na ibinigay ng ANG feet long. If he planned to cut them from a 6-foot
ALAMAT NI MARIANG MAKILlNG na ikinuwento ni piece of wood, how much of the piece would be left?
Jose Rizal? a. 4 ⅜ ft c. 4 ⅞ ft
a. Pagyamanin at pangalagaan ang ating bayan at b. 1 ⅛ ft d. 3 ft e. none of these
lahi pagka’t hiyas at yaman natin ito. 70. Four mangoes cost P29.00. At that price what will 2½
b. Pag-ibig ang makapagbabago sa mundong ito. dozen mangoes cost?
c. Kabanalan ang magpatawad at tulungan ang a. Php217.50 c. Php348.50
isang nagkasala. b. Php188.50 d. Php870.00 e. none of these
d. Dahil sa pagmamalabis at pagsasamantala, 71. If P75.00 is shared among three children in the ratio
maraming biyaya ang sa kanya'y nawawala. of 3:7:15. The size of the smaller share is
60. Alin sa mga sumusunod ang mensahe ng epiko ng a. Php9.00 c. Php25.00
IIokano na BIAG NI LAM-ANG? b. Php15.00 d. Php35.00 e. none of these
a. Pinatutunayan ng epiko ang yaman ng Ilokano sa 72. 4 1/5 + 3 2/7 = __________
lahat ng bagay. a. 7 3/12 c. 7 17/35
b. Kailangan paniniwalaan ang ukol sa bisa ng mga b. 7 3/35 d. 7 1/35 e. none of these
anting-anting dahil sa mga pangyayaring 73. 10 - 3 2/17 = __________
nagpapatunay dito. a. 7 2/1 7 b. 6 1/17
c. Dito nagpapatunay na walang kamatayan. b. 6 2/1 7 d. 6 15/17 e. none of these
d. Masasalat ang mga katutubong ugali at mga 74. How many whole numbers can divide 30 exactly?
tradisyong dapat pagyamanin at panatilihin a. Eight b. Six c. Five d. Four e. none of these
upang pakinabangan ng kabataan. 75. Which is the sum of the infinite progression 3/2, 1,
61. Ano ang pinakaangkop na kahulugan nito? 2/3, 4/9 . . . ?
"Nagsasaya tayo ngayon sapagkat ang iyong a. 6½ b. 5½ c. 4½ d. 7½ e. none of these
namatay na kapatid ay muling nabuhay; ang nawawala 76. In the progression 18, -12, 8 . . . which term is
ay muling nakita." 512/729?

18
a. the 8th c. the 9th a. 6 b. 8 c. 12 d. 24 d. none of these
b. the 6th d. the 7th e. none of these 92. One package is 100 pounds, and the other is 150
77. Which are the next three terms in the progression 1, pounds. The weight of the second package is how
4, 16 . . . 8 terms? many times that of the first?
a. 64, 256, 1024 c. 66, 258, 1026 a. 1½ times heavier d. 10 pounds heavier
b. 67, 259, 1027 d. 65, 257, 1025 e. none of these b. ½as heavy
78. Which are the next three terms in the progression c. none of these e. 20 pounds heavier
1/125, -1/25, 1/5 . . . 7 terms? 93. How many gallons of water will fill a fish tank that is
a. -2, 6, -26 c. -4, 8, -28 18 inches by 12 inches by 48 inches? (There are 231
b. -3, 7, -27 d. -1, 5, -25 e. none of these cubic inches per gallon). Round your answer to the
79. Which is the equivalent common fraction of the nearest gallon.
repeating decimal 3.242424…? a. 45 gallons c. none ofthese
a. 107/33 c. 109/33 b. 40 gallons d. 47 gallons e. 38 gallons
b. 110/33 d. 108/33 e. none of these 94. How much topsoil is needed to cover a garden 25
80. What is the difference between the largest 4-digit feet by 40 feet to a depth of 6 inches?
number and the smallest 4-digit number? a. 480 ft3 c. 500 ft3
a. 8999 c. 9998 b. 440 ft3 d. none of these e. 460 ft3
b. 8000 d. 8888 e. none of these 95. What are the next four numbers in this sequence 8, 5,
81. What is the sum of all the two digit numbers which 4, 9,17, ___, ___, ___, ___?
are divisible by 5? a. 4, 3, 2, 1 c. 6, 3, 2, 0
a. 945 b. 950 c. 960 d. 1050 b. 5, 4, 3, 2 d. none of these e. 3, 2, 1, 0
82. Find m in the proportion m/12 = 30/24. 96. What day follows the day before yesterday if 2 days
a. 30 b. 15 c. 20 d. 25 e. none of these from now will be Sunday?
83. The Miranda Family purchased a 250-pound side of a. Tuesday c. Wednesday
beef and had it packaged. They paid Php365.00 for b. none of these d. Saturday e. Thursday
the side of beef. During the packaging, 75 pounds of 97. How much larger is the supplement of a 57 degree
beef were discarded as waste. What was the cost per angle than the complementof a 75 degree angle?
pound for the packaged beef? a. 108 degrees c. 105 degrees
a. Php2.08 per pound d. Php2.06 per pound b. 18 degrees d. none of these e. 123 degrees
b. none of these 98. The hypotenuse of a right triangle is 25 feet. If one
c. Php2.50 per pound e. Php2.30 per pound leg is 24 feet, what is the length of the other leg?
84. If a baseball player hits 10 home runs in the first 45 a. 5 ft b. 5 ft c. 20 ft d. none of these e. 7 ft
games, at the same rate how many home runs can 99. Which is the length of the hypotenuse of a right
he expect to hit during the 162-game season? triangle with legs 5 inches and 12 inches?
a. 38 b. 42 c. 36 d. 40 e. none of these a. 17in. b. 13 in. c. 11in. d. 20 in e. none of these
85. During a recent shopping spree, Tomas and Nena 100. Which one should be TRUE if Earth's rotation
bought some new accessories for their apartment. axis did not tilt?
Nena chose a crocheted throw pillow at Php24.95, I. Days and nights would be the same length everywhere
and Tomas purchased a rural landscape painting for on Earth.
Php135.00. How much did they actually spend if they
paid 7% sales tax on their purchases? II. There would be no hours of darkness on points along
a. Php171.15 c. none of these the equator.
b. Php139.25 d. Php159.95 e. Php148.75
86. You buy a new refrigerator for Php12,800.00 and III. Earth would have no seasons,
make a down payment of Php2,500.00. If you finance
the remainder at 8% annually for three years, how IV. Each part of the Earth would have the same daily
much will you actually pay for the refrigerator?
temperature pattern.
a. Php12,190.00 c. none of these
b. Php10,300.00 d. Php12,772.00 e.Php15,272.00
a. II and IV c. IVand I
87. To pass her English Test, Lucille must get 75% of the
b. III and IV d. I and Ill
items correct. Out of 80 questions, how many must
101. Tides, caused by the moon's gravity, create a
she correctly answer?
frictional force that is gradually slowing down Earth's
a. 55 b. 60 c. none of these d. 65 e. 70
rotation speed. One million years from now, scientist
88. Employees at Shaira's Musicmart get a 20% discount
may discover that compared to today, Earth's
on all purchases. If Theresa buys three tapes at
a. day is shorter c. day is longer
Php7.49 each, how much will she have to pay after
b. year is longer d. year is shorter
her employee discount?
102. What indoor relative humidity range would
a. Php16.98 c. Php18.98
probably be comfortable when the outside
b. Php19.98 d. Php17.98 e. none of these
temperature and humidity levels are extremely low?
89. A car dealer is offering a rebate of Php7,500.00 on
a. 90 to 100% c. 20 to 30%
any new-car purchase. If the purchase price of a car
b. 50 to 70% d. 30 to 40%
is Php200,000.00 more than it was last year, what is
103. Which of the following facts support the Big Bang
the rate of discount offered by the rebate?
theory's explanation of the creation of the universe?
a. 10% c. 13.3%
a. The universe does not expand nor contract.
b. not enough information is given
b. The universe seldoms expand.
c. 7.5% e. 14.2%
c. The universe will have background radiation.
90. How many ways can a committee of 4 people be
d. The universe has no beginning nor end.
selected from a group of 7 people?
104. Which of the following is the BEST example of
a. 35 b. 70 c. 140 d. 210 e. none of these
self-preservation?
91. In how many ways can you arrange three
a. mouse runs when it sees a cat.
mathematics books (Algebra, Geometry,
b. A dog barks when it sees its owner.
Trigonometry) in order on a shelf?
c. A young man decides to quit smoking

19
d. A salmon swims back to the place of its birth to of students who are the same age, height, and
lay eggs. weight?
105. Which of the following internal forces interrupt the a. Athletes would tend to have higher rates of
external forces of erosion? cellular respiration than non-athletes.
I. Forces that cause volcanoes b. Africans would have a higher rate of cellular
respiration than Asians.
II. Forces that cause ocean trenchers c. Boys would have a higher rate of cellular
respiration than girls.
III. Forces that create mountains d. Non-athletes would have higher rates of cellular
respiration than athletes.
a. I, II and III c. II and III 117. What is the function of DIFFUSION in the human
b. I and III d. I and II body?
106. Which location should have most nearly twelve a. Regulates blood flow
hours of daylight and twelve hours of darkness during b. Plays an insignificant role in the body's
December? A town that is located functioning
a. halfway between the Equator and the South Pole. c. Allows an even distribution of substances
b. close to the Equator. throughout all cells of the body
c. close to the North Pole. d. Comes into play in times of extreme illness
d. close to the South Pole. 118. What is the MOST likely reason for a desert plant
107. At which time during the year does the ozone to have a few or no leaves?
level present a particular health threat in urban areas a. To increase photosynthesis
for people with respiratory problems? b. To decrease photosynthesis
a. Spring c. Summer c. To increase transpiration
b. Fall d. Winter d. To decrease transpiration
108. Which of the following procedures used by a 119. It was also discovered that the chemical 3m
farmer is NOT related directly to preventing erosion? butylphthalide can lower the blood pressure of rats.
a. Contour plowing around a hill This statement is classified as
b. Planting more seeds than are necessary to yield a. prediction c.finding
a bountiful crop. b. experiment d. nonessential fact
c. Planting grass in gullies to act as a filter 120. Which kingdom should STREPTOCOCCUS be
d. Planting crops in alternate rows (strip farming) classified?
109. A nation in which loess would likely be found a. Protista c. Fungi
today is b. Plantae d. Monera
a. Iceland c. Japan 121. Which of the following methods can all diabetics
b. United States d. Ecuador control their condition and avoid heart disease and
110. Which one explains why oxygen a gas, is the blindness?
largest component of the Earth’s crust? I. Regulating their intake of glucose
a. Oxygen gives Earth's crust its lightness
b. Oxygen is the most abundant element in the II. Increasing the levels of insulin in the body by
world taking insulin injections
c. Oxygen is capable of combining with most of the
elements in the Earth's crust. Ill. Maintaining a reasonable exercise regimen to
d. Oxygen is needed to sustain all life on Earth. keep weight down.
111. Which is the BEST evidence that helium gas is
lighter than air? a. I b. II c. I and II d. I and III
a. Helium has the lowest boiling point of all 122. Without the process of meiosis, we can infer that
elements. offspring from sexual reproduction would
b. Helium atoms do not combine with other air a. have a high degree of genetic variety.
atoms. b. have twice the assigned number of
c. Helium-filled balloons rise in air. chromosomes.
d. By volume, helium makes up only 0.0005% of air. c. be identical.
112. In which kingdom should MOLD be classified? d. have a number of mutations.
a. Protista c. Fungi 123. Which method of reproduction provides for the
b. Plantae d. Animalia most variety of offspring?
113. Which area of the brain controls feeling on the left a. Cloning c. Asexual reproduction
side of a person's face? b. Sexual reproduction d. Cellular reproduction
a. The left occipital lobe c. The right parietal lobe 124. What does a stick of unlit dynamite demonstrate?
b. The left parietal lobe d. The right occipital lobe a. Chemical Energy c. Kinetic Energy
114. Scientist also find that other stalky vegetables b. Nuclear Energy d. Potential Energy
such as carrots also help lower blood pressure. This 125. The principle under which a thermostat operates
statements is BEST classified as is the same when
a. experiment c. finding a. a gas expands to fill the container in which it is
b. nonessential fact d. prediction held.
115. The Jones family has had four children, all girls. b. a pendulum swings when it is set into motion.
The fifth child born is a boy. This change is the result c. a chemical reaction occurs when two substances
of combine.
a. conception classes taken by the parents. d. the level of mercury rises or falls in a glass tube.
b. the timing of the fertility cycles. 126. What do you predict will happen when you bring
c. the father's contribution of a "Y" chromosome. two bar magnets closer together?
d. the "law of averages" finally catching up. a. They will repel each other.
116. Which of the following should you expect to be b. They will create an alternating current
true about the rate of cellular respiration for a group c. Nothing will happen

20
d. They will attract each other. reproductive form resembles a
127. Which of the following BEST demonstrates the a. bulb b. runner c. seed d. bud
greenhouse principle? 137. Which antidote would have a similar effect if
a. A heated aquarium vinegar or citrus juice were not available?
b. A car with rolled-up windows a. Milk c. Vegetable oil
c. A microwave oven b. Raw egg white d. Water
d. A solar battery -powered calculator 138. Which is NOT among the hazardous effects of
128. Which law of force and motion explains this water pollution to health?
occurrence, when a rocket is propelled upward by the a. The epidemic threat of hepatitis and dysentery
powerful downward discharge of exhaust gases? b. The increase incidence of liver cancer
a. Universal Gravitation c. Applied Force c. The dumping of mercury in the sea causing
b. Action and Reaction d. Inertia blindness, brain damage, or death
129. Why does a pendulum in a grandfather clock d. The presence of certain bacteria in the digestive
once set in motion continue to swing, thereby tract causing Methemoglobinemia
regulating the clock's movement? This is due to the 139. A tightly coiled spring demonstrate?
Law of a. Steam energy c. Potential energy
a. Universal Gravitation c. Applied Force b. Kinetic energy d. Chemical energy
b. Action and Reaction d. Inertia 140. As a representative of the Urban Poor
130. Why does a bullet when discharged into the air Commission of the Association of Religious Superiors
eventually fall to the ground? This is due to the Law (ARS), which action will you most likely take to
of resolve the long-term roots of structural inequalities-
a. Universal Gravitation c. Inertia proliferation of child labor and child prostitution?
b. Applied Force d. Action and Reaction a. Raise views of human rights abuse.
131. Which of the following will occur if a cold bottle of b. Organize regular programs for information and
soda is left open on a kitchen counter? discussion of human rights.
a. The pressure that the soda exerts on the bottle c. Conduct skills training.
will increase. d. Raise questions over the government's
b. The temperature of the soda will decrease. commitment to rebuild human rights
c. The amount of dissolved carbon dioxide gas will 141. Which explains the reason why there are
decrease. continuous and increasing human rights violations?
d. The amount of dissolved carbon dioxide gas will a. The United Nation's General Assembly approved
remain the same. only resolutions on human rights and the basic
132. In an experiment, a drop of blue ink is placed on freedoms which are not binding.
the surface of a glass of water. In a few minutes, the b. The solutions used are ineffective.
drop of ink is dispersed throughout the water, turning c. The United Nations as an international body is
it light blue. The result of the experiment proves that rather slow in the exercise of its powers.
a. molecules of ink and molecules of water are in d. The United Nations uses a single solution on all
constant motion. forms of human rights violations.
b. heat causes the ink to disperse. 142. The following are legitimate children, EXCEPT
c. a new compound is formed by the combination of a. those born by artificial insemination.
ink and water. b. those legitimated.
d. ink molecules have less density than water c. those born during a valid marriage of parents.
molecule d. those born out a valid marriage of the parents.
133. Each of the following objects is designed to 143. Thousands of street children in large Brazilian
employ the buoyancy principle EXCEPT a cities were murdered by paramilitary death squads
a. life preserver c. submarine which included police officers. What could be the
b. kite d. canoe reason why these operations were not suppressed by
134. An elderly woman suffered a stroke, a restriction the government?
of blood flow to the brain. If the stroke caused to the a. Totalitarian governments do not give protection to
right side of her body to become temporarily their people.
paralyzed, she most likely experienced a decreased b. The business people even funded these
blood flow to operations to clean up their streets and
a. the left side of her body. neighborhoods.
b. the front of her brain. c. The Universal Declaration of Human Rights was
c. the left side of her brain. only lip service.
d. the right side of her brain. d. To how they treat their people was nobody else's
135. In an experiment, a vacuum is created when all business.
air is removed from a tube. A coin and bits of confetti 144. Spouses Jose S. Luz and Celerina Luz filed a
are released in the vacuum at the same time. They petition to adopt Gregorio Luz Ona, their nephew.
fall at the same rate and reach the bottom at the The spouses are childless and they reared Gregorio
same time. The experiment proves that from his birth in 1971 until 1975 and they continued to
I. in a vacuum, the rate of acceleration is the same support him. Gregorio had to be left in the Philippines
for all objects regardless of weight. when the spouses went to the United States where
II. outside a vacuum, air resistance is what makes Jose is employed. The MSSD recommended the
different objects fall at different rates. adoption to the court on the premise that petitioners
III. gravity has no effect at all on objects that fall in a are in a better position to provide for the minor child
vacuum. than the natural parents who are impoverished.
a. I and II c. I, II and III What could be the possible decision of the court on the
b. I and III d. II and III petition?
136. New peonies, perennial plants that produce
showy flowers can be propagated from the parent a. The court denied the petition because the
plant by dividing corns that grow underground. This spouses are already aliens

21
b. The petition for adoption was granted because base in Cuba is interpreted as a direct violation of
the court finds that it is to the best interest of the a. the Truman Doctrine
child. b. the Monroe Doctrine
c. The petition for adoption was denied because the c. the Strategic Arms Limitations Talks (SALT)
spouses are non-residents of the Philippines. d. the Declaration of Independence
d. The court denied the petition on the premise that 154. Which will solve poverty caused by capitalism?
the trial custody requiredby PD 603 cannot be a. Fascism c. Empirism
effected for spouses are non-residents b. Communism d. Socialism
145. Which one is the human right to life? 155. Which part of the Allied action has been detailed
a. Peace in this passage?
b. Live in national and international order After Saddam Hussein violated international
c. Own property agreements by sending Iraq troops to Kuwait and missiles
d. Fair trial into other neighboring countries, the Allies responded
146. Which one is the human right to dignity?
with military action.
a. Political independence
b. Honor and reputation
a. The Allied bases in Saudi Arabia
c. Form associations
b. The ground was in the desert
d. Social and economic reforms
c. The movement of Allied Troops with Iraq
147. Carolina Diaz filed a petition for habeas corpus
d. The air campaign in the Baghdad area
against Mr. and Mrs. Ramon Alde to recover custody
156. Carter's part in relinquishing U.S. control of the
of Lina Diaz Tan alias "Gracia Alde," the natural
Canal Zone to Panama is described as a victory for
daughter of Carolina Diaz, who was a hostess.
__________.
What could be the possible action of the court on the
a. conservatism c. isolationism
petition filed by Carolina Diaz? b. anti-imperialism d. imperialism
157. The undeclared war in Korea most closely
a. Her petition would be granted because she now resembled the situation of
works as a clerk in a prestigious office. a. The Spanish-American War c. World War I
b. Her petition would be denied because she was a b. The Vietnam War d. World War II
former hostess. 158. Which refers to FUNNEL EFFECT?
c. Her petition would be denied because when a. The belief that every criminal gets caught and is
Gracia was given to the Aldes it was tantamount punished.
to abandonment of the child, resulting to b. The belief that crime is under control in the
termination of parental authority. United States.
d. Her petition would be granted because she is the c. The idea that only a very few suspects arrested
natural mother. for committing a crime are actually punished.
148. Mary Rose, an 18 year old was sexually abused d. The idea that all crimes are put into the same
by 3 teenagers from well-to-do families from Makati. criminal justice system.
Despite pressures, she came out into the open to get 159. What values are being given priority by juries in
justice. Which need did Mary Rose satisfy? criminal cases?
a. Need for family unity a. The rights of the criminal over the strict
b. Need for civic responsibility interpretation of the law
c. Need for universal solidarity b. The safety of the community over symphathy for
d. Need for personal integrity the criminal
149. The following are defects present at the time of c. The needs of the criminals over the advice of the
marriage which is voidable and annullable EXCEPT judge
a. impotence c. fraud d. The punishment of the criminal over the safety of
b. deceit d. threat the community
150. Which one is the right to human dignity? 160. Two days after Japan attacked Pearl Harbor,
a. Choose the goals and means of development Roosevelt made the following statements:
b. Share in scientific and technological advances of "In the past few years and most violently in the past few
the world
days, we have learned a terrible lesson. We must begin
c. Right to information
d. Sovereignty over our natural resources the great task that is before us by abandoning once and
151. Which of the following foreign policy actions for all the illusion that we can never again isolate
today is a direct result of early American imperialist ourselves from the rest of humanity."
policy?
a. Stationing of American troops in West Germany In the statement, Roosevelt is expressing the ideas of
b. America's military support of Israel in the Middle __________.
East
c. America's patrolling of waters off the Libyan coast a. an internationalist c. an imperialist
d. America's establishment of military bases in the b. an anti-imperialist d. an isolationist
Philippines 161. The main message of the Moral Recovery
152. Which of the following represent, ethnocentric Program launched by Leticia Ramos Shahani starts
behavior? with the __________.
a. A tourist who lectures his foreign hosts on the a. world b. family c. self d. nation
"uncivilized" nature of their marriage customs 162. Which is NOT personal integrity?
b. A student who tutors an immigrant in English a. Time c. Order
c. A Hispanic community group demands that public b. Place d. Harmony
aid forms be published in English and Spanish 163. Which one BEST defines personal integrity?
d. A peace Corps volunteer who helps dig wells in a. The unity between ignorance and reality
Central Africa b. The unity of man’s deeds, words, thoughts and
153. The Soviet Union's attempt to establish a missile realities

22
c. The unity of man’s social, political and physical c. His loan was Php50,000.00
aspects d. His loan was Php40,000.00
d. The relationship between virtue and conduct 174. Julieta Villaruel was a landowner from Cabiao,
164. Which family obligation is especially valued? Nueva Ecija. Under the CARL, she was claiming 8
a. Supplying groceries to relatives in remote barrios hectares, 5 hectares of which represented the
b. Providing health assistance to relatives living in retention limit and the 3 hectares for her only child.
the same locale Why was her child denied the 3 hectares?
c. Sending to college relatives in remote barrios a. Her son was 15 years old who was actually tiling
d. Keeping immediate family members out of trouble the farm
165. Of the following changes in the socio-economic, b. Her son was 17 years old who was managing the
political cultural and physical that have occurred in farm
the Filipino family, which one remains to be TRUE? c. Her son was 13 years old who has been helping
a. The loss of the traditional evening prayer and the till the farm
ritual of blessing (mano) d. Her son was 19 years old who was actually tilling
b. The unity of the family despite competing or managing the farm
demands 175. One of the most outstanding accomplishments of
c. The continued parental influence over children's the cooperative movement is the encouragement of
language dress and other behavior thrift. Which maxim of God puts this into practice?
d. The continued support for parents and siblings a. "God helps those who help themselves"
166. Which is the MOST important perceived need b. "Look at the birds: they do not plant
and problem of the Filipino family? seeds,/gather a harvest and put it in barns; yet
a. Unemployment or financial problem your Father in heaven takes care of them!"
b. Proneness to vices c. "He is near to those who call to Him, who call to
c. Over protectiveness of children Him with sincerity"
d. Double standard on the roles of male and female d. "Happy are those who are merciful to others; God
167. The San Pascual Credit Cooperative of Quezon will be merciful to them!"
City wishes to appy for a loan of five million pesos 176. Mang Tacio has been unemployed for quite
from one of the financial institution, EXCEPT sometime due to his negative attitudes toward work.
a. Development Bank of the Philippines Which program of the Department of Social welfare
b. Central Bank of the Philippines. and Services will help him?
c. Philippine National Bank a. Income in Kind Program
d. Land Bank of the Philippines b. Anti-Medicancy program
168. If a farmer would want assistance like pricing, c. HRD Program
guarantee for all agricultural produce or cooperative d. Social Insurance Program
management training, where would he go? 177. Lucy's husband has been a drug dependent. She
a. Support Services of the Department of Agrarian wanted him rehabilitated to be economically
Reform productive. Where will she commit her husband?
b. Special Agrarian Court under the Regional Trial a. DARN c. DARE
court b. Bukang Liwayway Center d. NFPI
c. DAR adjudication board 178. Where would you commit a drug dependent for
d. Land bank him to achieve a natural, tensionless, and anxiety-
169. Why was San Andres Cooperative Association of free state?
Paco, Manila not exempted from taxation? a. NFPI c. DARE
a. It accumulated reserves and undivided net b. DARN d. Bukang Liwayway Center
savings of Php8,000,000.00 179. Which theory was asserted by the Pan-German
b. It accumulated reserves and undivided net belief in the superiority of the Aryan race and that the
savings of Php10,000.00 strength of the German culture came from a strong,
c. It accumulated reserves and undivided net healthy and rustic lineage?
savings Php9,000,000.00 a. DependencyTheory c. Social Darwinist Theory
d. It accumulated reserves and undivided net b. Culture of Poverty Theory d. Theory of Capitalism
savings of Php11,000.00 180. Which is the main goal of drug abuse education?
170. Which has become a prototype of other schemes a. Arrest c. Control
that defeated the real and true purpose of the CARL? b. Prevention d. Rehabilitation
a. The conversion of farmlands to industrial
complexes
b. The stock option scheme of Hacienda Luisita Situation 1 - Below is the poem written by Edgar Lee
c. The conversion from agricultural to subdivision Masters in 1915:
d. The voluntary offer to sell
171. Which are limited only to the sale of real property What Happened to Tom Merritt?
and stock transaction?
a. Business incomes c. Employment incomes At first I suspected something
b. Capital gains d. Passive incomes
172. Of the following, which is imposed a final tax of She acted so calm and absent-minded.
ten percent (10%)?
a. PCSO and lotto winnings And one day I heard the back door shut,
b. Books, literary works and musical compositions
c. Currency bank deposit As I entered the front, and I saw him slink
d. Royalties
173. Why did the register of deeds charge Lucio Cruz Back of the smoke house into the lot,
registration fee the instrument relative to his loan?
a. His loan was Php30,000.00 And across the field.
b. His loan was Php60,000.00

23
And I meant to kill him on sight. country and all who serve it, and the glow from that fire
can truly light the world.
But that day, walking near Fourth Bridge,
And so, my fellow Americans, ask not what your
Without a stick or a stone at hand, country can do for you; ask what you can do for your
country.
All of a sudden I saw him standing,
My Fellow citizens of the world ask not what
Scared to death, holding his rabbits,
America will do for you, but what together we can do for
And all l could say was, "Don't, Don't, Don't," the freedom of man.

As he aimed and fired at my heart. 185. In the speech, Kennedy paints a picture of the
United States as a nation that is
181. Which of the following techniques is used in the a. longing to return to the past
poem? b. on the brink of world war
a. Verse c. Free verse c. struggling to survive
b. Rhyme d. Personification d. the leading defender of freedom
182. The way in which the poet presents these words 186. One of the purposes of the speech is to motivate
in line 12 implies that Tom listeners to
a. tried to annoy the other man. a. serve their country
b. was shot before the finished the statement. b. enlist in the armed forces
c. did not want to hurt the other man. c. prepare themselves for battle
d. begged the man to stop seeing his wife. d. preserve the right to bear arms
183. Who is the speaker of this poem? 187. The speech is characterized by all of the
a. Tom Merritt c. God following stylistic devices EXCEPT
b. Merritt's wife d. The sheriff a. the use of the personal pronouns we and us to
184. The poet introduces the poem with lines 1-3 to build rapport with listeners
show us that Tom Merritt b. catchy turns of phrase in which subjects and
a. had suspected that his wife was seeing another objects are inverted
man c. a standard, predictable rhythm and the use of
b. was sure that his wife was ill rhyme
c. was a very suspicious person d. the repetition of key words
d. was sure that his wife still loved him 188. The tone of the speech can BEST be
characterized as
a. sad c. light-hearted
Situation 2 - Below is an excerpt from John F. b. uplifting d. sarcastic
Kennedy's "Inaugural Address." Read the excerpt
and answer the questions that follow. Situation 3 - Below is a Biographical sketch of an
American movies writer:

In your hands, my fellow citizens, more than


mine, will rest the final success or failure of our course. How Did Elvis Presley Achieve Recognition
Since this country as founded, each generation of
Americans has been summoned to give testimony to its Success often comes to those with humble
national loyalty. The graves of young Americans into beginnings, Elvis Aron Presley was born on January 8,
answered the call to service surround the globe. 1935, in Tupelo, Mississippi. He first sang in a church
choir and taught himself to play the guitar, but he never
Now the trumpet summons us again-not as a call learned to read music. By 1953, he had moved to
to bear arms, though, arms we need, not as a call to Memphis, Tennessee, graduated from high school, and
battle, though embattled we are; but a call to bear the enrolled in night school to become an electrician. That
burden of a long twilight struggle, year in and year out, year, at Sun Records, Presley recorded a personal record
"rejoicing in hope, patient in tribulation," a, struggle for his mother, a song that was heard by the company's
against the common enemies of man: tyranny, poverty, president. As a result of the president's recognition,
disease, and war itself. Presley's first record, "That's all Right, Mama," was out in
1954.
Can we forge against these enemies a grand and
global alliance, North and South, East and West, that can He toured the South, and in 1955 five of his
assure a more fruitful life for all mankind? Will you join me records were released simultaneously. His first national
in this historic effort? television appearance was that year on Jackie Gleason's
"The Stage Show," but Presley became known for his
In the long history of the world, only a few
appearance on "The Ed Sullivan Show," where the young
generations have been granted the role of defending
singer gyrated as he sang "rock n' roll" music. During the
freedom in its hour of maximum danger. I do not shrink
live television performance, Presley was photographed
from this responsibility; I welcome it I do not believe that
only from the waist up because his motions were
any of us would exchange places with any other people
considered obscene.
or any other generation. The energy, the faith, the
devotion which we bring to this endeavor will light our

24
"Elvis the Pelvis" began his film career in 1956
with LOVE ME TENDER and signed a long term film
contract. The movie critics were not always kind, but 193. How long does the action of the poem take
place?
teenagers flocked to Presley's films. Within a few short
a. A week c. An afternoon
years, Presley had established a career that would span b. Twenty-four horns d. A morning
twenty-five years of ups and downs and make him one of 194. The poet's use of words such as whine, voice,
the most popular entertainers in history. Long after his teeth, chatter and munch is to suggest that the
untimely death at age 42, Presley would be remembered power-mowers are
as "The King of Rock n' Roll” a. very powerful c. like cows
b. alive d. green
195. To what does the phrase "your blades" in line 8
refer?
189. The author uses the phrase ups and downs to a. Lawmowers c. Carpets
refer to Presley's b. Roots d. Monsters
a. gyrations as he performed 196. The imagery in the first stanza appeals to the
b. increasing and decreasing finances reader's sense of
c. successes and disappointments in his career a. sight b. touch c. smell d. hearing
d. use of drugs, "uppers and downers"
190. The statement that "success often comes to
those with humble beginnings" would apply best to Situation 5 - Ang sumusunod ay isang talumpati na
which of the following figures? may pamagat na SA KABATAAN na sinulat ni Onofre
a. Ramon Magsaysay c. Corazon C. Aquino Pagsanghan.
b. Gloria M. Arroyo d. Joseph E. Estrada
191. The main idea of the sketch is that
a. singers are more successful if they appear in
films Isa sa mga salitang napag-aralan natin sa wikang
b. there has always been obscenity on television Pilipino ay ang salitang NABANSOT.
c. opportunity and luck are often as important as
hard work Kapang ang isang bagay daw ay dapat pang lumaki
d. celebrities are usually more famous after their
ngunit ito'y tumigil na sa paglaki, ang bagay na ito raw ay
death
192. The last sentence reveals that the author's NABANSOT. Marami raw uri ng pagkabansot ngunit ay
attitude toward Presley is one of pinakamalungkot na uri raw ay ang pagkabansot ng
a. indifference c. disbelief isipan, ng puso at ng diwa.
b. admiration d. disgust
Ang panahon ng kabataan ay panahon ng
paglaki, ngunit ang ating paglaki ay kailangang paglaki, at
Situation 4 - The poem below is entitled "Suburban pag-unlad ng ating buong katauhan, hindi lamang ng
Prophecy" which is written by Howard Nemerov. ating sukat at timbang. Kung ga-poste man ang ating taas
at ga-pison man ang ating bigat ngunit kung ang pag-iisip
naman nati'y ga-kulisap lamang kay pangit na
On Saturday, the power-mowers' whine kabansutan. Kung tumangkad man tayong tangkad-
kawayan at bumilog man tayong bilog-tapayan, ngunit
Begins the morning. Over this neighbourhood kung tayo nama'y tulad ni "bondying" ay di
mapagkatiwalaan-anong laking kakulangan. Kung
Rises the keening, petulant voice, begin magkakatawan tayong katawang "Tarzan" at mapatalas
ang ating isipang sintalas ng kay Rizal, ngunit kung ang
Green oily teeth to chatter and munch the cud.
ating kalooban nama'y itim na duwende ng kasamaan-
Monsters, crawling the carpets of the world, anong kapinsalaan para sa kinabukasan.

Still send from underground against your blades Kinabukasan, kabataan, tayo raw ang pag-asa ng
Inang Bayan. Tayo raw ang maghahatid sa kanya sa
The roots of things battalions green and curled langit ng kasaganaan at karangalan o hihils sa kanya sa
putik ng kahirapan at kahihiyan. Ang panahon ng pagkilos
And tender, that will match your blades with blades ay ngayon, hindi bukas, hindi sa isang taon. Araw-araw
ay tumutuwid tayong palangit or bumabaluktod tayong
Till the revolted throats shall strangle on .
paputik. Tamang-tama ang sabi ng ating mga ninunong
The tickle of their dead, till straws shall break kung ano raw ang kinamihasnan ay siyang
pagkakatandaan. Huwag nating akalaing
Crankshafts like camels, and the sun go down makapagpapabaya tayo ng ating pag-aaral ngayon at sa
araw ng bukas ay bigla tayong magiging mga
On dinosaurs in swamps. A night attack dalubhasang magpapaunlad sa bayan. Huwag nating
akalaing makapaglulublob tayo ngayon sa kalaswaaan at
Follows and by the time the Sabbath dawns kahalayan at sa mahiwagang araw ng bukas sigla tayong
magiging ulirang mga magulang.
All armored beasts are eaten by their lawns.
Kabataan, tunay na pag-ibig sa bayan, ang tunay

25
na nasyonalismo, ay wala sa tamis ng pangarap wala rin d. Ask for a collateral for the cash he is loaning
sa pagpag ng dila. Ang tunay na pag-ibig ay nasa pawis
ng gawa. 3. Periodic checks on student seatwork with a smile and
pat on the shoulder effectively reinforce good study
197. Sa alin makikita ang tunay na
NASYONALlSMO? habit is an example of __________
a. Diwa b. Gawa c. Sulat d. Salita
198. Alin salita ang paulit-ulit na binabanggit ni Onofre a. Discrimination reinforcement
Pagsanghan?
a. Nabansot c. Kabataan b. Variable-ratio schedule
b. Bayan d. Kung
199. Bakit di dapat tumangkad tulad ni "bondying"? c. Continuous reinforcement
a. Di ito magpakakatiwalaan c. May kakulangan ito
b. Di totoo ito d. Magulo kasi ito
200. Alin sa mga sumusunod ang mensahe ng
d. Fixed interval and variable-interval schedule
talumpati?
a. Ang mataas na paniniwala at taimtim na 4. A person strives to work at a given task because of a
pananalig ay kailangang taglayin upang ang need. Which of the following situations can make a
hangarin sa buhay ay ating kamtin.
person strive to meet his needs?
b. Ang panahon ng kabataan ay panahon ng
paglaki at pagbabagong makabuluhan.
c. Ang gawa ang siyang sukat ng kadakilaan. a. Minimize the unpleasant consequences of student
d. Ang kabataan ay siyang pag-asa ng bayan. involvement

b. Utilize your own opinion as teacher in making final


decisions in the classroom

c. Use unfamiliar materials as examples in order to


“The three great things in the world are a mountain, initially arouse their curiosity

the ocean, and a man at his work. The potentialities d. Ask pupils to submit test questions or reactions which
each are beyond human calculation” you can select topics

5. The singing of the National Anthem in schools is an


offshoot of the philosophy of _________
PROFESSIONAL EDUCATION
a. Nationalism c. Naturalism
1. As a parent and at the same time a teacher, which of
the following will you do to show your cooperation to a b. Pragmatism d. Socialism
PTA project in your school to be financed with the
6. The environment in order to facilitate, learning must
proceeds of the sales of the school canteen where food
be interactive. Which of the following best typifies this
prices are little bit higher?
kind environment?
a. Bring food for you and your children, but always make
a. The child goes out and discovers for himself some rock
it a point to buy in the school canteen.
or fossil
b. Buy all you food in the school canteen but request for
b. The child listens to a lecture on fossils given by the
a discount
teacher
c. Bring food enough for you and your children but do
c. The child summarize the section on fossils in his
not eat in the canteen
science textbook
d. Buy all your food from the school canteen even if you
d. The child copies a list of facts concerning fossils on the
cannot afford to do everyday.
blackboard
2. How can you help a habitual borrower of money get
7. Social development means the acquisition of the
rid of his habit?
ability to behave in accordance with __________
a. Let him do something for you in return for the money
a. Social expectation c. social insight
you lent him
b. Stereotyped behavior d. universal norms
b. Direct him to others

c. Do not lend him anymore

26
8. When an adolescent combines ability to use deductive c. Filipino’s sense of humor
and inductive reasoning in the construction of realistic
rules that he can respect and live by, how does he d. Filipino’s love for “porma”
perceive his environment? 15. Which is NOT a characteristics of democratic
a. He views the world from his own perspective discipline?

b. He interprets events from a limited view a. Child has opportunity to express his/her opinion

c. He views events apart from himself and other people b. Child given punishment is related to the misdeed

d. He views the world and himself through the eyes of c. Child understands the meaning of rules
other people d. Child obeys blindly
9. Who introduced the technique of using the drawing of 16. Who among the following stressed the processes of
a man as a measure of intelligence? experience and problem solving?
a. Aristotle b. Herbart c. Good Enough d. Binet a. Dewey b. Aristotle c. Hegel d. Plato
10. Which Republic Act provides government assistance 17. Which of the following reasons of measuring student
to students and teachers in private education? achievement is NOT valid?
a. RA 7784 b. RA 6728 c. RA 7836 d. RA 6675 a. To prepare feedback on the effectiveness of the
11. The authoritarian setting in the Filipino home is learning process
reinforced by a classroom teacher who: b. To certify that students have attained a level of
a. Encourage pupils to ask questions competence in a subject area

b. Prescribes what pupils should do c. To discourage students from cheating during test and
getting high scores
c. Is open to suggestions
d. To motivate students to learn and master the
d. Ask open ended questions materials they think will be covered by the achievement
test
12. Who among the following believes that learning
requires disciplined attention, regular homework, and 18. Which characterizes the perfectionist type of
respect for legitimate authority? students?

a. Essentialist b. Perennialist c. Progressivist d. a. Does not volunteer or initiate


Reconstructionist
b. Give up easily
13. The Constitutional provision on language has the
c. Rarely complete tasks
following aims EXCEPT:

a. To make the regional dialect as auxilliary media of d. Often anxious fearful or frustrated about quality of
instructions in regional school work

b. To maintain English as a second language 19. When a school decides to work on a thematic
curriculum which should be out of the picture?
c. To make Filipino the sole medium of instruction
a. Peer collaboration c. team teaching
d. To make Filipino the national Language and medium
of instruction and communication b. Integration d. competition

14. The tendency to emphasize so much on school 20. Teacher wants to teach his pupils the technique on
beautification to the detriment of pupils performance reading for information. Which technique should be
illustrates the used?

a. Filipino’s lack of seriousness a. Text structure c. Story map

b. Filipino’s lack of reflection b. Prior knowledge d. SQ4R

27
21. In instructional planning, which among these three: d. Take the licensure examination for teacher and an
unit plan, course plan, lesson plan is (are) most specific? oath to do her best to help carry out the policies of the
_________ plans. state

a. Course and lesson c. lesson 26. Parents are up in arms on the telephone bills that
pay for sex calls. What is the solution to this problem?
b. Course d. unit
a. The telephone company is to blame for this
22. The use of drills in the classroom is rooted on
Thorndike’s law of b. The government restriction have no teeth

a. Readiness c. effect c. Parents allow this to make their children modern

b. Exercise d. belongingness d. Parents, school and students should discuss this


openly
23. Positive interdependence as an element of
collaborative learning means that the students must: 27. Cooperative are encouraged is as many groups as
possible. What agency controls the different
a. Learn to depend on each other to achieve a goal cooperatives?
b. Depend on the diligent students a. Security and Exchange Commission c. Commission on
c. Help one another in the individual test for everyone to Audit
pass b. Department of Local Government d. Bureau of
d. Be grouped heterogeneously Cooperative

24. Which of the following measures should a teacher do 28. Society and media know drinking starts off drug
to a principal whom she would like to file a case of sexual addiction. What should be discussed in schools?
harassment without violating the relationship of the a. Drug addiction has been traced to drinking wine
teacher and her superiors?
b. Nobody drinks at home except father
a. Present the case before competent authority and
prepare to prove the charge c. TV ads show drinking is a source of fellowship

b. Write an anonymous letter to a higher school official d. High taxes on liquor will be deterrent to eventual drug
to denounce the superior use

c. Call a parent-teacher meeting and denounce the 29. Cooperatives have branched out to consumer
superior cooperatives. Schools have included the concepts of
cooperatives. Where is it practiced?
d. Encourage the other teachers and students to hold a
demonstration to oust the superior a. School book stores c. Schools uniform purchases

25. Pick out the situation that illustrates the duty of a b. School canteen d. Class stores
new teacher to the state:
30. A student collapsed in her social studies class. It was
a. Take a long vacation which she firmly believes she found out that he did not eat her lunch. What principle is
deserves after four years of diligent study before taking shown in the situation
the examination for teachers
a. Psychological needs c. Somatotonic
b. Apply for teaching job where eligibility is not required
to gain teaching experience before taking the teachers b. Physiological need d. Safety need
board examination 31. The main function of a philosophy of education is to:
c. Prepare for the wedding she and her boyfriend have a. Aid the learner to build his own personal philosophy
long planned to able to raise a family with children which
they plan to rear as good citizen of our country b. Reconsider existing educational goals in the light of
society’s needs

28
c. Provide the academic background prerequisite to c. More than 50% of the score obtained is zero
learning
d. Less than 50% of the scores obtained is zero
d. Define the goals and set the direction for which
education is to strive 38. Which is the least authentic mode of assessment?

32. Which technique (s) enable (s) a teacher to identify a. Paper-and-pencil test in vocabulary
and eventually assists students with interpersonal b. Oral performance to assess student’s spoken
difficulties? communication ski8lls
a. Anecdotal record c. Cumulative record c. Experiments in science to assess skill in the use of
b. Personal inventory d. Sociogram scientific methods

33. Teachers and students can participate in levels of d. Artist production for music or art subject
computer use. Give the order of computer use from 39. In what period of a child is physical growth fastest?
simplest to complex?
a. Prenatal period c. Early childhood
a. Computer competency, computer literacy,
competency expertise b. Early adolescence d. Prenatal and early adolescence

b. Computer literacy, computer expertise, computer 40. How does fear affect the voluntariness of an act?
competency
a. Makes the act involuntary c. Increases voluntariness
c. Computer competency, computer expertise, computer
b. No effect at all d. Lessens but not destroy
literacy
voluntariness
d. Computer literacy, computer competency, computer
41. A group of people asserts that their culture is
expertise
superior to another. This exemplifies:
34. Which one is considered the “Brain” of the
a. Cultural gap c. norm conflict
microcomputer?
b. Cultural conflict d. Ethnocentrism
a. CPU b. Software c. Video Screen d. Keyboard
42. A test consist of a graph showing the relationship
35. A group activity wherein one group representative
between age and population. Follow a series of true-
presents the output to the bigger group rather than
false items based on the graph. Which type of test does
individual pupils presenting the output is known as:
this illustrate?
a. Consensus decision c. Jury trial
a. Laboratory exercise c. Performance
b. Composite report d. Agenda
b. Problem solving d. interpretative
36. In the formulation of classroom regulations, which of
43. Which curricular move served to strengthen spiritual
the following should a teacher refrain from doing?
and ethical values?
a. State classroom regulation as clearly as possible
a. Integration of creative thinking in all subject
b. Enlist student aid in the formation of classroom
b. Reducing the number of subject areas into the skill
regulation
subject
c. Enforce classroom regulations consistently and fairly
c. Introduction of Value Education as a separate subject
d. Teacher and the class should make as many area
regulations as possible
d. Re-introducing Science as all subject in Grade 1
37. Zero standard deviation means that:
44. Which computer seems to have the most potential
a. The students scores are the same for the classroom?

b. 50% of the scores obtained is zero a. Main frame computer c. Microcomputer

29
b. Minicomputer d. LPC a. Alienable rights c. Inalienable rights

45. A teacher notices glaring wrong pronunciation of b. Perfect rights d. Acquired rights
vowel sounds among her students necessitating more
practice. Which of the following activities would be a 52. In mastery learning the definition of an acceptable
most help? standard of performance is called:

a. Dictionary use c. Assignments a. Alienable rights c. Inalienable rights

b. Review d. Drill b. Perfect rights d. Acquired rights

46. When storage device is significantly more efficient in 53. A negative discrimination index means that:
holding information a. More from the lower group answered the test items
a. Hard disk c. Floppy disk correctly

b. The items could not discriminate between the lower


b. Software d. Audio cassette
and upper group
47. The Filipino tendency to resort to the easy way out
from a term paper as a course requirement by hiring a c. More from the upper group answered the test item
ghost writer or by passing a photocopied term paper correctly
provide which Filipino traits? d. Less from the lower group got the test item correctly
a. Anticipation c. Pakikisama 54. Your teacher is of the opinion that the world and
b. Ambivalence d. Lack of discipline everything in it are ever changing and so teaches you the
skill to cope with the changes. Which in his governing
48. “ No pain, no gain.” This means that philosophy?

a. Only those willing to carry the crosses that life impose a. Experimentation c. Realism
can share the joy of life
b. Existentialism d. Idealism
b. One should be penitent every Friday by carrying his
cross 55. For brainstorming to be effective which one should
be out?
c. The more suffering in this life, the more one is assured
a. Making use of the others ideas shared c.Non-
of heaven
threatening atmosphere
d. One should look for suffering to save himself/ herself
b. Teacher’s judge mental attitude d. Openness to idea
49. Which thrust on value formation is meant to help the
students make use of their thinking and scientific 56. Which statement on spaced and massed learning is
investigation to decide on topics and questions about CORRECT?
values? a. Massed learning is better than spaced learning
a. Value inculcation c. Value clarification b. Spaced learning is better than massed learning
b. Analysis d. Moral development c. Massed learning is as effective as spaced learning
50. Which interactive teaching should be AVOIDED? d. Both massed learning and spaced learning are not
a. Using “put down” strategy c. Asking more divergent effective
questions 57. A teacher is a facilitator of learning and of the
b. Using multiple response strategy d. Asking more development of the youth. Which practice is NOT in
evaluative questions keeping with his role as facilitator?

51. Rights which can not be renounced or transferred a. Considers the multiple intelligences of learners
because they are necessary for the fulfillment of man’s b. Humiliates misbehaving pupils
primordial obligations are called:

30
c. Dialogs with parents and with other members of the b. Noting specific details d. Drawing conclusion
community
64. In the Preamble of the Code of Ethics of Professional
d. Keeps himself abreast with educational trends Teachers, which is not mentioned about teachers?

58. Which one indicates a teacher’s genuine enthusiasm a. Dully licensed professionals c. LET passers
and pride in teaching?
b. Posses dignity and reputation d. With high moral
a. Sticking to teaching for the moment that there are no values
better offers
65. What does a skewed score distribution mean?
b. Telling everyone that he went to teaching for there
a. The scores are concentrated more at one end or the
was no other choice them
other end
c. Engaging himself in continuing professional education
b. The mode, the mean and the median are equal
d. Belittling the re-numeration one gets from teaching
c. The mean and median are equal
59. In writing performance objective which word is NOT
d. The scores are normally distributed
acceptable?
66. Which is implied by a negatively skewed score
a. Manipulate c. Delineate
distribution/
b. Integrate d. Comprehend
a. The scores are evenly distributed from the left to the
60. When is giving praise INEFFECTIVE? When IT? right

a. Uses the accomplishment of peers as the context for b. Most pupils are underachieves
describing a student’s present accomplishment
c. Most of the scores are high
b. Provides information to student’s about their
competence and the value of their accomplishment d. Most of the scores are low

c. Focuses students attention on her own task relevant 67. A teacher discovers that a product of a certain
bottling company brings about damage to teeth. Much
behavior
as he wants to share the products of his research, he
d. Shows spontaneity, variety and other signs of could not because of harassment from all sides. Which
credibility teacher’s right is violated?

61. Which statement applies when scores distribution is a. Right to property c. Academic freedom
negatively skewed?
b. Right to one’s honor d. Right to make a livelihood
a. The mode corresponds to a lower value
68. Why can the calculator do arithmetic? Because
b. The median is higher that the mode
a. A computer inside the calculator tells it how c. A
c. The mode and median are equal typewriter inside does inside does it

d. The mean corresponds to a high value b. A watch inside direct it d. A TV inside shows it

62. The use of the process approach gives the student 69. On which constitutional provision is the full or partial
the opportunity to: integration of capable deaf and blind students in the
classroom based? The provision on
a. Learn on their own c. Make use of laboratory
apparatuses a. Providing citizenship and vocational training to adult
citizen
b. Apply the scientific method d. Learn how to learn
b. Protecting and promoting the rights of all citizen to
63. A comprehension skill of higher level which may be quality education
inferred or implied from reading is
c. Academic freedom
a. Picking out the main idea c. Following direction

31
d. Creating scholarship for poor and deserving students b. Make sure that the students understand the pre-
requisite skills of the lesson
70. Teaching in the cognitive, psychomotor and affective
domains is based on the concept that the learner is a: c. Prepare the students for the mastery test

a. Moral and feeling being c. Thinking, feeling and acting d. Make learning interesting and enjoyable for students
being
76. Which is a selective reading technique meant at
b. Maternal and an acting being d. Spiritual and maternal getting at important facts very fast?
being
a. Skim reading c. Oral reading
71. Both Muslim and Christian value marriage but the
Muslim practices polygamous marriage while the b. Scanning d. Silent reading
Christian practices monogamous marriage. What is this 77. For counseling to be successful which assumption
called? must be AVOIDED?
a. Cultural relativism c. Ethical relativism a. The environment must provide assurance of
b. Acculturation d. Enculturation confidentiality

72. Teacher wants to compare 2 concepts. With which b. The student is willing to participate in the process
technique can accomplish this best? c. The counselor must be able to relate to the student
a. K-W-L technique c. Spider web d. The counselor tells the student what to do
b. Venn diagram d. Histogram 78. Which technique is most appropriate when a teacher
73. To build a sense of pride among Filipino youth which wants a group to agree on a plan of action?
should be done? a. Composite report c. Agenda
a. Re-study our history and stress on our achievements b. Symposium d. Consensus decision making
as a people
79. Which term applies to the search for related
b. Set aside the study of local history literature by computing access of data bases of discs
c. Re-study our history from the perspective of our kept in libraries?
colonizers a. On line research c. Compact discs computer research
d. Re0place the study of folklores and myths with b. Manual research d. Computer research
technical subjects
80. Which best indicates the effectiveness classroom
74. When necessary conditions are present, the use of activities?
inductive method is preferred because
a. The laughter and enjoyment of students
a. There is greater active participation on the part of
pupils b. The application of concept learned in daily life

b. It gives the teacher more time to rest c. The utilization of varied techniques and approaches

c. It needs only few instruction materials d. The variety of instructional materials used

d. Academic time is used wisely 81. The main purpose of the compulsory study of the
Constitution in Philippine schools is to
75. Which is the best reason why teacher begins a lesson
in Math by checking and reviewing on the previous day’s a. Develop the students into responsible thinking citizens
assignment and provides practice and drills?
b. Acquaint students with the historical development of
a. Check if parents guide their children in the making of the Philippine Constitution
assignment
c. Prepare students for law making

d. Make constitutional experts of the students

32
82. Some students who are high in the scholastic a. Taoism c. Confucianism
aptitude test have failed in college. Some who are below
the standards set for admission but who for various b. Shintoism d. Zen Buddhism
reasons were admitted, attained satisfactory standings. 87. A child refuse to obey orders or displays negativism
This proves that as a development trait. How may you best handle him?
a. Human beings are certainly predictable a. Take every opportunity to praise him for every positive
b. Admission tests are not accurate, hence should not be attitude displayed
used b. Detain him after office hours for him do to what he
c. Aptitude tests do not measure all factors important for has been ordered to do
success c. Insist on compliance to the same degree required of
d. Aptitude test can be perfectly relied on pupils

83. If the teachers pattern in questioning consists of d. Avoid giving him orders if you do and he objects take
calling on a student then asking the question back the order

88. Which term refers to the collection of students


a. All students may be encouraged to participate
products and accomplishment for a period of evaluation
b. The student called to answer may be able to think well purposes?
of his answer
a. Portfolio c. Anecdotal record
c. The rest of the class may just dictate the answer
b. Observation report d. Diary
d. The rest of the class may not engage themselves in
89. For comparing and contrasting which graphic
thinking of the answer
organizers is most appropriate?
84. In order to avoid disgrace, a pregnant, unmarried
woman takes drug to induce abortion. Is she morally a. Cycle c. Story map
justified to do that? b. Web d. Venn Diagram
a. Yes, it can save her and child from disgrace when he 90. If a resilient child with superior intelligence is reared
grows up in a poor environment the probable outcome would be
b. No, the unborn child can not be made to suffer the a. No change in IQ because environment deprivation has
consequences of the sins of his parents nothing has nothing to do with intelligence
c. No, the act of inducing abortion is bad in itself b. Slight change in IQ although he can overcome
d. No, it is better to prevent the child from coming into frustration and obstacle
the world who will suffer very much due to the absence c. Mental retardation since he is culturally deprived
of a father
d. Great change in IQ because he is culturally deprived
85. In which way does heredity affect the development
of the learner? 91. Which of the following is usually considered the most
important factor in a child’s observable classroom
a. By placing limits beyond which the learner can not behavior ?
develop
a. Intelligence c. Self concept
b. By compensating for what environment fails to
develop b. Heredity d. Cultural background

c. By blocking the influence of environment 92. Section 5, article XIV, of the Constitution states that
academic freedom shall be enjoyed in
d. By providing equal potential to all
a. Public assemblies c. All levels of learning
86. The cultivation of reflective and meditative skills in
teaching is an influence of b. State colleges and universities d. All institution of
higher learning

33
93. A teacher who subscribes to the pragmatic c. Health values development
philosophy of education believes that experience should
follow learning in her teaching, she therefore exerts d. National development requirement and reflects
effort in research based direction

a. Encouraging learners to memorize factual knowledge 99. The child cannot distinguish abstracts during the
sensory motor of development. Which of these
b. Equipping learners with the basic abilities and skills techniques should a teacher apply to accommodate
learning?
c. Requiring learners full mastery of the lesson
a. Make use of individualized instruction
d. Providing learners opportunities to apply theories and
principles b. Explain the lesson very well

94. Freud expounded that there is a period when young c. Utilize concrete objects to clarify concept
boys experience rivalry with their father for their
mother’s affection. This is d. Provide variety of educational toys

a. Oedipus complex c. Achilles syndrome 100. Which of these systems of learning includes ways
and methods which are used in preserving and building
b. Electra complex d. Cassandra syndrome certain within cultural communities?

95. Education is a life long process. This simply means a. Non-formal learning c. Cultural learning
that education
b. Multi-level learning d. Indigenous learning
a. May take place formally or informally to enable the
101. Which of the following statements is TRUE in the
individual to grow
use of experiments and demonstrations in teaching
b. May take place anywhere and any time the individual science
so desires
a. It is valuable if used in the context of a lesson that
c. Is a continuous process of experiencing and related observation to other information
reorganizing experiences
b. It should be encouraged in elementary school since
d. Take place in the school where the individual is the concept the encompass are difficult for your children
exposed, self contained experiences
c. It is as valuable as teaching by lecturing
96. The tendency to imitate elders is very strong in the
early childhood stage. Teachers should therefore be very d. It is less valuable than teaching through inquiry and
good discussion

a. Counselors c. Disciplinarians 102. Identical twins are more alike than fraternal twins.
Which of the following statements/ principle is
b. Role models d. Facilitators of learning supported by this?

97. How is Values Education offered in the National a. Environment affects both fraternal and identical twins
Secondary Education Curriculum?
b. Intelligence hinges in physical structure
a. Emphasized in Science and Technology
c. Heredity has a part in determining intelligence
b. As a separate subject
d. Intelligence is determined partly by pre-natal nutrition
c. Integrated in all subject areas
103. Which of theses philosophers is reflective of that of
d. Integrated with Technology and Home Economics Dewey’s which stresses the development of an individual
capable of reflective thinking specifically that of being
98. The NSEC orients secondary education to able to solve the problem he faces individually or
a. The teaching of the national symbols collectively

b. The development of competencies and values for a. Disciplinarianism c. Experimentation


social living

34
b. Developmentalism d. Rationalism c. Provide regular opportunity for socially acceptable
behavior
104. Which of the following abilities is stressed by
humanistic education? d. Consider peer approval and recognition

a. Learn the different philosophies of education 109. The trend of focusing attention on the child’s
interests, abilities and needs and on the improvement of
b. Develop man into a thinking individual community living necessitate the use of the
c. Enjoy the great works of man such as the classics a. Discovery approach c. Integrative approach
d. Make man distinctly civilized educated and refined b. Conceptual technique d. Project method
105. An appreciation lesson is one that is designed to 110. The best way the teacher can see the
lead the class to conduct and enjoy something. Which of
appropriateness of an instructional materials is to
the following statements closely approximate the
meaning of the above? a. Consider its technical quality c. Consider its availability

a. An appreciation lesson should be a lesson in values b. Try it out before using it in class d. Consider its cost

b. Appreciation lessons help pupils weigh and clarify 111. Tasks analysis involves the breaking down of a
values learning task into subtasks or sub skills. Given a task to
retell a story, which of the following skills is NOT
c. One cannot fully appreciate what one does not needed?
understand or enjoy
a. To disseminate information c. To identify topic
d. A teacher should plan lessons that will guide children
sentences
to appreciate what is beautiful
b. To outline a selection d. To arrange events in
106. Which of the following is the best time for a teacher sequence
to set up routine activities that will contribute to
effective classroom management? 112. You are assigned to teach students with varied
abilities. You want to teach a more homogenous
a. As soon as the students have established c. During his grouping. Which type of grouping will tend to benefit
homeroom days your students?
b. Daily at the start of the session d. On the every first a. Mixed ability grouping c. With-in class ability grouping
day of school
b. Low ability group d. High ability grouping
107. In large classes where little of the work pupils can
be individualized, the most effective and practical ways 113. Which of the following examples illustrate the use
to individualize instruction is to of questions to focus pupil attention on the key points of
the lesson?
a. Devise group activities which afford every pupil an
opportunity to work at his own a. Why are machine made goods cheaper than those
made by hand?
b. Give the pupils freedom to launch individual projects
b. What is Rizal Park known for?
c. Assign homework and check it regularly
c. Have you ever enjoyed watching the clouds on a bright
d. Assign program material for out-of-class hours day?
108. Which of these is the MOST important principle that
d. Who came while I was writing on the blackboard?
a teacher should follow in initiating program for positive
reinforcement? 114. The new teacher entered a noisy classroom. She
shouted immediately at the students desperately trying
a. Make sure the reward comes immediately after the to get order and discipline. Since then the teacher has
appropriate behavior not controlled the class. Which is the most probable
b. Punish negative behavior and reward positive cause of the teacher’s failure?
behavior

35
a. The students reaction to the teacher is the 119. When do test, inventories and career information
consequence of her behavior become effective for counseling services?

b. Rules are not defined and procedures to sustain order a. When the data generated are interpreted on time by
is not put into place professionally competent person

c. The new teacher wants to show the class who is b. The psychological test result are still valid and reliable
authority
c. When the records are updated
d. The class wants to test the ability and patience of the
d. When the records are kept for ready reference when
teacher.
needed
115. The educational implementation of research
findings relative to the ability of dull learners and bright 120. Which of the following is a major advantage in using
learners to organize and generalize is for teacher arithmetic mean?

a. To make the bright learners guide the dull ones in a. It is more commonly used than other measures
learning generalize b. It discriminates between the lowest and the highest
b. To make the bright learners to generalize and the dull c. It is simple to compute
ones to memorize
d. It is more than stable than the median
c. To give the dull learners more concrete experiences to
serve as basis for generalizing 121. In preparing a multiple choice test how many
options would be ideal?
d. To give both the dull and bright learners concrete and
abstract experiences to serve as basis for generalizing a. It is more commonly used than other measures

116. Which of the following will you do the FIRST to b. It discriminates between lowest and the highest
establish good class management?
c. It is simple to compute
a. Discuss the required rules for proper class behavior
d. It is more than stable than the median
b. Discuss the work plan for the year
122. Learners often find it much easier to fit into a new
c. Prepare a seat plan social situation when given encouragement and support.
How can this be done?
d. Train the class in the distribution of material
a. By giving him room responsibility
117. A student was diagnosed to have a high IQ but is
falling in his academic subject. What should the teacher b. By assigning “peers or Buddies” to him
do to help him?
c. By giving him special help
a. Talk to his parents
d. By discovering his new interest
b. Examine his study habits
123. Who expounded on the need to study the child
c. Talk the student and find out his problem carefully for individualized instruction?

d. Refer him to the guidance counselor a. Da Feltre b. Erasmus c. Boccacio d. Ascham

118. Which is the TRUE foundation of the social order? 124. Which of the following should a teacher do if she
cannot pay the monthly installment of an appliance she
a. Strong, political leadership got from a department store in their town?
b. The reciprocation of rights and duties a. Reject any notice of demand for payment to make the
c. Equitable distribution of wealth impression that she did not receive

d. Obedient citizenry b. Move to another neighborhood to escape payment

36
c. Inform the manager of the store personally and make 130. What should a teacher do before constructing items
a satisfactory arrangement of payment on or before the for a particular test?
due date of payment
a. Prepare a table specifications
d. Offer the return the used appliance to the store on the
condition that she will be refunded on the monthly b. Review the previous lessons
installment she paid c. Determine the length of time for answering it
125. Which of the following will you recommended to a d. Announce to students the scope of the test
senior high school scholar who is impregnated by a
fellow student? 131. Under which of the multiple choice type of test can
this question be classified? ‘ Which of the following
a. Tell her parent about her condition statements expresses this concept in different forms?
b. Stop schooling till after she gives birth a. Association b. Definition c. Difference d. Cause
c. Direct her to an abortion clinic 132. Of the following types of test which is the most
d. Force her boyfriend to marry her subjective in scoring?

126. The government prescribes a higher percentage on a. Matching type b. Simple recall c. Multiple choice d.
the administration of educational institution to Filipino Essay
citizens in order to 133. In which of these research methods can the
a. Minimize the unemployment problem researcher control certain variable?

a. Experimental b. Ex post facto c. Descriptive d.


b. Procedure globally competitive graduates
Historical
c. Protect the rights of the citizen
134. During the first grading period, a student obtained
d. Ensure the teaching of Filipino failing marks in five academic subjects. Which of the
following tests would best explain his performance?
127. Teacher should bear in mind that the period of
greatest mental development is from: a. Mental ability b. Personality c. Attitude d. Aptitude

a. 9 to 12 years b. 12 to 15 years c. 6 to 9 years d. 3 to 6 135. Measuring the work done by gravitational force is a
years learning task. At what level of cognition ?

128. Which of the following is the best situation wherein a. Application b. Knowledge c. Evaluation d.
you can balance responsibility and accountability? Comprehension

a. A teacher paid on an hour basis, takes her time with 136. Setting up criteria for scoring test is meant to
the subject matter till end of period increase their ________

b. A teacher paid on an hour basis, teaches as much as a. Objectively b. Reliability c. Validity d. Usability
she could for duration of the period
137. Which of the following you will do to an examinee
c. A teacher paid on an hour basis, spends most of the you caught cheating and who offered to a certain sum of
time on the latest gossips in showbiz money to keep quiet

d. A teacher paid on an hour basis, entertain her a. Motion him to keep quiet and watch for him after the
students with stories till the end of the period examination

129. You have a pupil who is so talkative, naughty and b. Confiscate his test paper and report him to the
aggressive that he is a burden to the entire members of examination supervisor
the class. How would you remedy this problem?
c. Announce to all examinees the name of the cheater
a. Call the parents for dialogue c. Reprimand him always
d. Ignore him but let him feel you saw him
b. Report the case to the principal d. Talk to him
seriously

37
138. Which of the following is the best situation wherein 145. Which of the following is the MOST important
you can balance rights and authority? purpose for using achievement test? To measure the
_________
a. Allow all their only daughter’s suitor to come and go
as she pleases a. Quality and quantity of previous learning c.
Educational and vocational aptitude
b. Censor all their only daughter’s suitor
b. Quality and quantity of previous teaching d. Capacity
c. Choose a life-partner for their only daughter
for future learning
d. Caution their only daughter’s choice of a boyfriend 146. Which of the different types of test covers a wide
139. In testing which of the following is referred to as variety of objectives?
cultural bias? a. True-false c. Matching
a. Test items are more familiar with some culture b. Multiple choice d. Essay
b. Some culture do better on test than others 147. In a multiple choice test, keeping the options brief
c. Test will show who is more cultured indicates ____________

d. Cultured people do better on tests a. Inclusion in the item irrelevant clues such as the use in
the correct answer
140. Which is the most obvious and familiar way of
reporting variability? b. Non inclusion of option that mean the same

a. Range between highest with some culture c. Standard c. Plausibility and attractiveness of the item
error of the mean d. Inclusion in the item any word that must otherwise
b. Standard deviation d. Distribution of raw scores repeated in each response

141. The theory of identical elements in learning holds 148. Which of these criteria is the most important in test
that transfer is facilitated when the ______ constructions?

a. Teacher uses different teaching devices a. The stem should contain the central problem

b. Learner has a memory of specific responses b. Items should be congruent with the objectives

c. Development task is easily identifies c. A table of specification should be prepared

d. Experience is similar to the application situation d. Options should be of almost the same length

142. If this need is not met the adolescent tends to be 149. Study this group of test which was administered to
critical and always tries to find fault. This is the need a class to whom Peter belongs, then answer the
question:
a. For recognition c. to belong
SUBJECT MEAN SD PETER’S SCORE
b. For adventure d. for material security
Math 56 10 43
143. The way a child talks, walks of manifest, gestures
may have been learned from models he had been Physics 41 9 31
exposed. This explains English 80 16 109
a. Affective b. Insight c. Social d. Cognitive In which subject(s) did Peter perform most poorly in
144. Audio-visual aids are used in classroom teaching to relation to the group’s mean performance?
__________ A. English C. English and Physics
a. Help make learning more permanent c. Help clarify
B. Physics D. Math
important concept
150. Based on the data given in #149, in which subject(s)
b. All of these d. Arouse and sustain student’s interest were the scores most widespread?

38
A. Math C. Cannot be determined A. independent thinking

B. Physics D. English B. social interaction

#ProfEd LET Reviewer C. individual mental work

150 Items with Answer Key D. scientific thinking

1. At the end of periodical examination, Teacher R 6. The design of the 2002 Basic Education Curriculum
administered a summative test in Filipino. After scoring (BEC) is based on the principles that the main sources for
the test papers she assigned grades to each test score contemporary basic education are the expert systems of
such as 95, 90, 85, 80 etc. What process did Teacher R knowledge and the learner's experience in their context.
use? This shows that the BEC is _____ in orientation.

A. Ranking I. constructivist

B. Computation II. behaviorist

C. Measurement III. essentialist

D. Evaluation A. I and III

2. It is the study of man’s pre­history through the buried B. III only


remains of ancient culture, skeletal remnants of human
beings. C. I only

A. Anthropology D. I, II, and III

7. Which assessment tool shows evidence of student’s


B. Archeology
writing skills?
C. Ethnology
A. project
D. Ethnography
B. portfolio
3. Teacher D claims: "If I have to give reinforcement, it
has to be given immediately after the response." Which C. critiquing sessions
theory supports Teacher D? D. daily journal
A. operant conditioning theory 8. Ask to do a learning task, Joe hesitates and says
"Mahirap. Ayaw ko. 'Di ko kaya!" (It's difficult. I don't like
B. social cognitive theory
it. I can't do it.) To which problem does the case of the
C. cognitive theory student allude?

D. humanist theory I. Unmotivated students

4. Visual imagery helps people store information in their II. Uncaring teachers
memory more effectively. Which is one teaching
implication of this principle? III. Extremely difficult learning tasks

A. You will not object when your students daydream in IV. incompetent teachers
class. A. I and III
B. Instruct students to take notes while you lecture. B. I and II
C. Encourage your students to imagine the characters C. II and III
and situations when reading a story.
D. I, II, and III
D. Tell them to read more illustrated comics.
9. Teacher S wants to determine immediately the
5. Which is essential in the cognitive development of a learning difficulties of her students. Which of the
person according to Vygotsky? following do you expect her to undertake?

39
A. Require her students to prepare a portfolio. above the minimum through _____ as provided in
Educational Act of 1982.
B. Administer an achievement test.
A. lifelong education
C. Administer a diagnostic test.
B. voluntary accreditation
D. Interview her students directly.
C. formal education
10. NSAT and NEAT results are interpreted against set
mastery level. This means that NSAT and NEAT fall under D. academic freedom
__________.
15. You intend to assess affective attributes such as
A. intelligence test capacity to feel, attitudes, and behavior. Which of the
following should you establish to ascertain the
B. aptitude test instrument's validity?
C. criterion-referenced test A. construct
D. norm-referenced test B. content
11. The result of the item analysis showed that item no. C. criterion-related
4 has a discrimination index of 0.67. What characteristic
could be true about this item? D. face

A. Difficult 16. Which will be the most authentic assessment tool for
an instructional objective on working with and relating to
B. Valid people?
C. Easy A. conducting mock election
D. Average B. home visitation
12. The theme of Vygotsky's socio-cultural theory
C. organizing a community project
emphasizes the role of appropriate assistance given by
the teacher to accomplish a task. Such help enables the D. writing articles on working and relating to people
child to move from the zone of actual developmeny to a
zone of proximal development. Such assistance is termed 17. As an effective classroom manager, what should a
_____. teacher do?

A. competency technique I. She uses instructional time wisely.

II. She uses her power to punish students for the sake of
B. scaffolding
discipline.
C. active participation
III. She puts to use the available and appropriate
D. collaboration materials.

13. Which of these activities is not appealing to the IV. She manipulates colleagues and students so she can
bodily-kinesthetic learners? meet her goals.

A. making math moves A. I and III

B. doing simple calisthenics B. II, III, and IV

C. sketching/illustrating events C. I, II, and III

D. joining extramural events D. I, II, III, and IV

14. Institutions of learning are required to meet the 18. Keeping track of assessment results from one
minimum standards for state recognition but are periodic rating to the next is useful in contributing to the
encouraged to set higher standards of quality over and development of a _____.

A. regional plan
40
B. annual implementation plan 24. The discrimination index of a test item is -0.35. What
does this mean?
C. school improvement plan
A. More from the upper group got the item correctly.
D. division plan
B. More from the lower group got the item correctly.
19. In the K- W- L technique K stands for what the pupil
already knows, W for he wants to know and L for what C. The test is quite reliable.
he _____.
D. The test item is valid.
A. failed to learn
25. His aim of education is individual not a preparation
B. he likes to learn for but participation in the life around the individual.

C. needs to learn A. Froebel

D. learned B. Spencer

20. Which is one characteristic of an effective classroom C. Herbart


management?
D. Pestalozzi
A. It quickly and unobtrusively redirects misbehavior
26. The following are characteristics of a child-friendly
once it occurs.
school except for _____.
B. It teaches dependence on others for self-control.
A. exclusive
C. It respects cultural norms of a limited group students.
B. child-centered
D. Strategies are simple enough to be used consistently.
C. gender-sensitive
21. When curriculum content is fairly distributed in each
area of discipline, this means that the curriculum is D. nondiscriminating
_____. 27. To make the lesson meaningful, systematic and
A. sequenced B. balanced motivating, teachers’ example should be ______.

C. integrated D. continued A. based on higher -level skills

22. Direct instruction is for facts, rules, and actions as B. interesting aided with illustrations
indirect instruction is for _____. C. easy, simple, and understandable
A. hypotheses, verified data, and conclusions D. relevant to students’ experience and knowledge
B. concepts, patterns, and abstractions 28. Which are the most important concerns about the
C. concepts, processes, and generalizations use of ICT in instruction?

D. guesses, data, and conclusions I. Developing appropriate curriculum materials that allow
students to construct meaning and develop knowledge
throught the use of ICT

23. The difficulty index of a test item is 1.0. This means II. Devising strategies to meaningfully integrate
that the test is _____. technology into the curriculum

A. a quality item III. Using pedagogical skills related to technology

B. very difficult IV. Providing teachers with skills for using software
applications
C. very easy
A. I and III
D. missed by everybody
B. II and III

41
C. III and IV 33. The difficulty index of a test item is 1.0. What does
this mean?
D. I, II, III, and IV
A. The test item is very good, so retain it.
29. Which of these non-threatening means of assessing
learning outcomes? B. The test item is very difficult.

I. Portfolio C. The test item is extremely easy.

II. Self-evaluation D. The test item is not valid.

III. Peer evaluation 34. Someone wrote: "Environment relates to the


profound relationship between matter, nature, and
IV. Learning journals society, and in such a context, ICTs bring new ways of
A. I and II living in a more interconnected society, all of which
reduces our dependency on matter and affects our
B. I and III relationship with nature." What does this convey?

C. I, II, and IV A. Environment and ICT are poles apart.

D. II and III B. ICT impacts on environment.

30. Teacher A's lesson is about the parts of the C. Environment affects ICT.
gumamela. He asked his pupils per group to bring a real
flower to study the different parts. After the group work D. ICT brings us away from an interconnected society.
labeling each part, the teacher gave a test. What would 35. Educational institutions’ effort of developing work
be the best type of test he can give? skills inside the school are aimed at _____.
A. essay type A. developing moral character
B. matching type B. inculcating love of country
C. diary C. teaching the duties of citizenship
D. journal D. developing vocational efficiency
31. Principal A wants her teachers to be constructivist in 36. Teacher B discovered that her pupils are weak in
their teaching orientation. On which assumption/s is the comprehension. To further determine in which particular
principal's action anchored? skills her pupils are weak which test should Teacher B
I. Students learn by personally constructing meaning of give?
what is taught. A. Aptitude Test
II. Students construct and reconstruct meanings based B. Placement Test
on experiences.
C. Diagnostic Test
III. Students derive meaning from the meaning that
teacher gives. D. Standardized Test

A. I and IIIB. I onlyC. I and II D. II only 37. That the quality of Philippine education is declining
was the result of a study by EDCOM which
32. The norms in a school culture are centered on the
recommended to _____ teachers and teaching.
_____.
A. regulate
A. learner
B. progessionalize
B. teacher
C. strengthen
C. principal
D. improve
D. supervisor

42
38. As provided in the Republic Act #4670, every teacher 43. Societal change requires continually deep-seated
shall enjoy equitable safeguards at each stage of any questions about "good" living. Which of these did
disciplinary procedures and shall the following except: Socrates recognize as the greatest of human virtues?

A. the right to be informed in writing, of the charges A. moral wisdom

B. the right to full access to the evidence in the case B. fair justice

C. the right to appeal to clearly designated authorities. C. courage

D. All of these D. piety

39. The subject matter or content to be learned must be 44. Among the following curriculum stakeholders, who
within the time allowed, resources available, expertise of has the most responsibility in curriculum
the teacher, and nature of learners. What criterion is implementation?
addressed?
A. the learners
A. validity B. significanceC. interest D. feasibility
B. the school heads
40. A child was shown an amount of water in a glass. The
teacher poured the whole amount to a much taller and C. the teachers
narrower glass and marked this glass as D. the parents
A. The same amount was poured in a shorter and wider 45. Study this group of tests which was administered
glass, marked glass with the following results, then answer the question.
B. When asked which has more water, the child's SubjectMathPhysicsEnglish
answer was "Glass A". In what stage of cognitive Mean564180SD10916Student′sScore4331109
development is the child and what is this ability called?
A. concrete operational stage; conservation B. formal In which subject(s) did Ronnel perform most poorly in
operational stage; deductive reasoning relation to the group’s performance?

C. sensorimotor stage; symbolic functions A. English

D. pre-operational stage; centration B. English and Math

41. In a multiple choice test item with four options and C. Math
out of 50 examinees, which was the least effective
D. Physics
distracter?
46. You like to show a close representation of the size
A. the option that was chosen by 13 examinees
and shape of the earth and its location in the entire solar
B. the option that was chosen by 2 examinees system. What is the best instructional aid?

C. the option that was chosen by 30 examinees A. pictureB. model

D. the correct answer that was chosen by 5 examinees C. realiaD. film

42. What is an alternative assessment tool for teaching 47. Which of the following techniques of curriculum
and learning consisting of a collection of work/artifacts implementation is fit to the objective of developing
finished or in-progress accomplished by the targeted cooperative learning and social interaction?
clientele?
A. buzz session
A. rubric
B. graded recitation
B. achievement test
C. individual reporting
C. evaluation instrument
D. lecture
D. portfolio
48. To determine your pupil's entry knowledge and skills,
which should you employ?
43
A. interview 54. Cooperative learning approach makes use of a
classroom organization where students work in teams to
B. focus group discussion help each other learn. What mode of grouping can
C. post-test facilitate the skill and values desired?

D. pre-test A. large group

49. We encounter people whose prayer goes like this: “O B. homogeneous


God, if there is a God; save my soul, if I have a soul” C. heterogeneous
From whom is this prayer?
D. Wear multicolored dress to catch the students'
A. Stoic attention.
B. Empiricist 55. To solve moral ambiguity among us Filipinos, we
C. Agnostic must _____.

D. Skeptic A. excuse ourselves whenever we do wrong

50. What is the implication of using a method that B. blame our government for not doing anything about
focuses on the why rather than the how? it

A. There is best method C. be aware and responsible about the problem

B. Typical one will be good for any subject D. be comfortable with the present state affairs

C. These methods should be standardized for different 56. In their desire to make schools perform, the DepEd
subjects. then published the ranking of schools in NAT results
nationwide. As an effect of this practice, what did
D. Teaching methods should favor inquiry and problem schools tend to do?
solving.
I. Taught at the expense of NAT
51. Which software can you predict changes in weather
pattern and or trends in the population of endangered II. Conducted review classes for NAT at the expense of
species? teaching

A. word processing III. Practiced the so-called teaching to the test

B. spreadsheet A. II and III

C. desktop publishing B. II only

D. database C. I and III

D. III only

52. You would like to assess students' ability to write a 57. Which one appropriately describes your lesson if you
portfolio. What type of test will determine their ability to use the cognitive approach?
organize ideas and think critically? A. promotes "find out for yourself" approach
A. long test B. essay test B. lecture-dominated
C. formative testD. summative test
C. rote learning dominated
53. Teacher F asks one student, "Rachel, can you D. highly directed teaching
summarize what we have just read? Remember, the title
of this section of the chapter." This is an example of a 58. Which apply/applies to extrinsically motivated
teacher's effort at _____. learners?

A. scaffolding B. inspiring I. Tend to process information superficially

C. directingD. giving feedback


44
II. Tend to be content with meeting minimum B. What educational purposes should the school seek to
requirements attain?

III. Achieve at high level C. How can these educational experiences be effectively
organized?
A. I and II
D. How can we determine whether these purposes are
B. II only attained or not?
C. I and III 63. To elicit more student’s response, Teacher G made
D. I only use of covert responses. Which one did she NOT do?

59. To make our children become like "little scientists", A. She had the students write their response privately.
which of the following methods should we employ more
B. She showed the correct answers on the overhead
often? after the students have written their responses.
I. InquiryII. Problem-solving C. She had the students write their responses privately
III. Laboratory then called each of them.

A. II and III D. She refrained from judging on the student's


responses.
B. I and II
64. Which is the ideal stage of moral development?
C. I, II, and III
A. social contractB. universal ethical principle
D. I and III
C. law and orderD. good boy/ good girl
60. Teacher B is a teacher of English as a Second
Language. She uses vocabulary cards, fill-in-the-blank 65. There is a statement that says, "No amount of good
sentences, dictation and writing exercises in teaching a instruction will come out without good classroom
lesson about grocery shopping. Based on this management." Which of the following best explains this
information, which of the following is a valid conclusion? statement?

A. The teacher is reinforcing learning by giving the same A. Classroom management is important to effect good
information in, a variety of methods. instruction.

B. The teacher is applying Bloom's hierarchy of cognitive B. There must be classroom management for instruction
learning. to yield good outcomes/ results.

C. The teacher wants to do less talk. C. Classroom management means good instruction.

D. The teacher is emphasizing listening and speaking D. Good instruction is equal to effective classroom
skills. management.

61. A mother gives her son his favorite snack every time
the boy cleans up his room. Afterwards, the boy cleans 66. What does a Table of Specification establish?
his room everyday in anticipation of the snack. Which
theory explains this? A. construct validity

A. operant conditioningB. social learning theory B. content related validity and criterion reference

C. associative learning D. Pavlovian conditioning C. content validity and construct validity

62. What is the primary fundamental question in D. content validity and content related validity
examining a curriculum?
67. A person who had painful experiences at the
A. What educational experiences can be provided that dentist's office may become fearful at the mere sight of
are likely to attain these purposes? the dentist's office. Which theory can explain this?

A. generalization B. classical conditioning

45
C. operant conditioning D. attribution theory 73. Who claimed that children are natural learners and
therefore, must be taught in natural settings?
68. Which one can help student develop the habit of
critical thinking? A. PiagetB. Froebel

A. Asking low level questions C. MontessoriD. Kohlberg

B. A willingness to suspend judgment until sufficient 74. the benefit of "reading aloud" is that children learn
evidence is represented _____.

C. Asking convergent questions A. new vocabulary in meaningful contexts

D. Blind obedience to authority B. to value the presence of their friends as they read
together
69. Multiple intelligences can be used to explain
children's reading performance. Which group tends to be C. to make predictions by examining pictures and
good readers? listening for clues

A. linguistically intelligent group D. to use their imaginations to explore new ideas as they
listen to books
B. spatially intelligent group
75. What does a negatively skewed score contribution
C. existentially intelligent group imply?
D. kinesthetically intelligent group A. The scores congregate on the left side of the normal
70. Teachers should avoid _____ in assigning student contribution curve.B. The scores are widespread.
performance-based ratings. C. The students must be academically poor.
A. arbitrarines and bias D. The score congregate on the right side of the normal
B. unnecessary deductions contribution curve.

C. partiality and calculation 76. A high school graduate was refused admission to a
university on the grounds that he failed the admission
D. unnecessary evaluation test. The student insisted that he had the right to be
admitted and the act of the univeristy was a violation of
71. Which of the following is NOT a guidance role of the
his right to education. Was the student correct?
classroom teacher?
A. No, the university may refuse the student in its
A. Psychological Test Administrator
exercise of academic freedom.
B. Listener-Adviser
B. Yes, education is everyone's right.
C. Human Potential Discoverer
C. Yes, especially if he belongs to the Indigenous
D. Total Development Facilitator Peoples' group.

72. Watson applied classical conditioning in his D. No, if the university is exclusively for girls.
experiments and the results showed that behavior is
77. Which of the following is a correct statement on
learned through stimulus-response associations,
service contracting scheme?
specifically the development of emotional responses to
certain stimuli. This helps is in _____. A. It increases access to education.

A. interpreting reflexes as emotions B. It works against quality education.

B. understanding fears, phobias, and love C. It discriminates against private schools.

C. connecting observable behavior to stimulus D. it is not cost-effective.

D. understanding the role of overt behavior 78. Student M obtained an NSAT percentile rank of 80.
This indicates that _____.

46
A. he surpassed in performance 80% of her fellow A. a large number of students receiving low grades and
examinees very few students with high marks

B. he got a score of 80 B. a large number of more or less average students and


very few students receiving low and high grades
C. he surpassed in performance 20% of her fellow
examinees C. a large number of students with high grades and very
few with low grades
D. he answered 80 items correctly
D. all of the students have average grades
79. Teachers are encouraged to make use of authentic
assessment. Which goes with authentic assessment? 84. Lecturer C narrates: "I observe that when there is an
English-speaking foreigner in class, more often than not,
A. unrealistic performances his classmates perceive him to be superior." To which
B. de-contextualized drills Filipino trait does this point?

C. real world application of lessons learned A. hospitality

D. answering high multiple choice test items B. friendliness

80. Global students learn with short bursts of energy. To C. colonial mentality
maintain concentration they require ______. D. lack of confidence
A. frequent reminder that they need to concentrate 85. Which violates this brain-based principle of teaching-
B. frequent and intermittent breaks learning: "Each child's brain is unique and vastly different
from one another."
C. short and easy reading materials
A. giving ample opportunity for a pupil to explore even if
D. music while studying the class creates "noise"

81. A child was punished for cheating in an exam. For B. making a left-handed pupil write with his right hand
sure the child wont cheat again in short span of time, but as it is better this way
this does not guarantee that the child won’t cheat ever
again. Based on Thorndike’s theory on punishment and C. allowing open dialogue among students
learning, this shows that _____. D. employing MI teaching approaches
A. punishment strengthens a response 86. The child fainted in your class because she has not
B. punishment doesn’t remove a response eaten her breakfast. What is the best thing for you to do
in this situation?
C. punishment removes response
A. Ignore the situation B. Comfort the child
D. punishment weakens a response
C. Give the child food D. Call the parent

87. Which one should teacher AVOID to produce an


82. Faith, hope, and love are values now and forever environment conducive for learning?
whether they will be valued by people or not. Upon what
philosophy is this anchored? A. Tests

A. Idealism B. Seat plan

B. Existentialism C. Individual competition

C. Realism D. Games

D. Pragmatism 88. You are convinced that whenever a student performs


a desired behavior, provide reinforcement and soon the
83. In a grade distribution, what does the normal curve student learns to perform the behavior on her own. On
mean? which principle is your conviction based?

47
A. cognitivism B. All children 6-12 years old are enrolled in elementary
schools.
B. behaviorism
C. All children complete their elementary education
C. constructivism within six years.
D. environmentalism D. All Grade 6 students pass the division, regional, and
89. “Men are built not born.” This quotation by John national tests.
Watson states that _____. 94. Thorndike's law of effect states that a connection
A. the ineffectiveness of training on a person’s between stimulus and response is strengthened when
development the consequence is _____.

B. the effect of environmental stimulation on a person’s A. repeated


development B. negative
C. the absence of genetic influence on a person’s
C. pleasurable
development
D. positive
D. the effect of heredity
95. Which is the most reliable tool of seeing the
90. Which order follows the basic rule in framing
development in your pupils' ability to write?
interaction?
A. portfolio asssessment
A. call on a student, pause, ask the question
B. scoring rubric
B. ask the question, call on a student, pause
C. interview of pupils
C. ask the question, pause, call on a student
D. self-assessment
D. call on a student, ask the question, pause
96. The Philippine constitution directs the teaching of
91. At the preoperational stage of Piaget's cognitive religion in public schools on the following conditions
development theory, the child can see only his point of
except for?
view and assumes that everyone also has the same view
as his. What is this tendency called? A. option is expressed in writing

A. transductive reasoning B. without cost to the government

B. animism C. with cost shouldered by the parents or guardians

C. egocentrism D. given only at the option of parents or guardians

D. conservatism

92. "Vox Populi Est Supreme Lex" is a Latin expression 97. These are also known as “combination classes”
that means what? organized in barrios/barangays where the required
number of pupils of the same grade levels has not met
A. The Supreme Being is God the required number to make a separate class thus the
B. No one is above the law teacher apportions class time for instruction to every
grade level within class. These are ______.
C. The voice of the people is the supreme law
A. extension classesB. heterogeneous classes
D. it is the popular choice
C. multi-grade classes D. homogeneous classes
93. Which is not among the major targets of the Child-
Friendly School System (CFSS)? 98. After reading and paraphrasing R. Frost's Stopping by
the Woods on a Snowy Evening, Teacher M asked the
A. All school children are friendly. class to share any insight derived from this poem. On

48
which assumption about the learner is Teacher M's act of III. The child has a certain degree of freedom not to allow
asking the class to share their insight based? himself to be shaped by his environment.

A. Learners are producers of knowledge, not only A. III only


passive recipients of information.
B. I and II
B. Learners are meant to interact with one another.
C. I and III
C. Learners are like empty receptacles waiting to be
filled up. D. II only

D. Learners have multiple intelligence and varied 103. _____ supports equitable access but on the other
hand, quality might be compromised. A. Open admission
learning styles.
B. School accreditation C. Deregulated tuition fee D.
99. "Do not cheat. Cheating does not pay. If you do, you Selective retention
cheat yourself." says the voiceless voice from within you.
In the context of Freud's theory, which is at work 104. If you want your students to develop reading
comprehension and learning strategies which one should
A. id alone B. superego alone you employ?

C. ego aloneD. Id and ego interaction A. reciprocal teaching

100. Which of the following assessment tools would you B. cooperative learning
recommend if one should adhere to constructivist theory
of learning? C. peer tutoring

D. mastery learning
I. Constructed response test
105. What statement is FALSE with reference to Article
II. Performance test
VIII “The Teacher and the Learners” of the Code of Ethics
III. Checklist of a motor screening test of Professional Teachers?

IV. Observation test A. A teacher shall not accept favors or gifts from
learners, their parents, or others in their behalf in
A. I and IIB. II and III exchange for requested concessions, especially if
C. I, II, and IIID. I, II, and IV undeserved,

101. The test in English and Mathematics showed poor B. A teacher shall not inflict corporal punishment or
results in comprehension and problem-solving questions. offending learners nor make deductions from their
How may the data be used for better learners' scholastic ratings as a punishment for acts which are
performance? clearly not manifestation of poor scholarship.

A. Use context clues in vocabulary building. C. In a situation where mutual attraction and
subsequent love develop between teacher and the
B. Give more exercises/situations on comprehension learner, the teacher shall exercise utmost professional
questions. discretion to avoid scandal, gossip, and preferential
treatment of the learner.
C. Determine weakness in grammatical structures.
D. A teacher shall maintain at all ties a dignified
D. Involve parents in guiding learners' developing good
personality, which could serve as model worthy of
study habits.
emulation by learners, peers and others.
102. Which of the following may be the assumption/s of
106. In the light of the modern concept of teaching,
behaviorists?
which is a characteristic of effective teaching?
I. The mind of a newborn child is a blank slate.
A. pouring information to the learners
II. All behaviors are determined by environmental
B. allowing learners to learn on their own
events.
C. developing abilities to address the future

49
D. removing the physical presence of the teacher 112. Professionalization of teachers and teaching as
promulgated in Presidential Decree No. 1006, defines
teaching as profession concerned with classroom
107. One facet of understanding, an evidence of learning institution ______.
is perspective. Which is an indicator of perspective? A. by teachers on full-time basis
A. A bright student refuses to consider that there is B. at the tertiary level in both public and private
another correct solution to the problem apart from hers.
institutions
B. A student explains the arguments for and against the C. at the elementary and secondary levels in both public
acquittal of Hubert Webb and group. and private schools
C. A teacher cannot accept opinions different from hers. D. by teachers of permanent status
D. A mother cannot understand why her child's 113. Teacher U asked her pupils to create a story out of
performance is below par. the given pictures. Which projective technique did
108. Which among the indicators could be most useful Teacher U use?
for assessing quality of schooling? A. Rorschach testB. narrative
A. participation rate C. thematic apperception testD. reflective
B. cohort survival rate 114. The Code of Ethics for Professional Teachers
C. net enrollment rate stipulates that educational institutions shall offer quality
education for all Filipino citizens. How is quality
D. drop-out rate education defined in R.A. 9155?

109. The discrimination index of a test item is +0.48. A. Relevance and excellence of education are
What does this mean? emphasized to meet the needs and aspirations of an
individual and society.
A. An equal number from the lower and upper group got
the item correctly. B. All school-aged children should be provided free and
compulsory education.
B. More from the upper group got the item wrongly.
C. Children with special needs should be mainstreamed
C. More from the lower group got the item correctly.
with regular classes in the public schools.
D. More from the upper group got the item correctly.
D. Public and private basic education schools should
110. Which of the following is NOT an example of a provide relevant education.
teacher’s nonverbal communication?
115. Teacher A discovered that his pupils are very good
A. eye contactB. gestures in dramatizing. Which tool must have helped him
discover his pupils’ strength?
C. pauses
A. Portfolio assessmentB. Performance test
D. voice
C. Journal entryD. Paper-and-pencil test
111. Which is a valid assessment tool if you want to find
out how well your students can speak 116. Which appropriate teaching practice flows from this
extemporaneously? research finding on the brain: The brain's emotional
center is tied into its ability to learn.
A. performance test in extemporaneous speaking
A. Create a learning environment that encourages
B. written quiz on how to deliver extemporaneous students to explore their feelings and ideas freely.
speech
B. Come up with highly competitive games where
C. display of speeches delivered winners will feel happy.

D. writing speeches

50
C. Tell students to participate in class activities or else C. co-curricular activities
won't receive plus points in class recitation. D. To
establish discipline, be judgmental in attitude. D. curricular activities

117. Your percentile rank in class is 60%. What does this 121. Which must be present for self-evaluation to
mean? succeed?

A. You got 40% of the test items wrongly. A. consensus between teacher and student regarding
evaluation results
B. You scored less than 60% of the class.
B. teacher's approval of self-evaluation results
C. You got 60% of the test items correctly.
C. teacher's monitoring of self-evaluation process
D. You scored better than 60% of the class.
D. student's intrinsic motivation to learn
118. Which test item is in the highest level of Bloom's
122. The instructions for a test are made simple, clear
taxonomy of objectives?
and concise. This is part of which of the following
A. Explain how a tree functions in relation to the characteristics of a good test?
ecosystem.
A. objectivityB. economy
B. Explain how trees receive nutrients.
C. administrabilityD. scorability
C. Rate three different methods of controlling tree
growth.

D. Write a paragraph that observes coherence, unity, 123. What is the mean of this score distribution: 4, 5, 6,
and variety. 7, 8, 9, 10?

119. In qualitative social and behavioral studies, "the A. 7B. 6


investigator is a part of the study." What are implied in C. 8.5D. 7.5
this statement?
124. The Early Childhood Care and Development Act
I. The researcher processes and analyzes the data provides for the promotion of the rights of children for
himself. survival and development. Which of the following
II Data interpretation depends on the orientation of the statements is not among its objectives?
researcher. A. Enhance the role of parents as the primary caregivers
III. The investigator is the only source of information. and educators of their children form birth onward.

IV. Data gathering may be done by others but the B. Facilitate a smooth transition from care and
analysis is done by the researcher. education provided at home to community or school-
based setting and primary schools.
A. I and IV
C. Assist the LGUs in their endeavor to prepare the child
B. II and III for adulthood.

C. I, II, and IV D. Enhance the physical, social, emotional, cognitive,


psychological, spiritual, and language development of
D. I, II, III, and IV
young children.
120. The observable manifestation of student's feelings, 125. Which of the following represents a minuscule
thoughts, or attitude are summed up as behavior. Every
curriculum?
high school teacher is expected to contribute to the
assessment of the student's behavior but the grade is A. textbook that learners use
reflected in _____.
B. lesson plan that teachers prepare
A. Good Manners and Right Conduct
C. reference materials that supplement the text
B. Values Education
D. lecture notes of the teacher

51
126. Bernadette enjoyed the roller coaster when they A. II onlyB. I only
went to Enchanted Kingdom. Just at the sight of a roller
coaster, she gets excited. Which theory explains C. I and IIID. I and II
Bernadette's behavior? 131. Here is a score distribution: 98, 93, 93, 93, 90, 88,
A. operant conditioning 87, 85, 85, 85, 70, 51, 34, 34, 34, 20, 18, 15, 12, 9, 8, 6, 3,
1. Which is the range?
B. attribution theory
A. 93
C. Pavlovian conditioning
B. 85
D. social learning theory
C. 97
127. What objectives do effective leaders foster?
D. between 51 and 34
I. AttainableII.Measurable
132. Which tool should a teacher use if she wants to
III.Results-orientedIV. SpecificV. Time-bound locate areas which are adversely affecting the
performance of a significant number of students?
A. II, III, and IV
A. problem checklist
B. I, II, III, IV, and V
B. self-report technique
C. I, IV, and V
C. autobiography
D. I, II, III, and IV
D. cumulative record
128. Thomasites are _____.
133. Which activity works best with self-expressive
A. the soldiers who doubted the success of the public
people?
educational system to be set in the Philippines
A. metaphors
B. the first American teacher recruits to help establish
the public educational system in the Philippines B. kinesthetic activities

C. the first religious group who came to the Philippines C. inquiry


on board the US transport named Thomas.
D. independent study
D. the devotees of St. Thomas Aquinas who came to
134. centralization : Education Act of 1901 ::
evangelize
decentralization : _____
129. Which is an underlying assumption of the social
cognitive theory? A. R.A. 9155

A. People are social by nature. B. R.A. 9293

C. R.A. 7836
B. People learn by observing others.
D. R.A. 7722
C. People learn by trial-and-error.
135. Piagetian tasks state that thinking becomes more
D. People learn by association.
logical and abstract as children reach the formal
130. Researchers found that when a child is engaged in a operations stage. What is an educational implication of
learning experience, a number of areas of the brain are this finding?
simultaneously activated. What is an application of this
in the teaching-learning process? A. Expect hypothetical reasoning from learners between
12 to 15 years of age.
I. Make use of field trips, guest speakers
B. Engage children in analogical reasoning as early as
II. Do multicultural units of study preschool to train them for HOTS.

III. Stick to the "left brain and right brain" approach

52
C. Learners who are not capable of logical reasoning D. technique to be used
from ages 8 to 11 lag behind in their cognitive
141. With which goals of educational institutions as
development.
provided for by the Constitution is the development of
D. Let children be children. work skills aligned?

136. This embodies the teacher’s duties and A. To develop moral character
responsibilities as well as proper behavior in performing
B. To teach the duties of citizenship
them.

A. Code of Ethics for Teachers C. To inculcate love of country

B. Magna Carta for Public School Teachers D. To develop vocational efficiency

C. Bill of Rights 142. Median is to point as standard deviation is to _____.

D. Philippine Constitution of 1987 A. area B. volume C. distanceD. square

143. Who is the forerunner of the presence of the


137. The class was asked to share their insights about the
poem. The ability to come up with an insight stems from Language Acquisition Device?
the ability to _____. A. WatsonB. Chomsky C. Gardner D. Piaget
A. comprehend the subject that is being studied 144. Which of the following statements best describes
B. analyze the parts of a whole metacognition as a strategy for curriculum
augmentation?
C. evaluate the worthiness of a thing
A. It is learning how to learn and thinking about how
D. relate and organize things and ideas one thinks.

138. Which is not a characteristic of authentic B. It is learning strategies for success.


assessment?
C. It is learning through interaction with the
A. focused on lifelike, meaningful, relevant types of environment.
student learning
D. It is learning through computer-aided instruction.
B. offers opportunities to study problem intensively
145. Which psychological theory states that the mind
C. easy to complete insists on finding patterns in things that contribute to the
development of insight?
D. fruitful in terms of genuine learning
A. Piaget's psychology
139. Schools should develop in the students the ability to
adapt to a changing world. This is adhereance to the B. Kohlberg's psychology
philosophy of _____.
C. Gestalt psychology
A. essentialism
D. Bruner's psychology
B. perennialism
146. Which is not a characteristic of an analytic learner?
C. progressivism
A. learns whole to part
D. reconstructionism
B. enjoys memorizing
140. The teacher’s first task in the selection of media in
teaching is to determine the ______. C. works on details

A. choice of the students D. sequences objects in order

B. availability of the media 147. The concepts of trust vs. maturity, autonomy vs.
self-doubt, and initiative vs. guilt are most closely related
C. objectives of the lesson with the works of __________.

53
A. Erikson 91. C92. C93. D94. D95. A96. B97. C98. A99. B100. A

B. Piaget 101. B102. B103. A104. A105. D106. B107. B108. B109. D

C. Freud 110. D111. A112. C113. C114. A115. B116. A117. D118. D

119. A120. B121. D122. C123. A124. C125. B126. C127. B


D. Jung
128. C129. B130. D131. C132. A133. B134. A135. A136. A
148. Ask to do a learning task, Joe hesitates and says
"Mahirap. Ayaw ko. 'Di ko kaya!" (It's difficult. I don't like 137. D138. C139. D140. C141. D142. B143. D144. A145. C
it. I can't do it.) Is it possible to motivate this type of
student? 146. A147. A148. A149. B150. C

A. Yes, he can do something with his ability. 1. "Life is what you make it" exemplifies best what
______:
B. Yes, he can change the nature of the job.
A. Existentialism✅✅✅
C. No, it is impossible to motivate a student who himself
is not motivated. B. Humanism

D. No, motivation is totally dependent on the student. C. Progressivism


No person outside him can influence him.
D. Essentialism
149. As of the Republic Act 7836 the licensure exam for
Answer: A
teachers is with the _____.
2. After listening to the homily of the Priest about
A. Commission on Higher Education
fidelity, Catherine has a moment of reflection. Her
B. Professional Regulation Commission understanding of the value of fidelity has become deeper
as she related this to her past experience. This typifies
C. Department of Education what kind of philosophy ?
D. Civil Service Commission A. Constructivism✅✅✅
150. Filipinization is violated if _____. B. Reconstructivism
A. an educational institution is owned by a corporation C. Humanism
of which 40% of the capital is owned by Filipino Citizens
D. Existentialism
B. an educational institution owned by a religious order
Answer : A
C. an American serving as president of the educational
institution 3. After studying the Principle of Identity, Teacher Beng
asks her students to determine which among the given
D. an educational institution owned by a charitable set of problems conforms to the said identity. This shows
institution that Teacher Beng upholds this kind of philosophy?
Answer Key A. Perennialism
1. B2. B3. A4. C5. A6. A7. D8. A9. C10. C11. D12. B13. C
B. Progressivism ✅✅✅
14. B15. D16. C17. A18. C19. D20. A21. B22. C23. C24. B
C. Essentialism
25. A26. A27. D28. D29. A30. B31. C32. A33. C34. B35. D
D. Naturalism
36. C37. B38. D39. D40. A41. B42. D43. A44. C45. A46. B
Answer : B
47. A48. D49. D50. D51. B52. B53. A54. C55. C56. A57. A
4. Joy listens to the advice given by her sister to end the
58. A59. C60. A61. A62. B63. B64. B65. B66. C67. B68. B
relationship that she has with Min Hoo . However, her
69. A70. A71. A72. B73. C74. B75. D76. A77. A78. A79. C sister learned that the advice she has given was not
followed and Joy decided to continue the relationship.
80. B81. B82. A83. B84. C85. B86. C87. C88. B89. B90. C

54
This action of Joy is a manifestation of what kind of A. Realism
philosophy?
B. Idealism ✅✅✅
A. Essentialism
C. Realism
B. Existentialism✅✅✅
D. Empiricism
C. Perennialism
Answer : B
D. Humanism
9. Lee Min Hoo learns well through active interplay with
Answer : B others. His learning increases when engaged I activities
that has meaning in him. Which philosophy can describe
5. After finishing the degree in Education, Teacher Mary
this ?
realizes that learning never stops. In fact, she
accumulates more knowledge after leaving the portal of A. Progressivism ✅✅✅
her alma mater. This typifies what kind of philosophy?
B. Idealism
A. Constructivism
C. Realism
B. Perennialism
D. Empiricism
C. Progressivism✅✅✅
Answer : A
D. Humanism
10. "Learning is the process of retrieving prior learning" ,
Answer: C this is a statement from ;

6. Mary Joy is an advocate of the principle "making the A. Constructivist ✅✅✅


most of your life". Live life to the fullest. She is indeed an
advocate of what kind of philosophy ? B. Reconstructivist

A. Humanism ✅✅✅ C. Progressivist

B. Idealism D. Idealist

C. Realism Answer : A

11. "The curriculum is a total learning experience for an


D. Existentialism
individual". This point of view is considered to be of :
Answer : A
A. Traditionalist
7. Teacher Mae Joy is a very dedicated teacher in the
nursery. Her foremost concern is for the students to B. Progressivist ✅✅✅
learn how to adapt themselves in the environment. This
C. Materialist
shows that Teacher Mae Joy upholds this kind of
philosophy? D. Cognitivist

A. Naturalism✅✅✅ Answer : B

B. Pragmatism 12. The principal shares the thought with his teachers
that the subject matter should help students understand
C. Progressivism and appreciate themselves as unique individuals who
accept complete responsibility of their thoughts,
D. Rationalism
feelings, and actions. From which point of view is this
Answer : A thought based ?

8. Teacher Sun Woo serves as an inspiration to his A. Essentialism


students because of his efficiency and effectiveness as a
teacher. The mind se of his students towards him is an B. Rationalism
instance of what kind of philosophy?

55
C. Progressivism 16. If you identify with Teacher Karylle, you adhere to
this philosophy
D. Existentialism ✅✅✅
A. Behaviorist
Answer : D
B. Perennnialism✅✅✅
13. During the faculty recollection, Mr. Duterte asked his
teachers to share their thoughts about their students as C. Existentialist
their primary customer. What followed were the gist of
what were shared : D. Progressivist

Teacher Vhong : The learner is a product of his Answer : B


environment. Sometimes he has no choice. He is 17. Student Park Shin Hye, after receiving the conceptual
determined by his environment. knowledge on multiplication of trinomials from his
Teacher Vice : The learner can choose what he becomes Algebra Teacher , immediately answers the exercises in
despite his environment. the book. This demonstrates what kind of philosophy ?

Teacher Anne : The learner is a social being who learns A. Existentialism


well through an active interplay with others. B. Idealism
Teacher Karylle : The learner is a rational being. School C. Constructivism
should develop his rational and moral powers.
D. Progressivism✅✅✅
Whose philosophical concept is that Teacher Vhong ?
Answer : D
A. Existentialist
18. Teacher Lee Jung Suk views his students as a unique ,
B. Progressivist
free choosing and responsible individuals. He encourages
C. Behaviorist ✅✅✅ them to develop their own individualities. What
philosophy does Teacher Lee Jung Suk adhere to ?
D. Rationalist
A. Naturalism
Answer : C
B. Humanism
14. Teacher Vice response comes from the mouth of ?
C. Existentialism✅✅✅
A. Essentialist
D. Contructivism
B. Perennialist
Answer : C
C. Existentialist ✅✅✅
19. In the Social Science class of Mr. Kim So Hyun,
D. Behaviorist students identify the various social and economic
problems that require urgent solutions. They not only
Answer : C discuss the ways to address it but also agreed to
15. If you agree with Teacher Anne, you are more of participate in solving them. What kind of philosophy
_______ does this class uphold ?

A. Essentialism
A. Progressivist ✅✅✅

B. Perennialist B. Reconstructivism✅✅✅

C. Essentialist C. Naturalism

D. Rationalist D. Behaviorism

Answer : A Answer : B

20. Teacher Park Bo Gum, unlike other teachers in his


school, practices the non-traditional method of

56
discussing the lessons using not only the book prescribed D. Idealism✅✅✅
by the school as the main source of information and
knowledge. Instead , he went out of his way to devise Answer : D
other means that will help his students to understand
24. If a teacher states that "specialization is knowing
their lessons. To what philosophy does Teacher Park Bo
more and more about less and less, hence it is better to
Gum submit to ?
be a generalist". What kind of philosophy does they
A. Behaviorism uphold?

B. Contructivism A. Progressivism

C. Progressivism✅✅✅ B. Essentialism✅✅✅

D. Humanism C. Naturalism

Answer : C D. Perennialism

21. Teacher Soong Jong Ki class is engage in problem Answer : B


solving activity which in a way is a reflection of the
25. In social trends, his do you call those who are for the
personal and social experiences. The purpose of this
revival of the classics ?
activity is for the students to acquire the skills that can
help them in solving their own real-life problems. What A. Progressivist
philosophy can best described this ?
B. Essentialist
A. Existentialism
C. Humanist✅✅✅
B. Progressivism✅✅✅
D. Behaviorist
C. Idealism
Answer : C
D. Realism
26. A teacher puts emphasis on making her students
Answer : B master the rules of multiplying fractions, and not so
much on the strategies and execution. This is an
22. In his class, Teacher Goong Yu always presents
illustration of _______?
principles and values so as to encourage his students to
examine them and decide for themselves whether to A. Existentialism
accept it or not . What kind of philosophy does Teacher
Goong Yu practice ? B. Progressivism

A. Idealism C. Essentialism✅✅✅

B. Existentialism✅✅✅ D. Naturalism

C. Humanism Answer : C

D. Essentialism 27. Students are the product of their environment. To


which philosophy does this statement belong ?
Answer : B
A. Naturalism
23. Teacher Kim Woo Bin is an inspiration to almost all of
his students. His efficiency and effectiveness in the B. Essentialism
profession is truly outstanding. Which of the following
C. Behaviorism✅✅✅
describes this attitude of the students toward him ?
D. Existentialism
A. Existentialism
Answer : C
B. Naturalism
28. What philosophy posses that a social teacher is a
C. Progressivism
revered person, central to the educative process and

57
therefore must be excellent mentally and morally in B. Naturalism
personally conduct and conviction ?
C. Essentialism✅✅✅
A. Idealism✅✅✅
D. Behaviorism
B. Progressivism
Answer : C
C. Realism
33. The philosophy believes that the schools should
D. Essentialism originate policies and progress which would bring about
reform of the social orders
Answer : A
A. Behaviorism
29. This philosophy emphasizes the teaching of
controversial issues to prepare students for a progressive B. Progressivism
structuring of the social order.
C. Reconstructionism✅✅✅
A. Progressivism
D. Perennialism
B. Social Experimentation
Answer : C
C. Behaviorism
34. This philosophy contends that the intellectual faculty
D. Reconstructionism✅✅✅ of man is higher that the physical faculties.

Answer : D A. Perennialism

30. The philosophy contends that truth is universal an B. Idealism✅✅✅


unchanging and therefore a good education is also
universal and constant. C. Realism

A. Existentialism D. Existentialism

B. Essentialism Answer : B

C. Perennialism✅✅✅ 35. The philosophy proclaims the spiritual nature and the
universe. It holds that the good, the true anted the
D. Progressivism beautiful are permanently part of the structure of an
unchanging universe.
Answer : C
A. Realism
31. Which philosophy of education has for its curriculum
a body intellectual subject matter that are conceptual on B. Existentialism
subject, essential for the realization of mental and moral
development? C. Naturalism

A. Pragmatism D. Idealism✅✅✅

B. Realism✅✅✅ Answer : D

C. Existentialism 36. This philosophy posses the known ability of the world
and everything in its as they are in themselves and their
D. Idealism existence independent of human mind.

Answer : B A. Realism✅✅✅

32. The educational theory, in response to progressivism, B. Idealism


which concerns with the revival of effort in the direction
of teaching the fundamental tools of learning as the C. Naturalism
most indispensable type of education .
D. Existentialism
A. Perennialism
Answer : A

58
37. "The central aim of education should be develop the questioning and discussion, lecture and the project,
power of thought " ; this would likely be the aim of what whether methods done singly or in group.
educational practitioner ?
A. Behaviorism
A. Reconstructivism
B. Essentialism
B. Progressivism
C. Idealism ✅✅✅
C. Perennialism✅✅✅
D. Existentialism
D. Existentialism
Answer : C
Answer : C
42. This theory views education as a recurring process
38. This educational theory is based on the philosophy based on eternal truths; thus, the schools curriculum
grounded on experience and the interaction of the should emphasize the recurrent themes of human life.
person with his environment. It posist that the education
must use past experiences to direct future experiences. A. Naturalism

B. Progressivism
A. Progressivism✅✅✅
C. Perennialism ✅✅✅
B. Idealism
D. Pragmatism
C. Existentialism

D. Realism Answer : C

Answer : A 43. Socrates : Idealism

39. This philosophy is a way of viewing and thinking Dewey : ???????


about life in the world so that priority is given into A. Essentialism
individualism and subjectivity. It believes that human
beings are the creator of their own experiences. B. Progressivism ✅✅✅

A. Idealism C. Existentialism

B. Reconstructionism D. Naturalism

C. Realism Answer : B

D. Existentialism✅✅✅ 44. Hutchins : ?????

Answer : D Watson : Behaviorism

40. It is belief that when an idea agrees with its object, it A. Progressivism
is proof of its truth. However, it is definition of truth not
criteria . B. Perennialism ✅✅✅

A. Idealist C. Behaviorism

B. Naturalism D. Naturalism

Answer : B
C. Naive Realism✅✅✅
45. Essentialism : Bagley ;_____: Jean Paul Sarte
D. Behaviorist
A. Naturalism
Answer : C

41. This educational philosophy encourages B. Existentialism ✅✅✅


accumulation of knowledge and thinking and must apply C. Behaviorism
criteria for moral evaluation. Suggested methods are
D. Progressivism

59
Answer : B C. Hedonism

46. The establishment of the school of the future (SOF) D. Deism or Theism✅✅✅
introduces improvement and reform in education to
respond to both human and social needs. What Answer : D
philosophy is related to it ?
ASSESSMENT
A. Perennialism
1. Who among the teachers described below is
doing assessment?
B. Idealism
a. Mrs. Bautista who is administering a test to her
C. Existentialism students.
b. Mr. Ferrer who is counting the scores obtained
D. Reconstructivism✅✅✅ by the students in his test.
c. Ms. Leyva who is computing the final grade of
Answer : D the students after completing all their
requirements.
47. The Department of Education gives greater emphasis d. Prof. Cuevas who is planning for a remedial
on the development of basic skills. What is the instruction after knowing that students perform
poorly in her test
philosophical basis for this ?
2. Mr. Fernandez is judging the accuracy of these
A. Pragmatism
statements. Which statements will he consider as
correct?
B. Existentialism
I. Test is a tool to measure a trait.
C. Naturalism II. Measurement is the process of qualifying a
given trait.
D. Essentialism✅✅✅ III. Assessment is the gathering of quantitative
and qualitative data.
Answer : D IV. Evaluation is the analysis of quantitative and
qualitative data for decision making
48. Teacher Sandara views her students as unique , free- A. I and II only
choosing and responsible individuals. All classroom b. III and IV only
c. I, II, and III
activities revolve the said premise. What theory
d. I, III and IV
underlines this ?
3. If I have to use the most authentic method of
A. Naturalism assessment, which of these procedures should I
consider?
B. Essentialism a. Traditional Test
b. Performance-based Assessment
C. Behaviorism
c. Written Test
d. Objective Assessment
D. Existentialism ✅✅✅
4. After doing the exercise on verbs, Ms. Borillo
Answer : D
gave a short quiz to find out how well students
49. The most influential movement in the 18th century. have understood the lesson. What type of
assessment was done?
Acts according by the nature . a. Summative Assessment
b. Formative Assessment
A. Idealism
c. Diagnostic Assessment
d. Placement Assessment
B. Existentialism
5. Who among the teachers below performed a
C. Naturalism✅✅✅
diagnostic assessment?
D. Realism a. Ms. Santos who asked questions when the
discussion was going on to know who among h er
Answer : C students understood what she was trying to
emphasize.
50. Admits the existence of GOD. b. Mr. Colubong who gave a short quiz after
discussing thoroughly the lesson to determine the
A. AgnosticismB. Atheism programs of learning.
c. Ms. Ventura who gave 10-item test to find out

60
the specific lessons which the students failed to a. No, the giving of an objective test is more
understand. appropriate than the use of essay.
d. Mrs. Lopez who administered a readiness test b. No, such method of assessment is
to the incoming grade one pupils. inappropriate because essay is difficult.
c. Yes, essay test could measure more than what
6. You are assessing for learning. Which of these other tests could measure.
will you likely do? d. Yes, essay test is the best in measuring any
a. Giving grades to students type of knowledge.
b. Reporting to parents the performance of their
child. 12. What type of validity does the Pre-board
c. Recommending new policies in grading examination possess if its results can explain how
students. the students will likely perform in their licensure
d. Assessing the strengths and weaknesses of examination?
students. a. Concurrent
b. Predictive
7. Ms. Saplan is planning to do an assessment of c. Construct
learning. Which of these should she include in her d. Content
plan considering her purpose for assessment?
a. How to give immediate feedback to student's 13. Ms. Aviz wants to determine if the students'
strengths and weaknesses scores in their Final Test is reliable. However, she
b. How to determine the area of interest of has only one set of test and her students are
learners already on vacation. What test of reliability can
c. How to certify student's achievement she employ?
d. How to design one's instruction a. Test-Retest
b. Kuder Richardson Method
c. Equivalent Forms
8. You targeted that after instruction, your d. Test-Retest with Equivalent Forms
students should be able to show their ability to
solve problems with speed and accuracy. You Refer to this case in answering items 14-15
then designed a tool to measure this ability. What Two teachers of the same grade level have set
principle of assessment did you consider in this the following objectives for the day's lesson. At
situation? the end of the period, the students should be able
a. Assessment should be based on clear and to:
appropriate learning targets or objectives. a. Construct bar graph, and
b. Assessment should have a positive b. Interpret bar graphs
consequence on student's learning
c. Assessment should be reliable. To assess the attainment of the objectives,
d. Assessment should be fair. Teacher A required the students to construct a
bar graph for the given set of data then she asked
9. Ms. Ortega tasked her students to show how to them to interpret this using a set of questions as
play basketball. What learning target is she guide. Teacher B presented a bar graph then
assessing? asked them to interpret this using also a set of
a. Knowledge guide questions.
b. Reasoning
c. Skills 14. Whose practice is acceptable based on the
d. Products principles of assessment?
a. Teacher A
10. Mr. Ravelas made an essay test for the b. Teacher B
objective "Identify the planets in the solar system". c. Both Teacher A and B
Was the assessment method used the most d. Neither Teacher A nor Teacher B
appropriate for the given objective? Why?
a. Yes, because essay test is easier to construct 15. Which is true about the given case?
than objective test. a. Objective A matched with performance-based
b. Yes, because essay test can measure any type assessment while B can be assessed using the
of objective. traditional pen-and-paper objective test.
c. No, he should have conducted oral questioning. b. Objective A matched with traditional
d. No, he should have prepared an objective test. assessment while B can be assessed using a
performance-based method.
11. Mr. Cidro wants to test students' knowledge of c. Both objective A and B matched with
the different places in the Philippines, their capital performance-based assessment.
and their products and so he gave his students an d. Both objective A and B matched with traditional
essay test. If you were the teacher, will you do the assessment.
same?
61
16. In the context of the Theory of Multiple a. Diary
Intelligence, which is a weakness of the paper- b. Portfolio
pencil test? c. Anecdotal record
a. It puts non-linguistically intelligent at a d. Observation report
disadvantage.
b. It is not easy to administer. 22. Mrs. Catalan allowed the students to develop
c. It utilizes so much time. their own portfolio in their own style as long as
d. It lacks reliability. they show all the non-negotiable evidences of
learning. What principle in portfolio assessment
17. Mr. Umayam is doing a performance-based explains this practice?
assessment for the day's lesson. Which of the a. Content Principle
following will most likely happen? b. Learning Principle
a. Students are evaluated in one sitting. c. Equity Principle
b. Students do an actual demonstration of their d. Product Principle
skill.
c. Students are evaluated in the most objective 23. How should the following steps in portfolio
manner. assessment be arranged logically?
d. Students are evaluated based on varied I. Set targets
evidences of learning II. Select evidences
III. Collect evidences
18. Ms. del Rosario rated her students in terms of IV. Rate Collection
appropriate and effective use of some laboratory V. Reflect on Evidences
equipment and measurement tools and the a. I, II, III, IV, V
students ability to follow the specified procedures. b. I, III, II, V, IV
What mode of assessment should Miss del c. I, II, III, V, IV
Rosario use? d. I, III, V, II, IV
a. Portfolio Assessment
b. Journal Assessment 24. Which could be seen in a rubric?
c. Traditional Assessment I. Objective in a high level of cognitive behavior
d. Performance-based Assessment II. Multiple criteria in assessing learning
III. Quantitative descriptions of the quality of work
19. Mrs. Hilario presented the lesson on baking IV. Qualitative descriptions of the quality of work
through a group activity so that the students will a. I and II only
not just learn how to bake but also develop their b. II, III and IV only
interpersonal skills. How should this lesson be c. I, II and III
assessed? d. I, II, III and IV
I. She should give the students an essay test
explaining how they baked the cake. 25. The pupils are to be judged individually on
II. The students should be graded on the quality of their mastery of the singing of the national anthem
their baked cake using a rubric. so their teacher let them sing individually. What
III. The students in a group should rate the should the teacher use in rating the performance
members based on their ability to cooperate in of the pupils considering the fact that the teacher
their group activity. has only one period to spend in evaluating her 20
IV. She should observe how the pupils perform pupils?
their tasks. a. Analytic
b. Holistic
a. I, II, and III only c. Either holistic or analytic
b. I, III, and IV only d. Both holistic and analytic
c. I, II and IV only
d. I, II, III, and IV Answers: 1C 2D 3B 4B 5C 6D 7C 8A 9C 10D
11A 12B 13B 14A 15A 16A 17B 18D 19C 20C
20. If a teacher has set objectives in all domains 21B 22C 23B 24B 25B
or learning targets and which could be assessed
using a single performance task, what criterion in
selecting a task should she consider? 1. Mrs. Pua is judging the worth of the project of
a. Generalizability the students in her Science class based on a set
b. Fairness of criteria. What process describes what she is
c. Multiple Foci doing?
d. Teachability a. Testing
b. Measuring
21. Which term refers to the collection of students' c. Evaluating
products and accomplishments in a given period d. Assessing
for evaluation purposes?
62
2. Mrs. Acebuche is comparing measurement
from evaluation. Which statement explains the
difference? 8. The grade six pupils were given a diagnostic
a. Measurement is assigning a numerical value to test in addition and subtraction of whole numbers
a given trait while evaluation is giving meaning to to find out if they can proceed to the next unit.
the numerical value of the trait. However, the results of the test were very low.
b. Measurement is the process of gathering while What should the teacher do?
evaluation is the process of quantifying the data a. Proceed to the next lesson to be able to finish
gathered. all the topics in the course.
c. Measurement is the process of quantifying data b. Construct another test parallel to the given test
while evaluation is the process of organizing data. to determine the consistency of the scores.
d. Measurement is a pre-requisite of assessment c. Count the frequency of errors to find out the
while evaluation is the pre-requisite of testing. lessons that the majority of students need to
relearn.
3. Ms. Ricafort uses alternative methods of d. Record the scores then inform the parents
assessment. Which of the following will she not about the very poor performance of their child in
likely use? mathematics.
a. Multiple Choice Test
b. Reflective Journal Writing 9. Mrs. Nogueras is doing an assessment of
c. Oral Presentation learning. At what stage of instruction should she
d. Developing Portfolios do it?
a. Before instruction
4. Ms. Camba aims to measure a product of b. After instruction
learning. Which of these objectives will she most c. Prior to instruction
likely set for her instruction? d. During the instructional process
a. Show positive attitude towards learning
common nouns 10. Mr. Cartilla developed an Achievement Test in
b. Identify common nouns in a reading selection Math for her grade three pupils. Before she
c. Construct a paragraph using common nouns finalized the test she examined carefully if the test
d. User a common noun in a sentence items were constructed based on the
competencies that have to be tested. What test of
5. The students of Mrs. Valino are very noisy. To validity was she trying to establish?
keep them busy, they were given any test a. Content-validity
available in the classroom and then the results b. Concurrent validity
were graded as a way to punish them. Which c. Predictive validity
statement best explains if the practice is d. Construct validity
acceptable or not?
a. The practice is acceptable because the 11. Mrs. Robles wants to establish the reliability of
students behaved well when they were given a her achievement test in English. Which of the
test. following activities will help achieve her purpose?
b. The practice is not acceptable because it a. Administer two parallel tests to different groups
violates the principle of reliability. of students.
c. The practice is not acceptable because it b. Administer two equivalent tests to the same
violates the principle of validity. group of students
d. The practice is acceptable since the test results c. Administer a single test but two different groups
are graded. of students.
d. Administer two different tests but to the same
6. Ms. Delos Angeles advocates assessment for group of students.
learning. Which will she NOT likely do?
a. Formative Assessment Refer to the situation below in answer items 12
b. Diagnostic Assessment and 13
c. Placement Assessment A teacher set the following objectives for the day's
d. Summative Assessment lesson:
At the end of the period, the students should be
7. At the beginning of the school year, the 6-year able to:
old pupils were tested to find out who among a. Identify the parts of friendly letter
them can already read. The result was used to b. Construct a friendly letter using the MS Word,
determine their sections. What kind of test was and
given to them? c. Show interest towards the day's lesson
a. Diagnostic To assess the attainment of the objectives, Ms.
b. Formative Cidro required the students to construct friendly
c. Placement letter and have it encoded at their Computer
d. Summative Laboratory using the MS Word. The letter should
63
inform one's friend about what one has learned in b. Mrs. Mandia who asked her pupils to construct
the day's lesson and how one felt about it. a word problem for a given number sentence that
involves four fundamental operations and then
12. Which is NOT true about the given case? asked them to solve the word problem they
a. Ms. Cidro practices a balanced assessment. constructed.
b. Ms. Cidro's assessment method is c. Mrs. Malang who asked her pupils to construct
performance-based. any word problem that involves the four
c. Ms. Cidro needs a rubric in scoring the work of fundamental operations and then asked them to
the students. show how to solve it.
d. Ms. Cidro's assessment targets are all in the d. Mrs. Pontipedra who asked her pupils to
cognitive domain. construct any word problem that involves the four
fundamental operations then formed them by twos
13. If Mr. Paraiso will have to make a scoring so that each pair exchanged problems and help
rubric for the student's output, what format is solve each other's problem.
better to construct considering that the teacher
has limited time to evaluate their work? 18. Which is wrong to assume about traditional
a. Analytic Rubric assessment?
b. Holistic Rubric a. It can assess individuals objectively.
c. Either A or B b. It can assess individuals at the same time.
d. Neither A nor B c. It is easier to administer than performance test.
d. It can assess fairly all the domains of
14. The school principal has 3 teacher applicants intelligence of an individual
all of whom graduated from the same institution
and are licensed teachers. She only needs to hire 19. Which statement about performance-based
one. What should she do to choose the best assessment is FALSE?
teacher from the three? a. It emphasizes merely process.
I. Give them a placement test. b. It also stresses doing, not only knowing.
II. Interview them on why they want to apply in the c. It accentuates on process as well as product.
school. d. Essay tests are an example of performance-
III. Let them demonstrate how to teach a based assessments.
particular lesson.
IV. Study their portfolios to examine the qualities 20. Under which assumption is portfolio
of the students' outputs when they were in assessment based?
College. a. Portfolio assessment is a dynamic assessment.
a. I and II. b. Assessment should stress the reproduction of
b. II and III. knowledge.
c. I and III, IV c. An individual learner is adequately
d. II, III and IV characterized by a test score.
d. An individual learner is inadequately
15. What should be done first when planning for a characterized by a test score.
performance-based assessment?
a. Determine the &table of specifications& of the 21. Which is a good portfolio evidence of a
tasks student's acquired knowledge and writing skills?
b. Set the competency to be assessed. a. Project
c. Set the criteria in scoring the task. b. Test Results
d. Prepare a scoring rubric. c. Reflective Journal
d. Critiqued Outputs
16. To maximize the amount of time spent for
performance-based assessment, which one 22. When planning for portfolio assessment,
should be done? which should you do first?
a. Plan a task that can be used for instruction and a. Set the targets for portfolio assessment.
assessment at the same time. b. Exhibit one's work and be proud of one's
b. Assess one objective for one performance task. collection
c. Set objectives only for cognitive domains. c. Select evidences that could be captured in
d. Limit the task to one meeting only. one's portfolio
d. Reflect on one's collection and identify
17. Who among the teachers below gave the strengths and weaknesses
most authentic assessment task for the objective
&Solve word problems involving the four basic 23. Which kind of rubric is best to use in rating
operations& students' projects done for several days?
a. Mrs. Juliano who presented a word problem a. Analytic
involving a four fundamental operations and then b. Holistic
asked the pupils to solve it. c. Either holistic or analytic
64
d. Both holistic and analytic c. Mean=63, median=63, mode=63
d. The distribution is mesokrutic
24. Which is not true of an analytic rubric?
a. It is time consuming
b. It is easier to construct than the holistic rubric
c. It gives one's level of performance per criterion
d. It allows one to pinpoint the strengths and
weaknesses of one's work.

25. Mrs. Bacani prepared a rubric with 5 levels of


performance described in 5-excellent, 4-very
satisfactory, 3-satisfactory, 2 needs improvement,
1-poor. After using this rubric with these
descriptions, she found out that most of her
students had a rating of 3. Even those who are 5. If quartile deviation is to median, what is to
evidently poor in their performance had a rating of mean?
satisfactory. Cold there be a possible error in the a. Standard deviation
use of the rubric? b. Mode
a. Yes, the teacher could have committed the c. Range
generosity error. d. Variance
b. Yes, the teacher could have committed the
central tendency source of error. 6. In a normal distribution, which of the following
c. No, it is just common to see more of the is true?
students having grade of 3 in a 5-point scale. a. median=mode=mean
d. No, such result is acceptable as long as it has a b. median≠mode=mean
positive consequence to the students. c. median≠mode≠mean
d. Mean=median=mode
Answers: 1C 2A 3A 4C 5C 6A 7C 8C 9D 10A
11B 12D 13B 14D 15B 16A 17D 18D 19A 20D 7. Which of the following situations may lower the
21C 22A 23A 24B 25B validity of test?
a. Mrs. Josea increases the number of items
measuring each specific skill from three to five.
1. In a positively skewed distribution, the following b. Mr. Santosa simplifies the language in the
statement are true except directions for the test.
a. Median is higher than the mode. c. Miss. Lopeza removes the items in the
b. Mean is higher than the Media. achievement test that everyone would be able to
c. Mean is lower than the Mode. answer correctly.
d. Mean is not lower than the Mode. d. None of the above.
2. Which of the following questions indicate a
norm - referred interpretation? 8. In a negatively skewed distribution, which of the
a. How does the pupils test performance in our following statements is true?
school compare with that of other schools?: a. Mode is lower than the mean.
b. How does a pupil's test performance in reading b. Mean is lower than the mode.
and mathematics compare? c. Median is higher than the mode.
c. What type of remedial work will be most helpful d. Mode is lower than the median.
for a slow- learning pupil?
d. Which pupils have achieved master of 9. In a negatively skewed distribution, the
computational skills? following statements are true EXCEPT?
a. Mean is not higher than the median
3. What is the performance of a student in the b. Median is lower than the mode.
National Achievement Test (NAT) if he c. Mean is lower than the mode.
obtained/got a stanine score of 5? d. Mode is less than the median.
a. Between average and above average
b. Between average and below average 10. Miss Cortez administered a test to her class
c. Below average and the result is positively skewed. What kind of
d. Average test do you think Miss Cortez gave to her pupils?
a. Post test
4. Based on the figure, which is true about the b. Pretest
distribution? c. Mastery test
a. Mean=55, median=48, mode=34 d. Criterion-referenced test
b. Mean=46, median=40, mode=37

65
11. The result of the test given by teacher A Set 3 0,2,8,15,20
showed a negatively skewed distribution. What Set 4 505,501,503
kind of test did Teacher A give? a. Set 1
a. The test is difficult b. Set 2
b. It is not too easy nor too difficult c. Set 3
c. It is moderately difficult d. Set 4
d. It is easy
19. Standard deviation is to variability as mode
12. When the distribution is skewed to the right, to?
what kind of test was administered? a. Correlation
a. Difficult b. Discrimination
b. Easy c. Central tendency
c. Average/moderately difficult d. Level of difficulty
d. Partly easy- partly difficult
20. Goring performed better than 65% of the total
13. In a negatively skewed distribution, what kind number of examinees in the district achievement
of students does Teacher B have? test. What is his percentile rank?
a. Very good a. P35
b. Very poor b. P65
c. Average c. P66
d. Heterogeneous d. P75

14. In a positively skewed distribution, the 21. Which is a guidance function of a test?
students are? a. Identifying pupils who need corrective teaching
a. Very good b. Predicting success in future academic and
b. Very poor vocational education
c. Average c. Assigning marks for courses taken
d. Normally distributed d. Grouping pupils for instruction within a class

15. In a positively skewed distribution, which of 22. Mr. Reyes, an elementary school teacher in
the following statements is true? Science found out that many of his pupils got very
a. Mode = 67 while Media = 54 high scores in the test. What measure of central
b. Median = 53 while Mean = 41 tendency should he use to describe their average
c. Mean = 73 while Mode = 49 performance in the subject?
d. Median = 34 while Mode = 42 a. Mean
b. Median
16. Which statements represent criterion- c. Mode
referenced interpretation? d. Range
a. Lucresia did better in solving the linear
equation than 80% of representative Algebra 23. Which of the following indicates how
students. compressed or expanded the distribution of
b. Lucresia's score indicates that she is able to scores is?
solve about two thirds of all one-variable linear a. Measures of position
equations of such complexity. b. Measures of central tendency
c. Students who have reached Lucresia's level on c. Measures of correlation
linear equations usually succeed in the d. Measures of variability
subsequent unit on simultaneous equations with
special help or extra time; i.e., Lucresia is ready to 24. The proportion passing the upper and lower
move ahead. group is .80 and .95, respectively. What is the
d. All of the above index of difficulty?
a. .38
17. Bernard obtained a 97 percentile rank in an b. .40
aptitude test. This means c. .58
a. He answered 97% of the items correctly. d. 1.02
b. He belongs to the 97% of the group who took
the test. 25. Mr. Gringo tried to correlate the scores of his
c. 79% of the examinees did better than her on pupils in the Social studies test with their grades
the test. in the same subject last 3rd quarter. What test
d. He surpassed 97% of those who took the test. validity is he trying to establish?
a. Content validity
18. Which set of scores has the least variability? b. Construct validity
Set 1 0,5,10,15,20 c. Concurrent validity
Set 2 25,35,45,55 d. Criterion related validity
66
8. a. mean > median > mode
Answers: 1C 2A 3D 4C 5A 6D 7D 8B 9D 10B b. mean < mode > median
11D 12A 13A 14B 15C 16B 17D 18D 19C 20B c. mean > mode < median
21B 22B 23D 24C 25C d. mean < median < mode
1. If a test item has a difficulty index of 0.06, how
would you describe the test item?
a. It is very easy.
b. It is moderately difficulty.
c. It is very difficult
d. It is difficult

2. Two sections have the same mean but the standard


deviation of section 2 is higher than section 1. Which
of the two sections is more homogeneous?
a. Section 1
b. Section 2
c. Both A and B 9. a. mode < mean < median
d. None of the above b. mode > mean > median
c. median < mode > mean
3. Miss Corteza administered a test to her class and d. none of the above
the result is positively skewed. What kind of test do
you think Miss Corteza gave to her pupils?
a. Posttest
b. Pretest
c. Mastery test
d. Criterion-referenced test

4. In his second item analysis, Mr. Gonzales found


out that more from the lower group got the test item
15 correctly. What does this mean?
a. The item has become more valid
b. The item has become more reliable
10. a. equal means, unequal standard deviations
c. The item has a positive discriminating power b. equal means, equal standard deviations
d. The item has a negative discriminating power c. unequal means, equal standard deviations
d. unequal means unequal standard deviations
5. Q1 is 25th percentile as media is to what
percentile?
a. 40th percentile
b. 60th percentile
c. 50th percentile
d. 75th percentile

6. Which is implied by a positively skewed scores


distribution?
a. The mean, the median, and the mode are equal.
b. Most of the scores are high
c. Most of the scores are low.
d. The mode is high 11. a unequal means, equal standard deviations
b. unequal means, equal standard deviations
7. In a normal distribution curve, what does a T- c. equal means, equal standard deviations
score of 60 mean? d. equal means, unequal standard deviations
a. Two SDs below the mean
b. Two SDs below the mean
c. One SD below the mean
d. One SD above the mean

For items 8 to 13, what does each


figure/distribution on the right indicate?

67
a. 11.0
b. 10.0
c. 9.0
d. 5.0

17. Given a mean of 55 and a standard deviation


of 8, what two scores include one standard
deviation below and above the mean?
a. 46 and 63
b. 47 and 64
c. 47 and 63
d. 46 and 64
12. a. unequal variability, equal means, different
shapes 18. Given the same mean of 55 and standard
b. unequal means, equal variability, different deviation of 8, what score corresponds to two
shapes standard deviation above the mean?
c. equal variability, equal means, different shapes a. 70
d. unequal variability, unequal means, different b. 71
shapes c. 72
d. 73

19. What principle of test construction is violated


when one places very difficult items at the
beginning; thus creating frustration among
students particularly those of average ability and
below average?
a. All the items of particular type should be placed
together in the test.
b. The items should be phrased so that the
content rather than the form of the statements will
determine the answer.
c. All items should be approximately 50 percent
13. a. unequal means, equal standard deviations difficulty.
b. equal means, unequal standard deviations d. The items of any particular type should be
c. equal means, equal standard deviations arranged in an ascending order of difficulty.
d. unequal means, unequal standard deviations
20. Mrs. Reyes would like to find out how well her
students know each other. What assessment
instrument would best suit her objective?
a. Self-report instrument
b. Sociometric technique
c. Guess-who technique
d. All of the above

21. Mr. Reyes asked his pupils to indicate on the


14. In conducting a parent- teacher conference, piece of paper the names of their classmates
which of the following is NOT true? whom they would like to be with for some group
a. Be friendly and informal activity, what assessment technique did Mr.
b. Be a know-it-all person Reyes use?
c. Be willing to accept suggestions a. Self-report technique
d. Be careful in giving advice b. Guess-who technique
c. Sociometric technique
15. In a frequency distribution, what is the d. Anecdotal technique
midpoint of the class interval whose lower and
upper limits are 99.5 and 109.5? 22. Which of the following assessment
a. 107.0 procedures/tools is useful in assessing social
b. 105.0 relation skills?
c. 104.5 a. Anecdotal record
d. 102.5 b. Attitude scale
c. Peer appraisal
16. In a frequency distribution, what is the interval d. any of the above
size of the class whose lower and upper limits are
9.5 and 19.5? 23. If the proportion passing for the upper and
68
lower group is .90 and .30 respectively, what is 3. Which statements below best describes
the discrimination index? development?
a. .40 a. A high school student's height increased by
b. .50
5'2& to 5'4&
c. .60
d. .70 b. A high school student's change in weight from
110 lbs. to 125 lbs.
24. Which is an example of affective learning c. A student had learned to operate the computer
outcome? d. A student's enlargement of hips
a. Interpret stimuli from various modalities to
provide data needed in making adjustments to the 4. What concept can best describes Francisco's
environment
ability to walk without a support at age of 12
b. Judge problem and issues in terms of situations
involved than in terms of fixed dogmatic thinking months because of the &internal ripening& that
c. Appreciate the quality and worth of the story occured in his muscles, bones and nervous
read system development?
d. None of the above a. Development
b. Growth
25. Mr. Mirasol who is a high school teacher in c. Learning
English conducted an item analysis of her test.
d. Maturation
She found out that four of the items of the test
obtained the following difficulty and discrimination
indices and as follows: 5. Teacher Jesus in now 69 years old has been
observing changes in himself such as the aging
process. Which term refers to the development
Which of the above items should be discard in her change in the individual?
item pool? a. Development
a. Item 1
b. Growth
b. Item 2
c. Item 3 c. Learning
d. Item 4 d. Maturation

Answers: 1C 2A 3B 4D 5C 6C 7D 8D 9D 10A 6. Manuel, a five-year old boy can hold his pen
11A 12C 13D 14B 15C 16B 17C 18B 19D 20C and write his name with his right hand. Which
21C 22C 23C 24B 25D
term describes Manuel's action/ behavior?
a. Development
CHILD AND ADOLESCENCE
b. Growth
1. Dr. Escoto, the school physician conducted a c. Learning
physical examination in Ms. Manuel's class. What d. Maturation
concept best describes the quantitative increase
observed by Dr. Escoto among learners in terms 7. Which of the following theory can help Miss
of height and weight? Samson determine the readiness of her learners
a. Development by administering a readiness test?
b. Growth a. Conditioning Theories
c. Learning b. Cognitive Development Theory
d. Maturation c. Maturation Theory
2. Which situation best illustrates the concept of d. Ethological Theory
growth?
a. A kinder pupil gains 2 pounds within two
months. 8. Mr. Francisco was very much worried about the
b. A high school student gets a score of 85 in thumb sucking of his son. A friend of him says
mental ability test. that certain behavior among infants. Who
c. An education student has gained knowledge on presented that notion that certain behavior like
approaches and strategies in teaching different thumb-sucking is normal behavior?
subjects A. Sigmund Freud
d. An elementary grader has learned to play b. Erick Ericson
piano. c. John Bowlly
d. Urie Bronfrenbenner

69
9. A newborn infant move his whole body at one preschool learners. What stage in the cognitive
time, instead of moving a part of it. Which of the theory of development explains this?
following principles is illustrated by his behavior? a. Sensorimotor
a. Development proceeds from specific to b. Preoperational
general. c. Concrete operation
b. Development proceeds from general to d. Formal operation
specific.
c. Development follows an orderly pattern. 16. Connie develops concepts necessary for
d. Development follows a general pattern. everyday living, builds healthy attitudes towards
oneself, and achieve personal independence.
10. Train up a child in the way he should be; when These are among the attributes of an individual in
he grows up, he will not depart from it. Which what particular stage?
principle supports this? a. Infancy and early childhood
a. Development is determined by his heredity b. Middle childhood
b. Development is determined by the environment c. Adolescence
c. Early development is more critical than the late d. Early adulthood
development
d. Early development is less critical than late 17. Some children are more active than others, as
development. everyone knows-extremely highlevels of activity or
hyperactivity are considered problematic. How
11. Which state of the psycho-sexual theory does may a teacher help a child who is hyperactive?
young boys experience rivalry with their father for a. Make him the leader of the class
their mother's attention and affection? b. Transfer him to another class
a. Oral c. Give him challenging activities that are
b. Anal appropriate to his ability level and interests.
c. Phallic d. Allow him to spend longer at the playground
d. Latency until he gets tired.

12. Angela focuses her attention on the school 18. Tessa gets jealous whenever she sees her
work and vigorous play that consume most of her father showing love and affection to her mother.
physical energy. Which stage of psychosexual Which of the following is she showing according
theory illustrates her behavior? to Freud?
a. Oral a. Complex
b. Anal b. Phallic
c. Phallic c. Electra Complex
d. Latency d. Oedipus Complex

13. Which of the following is likely to be 19. In Piaget's Theory of Cognitive Development,
developed if infants are shown genuine affection? which of the following statements would illustrate
a. Trust Edward who is 11 years old?
b. Autonomy a. Able to see relationships and to reason in the
c. Initiative abstract.
d. Industry b. Unable to breakdown a whole into separate
parts.
14. Christian develops an integral and coherent c. Differentiates goals and goal-directed activities.
sense of self. He seeks answers to the question. d. Experiments with methods to reach goals.
&Who am I&? Which of the following is Christian 20. Trisha goes with her mother in school. She
likely to develop? enjoys the workplace of her mother. Which of the
a. Initiative following ecological theories is illustrated by the
b. Identity and Role Confusion situation?
c. Intimacy a. Microsystem
d. Autonomy b. Mesosystem
c. Exosystem
15. Ms. Reyes uses images and language to d. Macrosystem
represent and understand her various lessons to
70
21. Lito, a student in secondary level tends to b. Development refers to the progressive series of
spend more time with his friends and his family, changes of an orderly coherent type toward the
thus, his behavior is greatly affected by them. In goal of maturity.
which stage in the Psychosocial Stages of c. Development is the gradual and orderly
Development does Lito belong? unfolding of the characteristics of the individuals
a. Autonomous vs Shame and Doubt as they go through the successive stages of
b. Identity vs. Role Confusion growth.
c. Intimacy vs. Isolation d. All of the above
d. Initiative vs. Guilt
2. What do you call the quantitative increase in
22. Anna believes that authority is respected. She terms of height and weight as observed by the
is now in what particular level in moral school physician during the physical examination
development theory of Lawrence Kholberg? of the students?
a. Social contract a. Development
b. Law and order orientation b. Growth
c. Interpersonal concordance c. Learning
d. Universal ethics orientation d. Maturation

23. What level has a four year old learner like 3. Mrs. Gomez conducts research on the
Maryann reached when she acquired new skills psychosocial domain of development. In what
such as putting the same shapes and the same particular area of the child's development is Mrs.
colors together? Gomez most likely to be interested with?
a. Development a. Perceptual abilities
b. Maturation b. Brain-wave patterns
c. Zone of Proximal Development c. Emotions
d. Learning d. Use of language

24. Which of the following principles can be the 4. Which of the following is the correct order of
basis of the growing realization of the significance psychosexual stages proposed by Sigmund
of the early childhood education? Freud?
a. The young children are capable of doing many a. Oral stage, anal stage, phallic stage, latency
things at an early stage. stage, genital stage
b. The child should be seen and should learn. b. Anal stage, oral stage, phallic stage, latency
c. The first five years of life are the formative stage, genital stage
years of the child. c. Oral stage, anal stage, genital stage, latency
d. Early childhood experiences can be interesting stage, phallic stage
and challenging. d. Anal stage, oral stage, genital stage, latency
stage, phallic stage
25. Which of the following learner's characteristics
will affect most of the learners learning in the 5. What is the best description of Erickson's
academic class? psychosocial theory of human development?
a. His affective characteristics a. Eight crises all people are thought to lace
b. His cognitive characteristics b. Four psychosocial stages in latency period
c. His psychomotor characteristics c. The same number of stages as Freud's, but
d. His socio-emotional characteristics with different names
d. A stage theory that is not psychoanalytic
Answers: 1B 2A 3C 4D 5D 6A 7C 8A 9B 10B
11C 12D 13A 14B 15B 16B 17C 18C 19A 20C 6. In Erickson's theory, what is the unresolved
21D 22A 23C 24B 25 crisis of an adult who has difficulty establishing a
secure, mutual relationship with a life partner?
a. Initiative vs. Guilt
1. Which of the following is true about human b. Autonomy vs. Shame and Doubt
development? c. Intimacy vs. Isolation
a. Human development considers both maturation d. Trust vs. Mistrust
and learning.

71
7. Alyssa is eight years old, and although she d. Penis envy complex
understands some logical principles, she still has
troubles in understanding hypothetical concepts. 13. When a little girl who says she wants her
According to Piaget, Alyssa belongs to what mother to go on vacation so that she can marry
particular stage of cognitive development? her father, Freud believes that he is voicing a
a. Sensorimotor fantasy consistent with?
b. Preoperational a. Oedipus complex
c. Concrete operational b. Electra complex
d. Formal operational c. Theory of mind
d. Crisis of initiative vs. Guilt

8. Which of the following provides the best broad 14. Which of the following can best describe the
description of the relationship between heredity preschooler's readiness to learn new task and
and environment in determining height? play activities?
a. Heredity is the primary influence, with a. Emerging competency and self-awareness
environment affecting development only in severe b. Theory of the Mind
situations. c. Relationship with parents
b. Heredity and environment contribute equally to d. Growing identification with others
development
c. Environment is the major influence on physical 15. James noted that when the preschoolers
characteristics. eagerly begin many new activities but are
d. Heredity directs the individual's potential and vulnerable to criticism and feelings of failure, they
environment determines whether and to what are experiencing what particular crisis?
degree the individual reaches the potential. a. Identity vs. Role Confusion
b. Initiative vs. Guilt
9. What is the correct sequence of prenatal c. Basic trust vs. mistrust
stages of development? d. Efficacy vs. Helplessness
a. Embryo, germinal, fetus
b. Germinal, fetus, embryo 16. What stage of Piaget's Cognitive Development
c. Germinal, embryo, fetus does a person belong to when he can understand
d. Embryo, fetus, germinal specific logical ideas and apply them to concrete
problems?
10. When a baby realized that a rubber duck a. Preoperational thought
which has fallen out of the tub must be b. Operational thought
somewhere on the floor, he is likely to achieved c. Create operational thought
what aspect of cognitive development? d. Formal operational thought
a. Object permanence
b. Deferred imitation 17. What is the best explanation of Piaget's
c. Mental combinations concrete operational thought to describe the
d. Goal-directed behavior school-age child's mental ability?
a. A child can reason logically about things and
11. Which of the following will be Freud's events he or she perceives.
description of the child's behavior if he he has b. A child's ability to think about how he thinks
biting, sarcastic manner? c. Can understand that certain characteristics of
a. Anally expulsive an object remain the same when other
b. Anally retentive characteristics are changed
c. Fixated in the oral stage d. Can understand that moral principles may
d. Experiencing the crisis of trust vs. mistrust supersede the standards of society.

12. What is Freud's idea about a young boy's guilt 18. Elisa who is in between 9 and 11 years of age
feelings brought about by jealousy of his father's are most likely to demonstrate moral reasoning at
relationship with his mother? which Kohlberg's stage?
a. Electra complex a. Pre-conventional
b. Oedipus complex b. Conventional
c. Phallic complex c. Post-conventional
72
d. None of the above 24. According to Erikson, what is the primary task
of adolescent?
19. According to Kohlberg, a dutiful citizen who a. To establish trust
obeys the laws set down by society is at which b. To search for his identity
level of moral reasoning? c. To be more intimate with others
a. Pre-conventional Stage One d. To establish integrity
b. Pre-conventional Stage Two
c. Conventional 25. What is the main source of emotional support
d. Post-conventional for most young people who are establishing
independence from their parents?
20. Ana, who is low-achieving, shy and a. Older adolescents of the opposite sex
withdrawn, is rejected by most of her peers. her b. Older sibling
teacher wants to help Ana increase her self- c. Teachers
esteem and social acceptance. What can Joy's d. Peer groups
teacher suggest to her parents?
a. Transfer her to a different school Answers: 1D 2B 3C 4A 5A 6C 7C 8A 9C 10A
b. Help their daughter improve her motor skills 11C 12B 13B 14A 15B 16C 17C 18B 19C 20D
c. Help their daughter learn to accept more 21C 22D 23D 24B 25D
responsibility for her academic failures
d. Help their daughter improve her skills in relating
to peers CURRICULUM DEV’T

1. Which is NOT a provision for the development


21. What is the most accurate definition of puberty of each learner in a good curriculum?
stage? a. Extensive arrangements are made for the
a. Rapid physical growth that occurs during educational diagnosis of individual learners.
adolescence b. Self-directed, independent study is encouraged
b. Stage when sexual maturation is attained. wherever possible and advisable.
c. Rapid physical growth and sexual maturation c. Self-motivation and self-evaluation are
that ends childhood stimulated and emphasized throughout the
d. Stage when adolescents establish identifies learning opportunities of the school.
separate from their parents. d. The program provides a wide range of
opportunities for individuals with same abilities,
22. Fifteen year old Marie is preoccupied with her needs and interests.
&disgusting appearance& and seems depressed
most of the time. What is the best thing her 2. Teacher Lily would like to take part in
parents can do to help her get through this difficult developing a subject-centered curriculum
time? because she believes that all subjects in this type
a. Ignore her self-preoccupation because their of curriculum are geared towards the hollistic
attention would only reinforce it. development of the learner. Is her belief about the
b. Encourage to &shape up& and not give in the subject-centered curriculum true?
self-pity a. Yes, because the subject-centered curriculum
c. Kid her about her appearance in the hope that focuses on the learners needs, interests and
she will see how silly she is acting. abilities.
d. Offer practical advice, such as clothing b. No, because it is the experience-centered
suggestions, to improve her body image. curriculum that emphasizes the teaching of facts
and knowledge for future use.
23. What can be the best comparison of the c. Yes, because the subject-centered curriculum
behavior of a 17 year old girl to that of her 13 year involves cooperative control.
old brother? d. No, because it is the experience centered and
a. She is more likely critical about herself not the subject-centered curriculum that
b. She tends to be more egocentric. emphasizes integration of habits and skills in
c. She had less confidence in her abilities. learning the knowledge component of subject
d. She is more capable of reasoning areas.
hypothetically.

73
3. In the elementary level, English literature and 9. Ms. Mateo, a History teacher considers the
Social studies relate well. While history is being element of time in arranging content of her
studied, different literary pieces during the lessons in World History. What way of
historical period is being studied as well. What establishing sequence is given emphasis by Ms.
curriculum design is shown here? Mateo?
a. Separate subject design a. Simple to complex
b. Correlation design b. Part to whole
c. Discipline design c. Concrete to abstract
d. Broad field design d. Chronological

4. This phase of curriculum development involves 10. Mr. Rivera, a new teacher believes that
decisions, among other things, on grade education is a process of development and is life
placement and sequencing of content. Which itself; therefore, experience related to the child's
phase is this? need and interest should be given primary
a. Curriculum planning consideration. What educational philosophy is
b. Curriculum evaluation being exhibited by Mr. Rivera?
c. Curriculum organization a. Idealism
d. Curriculum implementation b. Reconstructionism
c. Progressivism
5. One example of this design of subject-centered d. Realism
curriculum is that which shows social studies
being combined with geography, civics, culture 11. A stakeholder in curriculum development, Mr.
and history to comprises subject area. Which Cruz, a district supervisor and a member of the
design is this? school board has one of the following primary
a. Correlated roles.
b. Broadfields a. Support and participate in parent-school
c. Separate Subject organization activities.
d. Core b. Authorize school expenditures for curriculum
development, implementation and evaluation
6. Ms. Ortiz, as Science teacher tries to enrich the c. Enact legislation to effect curriculum
content of her lesson by identifying related improvement.
concepts in Math. What pattern of organizing d. Recommend changes in curriculum.
subjects did Ms. Ortiz consider?
a. Broadfield 12. The schools in the first District plan to adopt
b. Correlated the reading program used in the third district.
c. Core What level of curriculum improvement is used?
d. Separate Subject a. Variation
b. Value orientation
7. Which design is easy to deliver because c. Substitution
complementary books and materials are d. Restructuring
commercially available?
a. Experience centered design 13. Mr. Bernardo, a curriculum consultant on
b. Problem design Economics insists that in selecting the curriculum
c. Process design content, it is better that throughout the high school
d. Subject centered design years, economic geography concepts be used to
recur and be repeated with depth for effective
learning. What criterion in content selection is
8. What refers to the matching between shown here?
curriculum and test to be used to assess the a. Validity
learners? b. Continuity
a. Alignment c. Significance
b. Auditing d. Learnability
c. Articulation
d. Delivery
14. The Filipino learners envisioned by the
74
Department of Education (DepEd) in the light of connect the lessons he learned in a subject area
K-12 Curriculum is to a related content in another subject area?
a. Technologically literate or logistically developed a. Articulation
Filipino b. Balance
b. Functionally literate or logistically developed c. Continuity
Filipino d. Integration
c. Scientifically Advanced and Values Oriented
Filipino 20. The following curricular changes took place in
d. National Oriented and Internationally what particular period? Restore Grade VII,
Competitive Filipinos double-single session was abolished and more
textbooks were written by Filipino authors.
15. Teacher Dominguito believes that a new a. American Period
respect for the child is fundamental in curriculum. b. Philippine Republic
Thus, all activities in the classroom are geared c. Japanese Occupation
towards the development of the child - the center d. New Society
of the educative process. To which approach in
curriculum does Teacher Dominguito adhere? 21. This concept includes the sub-processes of
a. Learner-centered curriculum planning, organization, implementation
b. Subject-centered and evaluation. Which concept is this?
c. Problem-centered a. Curriculum development
d. Pragmatic b. Curriculum assessment
c. Curriculum management
16. Mrs. Manuel, the Principal of Bagong Barrio d. Curriculum and instruction
Elementary School invited the Brgy. Captain in
the school to solicit inputs for a new curriculum in 22. If curriculum is the &means&, what is the
Social Science which highlights indigenous &end&?
knowledge in the community. What is shown in a. Strategies
this situation? b. Instruction
a. Community members as supporters of c. Technique
curriculum d. Approaches
b. Community members as curriculum resources
c. Community members as managers of 23. The curriculum used during the period in
curriculum Philippine history terminated the use of English as
d. Community members as beneficiaries of a medium of instruction, What period is this?
curriculum a. American
b. Spanish
17. Teacher Bert puts emphasis on the immediate c. Commonwealth
felt interests and needs of his students and not on d. Japanese
the anticipated needs and interests. What type of
curriculum does teacher Bert adheres? 24. Which of the following statements about the
a. Subject-centered concept of curriculum is NOT quite acceptable?
b. Learner-centered a. It refers to all experiences that both the school
c. Experience-centered and the teacher provide the students with.
d. Culture-based b. It is the set of acquired knowledge, habits and
skills
18. What type of curriculum divides the school c. It consists of everything that goes within the
day into different periods such as language arts, school.
social studies, science and health, arithmetic, d. It is a planned action for instruction
etc.?
a. Correlated 25. What process is being undertaken by
b. Broad fields curriculum developers when they enrich or modify
c. Integrated certain aspects of a particular program without
d. Separate Subject changing its fundamental conceptions?
19. Which curriculum design element is taking a. Curriculum improvement
place when Eduardo, a 4th year student can b. Curriculum change
75
c. Curriculum design response to society's demand for integration of
d. Curriculum implementation knowledge and enables the learner to see
relationship among various aspects?
Answers: 1D 2D 3C 4C 5B 6B 7D 8A 9D 10C a. Broadfield
11D 12C 13B 14B 15A 16B 17C 18D 19D 20B b. Correlated
21A 22B 23D 24C 25A c. Core
d. Separate subjects

1. What design element establishes the vertical


linkage from level to level to avoid glaring gaps 8. Who controls the subject centered-curriculum?
and wasteful overlaps? a. Learner
a. Articulation b. Teacher
b. Balance c. Parent
c. Scope d. Teacher and parent
d. Sequence
9. To provide individual differences in the
2. What refers to the authenticity of the content classroom, how is curriculum designed?
selected by the curriculum developer? a. Minimum learning competencies are included
a. Feasibility b. Realistic and meaningful experiences are
b. Learnability provided
c. Significance c. Some degree of flexibility is provided
d. Validity d. Social skills are emphasized

3. What do we call the allocation of content to a 10. To ensure success in curriculum


definite grade capable of learning? development, which of the following specific
a. Time allotment actions should a curriculum leader avoid?
b. Grade placement a. Work with people over them.
c. Grade level b. Use your status frequently to establish
d. Maturity level discipline
c. Keep channels of communication open
4. Which pattern of experience-centered d. Show that you too desire to improve
curriculum centers around the normal activities of
children and is based on each child's needs, 11. Which of the following is a reason for the
interests and potentials? continuous appraisal of the existing curriculum in
a. Child-centered all levels?
b. Activity a. New national policies in government
c. Social function b. Changing needs and condition of society
d. Specific competencies c. Economic status of the people
d. Political trust of the country
5. Which curriculum development phase focuses
on the change which will take place in certain 12. Which of the following best defines curriculum
aspects of the curriculum without changing the development?
fundamental conceptions? a. The total mental phenomena directly received
a. Curriculum planning at any given time
b. Curriculum design b. The planning of learning opportunities intended
c. Curriculum improvement to bring about certain desired changes in pupils
d. Curriculum evaluation and the assessment of the extent to which these
6. Which is not a component of curriculum changes have taken place.
designing? c. A continuous cycle of activities in which all
a. Objective elements of curriculum are considered.
b. learning content d. Education is aiding each child to be socially
c. learning experiences creative individuals.
d. Diagnosis of needs
13. To build a sense of pride among Filipino
7. Which type of curriculum design serves as a youth, which should be done in the curriculum?
76
a. Re-study our history and stress on our c. Promoters
achievements as a people. d. Incorporators
b. Re-study our history from the perspective of our
colonizer. 20. What refers to the appropriateness of the
c. Replace the study of folklore and myths with content in the light of the particular students who
technical subjects. are to experience the curriculum?
d. Set aside the study of local history. a. Significance
b. Validity
14. What do you call the curriculum when the c. Interest
teacher puts into action all the different planned d. Learnability
activities in the classroom?
a. Recommended Curriculum 21. Which of the following statements is NOT
b. Written Curriculum acceptable?
c. Taught Curriculum a. Instruction is the actual engagement of learners
d. Supported Curriculum in the planned learning activities.
b. Curriculum determines what assessment
15. Which statement about the subject-centered should be done, and how to do it.
curriculum is NOT true? c. Instruction requires teachers to use a variety of
a. There is a high level of cooperative interaction action to accomplish a variety of functions.
b. It covers much content in a short period of time d. Assessment establishes what is to be
c. The teacher has full control of the classroom accomplished in teaching and learning.
activities
d. The main task is mastery of learning 22. Which characteristic of a good curriculum
highlights the psychological nature of the learner?
16. Schools divide the school hours to different a. Provisions are made for the smooth transition
subjects such as reading, grammar, literature, and continuing achievement of pupils.
math, science, history and geography. What b. Curriculum plans in areas which extend over
curriculum design is referred here? several years are developed vertically.
a. Problem-centered c. Classroom practices give attention to the
b. Learner-centered maturity and learning problems of each pupil.
c. Subject-centered d. Cooperative planning and teaching provide for
d. Culture-based exchange of information about pupil's learning
experiences.
17. Which is NOT a description of the learner-
centered curriculum? 23. Objectives must be evaluated in the light of
a. Emphasis is on the total growth and practical considerations, including teacher
development of the learners competence, availability of instructional materials,
b. Controlled and cooperatively directed by time allotment, etc. What characteristic of
learners, teachers and parents educational objective is defined by the
c. Education is a means to develop social creative aforementioned statement?
individual a. Comprehension
d. Emphasis upon facts and knowledge for future b. Attainability
use c. Consistency
d. Feasibility
18. The K-12 curriculum is otherwise called as
a. 2002 Basic Education Curriculum 24. &Knowledge is true if it is workable&. What
b. Revitalized Basic Education Curriculum philosophical foundation supports this statement?
c. Enhanced Basic Education Curriculum a. Idealism
d. Extended Basic Education Curriculum b. Realism
c. Pragmatism
19. What refers to an individual or group of d. Essentialism
individuals who have a direct and indirect
influence in curriculum development? 25. As a member of the curriculum committee,
a. Stockholders your chief concern is to give the child freedom to
b. Stakeholders choose what to learn and believe, as you allow
77
them to set their own identities and standards. 5. Which group of technologies has the highest
What philosophy will you consider? degree of concreteness?
a. Existentialism a. Realia and computer
b. Realism b. Video, picture and television
c. Idealism c. Digital video, film, versatile compact disc
d. Pragmatism d. Book, imaginative literature, programmed
instruction
Answers: 1A 2D 3B 4A 5C 6D 7A 8B 9C 10B
11B 12B 13A 14C 15A 16C 17D 18C 19B 20D 6. Mrs. Del Prado placed text together with the
21D 22C 23D 24C 25A relevant graphics on the same page in her
multimedia presentation. Which principle did she
apply?
EdTech a. Split attention
1. Which of the following statements has a very b. Spatial contiguity
limited definition of educational technology? c. Cost effectiveness
a. It is a profession composed of various job d. Communication effectiveness
categories.
b. It refers to the computers used for teaching and 7. Mrs. Olivarez presented real samples of rocks
learning. in her General Science class. What principle did
c. It includes audiovisual materials, interactive she apply?
multimedia and self-instructional materials. a. Appropriateness
d. It is the development, application and b. Authenticity
evaluation of system, techniques and aids to c. Responsiveness
improve human learning d. Simplicity

2. Which of the following statements is correct


about the domains of educational technology? 8. Which is the best reason why teachers state
a. Design is the production stage while the objectives before using instructional media?
development is the planning stage. a. To secure materials
b. Both the design and development are the b. To prepare the materials beforehand.
planning stage. c. To determine which media to use best.
c. Evaluation is synonymous with implementation. d. To be able to practice how to operate the
d. Utilization is the action phase. equipment

3. Ms. Gomez is planning to integrate technology 9. Which of the following should Mr. Rivera
in her Mathematics class. Which of the following primarily consider in determining the teaching-
would be the logical steps in doing this? learning objectives and use of instructional
I. Set the objectives media?
II. Analyze the learners a. The assessment tool to be used
III. Utilize the materials with showmanship b. The learning activities
IV. Evaluate the performance of the students c. The learner
a. I, II, III, IV d. The teacher
b. II, I, III, IV
c. I, II, IV, III 10. Which of the following technologies provide
d. II, I, IV, III iconic experiences to students/ children?
a. Video and books
4. Which of the following is a limitation of models b. Pictures and videos
and real objects in teaching and learning? c. Radio and recording
a. They pose problems on storage d. Modules and periodicals
b. They make learning more concrete. 11. Which of these technologies used in the
c. They provide hands-on learning experiences. classroom are arranged from the most symbolic to
d. They are readily available in the environment, multisensory?
around school and in the home. a. Real objects, print, audio-visual materials and
visual materials
b. Visual materials, audio visual materials, print
78
and computers Which is true about her?
c. Visual materials, print, audio-visual materials a. She models digital-age work and learning
and realia b. She facilitates and inspires student learning
d. Print, audio-visual materials, computers and and creativity.
realia c. She promotes and models digital citizenship
and responsibility.
12. Which of the following is inappropriate in using d. She designs and develops digital-age learning
printed visuals such as charts, graphs and experiences and assessments
drawings?
a. Provide written or verbal cues to highlight 17. With the fast-paced evolution of technologies
important aspect of visuals nowadays, why are teachers encouraged to shift
b. Present the instructional materials gradually from a teacher-centered instruction to a
simultaneously learner-centered instruction?
c. Use materials that everyone can see I. A learner-centered instruction focuses on
d. Make the presentation suspenseful transformation of facts.
a. II and IV only
13. Susan wants to learn more English. b. I, II and IV only
Specifically, she wants to improve her listening c. I, III and IV only
skills. She has a CD player, a tape recorder and d. II, III and IV only
has internet access. As an English teacher, what
do you suggest? 18. Ms. Hernandez employs student-centered
I. CDs with English listening drills instruction as the learners create their digital
II. Tapes with English listening drills portfolios in her computer class. What could be
III. Internet website such as Go4English, English developed among them through this approach?
Language Listening Lab or Randall's listening Lab a. Repetition and active learning
a. I and II b. Mastery of skills and information delivery
b. II and III c. Information processing and passive learning
c. I or III d. Construction of knowledge and information
d. I, II and III exchange

14. Which of the following statements is incorrect 19. Mr. Torres will have a multimedia presentation
about the contributions of technology to student in his Science class. Which of the following should
learning? he avoid?
a. The quality of learning can be improved. a. Consider technical quality.
b. The delivery of instruction can be more b. Apply different computer effects per slid.
interesting. c. Present information through graphic organizers
c. The method of teaching and learning becomes d. Use contrasting colors for text and background.
more interactive.
d. The role of the teacher can be changed into 20. Mrs. Sison would like to integrate technology
knowledge dispenser. in writing a friendly letter. Which of the following is
the most effective way of doing it?
15. Mr. Tarnate, an ICT teacher takes into a. Let the pupils surf a friendly letter from the
account technology standards to address the internet
needs of the students and help them adapt with b. Have the pupils write a friendly letter and send
the changing society and technology Which of the it through an email.
following standards is an exception? c. have the pupils forward a downloaded friendly
a. Creativity and innovation letter to others via email.
b. Research and information literacy d. Let the pupils write a friendly letter using word
c. Model digital-age work and learning processing and have it critiqued by their peers.
d. Technology operations and concepts
21. Which of the following computer-based
16. Ms. Vinluan, a computer teacher instructional materials can be used to learn new
demonstrates understanding of local and global concepts?
issues and exhibits ethical and legal use of a. Games
information and communications technology tools. b. Tutorial
79
c. Simulation d. Development, utilization, evaluation, design
d. Drill and practice
2. Which is the most important reason why teachers
22. Professor dela Cruz would like to create a need to preview their instructional materials that will be
presentation material for her lesson on the types used in the class?
of computer-assisted instruction. To make her a. To gain confidence in using them
presentation effective, which? b. To encourage viewers to be more focused
a. Situating tool c. To avoid potential problems that might occur while
b. Informative tool materials are in use
c. Productivity tool d. To ensure appropriateness of the materials with the
d. Communicative tool objectives and target audience

23. Professor Reyes is thinking of an online 3. After Ms. Rivas planned her lesson in English, she
learning approach by which content provides links found out that the materials at hand do not match her
to information at other locations and serves as a objectives. Which is the best thing that she can do?
focal point for a distance education experience. a. Modify the available materials
Which of the following should she use? b. Teach the lesson the following day.
a. Teleconferencing c. Change the objectives to match with available
b. Self-paced program materials.
c. Web-based instruction d. Carry out the lesson as planned and use the materials
d. Computer-aided instruction at hand.

24. Which is NOT a basic consideration in 4. Prof. Balagtas used worksheets, manipulatives, and
selecting and evaluating the content of an models in teaching math to help her students
educational technology tool? understand the lessons and love the subject. What did
a. Does it match the content? she bear in mind when she used these materials?
b. Can it be easily dismantled?
a. Appropriateness
c. Will it motivate and maintain interest?
b. Balance
d. Is there evidence of its effectiveness?
c. Breadth
d. Variety
25. Your father wanted to finish his long dreamed
course but he wanted to do it at home during his
5. With increasing use of educational technology inside
free time. Would you recommend an online
the classroom, what role is expected of the teacher?
learning?
a. Facilitator
a. Yes, because online learning is the &in& thing
b. Researcher
b. No, because online learning inhibits student-
c. Knowledge giver
teacher interaction.
d. Source of information
c. No, because hiring a helper would enable him
to attend regularly in his class.
6. It is impractical to bring real objects to the classroom
d. Yes, because he could learn at his own pace
so Aaron constructed a three-dimensional visual instead.
using a wide spectrum of technologies.
Which of the following did he construct?
a. Chart
Answers: 1B 2D 3B 4A 5A 6A 7B 8C 9B 10B
b. Cartoon
11D 12B 13D 14D 15C 16C 17C 18D 19B 20D
21B 22C 23C 24B 25D c. Model
d. Graphic organizer

1. Ms. Delos Santos is a fresh graduate teacher who was 7. You asked your students to show a two-dimensional
hired to teach in an elementary school where there are illustration of what they have understood from what
enough resources for instruction. She wanted to start they have read. Which of the following non-projected
preparing her instructional materials. Which is the most visuals are you referring to?
systematic process in doing this? a. Graphic organizer
a. Design, utilization, evaluation, development b. Print materials
b. Design, development, utilization, evaluation c. Model
c. Development, design, utilization, evaluation d. Realia

80
8. There are several reasons why teachers are learners
reluctant in using electronic media in the teaching-
learning process. Which is the most common 13. Your principal purchased new computer units
reason? for your Learning Resource Center. Which of the
a. The limited exposure of teachers to new following should be your last consideration in
equipment using the technology?
b. Their incompatibility to diverse needs of the a. Computers can be used for entertainment.
learners b. Computers can be used for research activity.
c. The difficulty in integrating technology in the c. Computers can be used for interactive
curriculum presentation.
d. The excessive availability of local technology in d. Computers can be used to reinforce discussion
the community of difficult concepts in class.

9. Prof. Mandanas would like to use an audio 14. Computers can be classified according to the
compact disc in teaching a lesson in Filipino. In roles they play namely communicative tool,
which activity in the teaching-learning process is it informative tool, and constructive tool. What is the
very effective? other role of computers not mentioned in this
a. In developing listening skills item?
b. In teaching creating thinking a. Instructional tool
c. In composing poems b. Situating tool
d. In building concepts c. Utility tool
d. Application tool
10. Plants, pebbles, and blocks are just some of
the effective instructional materials readily found 15. Which of the following categories of CAI will
in the environment if they are utilized properly. you use in your class if your objective is to
Which of the following is INCORRECT about their increase proficiency in a newly learned skill or
classroom use? refresh an existing one?
a. Pass a single object around the class. a. Tutorial
b. Familiarize yourself with the object or model b. Simulation
before its actual utilization. c. Drill and practice
c. Use the objects as springboard in encouraging d. Instructional game
students' active participation.
d. Make sure that the realia and model are large 16. Which of the following is an ineffective use of
enough to be seen by the whole class. Presentation software?
a. Darken the room.
11. Prof. Ruiz uses projected visuals such as b. Use appropriate pacing.
Digital Liquid Projector (DLP) in presenting her c. Read directly from the slides.
lesson. What could be her main reason for using d. Allow interaction with the learner.
it?
a. The projected materials are readily available. 17. Which of the following is NOT an example of a
b. They are more abstract than any other visuals. communicative tool?
c. Most projected visuals can be obtained at no a. Chat
cost. b. Electronic mail
d. She can easily prepare for own transparencies c. Teleconferencing
in advance. d. Multimedia encyclopedia

12. Ms. Samonte used a film clip in teaching 18. Why is one-way delivery of information a
Social Studies concepts to her First Year High misuse of communication tools?
School class. However, she found out that it was a. Because the teacher expects the student to
ineffectively used in the classroom. When is study more
technology considered INEFFECTIVE? b. Because it requires activities that focus on
a. When it promotes mastery of the lesson thinking than responding
b. When it makes viewing more interesting c. Because it enables the users to focus more on
c. When it helps attain the objectives of the lesson higher level cognitive activities
d. When it induces alienation on the part of the d. Because this kind of practice lessens
81
interaction capabilities of communication tools traditional instruction?
a. When the method, technologist and
19. Internet consists of thousands of connected assessment used are appropriate to the required
computer networks around the world. Which term competencies.
does not refer to internet? b. When the course requires more face-to-face
a. NET communication between the students and
b. On-line teachers.
c. Cyberspace c. When students depend more on their online
d. Information Superhighway mentor.
d. When there is student to student interaction.
20. Which technology tool can Prof. Soriano use
to communicate asynchronously with her Answers: 1B 2D 3A 4D 5A 6C 7A 8A 9A 10A
students? 11A 12D 13A 14B 15C 16C 17D 18D 19B 20D
a. Chat and blog 21B 22A 23C 24B 25A
b. Chat and instant messaging
c. Blog and video conferencing 1. Sinabi ni Carlos P. Romulo sa isa niyang akda,
“Ang Pilipino ay may dugong maharlika.” Ano ang
d. Electronic bulletin board and email
kahulugan nito?
a. Ang Pilipino ay nanggaling sa malayang lahi
21. In your computer subject, you allow your class b. Ang Pilipino ay sadyang mabuti ang budhi.
to chat as part of your motivation before c. Ang Pilipino ay galing sa mayamang lahi
discussing to them the roles of computer as a d. Ang Pilipino ay madaling maipagbili
tool. How is chat used in t his context? 2. Alin sa mga sumusunod ang pinakatamang
pangungusap?
a. Information tool
a. Ang mga kabataan sa lansangan ay naglalaro
b. Application tool at nagtatakbuhan sa lansangan.
c. Communicative tool b. Ang kabataan ay naglalaro kung maliwanag
d. Situating tool ang buwan at nagtatakbuhan sa lansangan.
c. Ang mga kabataan ay naglalaro at
22. Which statement is INCORRECT about nagtatakbuhan sa lansangan kung maliwanag
computer conferencing? ang gabi.
d. Ang mga kabataan kung maliwanag ang buwan
a. It refers to live student interaction with an
ay nagtatakbuhan sa lansangan at naglalaro.
expert. 3. Piliin ang gawi ng pagsasalita: Kasiyahan ko
b. It is also known as discussion forum or bulletin nang makitang kayo’y nagmamahalan.
board. a. Pangarap
c. It also refers to online class discussions, forums b. Pagkontrol ng kilos
or debates. c. Pagkuha ng impormasyon
d. It permits two or more individuals d. Pagbabahagi ng damdamin
4. Ang kaugnayan ng pagkakapatay kina Burgos,
Gomez at Zamora sa panitikang Pilipino ay
23. Which instructional application will you a. Nanatiling masigla ang diwang Pilipino
introduce to your class if your objective is to help b. Nakagising sa damdaming makabayan ng mga
them find and use information resources available Pilipino
in the internet? c. Natutong lumabag sa batas at lumaban sa may
a. Webquests kapangyarihan ang mga Pilipino
b. Hybrid course d. Naimpluwensyahan ang diwang alipin ng mga
Pilipino
c. Scavenger Hunt 5. Laging UMUUKILKIL sa isipan ng ama ang
d. Distance education nasirang pangako ng anak.
24. In the delivery of distance education, what a. Sumasagi
computer application is used to organize b. Gumugulo
instructions and track students records and c. Bumubuhay
progress? d. Sumasapi
6. Ang Kagawaran ng Ugnayang Panlabas ng
a. Computer-based Multimedia
Pilipinas ay gulong-gulo kapag may giyera sa
b. Computer-assisted Instruction ibang bansa. Ano ang unang ginagampanan ng
c. Computer-mediated Education ambassador ng bansa?
d. Computer-managed Instruction a. Bilangin ang mga nasugatan at nasawi
b. Alamin ang mga tirahan ng mga Pilipino sa
25. When is distance education as effective as the bayang iyon.

82
c. Ipunin ang mga maykaya at ipalipad pauwi. 15. Ang butong tinangay ng aso, walang
d. Bayaang magsipag-uwian sa sariling sikap ang pagsalang nalawayan ito. Ang kaisipang ito ay
bawat isa tumutukoy sa katotohanan ng ______.
7. Ano ang pokus ng pandiwa sa pahayag na a. Pagnanakaw
“Bumili ng bagong sasakyan si Angelo”? b. Pagtatanan
a. Pokus sa direksyon c. Pagpapakasal
b. Pokus sa kagamitan d. Pakikipagkaibigan
c. Pokus sa sanhi 16. Anong uri ng pagbigkas ang salitang
d. Pokus sa aktor “dambuhala”?
a. Malumi
8. Kabaliwan at paglulustay ang inyong ginagawa b. Mabilis
taon-taon. Higit na marami ang maralitang c. Maragsa
nangangailangan ng salapi at dunong. Ang d. Malumay
nagsasalita ay 17. Ang katawagan sa pangngalan, pang-abay,
pang-uri at pandiwa ay?
a. Kuripot a. Palabuuan
b. Matipid b. Pangkayarian
c. Maramot c. Pangnilalaman
d. Praktikal d. Palaugnayan
9. Nasa anong kaganapan ng pandiwa ang 18. Ang panukalang inihain niya ay lubhang
pangungusap? malalim at mahirap arukin.
Naglaro ng basketball sa Rizal Stadium ang a. Abutin
koponan ng aming pamantasan. b. Unawain
a. Sanhi c. Sukatin
b. Tagaganap d. Tanggalin
c. Kagamitan 19. Ang wikang Filipino ay hawig sa mga wika sa
d. Ganapan Asya. Alin dito ang pinagmulan ng wikang
10, Sa aling salita magkakaroon ng saglit na Filipino?
paghinto kung pinagpipilitang si Rose ang a. Bahasa
nakabasag ng pinggan? c. Nihonggo
Hindi si Rose ang nakabasag ng pinggan. d. Mandarin
d. Malayo-Polinesyo
a. Rose 20. Ano ang katumbas ng “Dekalogo” ni
b. Hindi Apolinario Mabini na nagsasaad ng aral sa
c. Nakabasag Filipino?
d. Pinggan a. Mosaic Law
11. Anong tayutay ang tinutukoy sa pahayag. d. Code of Ethics ni Kalantiaw
Durog ang katawang bumagsak sa semento si c. New Society ni Pres. Marcos
Miguel. d. Code of Citizenship ni Pres. Quezon
21. Siya ay hinirang na taga-sensus ng bahay-
a. Pagtutulad
bahay. Ano ang kanyang nalikom?
b. Pagbibigay katauhan
a. Ang bilang ng tao sa bahay
c. Pagmamalabis
b. Ang kayamanan ng may-bahay
d. Pagwawangis
c. Ang datos tungkol sa mga bata sa bawat bahay
12. Sino ang pinagkalooban ng karangalan bilang
d. Ang datos tungkol sa mga naninirahan sa
“Unang Tunay na Makata” noong 1708?
bawat bahay
a. Jose dela Cruz
22. Kami ang kabataang siyang magiging pag-asa
b. Felipe de Jesus
ng bayan. Paano ginamit ang salitang may
c. Francisco Balagtas
salungguhit?
d. Jose Corazon de Jesus
a. Pagtukoy
13. “Magtatrabaho ako at ikaw ay mag-aaral
b. Pagpuri
upang makatapos ka ng pag-aaral.” Anong uri ng
c. Panghalip
pangungusap ito?
d. Pagmamalaki
a. Payak
23. Walang tubig kahapon. Ito ay pangungusap
b. Tambalan
na:
c. Hugnayan
a. May paksa
d. Langkapan
b. Walang pandiwa
14. Mag-aalas-singko na _____ umaga _____
c. May panaguri
magising siya.
d. Walang paksa
a. ng – ng
24. Ipinagmamalaki mo siya, BAHAG naman pala
b. nang – nang
ang kanyang BUNTOT. Ang ibig sabihin ng
c. ng – nang
salitang may malaking titik ay:
d. nang – kapag
a. Kuripot
83
b. Traydor C. Nagtago
c. Duwag D. Nagmaktol
d. Mahiyain
25. Sabihin ang gawi ng pananalitang ito: “Bawal 3. Ang ________________ ng mga Nars ay dininig
tumawid, may namatay na dito!” ng komite kahapon.
a. Pananakot A. Pakiusapan
b. Pagtukoy B. Pakikipag-usap
c. Babala C. Ipakiusap
d. Paalala D. Pakiusap
26. Pinakamahalagang nobelang Pilipino sa
maraming taon na nalimbag noong 1906 at
4. Lumapit si Prink kay Eadji at sinabing, “
tumalakay nang masinsinan sa paksang puhunan
__________ mo si Landon ng pagkain sa kusina.”
laban paggawa at sa sosyalismo ang _____.
A. Utusan
a. Luha ng Buwaya
b. Banaag at Sikat B. Kunin
c. Ibong Mandaragit C. Kunan
d. Pangginggera D. Hanapan
27. Ayon kay Balagtas, “ang laki sa layaw,
karaniwa’y hubad” kaya ang mga bata ay 5. Nakatulog si Jana sa kanilang opisina dahil sa
a. jeproks HIMINGTING ng kapaligiran. Ano ang kahulugan
b. nag-aartista ng salita na nasa malalaking letra?
c. nakapagtatapos sa pag-aaral A. Kaingayan
d. hindi sumusunod sa magulang B. Kapayapaan
28. Noong taong 1962, ano ang pagbabago sa C. Lakas ng hangin
paglimbag ng diploma at sertipiko ng pagtatapos? D. Katahimikan
a. Pinahihintutan ang pribadong paaralan na
maglimbag sa wikang Ingles 6. Ang sentro ng pagdiriwang ng SENTENARYO ay
b. Nilimbag sa Tagalog ang diploma sa di- sa Kawit, Cavite. Ang kasing-kahulugan ng salitang
Tagalog na bayan nasa malalaking letra ay:
c. Nilimbag sa Filipino ang diploma ngunit may A. Ika-50 taon
Ingles B. Panghabang panahon
d. Nalimbag sa Filipino ang diploma C. Ikasandaang taon
29. Ang gintong panahon ng mga manunulat D. Ika- 25 taon
noong panahon ng Amerikano ay batid sa uring
a. Sanaysay
7. “Ang sinuman ay makabubuo ng matibay na lubid
b. Nobela
kung pagsasama-samahin ang sinulid.” Ang ibig
c. Panulaan
sabihin nito ay?
d. Maikling kwento
30. Alin sa mga sumusunod ang may wastong A. Ang lakas ng tao ay nasa pagkakaisa nila.
gamit ng tinig ng pandiwa? B. Kailangan magkaisa tayong lahat.
a. Ang hinog na papaya na kinuha sa puno ni C. Magkakaiba ang mga tao kaya hirap magkaisa.
Marie. D. Nagkakaisa ang mga tao.
b. Kinuha ni Marie ang hinog na papaya sa puno.
c. Kinuha sa puno ang hinog na papaya ni Marie. 8. MALIIT ANG SISIDLAN ni Dian kaya iniiwasan
d. Papayang hinog ang kinuha sa puno ni Marie. siyang tuksuhin ng mga kasama. Ano ang ibig
sabihin nito?
Answers: 1A 2C 3D 4B 5B 6B 7D 8D 9D 10B A. Walang pasensya
11C 12B 13D 14C 15B 16A 17C 18B 19D 20A B. Walang pagtitimpi
21D 22A 23D 24C 25C 26B 27D 28D 29D 30B C. Walang galang
D. Walang lakas ng loob
1. Hindi totoo ang katapangan na ipinapakita ni
Vincent sa harap ni Lalie sapagkat BAHAG ANG 9. Siya’y isang bulag ngunit kaya niyang gumuhit ng
BUNTOT niya sa harap ng paghihirap. larawan. Siya’y isang ___________________
A. Matapang A. Imbentor
B. Matiyaga B. Manunulat
C. Duwag C. Dalubhasa
D. Malakas ang loob D. Pintor

2. NAG-ALSA BALUTAN si Claudia dahil sa 10. Anong hukuman ang siyang _____________ ng
malimit umanong pananakit ni Reymar. mga kaso ng korupsyon.
A. Lumayas A. Court of Appeals-manglilitis
B. Nagtampo B. Sandigan Bayan-naglilitis

84
C. Korte Suprema-maglilitis bagay ay maging kapaki-pakinabang. Anong uri
D. Ombudsman-tagapaglitis ng texto ang seleksyong binasa?
A. Informativ
11. _________________ mo naman sa kanila na B. Argumentativ
magdala ng mapagsasaluhan. C. Prosijural
A. Usap-usapan D. Narativ
B. Usapin
C. Ipakiusap 16. ____________ magtrabaho sina Fred at
D. Pakiusap Jose.
A. Magkasinbilis
B. Magkasingbilis
12. “Gamitin ninyo sa pakikipagkaibigan ang
C. Napabilis
kayamanan ng sanlibutang ito. Maubos man ito ay
D. Magkasimbilis
may tatanggap naman sa inyo sa tahanang walang
hanggan.” (Luk 16:9) 17. Samahan mo si Lola sa palengke _____ hindi
Ano ang kahulugan ng sanlibutan? maligaw.
A. Mga israelita lamang A. kung saan
B. Apostoles B. noong
C. Dukha C. ng
D. Katauhan D. nang

18. Upang lalong maging ______________ ang


patakaran ng Pangulo na mabago ang
13. Ano ang pangungusap na dapat mauna? pamamalakad sa bansa, ipinasya niyang
I. Ngunit ang pagtatalo ay hindi dapat nauuwi sa magkaroon ng pagbabagong tatag sa lahat ng
pagkakagalit. sangay ng pamahalaan.
II. Natural lamang ang pagkakaroon ng A. marangal
magkakaibang pala-palagay ang mga tao. B. mabisa
III. Sa mga mag-asawa man ay hindi maiiwasan C. tanyag
ang pagkakaiba ng opinyon. D. malinaw
IV. Kahit sa magkakapatid, karaniwan na ang
hindi pagkakasundo. 19. Nakalulungkot isipin na ang karamihan sa
V. Upang maiwasan ito, nararapat lamang na mga kabataan ngayon ay ______________ sa
maging bukas ang ating isipan sa paniniwala ng masamang bisyo.
iba. A. nababalisa
B. nalalayo
A. I. C. nabubuyo
B. II D. nababalot
C. III
D. IV 20. ________ ng mga pulis ang kadena sa
kanyang mga kamay.
14. Alin sa mga sumusunod ang hindi nabibilang A. Nalagyan
na kasingkahulugan ng salitang pala-palagay? B. Nilagyan
A. Opinyon. C. Inilagay
B. Kuru-kuro D. Naglagay
C. Paniniwala
D. Akala Answers: 1C 2A 3D 4C 5D 6C 7A 8A 9D 10B
11C 12D 13B 14C 15A 16B 17D 18B 19C 20C
15. May pera sa basura. Huwag mong pagtakhan
iyan. Isa-isahin mo ang laman ng iyong FOUNDATION OF EDUCATION
basurahan. Tiyak na may papel na walang sulat
sa likod at maaari mo pa itong sulatan. Kung 1. The Department of Education gives greater
minsan, may mga papel at notebook na may sulat emphasis on the development of basic skills.
pero pwede mo namang ipagbili. May mga bukas What is the philosophical basis for this?
na lata na maaari mo rin namang tamnan o kaya’y a. Essentialism
pwedeng balutin ng wrapping paper upang b. Existentialism
paglagyan ng lapis, bolpen, krayola, aspile o c. Perennialism
kaya’y pako. Ang pinagbalatan ng sibuyas, d. Pragmatism
patatas at saging, sanga ng kangkong, tira-tirang
pagkain ay maaaring maging pagkain ng baboy at 2. Teacher M views his students as unique, free-
maaari rin itong maging pataba sa lupa. Tunay na choosing and responsible individuals. All
may pera sa basura kung magtitiyaga lamang at classroom activities revolve around the said
magiging malikhan upang ang patapong mga premise. What theory underlies this?
85
a. Essentialism of the child. What is the concern of the
b. Existentialism reconstructionists?
c. Progressivism a. Experiential learning
d. Realism b. Socialization
c. Social problem
3. Religious rituals in the classroom and in the
school programs prove the deep natural religiosity 10. One of the following quotations does not
of the Filipinos. Which philosophy has greatly conform to the Christian doctrine of Education for
contributed to the tradition? Humanitarianism. Which one is it?
a. Buddhism a. Do unto others as you would like others do unto
b. Confucianism you
c. Hinduism b. Love they neighbor as thyself
d. Islam c. Not on bread alone is man to live but on every
utterance that comes from mouth of God
4. In order to make Roman education truly d. Whatever good things we do to our poor,
utilitarian, how should the day-to-day lessons be helpless brothers, we do it for God.
taught?
a. Taught in the students' native dialect 11. Scouting and Citizen's Army Training (CAT)
b. Taught interestingly through the play way give training in character-building, citizenship
method training, etc. Which leads to the creation of a new
c. Related and linked to the events happening in social order and a new society eventually. What
everyday life philosophy supports this?
d. Practiced at home under the guidance of their a. Existentialism
respective parents. b. Perennialism
c. Progressivism
5. Which influenced the military training d. Social reconstructionism
requirements among students in the secondary
and tertiary levels? 12. Teacher V demonstrated the technique on
a. Chinese how to group students according to their needs
b. Greeks and interests and how to use self-paced
c. Orientals instructional materials. Which philosophy is
d. Romans manifested in this activity?
a. Essentialism
6. Which philosophy has the educational objective b. Progressivism
to indoctrinate Filipinos to accept the teachings of c. Realism
the Catholic church which is foster faith in God? d. Social Reconstructionism
a. Realism
b. Pragmatism 13. Teacher G, a Christian Living teacher, puts so
c. Idealism much significance on values development and
d. Existentialism discipline. What could be her educational
philosophy?
7. Virtue as one component in the teaching of a. Idealism
Rizal as a course focuses on the teaching of good b. Pragmatism
and beauty consistent with the good and beauty in c. Progressivism
God. What philosophy supports this? d. Realism
a. Existentialism
b. Idealism 14. Which one does not illustrate the principle that
c. Progressivism rights and duties are correlative?
d. Social Reconstructionism a. The right of an unmarried pregnant teacher to
abort her baby in relation to her duty to protect her
name and her job as a teacher
8. Giving education the highest budgetary b. The right of a state to compel students to
allocation, the Philippine government recognizes military service is reciprocated by the duty of the
the possible contribution of its future citizens to state to protect them.
the national development goals of the Philippine c. The right to a living wage involves the duty of
society. Which stressed this goal of education for the school administrators to give the salary
social transformation? agreed upon and the duty of the teachers to give
a. Athenian education a fair amount of work.
b. Followers of Christ d. The right to life of children and to be given
c. Greek education respect of such right.
d. Roman education
15. Why should a teacher take the obligation upon
9. The progressivists emphasized the individuality himself to study and understand the custom and
86
traditions of the community where he works? relative to the state are schools seemed to be
a. To change the culture of the community. failing?
b. To have a sympathetic attitude for the people of a. Instill allegiance to the constitutional authorities
the community. b. Promote national pride
c. To identify the weaknesses of the culture of the c. Promote obedience to the laws of the state
community. d. Respect for all duly constituted authorities.
d. To please the people of the community.
22. Which subject in the elementary and likewise
16. A teacher who is a recognized expert in in the secondary schools are similar to the goal of
carpentry works, taught his students how to Rome to train the students for citizenship?
prepare and construct good and aesthetic a. Communication ARts
furniture from local resources. What cultural b. MAPEH/PEHMS
transmission process is this? c. Science
a. Acculturation d. THE/TLE
b. Enculturation
c. Indoctrination 23. Which of the following schools practices is not
d.Observation based on Social Reconstructionism?
a. Establishment of SDF
17. Every first day of the school year, Miss Reyes b. Exemption of Scouts from CAT
prepared activities which will make her Grade 2 c. Promoting culture and arts in schools
children, sing, plan, learn and introduce d. Promoting project WOW
themselves to the class. What process did the
teacher emphasize? 24.Which of the following is the focus of the
a. Acculturation Japanese education in the Philippines?
b. Enculturation a. Democratic ideals and nationalism
c. Indoctrination b. Love and service to one's country
d. Socialization c. Religion and love for Asian brothers
d. Vocational and health education
18. Which program in the educational system
seems to be aligned to the Christian humanitarian 25. According to reconstructionism, the goal of
principle respect for the human personality? education is to bring about a new social order.
a. The alternative learning system delivery Which practice best manifests this view?
b. The functional literacy program for the out-of- a. The class conducts scientific experiments to
school youth and adults discover or verify concepts.
c. The promotion of the basic human rights of the b. The class discusses role models and their
Filipino impact on society.
d. The study of the Philippine Constitution c. The class allowed to engage in divergent
thinking.
19. With a death threat over his head, Teacher d. The class undertakes well-planned projects in
Liza is directed to pass an undeserving student, if the community.
she is a hedonist, which of the following will she
do? Answers: 1A 2B 3B 4C 5D 6C 7B 8D 9C 10C
a. Don't pass him, live her principle of justice. She 11D 12B 13A 14A 15B 16B 17D 18C 19D 20B
will get reward, if not in this life, in the next. 21B 22D 23C 24D 25D
b. Don't pass him. She surely will not like 1. Teacher A, a Values Education teacher
someone to give you a death threat in order to emphasizes ethics in almost all her lessons.
pass. Which of the following emphasizes the same?
c. Pass the student. That will be of use to her, the a. Liberal Education
student and his parents. b. Moral Education
d. Pass the student. Why suffer the threat? c. Religious Training
d. Social Education
20. Which philosophy approves a teacher who
lectures most of the time and requires his 2. Which reform in the Philippine Educational
students to memorize the rules of grammar? System advocates the use of English and Filipino
a. Existentialism as media of instruction in specific learning areas?
b. Idealism a. Alternative Learning
c. Pragmatism b. Bilingual Education
d. Realism c. K-12 Program
d. Multilingual Education
21. In a student conducted, the pupils were asked
which nationality they would prefer if given a 3. Activities planned by school clubs/
choice. Majority of the pupils wanted to be organizations show school-community connection
Americans. In this case, in which obligation geared towards society's needs. What philosophy
87
is related to this?
a. Existentialism 10. Which move liberalized access to education
b. Progressivism during Spanish period?
c. Realism a. The education of illiterate parents
d. Social reconstructionism b. The establishment of at least one primary
school for boys and girls in each municipality
4. What philosophy is related to the practice of c. The hiring of tribal tutors to teach children
schools acting as laboratory for teaching reforms d. The provision of vocational training for school
and experimentation? age children
a. Essentialism
b. Existentialism 11. Which of the following is the chief aim of
c. Progressivism Spanish education?
d. Social Reconstructionism a. Conformity and militarism
b. Perpetuation of culture
5. Which of the following situations presents a c. Propagation of the Catholic religion
value conflict? d. Utilitarianism and conformity
a. The teacher and his students have class
standing as their priorities. 12. Which of the following is the aim of our
b. The teacher and the administrator follow a set education during the Commonwealth period?
of criteria in giving grades. a. Designed after Japanese education
c. The teacher has students whose parents want b. Patterned after the American curriculum
their children to obtain higher grades than what c. Predominantly religious
they are capable of getting. d. Purely nationalistic and democratic
d. The teacher sets high expectations for
intelligent students such as getting higher grades. 13. Which of the following is not a reason why the
basic education curriculum has been
6. Which situation shows that a sense of restructured?
nationhood is exemplified? a. To become globally competitive during this
a. The class conducted a debate using Filipino as industrial age
medium b. To be relevant and responsive to a rapidly
b. The class is required to watch the TV sitcom of changing world
Oprah to improve their English communication c. To empower the Filipino learners for self-
skills. development throughout their life.
c. The class opted to make a choral rendition of d. To help raise the achievement level of students
the theme song of a foreign movie.
d. When Teacher Chris asked her Grade 2 14. Which philosophy of education influence the
students in what country they wish to live, most of singing of the National Anthem in schools?
them chose United States. a. Nationalism
b. Naturalism
7. A teacher who believes in the progressivist c. Pragmatism
theory of education would embrace certain d. Socialism
reforms on methodology. Which reform would be
consistent with this theory? 15. Who among the following believes that
a. Active participation of teachers learning requires disciplined attention, regular
b. Formal instructional pattern homework, and respect for legitimate authority?
c. Strict external discipline a. Essentialist
d. Teacher domination of class activities b. Progressivist
c. Realist
d. Reconstructionist
8. What philosophy of education advocates that
the curriculum should only include universal and 16. Which of the following is the main function of
unchanging truths? the philosophy of education?
a. Essentialism a. Reconsider existing educational goals in the
b. Idealism light of society's needs
c. Perennialism b. Provide the academic background prerequisite
d. Pragmatism to learning
c. Define the goals and set the direction for which
9. Which of the following is not a function of the education is to strive
school? d. Aid the learner to build his own personal
a. Changing cultural practices philosophy
b. Development of attitudes and skills
c. Reproduction of species 17. Homeroom advisers always emphasize the
d. Socialization among children importance of cleanliness of the body. Children
88
are taught how to wash their hands before and theories and principles
after eating. What is this practice called? d. Requiring learners full master of the lesson.
a. Folkway
b. Laws 24. Which philosophy influenced the cultivation of
c. Mores reflective and meditative skills in teaching?
d. Social norm a. Confucianism
b. Existentialism
18. Which curricular move served to strengthen c. Taoism
spiritual and ethical values? d. Zen Buddhism
a. Integration of creative thinking in all subject
b. Introduction of Values education as a separate 25. Which of the following situation manifests a
subject area balance between teachers responsibility and
c. Reducing the number of subject areas into skill accountability?
subject a. She entertains her students with personal
d. Re-introducing science as a subject in Grade 1 stories until the end of the period.
b. She spends most of the time on the latest
19. The greatest happiness lies in the gossips in showbiz.
contemplative use of mind, said Plato. Which of c. She teaches as much as she could for duration
the following activities adheres to this? of the period.
a. Cooperative learning d. She teaches as well as entertains the students
b. Instrospection with per personal stories.
c. Role Playing
d. Social Interaction Answers: 1B 2B 3D 4C 5C 6A 7A 8C 9C 10B
11C 12D 13D 14A 15A 16C 17D 18B 19B 20A
20. Your teacher is of the opinion that the word 21C 22B 23C 24D 25C
and everything in it are ever changing and so
teaches you the skill to cope with the changes. MATH
Which in his governing philosophy?
a. Experimentalism 1. Two buses leave the same station at 8:00 pm.
b. Existentialism One bus travels north at the rate of 30 kph and
c. Idealism the other travels east at 40 kph. How many
d. Realism kilometers apart are the buses at 10 pm?
a. 140 km
21. Teacher Myra says: &If it is billiard that brings b. 100 km
students out of the classroom, let us bring it into c. 70 km
the classroom. Perhaps, I can use it to teach d. 50 km
Math&. To which philosophy does teacher Myra
adheres to? 2. Calculate the mean absolute deviation of the
a. Essentialism following numbers: 60, 80, 100, 75 and 95
b. Idealism a. 12.4
c. Progressivism b. 14.2
d. Reconstructionism c. 16.1
d. 18.9
22. Which of the following should be done to build
a sense of pride among Filipino youth? 3. Which of the following is the factorization of the
a. Replace the study of folklores and myths with binomial x2 - 42?
technical subjects a. (x + 4)(x + 2)
b. Re-study our history and stress on our b. (x – 4)2
achievements as people c. x(x + 2x + 2)
c. Re-study our history from the perspective of our d. (x – 4)(x + 4)
colonizers
d. Set aside the study of local history 4. What value of x will satisfy the equation: 0.4(5x
- 1470) = x?
23. A teacher who subscribes to the pragmatic a. 490
philosophy of education believes that experience b. 2,130
study should follow learning in her teaching. c. 1470
Which of the following does she do to support her
belief? 5. Which of the following has the greatest value:
a. Encouraging learners to memorize factual a. 3 + 32 + (3 + 3)2
knowledge b. 33
b. Equipping learners with the basic abilities and c. [(3 + 3)2]2
skills d. (3 + 3 + 3)2
c. Providing learners opportunities to apply
89
counting numbers greater than the sum of the first
100 counting numbers?
6. The average of 5 different counting numbers is a. 110
20. What is the highest possible value that one of b. 3,775
the numbers can have? c. 3,155
a. 20 d. 1200
b. 40
c. 30 15. Which of the following has the largest value?
d. 90 a. 85
b. 39
7. Three brothers inherited a cash amount of c. 65
P62,000 and they divided it among themselves in d. 94
the ratio of 5:4:1. How much more is the largest
share than the smallest share? 16. A water tank contains 18 liters when it is 20%
a. P75,000 full. How many liters does it contain when 50%
b. P30,000 full?
c. P24,800 a. 60
b. 30
8. What is the missing terms in the series 5, 20, c. 58
80, ___,1280, ___, 20, 480? d. 45
a. 50;210
b. 40;160 17. The edges of a rectangular solid have these
c. 35;135 measures: 1.5 feet by 1½ feet by 3 inches. What
d. 320;5120 is its volume in cubic inches?
a. 324
9. At what rate per annum should P2400 be b. 225
invested so that it will earn an interest of P800 in c. 972
8 years? d. 27
a. 6 ½ %
b. 5 ½ % 18. In a certain school, the ratio of boys to girls is
c. 4.17 % 5 is to 7. If there are 180 boys and girls in the
d. 6 % school, how many boys are there?
a. 105
10. The area of a rectangle is (x2 + 2x - 8). If its b. 90
length is x + 4, what is its width? c. 45
a. x + 2 d. 75
b. x - 2
c. x + 1 19. Ruben’s grades in 6 subjects are 88, 90, 97,
d. x + 6 90, 91 and 86? What is the least grade that he
should aim for in the 7th subject if he has to have
11. What is the value of 12⅙ - 3 ⅜ - 5 ⅔ + 20 ¾? an average of 88?
a. 21 1/8 a. 92
b. 22 b. 74
c. 23 7/8 c. 88
d. 21 d. 85

12. The vertex angle of an isosceles triangle is 20. On a certain day, three computer technicians
20°. What is the measure of one of the base took turns in manning a 24-hour internet shop.
angles? The number of hours Cesar, Bert, and Danny
a. 150° were on duty was in the ratio 3:4:5, respectively.
b. 60° The shop owner pays them P50 per hour. How
c. 75° much would Danny receive for that day?
d. 80° a. P 230
b. P500
13. Ana and Beth do a job together in three hours. c. P160
Working alone, Ana does the job in 5 hours. How d. P480
long will it take Beth to do the job alone?
a. 3 and 1/3 hours 21. A retailer buys candies for P90.25. The pack
b. 2 and 1/3 hours has 35 pieces of candies. If she sells each candy
c. 3 hours for P2.25, how much profit does she make?
d. 7 and 1/2 hours a. P11.50
b. P56.25
14. How much greater is the sum of the first 50 c. P37.50
90
d. P18.75 late for school?
a. 6:42 am
22. An online shop sells a certain calculator for b. 6:27 am
P950 and charges P150 for shipping within c. 6:57 am
Manila, regardless of the number of calculators d. 7:02 am
ordered. Which of the following equations shows
the total cost (y) of an order as a function of the 30. Which common fraction is equivalent to
number of calculators ordered (x)? 0.215?
a. y = (950 + 150)x a. 43/200
b. y = 150x +950 b. 27/125
c. x = 950y + 150 c. 21/50
d. y = 950x + 150 d. 108/375

23. One side of a 45° - 45° - 90° triangle Answers: 1B 2A 3D 4D 5C 6D 7C 8D 9C 10B


measures x cm. What is the length of its 11C 12D 13D 14 15A 16D 17C 18D 19B 20B 21
hypotenuse? 22D 23D 24C 25C 26D 27D 28C 29B 30A
a. X √3 cm
b. X cm
c. (X √3)/2 cm 1. 120 is 4/5 of what number?
d. X √2 cm A. 150
B. 96
24. The legs of one right triangle are 9 and 12, C. 30
while those of another right triangle are 12 and D. 24
16. How much longer is the perimeter of the larger
triangle than the perimeter of the smaller triangle? 2. 45 is 60% of what number?
a. 84 A. 75
b. 7 B. 27
c. 12 C. 15
d. 14 D. 18
25. Determine the midpoint of the line segment
3. What is the ratio of 1/5 to 5/15
joining the points (7, -3) and (-1, 6).
A. 3:5
a. (2, 3/2)
B. 1:5
b. (2, -3/2)
c. (3, 3/2) C. 5:15
d. (1, 5/2) D. 15:75

26. Which of these has the longest perimeter? 4. What is the value of y is 2x + 2xy + 3y = 25, and x
a. A square 21 cm on a side = 2?
b. A rectangle 19 cm long and 24 cm wide A. 3
c. An equilateral triangle whose side is 28 cm B. 25
d. A right triangle whose two legs are 24 and 32 C. 2
cm D. 12

27. How many square inches are in 2 square 5. Ten nurses earn a total average monthly salary of
yard? P120, 000. The total average income of 6 of them
a. 900 amounts to P80, 000. What is the average total
b. 144 income of each of the remaining workers?
c. 1296 A. P10, 000
d. 2,592 B. P40, 000
C. P400
28. In a playground for Kindergarten kids, 18 D. P1, 000
children are riding tricycles or bicycles. If there are
43 wheels in all, how many tricycles are there? 6. 2 tablespoons is equivalent to 1 liquid ounce while
a. 8
10 tablespoons is equivalent to ½ cup. How many
b. 9
liquid ounces are there in two cups?
c. 7
A. 20 ounces
d. 11
B. 10 ounces
29. Nelia takes ¾ hour to dress and get ready for C. 16 ounces
school. It takes 4/5 hour to reach the school. If her D. 8 ounces
class starts promptly at 8:00 am; what is the latest
time she can jump out of bed in order not to be 7. A taxi charges P50 for the first kilometer and

91
charges P3 for each additional mile. How far could C. 100
P200 go? D. 200
A. 50 kilometers
B. 51 kilometers 15. It takes John 25 minutes to walk to the car
C. 66. 67 kilometers park and 45 to drive to work. At what time should
D. 4 kilometers he get out of the house in order to get to work at
9:00 a.m.?
8. 20, 10, 40, 10, 60, 20, 80, 60, 100, 240, 120, ____, A. 7:50 am
____ B. 8:00 am
A. 1200, 140 C. 8:10 am
D. 8:20 am
B. 480, 140
C. 140, 480
16. Kim can walk 4 kilometers in one hour. How
D. 1200, 240
long does it take Kim to walk 18 kilometers?
A. 4 ½ hours
9. What is the value of n in the statement 2mn = 50, B. 4 hours
if m = 1? C. 5 hours
A. 25 D. 8 hours
B. 2
C. 50 17. A factory produced 2300 TV sets in its first
D. 1 year of production. 4500 sets were produced in its
second year and 500 more sets were produced in
its third year than in its second year. How many
TV sets were produced in three years?
10. One store sold twice more apples in the A. 11,800
afternoon that in the morning. The store sold 450 B. 12, 800
pieces of apples. How many apples are sold in C. 10, 800
the morning? D. 13, 800
A. 150
B. 300 18. Tom and Bob have a total of 49 toys. If Bob
C. 200 has 5 more toys than Tom, how many toys does
D. 350 Tom has?
A. 22
11. Joey gathered twice more marbles than Vic B. 27
and Tito gathered 2 more marbles than Vic. They C. 23
gathered 26 marbles. How many marbles did Vic D. 26
gather?
A. 6 19. John can eat a quarter of a pizza in one
B. 12 minute. How long does it take John to eat one
C. 8 pizza and a half?
D. 14 A. 6 minutes
B. 12 minutes
12. A child has read 2/3 of a book. The read part C. 9 minutes
of the book has 100 pages more than the unread. D. 3 minutes
How many pages is the book?
A. 300 20. What is 150% of 298?
B. 200 A. 447
C. 500 B. 44.7
D. 150 C. 4.47
D. 4470
13. A number is divided by 5. The result was
subtracted by 154 and the difference is 6. What is Answers: 1A 2A 3A 4A 5A 6A 7B 8A 9A 10A 11A
the number? 12A 13A 14D 15A 16A 17A 18A 19A 20A
A. 800
B. 160 PRINCIPLE OF LEARNING&MOTIVATION
C. 200
D. 400 1. Which theory operates on the &stimulus-
response principle&, which means all behaviors
14. A container contains 23bottles and each bottle are caused by external stimuli?
contains 20 marbles. How many marbles are a. Contextual theory
there in 10 bottles? b. Behaviorist theory
A. 460 c. Cognitive theory
B. 400 d. Constructivist theory
92
b. Materials presented are difficult and
2. Ms. Erika in her Biology class accompanies her challenging to the students
discussion with interesting visual aids. She c. When the materials to be learned is related to
strongly believes that students learn better when what students already know
lessons are presented with images, real or d. Students find the lessons easy and relevant to
imagined aside from mere lecture method. Which what was assigned to them
learning theory does she upholds?
a. Dual-Coding Theory 9. Rita easily remember dates and events in
b. Information Processing Theory history. What component of LTM does Rita have?
c. Meaningful Reception Learning Theory a. Creative thinking
d. Social Cognitive Theory b. Critical thinking
c. Reflective thinking
3. Miss Rita is an excellent Physical Education d. Logical thinking
teacher. She started teaching volleyball to her
Grade 2 class. Despite all her efforts, her class 10. An Earth Science has just completed a unit on
does not seem to learn how to play the game. the sun. As she recognizes her next unit on other
What law of learning was disregarded? stars, she uses the sun as a frame of reference.
a. Law of Disuse What view of learning was used?
b. Law of Effect a. Discovery learning
c. Law of Exercise b. Informative learning
d. Law of Readiness c. Meaningful learning
d. Transfer learning
4. Teacher jay, a physical education teacher,
demonstrates the new skill to be learned so that 11. Which is an application of cognitive approach
his students can watch him and later reproduce to motivation?
the skill. What learning theory is associated with a. Explain the reasons for studying the topic
the situation? b. Create a supportive classroom climate for
a. Dual-Coding Learning Theory students
b. Information Processing c. Provide clear and prompt feedback on
c. Schema Learning Theory assignments
d. Social Learning d. Begin lessons with challenging questions and
conflicting events
5. Patrice is always fearful of freely roaming dogs
but does not mind dogs in a pen or on a leash. 12. The first people power was held in February
What feature of classical conditioning is 25, 1986. What kind of knowledge is presented?
exhibited? a. Conditional Knowledge
a. Discrimination b. Cognitive Knowledge
b. Extinction c. Domain-Specific Knowledge
c. Generalization d. Procedural Knowledge
d. Practice
13. The students of Mrs. Reyes were not able to
6. A music teacher is careful in planning activities learn the concepts that she presented yesterday
for each lesson. He praises liberally and rewards so she taught the same concepts again but this
correct answers. What view of learning is time using a different teaching method. What
exhibited? principle of learning was applied?
a. Classical conditioning a. Concepts should be presented in varied and
b. Meaningful learning different ways
c. Operant conditioning b. Effort was put forth when tasks are challenging
d. Social learning c. Learning by doing is more effective than just by
sitting and listening
7. Which of the theories of learning presents or d. Learning is aided by formulating and asking
states that learning skills are hierarchically questions
arranged?
a. Cumulative Learning 14. Alvin is a transferee and feels uneasy with his
b. Meaningful Learning new school. His teacher is very accommodating,
c. Social Cognitive Learning warm and caring. Alvin felt comfortable with the
d. Theory of Instruction teacher display of genuine warmth. The teacher is
consistent in his manner and Alvin began to
associate school with the teacher's warmth.
8. Which of the following best describes what Which theory is being illustrated?
meaningful learning is? a. Meaningful learning
a. When what is to be learned is new and easy for b. Operant conditioning
the students c. Classical conditioning
93
d. Observational learning learn psychomotor skills?
a. Teacher uses verbal explanation and
15. After just being introduced to another guest in description of the movements in addition to live
the party, Tom cannot remember the name of the demonstration of the movements
guest he was introduced to. In what memory b. Teacher provides feedback to the learner about
stage was the information stored in? his/her progress
a. Episodic memory c. Teacher encourages the learner to practice, in
b. Semantic memory order to maintain his/her sharpness of the
c. Sensory memory movements
d. Working memory d. All of the above

16. Vygotsky claimed that social interaction is 22. The teacher presented a new lesson where in
important for learning. What does this imply? the students were asked to work on a new project
a. Children are independent problem solvers which was somewhat complicated. The students
b. Children learn from adults and other children showed interest while working on the project.
c. Children learn by passive presentation of What principle applies to the situation?
information a. Effort was put forth when tasks are challenging
d. Children in the crib has no learning yet, since b. Lessons should be presented in varied and
they are not capable of interaction different ways
c. Meaningful materials are readily learned than
17. How would you help a student who is nonsense materials
intelligent but is underachieving in class? d. Teachers should provide opportunities for
a. Provide challenging activities which he/she can meaningful and appropriate practice
accomplish
b. Recognize his talents by asking him/her to help 23. Maturation should precede certain types of
other students with their work learning. How is this applied in the classroom?
c. Identify the immediate causes of difficulties that a. Concepts should be taught from simple to
cause his/her being an underachiever complex
d. Allow him/her to work with the slow learner b. Consider the age level of students in assigning
group to cope with the academic needs of the tasks
lesson. c. Follow the interest of students in assigning
tasks
18. Mrs. Corpuz always makes sure that her pre- d. Give the same task to all students in a
school classroom is well organized and clean. particular grade level
She puts up interesting and colorful visuals on the
bulletin boards. What principle of motivation was 24. Luz easily learns a lesson when she is
applied? working with laboratory equipment but hardly
a. Incentives motivate learning remembers a lesson the teacher lectured on.
b. Internal motivation is longer lasting and more What type of learner is Luz?
self-directive than is external motivation a. Auditory Learner
c. Motivation is enhanced by the way in which b. Kinesthetic Learner
instructional material is organized. c. Tactile Learner
d. The environment can be used to focus the d. Visual Learner
student's attention on what needs to be learned.
25. Which of the following statements about
19. For every correct answer, the teacher would motivation is false?
give a star to her students. What schedule of a. External motivation is longer lasting and more
reinforcement was used? self-directive than internal motivation
a. Fixed interval b. Internal motivation is fueled by one's goals or
b. Fixed ratio ambitions
c. Variable interval c. Motivation is enhanced by the way in which the
d. Variable ratio instructional material is organized
d. Motivation to perform is affected by expectancy
20. Marga, a six year old, always asked her and value
playmates to sit in front of her small black board
and she plays teacher. Her mother is a teacher. Answers: 1B 2A 3D 4D 5A 6C 7A 8C 9C 10C
What theory explains Marga's behavior? 11A 12B 13A 14B 15C 16B 17C 18D 19B 20C
a. Classical Conditioning 21D 22A 23A 24D 25A
b. Operant Conditioning
c. Social Learning
d. Information Processing 1. Based on Bandura's theory, which conditions
must be present for a student to learn from a
21. What should the teacher do to help students model?
94
I. Attention II. Retention III. Motor Reproduction a. 2-1-3-4
IV. Motivation b. 2-1-4-3
a. I and II c. 2-3-4-1
b. I, II and III d. 2-4-3-1
c. I, II, III and IV
d. III and IV 9. Which is essential in meaningful reception
learning?
2. Which of the following principles of learning a. Concepts are presented to learner and
applies to considering student's age in presenting received by them.
certain content and cognitive processes? b. Concepts are discovered by the learner
a. Principle of readiness c. Concepts are related to one another
b. Principle of learning by doing d. Concepts are solicited from the learners
c. Principle of presenting challenging tasks
d. Principle of learning aided by formulating and 10. Grace is bilingual. She speaks both English
asking questions and Filipino fluently. She begins to study Spanish
and immediately recognizes many similarities
3. In classical conditioning, which are paired between the Spanish and Filipino languages and
together in order to elicit the desired response? uses this information to acquire the new language
a. UCS and NS faster. What kind of transfer was Grace able to
b. CS and NS use?
c. UCS and CS a. Lateral Transfer
d. UCR and NS b. General Transfer
c. Specific Transfer
4. According to Thorndike, what law states that d. Vertical Transfer
the strength of a connection is influenced by the
consequences of the response? 11. Cristina is almost asleep when she felt the
a. Law of disuse need to go to the bathroom. She tried to sleep it
b. Law of effect off but after a while, she was forced to stand up
c. Law of exercise and go to the bathroom. What theory of motivation
d. Law of readiness explains Cristina's behavior?
a. Attribution Theory
5. A burglary occurred in Jason's neighborhood. b. Drive Theory
Since then, Jason is very careful of locking their c. Expentancies and Values Theory
doors and closing windows. What theory best d. Solomon's Opponent Theory
explains Jason's behavior?
a. Observational learning 12. Marko excels in adding numbers. He learned
b. Imitation learning this skill in his Math class. He is now able to apply
c. Self-regulated learning this skill in his Music class. What type of transfer
d. Vicarious learning was used?
a. Lateral transfer
6. In observation and imitation learning, what b. General transfer
should be the learner's response when the c. Specific transfer
teacher initially models the behavior? d. Vertical transfer
a. Reproduce and match
b. Pay Attention 13. Mr. Lorenzo would always give the chapter
c. Imitate and practice test on a Friday. What schedule of reinforcement
d. Shows satisfaction is used by Mr. Lorenzo?
a. Fixed interval
7. What is the correct sequence of information b. Fixed ratio
processing? c. Variable interval
a. Sensory register-STM-LTM d. Variable ratio
b. STM-sensory register-LTM
c. Sensory register-LTM-STM 14. To remember the six digits, 8, 4, 3, 9, 4, 5, the
d. LTM-sensory register- STM Math teacher grouped the numbers in two's 84,
39, 45 or in threes 843, 945. What control process
of retaining information is referred to?
8. What should be the hierarchy of the types of a. Chunking
learning according to the cumulative learning b. Interfering
theory? c. Rehearsing
1. Problem solving learning d. Remembering
2. Rule learning
3. Discrimination learning 15. Rob regularly practice playing the guitar so he
4. Concept learning can finish Book I. His mother promised to buy him
95
a Nintendo when he finishes Book I. How is Rob 22. Instead of asking her students to write about
motivated? their reaction to a story, the teacher asked her
a. Extrinsically students to interpret the story in dance form. What
b. Intrinsically principle of learning is considered?
c. with Self-Determination a. Concepts should be presented in varied and
d. with Self-Efficacy different ways
b. Effort if put forth when tasks are challenging
16. Lara excels in dancing and in certain sports. c. Learning by doing is more effective than just
According to Gardner what intelligence is sitting and listening
dominant in Lara? d. Learning is aided by formulating and asking
a. Bodily kinesthetic questions
b. Intrapersonal
c. Musical 23. In performing the minuet, first raise your heel,
d. Spatial second make three steps forward, third step make
a point. What kind of knowledge was exhibited?
17. According to Bronfenbrenner, what system a. Conditional Knowledge
contains structures that has direct contact with the b. Declarative Knowledge
child? c. Domain-Specific Knowledge
a. Chronosystem d. Procedural Knowledge
b. Exosystem
c. Mesosystem 24. Why should teachers provide positive
d. Microsystem feedback and realistic praise?
a. To motivate the students to study
18. Some learners like to find specific and b. So the students will know what to do
concrete answers. What kind of learners are c. To be liked and loved by the students
they? d. So the students will praise him/her
a. Accomodators
b. Assimilators 25. Which statement does not refer to cognitive
c. Convergers theories?
d. Divergers a. Prefer to concentrate on analyzing cognitive
process
19. Mrs. Mercado, the Home Economics teacher, b. Conclusions are based on observation of
constantly gives verbal guidance to her pupils external manifestations on learning
while practicing a sewing skill. What is the value c. Study of the structures and components of
of giving verbal guidance in improving pupils information processing
learning behavior? d. Believe in non-observable behavior
a. It promotes the growth of interest in the new
learning tasks Answers: 1C 2A 3A 4B 5D 6B 7A 8D 9C 10C
b. It serves as informational feedback 11C 12B 13A 14A 15C 16A 17D 18C 19B 20B
c. It facilitates perfection of skills 21B 22B 23D 24B 25B
d. It directs pupils' attention to more adequate and
better techniques
PRINCIPLES OF STRATEGIES AND
20. Why should learning be aided by formulating TEACHING
and asking questions? 1. To ensure the lesson will go smoothly, Teacher
a. Students will have a grade in recitation A listed down the steps she will undertake
b. Students will develop their self-confidence together with those of her students. This practice
c. The teacher will know who among the students relates to?
can communicate very well a. Teaching style
d. The teacher will not always do the talking but b. Teaching method
the students will be given a chance to do the c. Teaching strategy
same thing. d. Teaching technique

21. Ms. Baquiran, the VE teacher, and her pupils, 2. The class of Grade 6 - Einstein is scheduled to
while working on the concept of honesty, agreed perform an experiment on that day. However, the
that no cabinets and book cases would be locked chemicals are insufficient. What method may then
throughout the day. Which principle in affective be used?
learning is being implemented? a. Project
a. Provide exemplary models b. Laboratory
b. Provide appropriate practice c. Lecture
c. Provide for pleasant emotional experience d. Demonstration
d. Provide for independent attitude cultivation
3. Teacher C gives the class specific topic as
96
assignment which they have to research and pass 9. Devices can make a lecture more
the following day. However, the students could understandable and meaningful. What is the most
not find any information about it. What method important thing a teacher should consider in the
should Teacher C use to teach the assignment? selection and utilization of instructional materials?
a. Project method a. Objectives of the lesson
b. Discovery approach b. Availability of instructional materials
c. Lecture method c. Attractiveness of instructional materials
d. Demonstration method d. Degree of interest on the part of the students

4. Pictures, models and the like arouse students 10. Teacher E asks student A to identify and
interest on the day's topic, in what part of the analyze events, ideas or objects in order to state
lesson should the given materials be presented? their similarities and differences. In which part of
a. Initiating activities the lesson does said activity take place?
b. Culminating activities a. Preparation
c. Evaluation activities b. Generalization
d. Developmental activities c. Application
d. Comparison and Abstraction
5. In Bloom's taxonomy of educational objectives,
the domains are stated from lowest to highest 11. Which part of the lesson is involved in the
level. Which of the following objectives belongs to giving of situation or activities based on the
the lowest level? concepts learned?
a. To identify the characters of the story. a. Preparation
b. To differentiate active from passive voice. b. Generalization
c. To give the available resources that could be c. Application
recycled to useful things. d. Comparison and Abstraction
d. To explain the procedure in changing improper
fraction to mixed number 12. Teacher F wants the class to find out the
effect of heat on matter. Which method will help
6. The class of IV - Kalikasan is tasked to analyze him accomplish his objective?
the present population of the different cities and a. Project Method
municipalities of the National Capital Region for b. Laboratory Method
the last five years. How can they best present c. Problem Method
their analysis? d. Expository Method
a. By means of a table
b. By looking for a pattern 13. In Math, Teacher G presents various
c. By means of a graph examples of plane figures to her class.
d. By guessing and checking Afterwards, she asks the students to give
definition of each. What method did she use?
7. There are several reasons why problem-solving a. Inductive
is taught in Math. Which is the LEAST important? b. Laboratory
a. It is the main goal for the study of Math c. Deductive
b. It provides the content in which concepts and d. Expository
skills are learned and applied
c. It provides an opportunity to develop critical and 14. Teaching Tinikling to I-Maliksi becomes
analytical thinking possible through the use of?
d. It provides pupils an opportunity to relate Math a. Inductive Method
in the real world b. Expository Method
c. Demonstration Method
d. Laboratory Method
8. Teacher D teaches in a remote high school
where newspapers are delivered irregularly. 15. What is the implication of using a method that
Knowing the importance of keeping the students focuses on the why rather than the how?
aware of current affairs, what is probably the best a. There is best method
way to keep the students updated? b. Typical one will be good for any subject
a. Gather back issues of newspapers and let c. These methods should be standardized for
pupils compile them. different subjects.
b. Urge the pupils to listen to stories circulating in d. Teaching methods should favor inquiry and
the community. problem solving.
c. Encourage the pupils to listen to daily
broadcast from a transistor radio. 16. When using problem solving method, the
d. The teacher should try all available means to teacher can
get the newspaper delivered to the school a. Set up the problem
b. Test the conclusion
97
c. Propose ways of obtaining the needed data revision.
d. Help the learners define what is it to be solved b. A good daily lesson plan ensures a better
discussion.
17. Which of the following characterizes a well- c. Students should never see a teacher using a
motivated lesson? lesson plan.
a. The class is quiet. d. All teachers regardless of their experience
b. The children have something to do. should have daily lesson plan.
c. The teacher can leave the pupils
d. There are varied procedures and activities 24. In Music, Teacher 1 wants to teach the class
undertaken by the pupils. how to play the piano in the Key of C. Which of
the following should be his objective?
18. Learners must be developed not only in the a. To play the piano in the key of C chords
cognitive, psychomotor but also in the affective b. To improve playing the piano in the key of C
aspect. Why is development of the latter also c. To interpret property of chords of Key of C in
important? the piano
a. It helps them develop a sound value system. d. To exhibit excellent playing of piano in the key
b. Their actions are dominated by their feelings. of C
c. It helps them develop an adequate knowledge
of good actions. 25. When using instructional material, what
d. Awareness of the consequences of their action should the teacher primarily consider?
is sharpened. a. The material must be new and skillfully made.
b. It must be suited to the lesson objective.
19. Which of the following attributes characterizes c. The material must stimulate and maintain
a learner who is yet to develop the concept? students' interest
a. The learner can identify the attributes of the d. It must be updated and relevant to Filipino
concept. setting.
b. The learner can summarize the ideas shared
about the concept. Answers: 1B 2D 3C 4A 5A 6C 7A 8C 9A 10D
c. The learner can distinguish examples from non- 11C 12B 13A 14C 15D 16D 17D 18A 19A 20D
examples. 21C 22A 23C 24A 25B
d. The learner gets a failing grade in the tests
given after the concept has been discussed.
1. Of goals of education, which relates to the
20. The strategy which makes use of the old strengthening of our society’s sense of belonging
concept of &each-one-teach-one& of the sixty's is and identity?
similar to? A. Autonomy
a. Peer learning B. Enculturation
b. Independent learning C. Moral character
c. Partner learning D. Citizenship
d. Cooperative learning
2. Of the following interventions, which is directly
21. Which part of the lesson does the learner give aimed at responding to the transitional gap
a synthesis of the things learned? between academic achievement and
a. Motivation employment?
b. Application A. Deregulation of tuition fees
c. Evaluation B. Voluntary accreditation of schools
d. Generalization C. School networking with business and industry
D. Identification of centers of excellence
22. Educational objectives are arranged from
simple to complex. Why is this? 3. Teacher Ernie makes sure that he covers the
a. Each level is built upon and assumes essential subject content, while treating them
acquisition of skills from the previous level. sufficiency or in-depth. What guiding principle is
b. Objectives are broad and value-laden he following for lesson preparation?
statements that lead to the philosophy of A. Balance
education. B. Feasibility
c. Be idealistic and ambitious to begin with C. Significance
grandiose scheme for using taxonomy in all D. Self-sufficiency
levels.
d. These are guidelines to be taught and learned 4. As preventive measure for classroom
where teachers and students evaluate learning. discipline, the teacher may restructure the
program. How is this not done?
23. Which of the following is NOT true? A. Reteach lessons difficult to understand
a. Lesson plan should be in constant state of B. Remove tension level before proceeding with
98
lesson well to the school environment. What should be
C. Modify lesson the policy for assigning said physical facilities?
D. Skip whole lesson unit altogether A. Needs of student’s basis
B. Position ranking basis
5. Teacher Jose talks to students about their C. First-come, first-served basis
interests, what they did over the weekend, their D. Service seniority basis
progress in school work, etc. What positive
approach to classroom management did Teacher 12. According to the guidelines on punishment,
Jose apply? what does it mean if the teacher should give the
A. Trusting students student the benefit of the doubt?
B. Being fair and consistent A. Make sure facts are right before punishing
C. Expressing interest for students B. Doubt the incident really happened
D. Being positive C. Don’t punish and doubt effectiveness of
punishment
D. Get the side of the students when punishing
. Of the following, which is a non-threatening style
of disciplining unruly students? 13. Of subcategories of movement behavior, what
A. Stand under heat of the sun is happening when the teacher ends an activity
B. Do push-ups abruptly?
C. Send to guidance office A. Thrust
D. Squat before the class B. Truncation
C. Stimulus-bounded
7. Among mistaken goals in the Acceptance D. Flip-flop
Approach to discipline, what happens when
students seek to hurt others to make up being for 14. Which of the following is true of a democratic
being hurt or rejected? classroom?
A. Revenge seeking A. Teacher acts as firm decision maker
B. Power seeking B. Students decide what and how to learn
C. Withdrawal C. Consultation and dialogue
D. Attention getting D. Suggestions are sent to higher officials for
decisions
8. Among mistaken goals in the Acceptance
Approach to discipline, what happens when 15. Which of the following steps should be
students are not getting the recognition they completed first in planning an achievement test?
desire, continually seek help, and refuse to work A. Set up a table of specifications
unless the teacher hovers over them? B. Define the instructional objective
A. Attention getting C. Select the types of test items to use
B. Withdrawal D. Decide on the length of the test
C. Revenge seeking
D. Power seeking 16. Teacher Francis organized a structured class
discussion with two opposing sides and assigned
9. Among mistaken goals in the Acceptance speakers on the issue of contraceptives. What
Approach to discipline, what happens when was this kind of class?
students feel helpless and rejected so that they A. Brainstorming
remove themselves rather than confront the B. Debate
situation? C. panel discussion
A. Withdrawal D. symposium
B. Power seeking
C. Revenge seeking 17. Problems of discipline (misdeeds, lapses,
D. Attention getting minor offenses) can be reduced through
enthusiasm which can be matched by the
10. To manage behavior, the teacher needs to be enthusiasm of learners. What can draw a laugh
able to identify the mistaken goals of students. and reduce tension from all?
What is the hidden goal of students who become A. Verbal reinforces
violent? B. Nonverbal gestures
A. Goal is to seek power C. Dialogues
B. Goal is to get attention D. Sense of humor
C. Goal is to isolate self
D. Goal is to get revenge 18. What best describes “puwede na” mentality
vs. excellence in service/work?
11. Facilities such as classrooms, fixtures, and A. Arduous preparation
equipment can often damage the morale of new B. Resignation to mediocrity
teachers and become an obstacle for adapting C. Committed work
99
D. Striving to be the best b. A sealed aerosol can explodes when thrown into a
fire.
19. If the children are cooperatively engaged with c. A balloon expands and bursts when exposed to
the teacher in a group project the children will direct sunlight.
discipline themselves as each member of the d. A scuba divers stops at certain depths as he
group exercises ascends to the ocean’s surface.
A. Obedience to the teacher
B. Special interest 5. Three liquids A, B, C were studied in a laboratory.
C. Peer influence Liquid A was found to float over B and C. It was
D. Moral compulsion also found that liquid A flows fastest among the
three. What can be said about liquid A?
20. Which of these “combination of classes” is
a. Densest and most viscous
organized in places where the required number of
b. Densest and least viscous
pupils of the same grade levels has not met the
required number to make up a separate class c. Least dense and most viscous
thus the teacher apportions class time for d. Least dense and least viscous
instruction to every grade level within the class?
A. Mutli-grade 6. Which of the following statements represents a
B. Heterogeneous physical change?
C. Extension a. An antacid tablet forms bubbles when dissolved in
D. Homogeneous water.
b. A flashlight beam slowly gets dimmer and finally
Answers: 1D 2C 3A 4D 5C 6C 7A 8A 9A 10D dies out over time.
11A 12D 13B 14C 15B 16B 17D 18B 19C 20A c. The lawn grows thicker every day because
fertilizers were added into the soil.
d. Frozen mango juice melted when left standing at
SCIENCE1 room temperature for 30 minutes.
1. Which of the following statements best describes a 7. Which of the following best explains why farmers
hypothetical element with an electron configuration burn rice straw and hull during seasons of harvest?
of 1s22s22p63s23p5? a. Burning rice hulls and straws produce compounds
a. The hypothetical element has an atomic number of that act as repellant for pests which may damage
11. plantation.
b. The hypothetical element is a member of Group b. The smoke produced by burning rice hulls and
V, otherwise called the Nitrogen Group. straws stimulate growth and fruit bearing of trees.
c. The hypothetical element is in the fifth position in c. Rice hulls and straws are burned so that more
the p-block, along the third period of the periodic spaces will be available for planting next set of
table. crops.
d. The hypothetical element is located at the third d. Ash from burnt rice hulls and straws are rich in
position of the p-block, along the fifth period of the compounds that could neutralize acidic soil so that
periodic table. more crops will grow
2. Despite the observed diversity among organisms,
they are all made from the same set of biomolecules
composed of monomeric units except: 8. Acid rain occurs when ____________.
a. Proteins
b. Carbohydrates 8. Acid rain occurs when _________.
c. Nucleic Acids a. carbon dioxide combines with water in the
d. Lipids and Fats atmosphere.
b. phosphorus-rich water in lakes evaporates to
3. What will be formed when radium isotope, with form phosphoric acid.
88 protons and 138 neutrons undergoes alpha decay? c. sulfur released in burning fossil fuels combines
a. Radon Atom (Rn222) with 86 Protons with water in the atmosphere.
b. Francium Atom (Fr222) with 87 Protons d. excess hydrogen is released into the
c. Actinium Atom (Ac222) with 89 Protons atmosphere to produce acids.
d. Thorium Atom (Th232) with 90 Protons
9. Why is it difficult to integrate nitrogen gas from
4. Which of the following examples best illustrates the atmosphere into the nitrogen cycle of the
application of Boyle’s Law? biosphere?
a. A tire becomes harder as more air is pumped into a. Nitrogen is very abundant in the atmosphere
it. b. Living organisms quickly absorb nitrogen gas
c. Oceans quickly absorb nitrogen gas

100
d. Few organisms can directly utilize atmospheric Amihan and Habagat, internationally known as
nitrogen ________ and ________ respectively.
a. Northeast and southwest
10. Which of the following sentences about b. Trade wind and easterlies
greenhouse effect is INCORRECT? c. Southwest and northeast
a. Greenhouse gases trap heat in the atmosphere d. Westerlies and easterlies
which are returned to the earth’s surface.
b. Greenhouse effect is important in maintaining 17. Fog is a cloud with its base at or very near the
the temperature of the earth. ground. The formation of fog generally occurs
c. Greenhouse effect is due to gases that absorb after the ground has lost heat by:
the green region of light from the sun. a. Evaporation
d. Greenhouse effect increases the overall b. Convection
surface temperature of the earth. c. Conduction
d. Radiation
11. When a gardener propagates a plant by taking
cuttings, he plants his cutting in a well-watered 18. Why do we see the sun rise in the east?
soil in a plant pot. What is the most likely reason a. The earth revolves eastward.
why he may then cover the plant and pot with a b. The earth rotates from west to east.
lightly perforated polythene bag? c. We are located in the 20th meridian.
a. To reduce the water demand of the cutting. d. On the globe, we are located in the east.
b. To decrease the rate of gaseous exchange by
the plant. 19. If a voltage of 100 volts produces a current of
c. To reduce the chance of attack by pests. 5 amperes in an electrical device, what is the
d. To protect the plant from cold weather. resistance?
a. 95 Ohms
12. It is a common observation that mushrooms b. 20 Ohms
thrive few days after lightning strikes. Which c. 105 Ohms
among the following biogeochemical cycles is d. 500 Ohms
involved in this process?
a. Nitrogen cycle 20. Which of the following best differentiates an
b. Carbon-Oxygen cycle earthquake's intensity from its magnitude?
c. Phosphorus cycle a. Intensity describes 'the depth from which the
d. Sulfur cycle earthquake originated’ while magnitude refers to
‘the energy of the earthquake’.
13. Which of the following factors contribute to an b. Intensity cannot be measured while magnitude
increase in human population? can be measured using a seismograph.
I. Immigration II. Emigration c. Intensity refers to the strength of the quake
III. Natality IV. Mortality while magnitude refers to the degree of
a. III and IV destruction it caused at the epicenter.
b. II and III d. Intensity is a measure of how much damage an
c. I only earthquake cause at the surface while magnitude
d. I and III is the strength of the quake.

14. Shown below is a simple food web in a grassy 21. Comparing the speed of sound in liquids,
community. The arrow symbol means eaten by. gases, and solids, the speed of sound is usually
What would happen if all snakes are killed? lowest in ____ and highest in ____.
Grass>Grasshopper>Frog>Snake a. solids, gases
Bird>Snake b. gases, liquids
a. Grass population would increase. c. liquids, solids
b. Grasshopper population would increase. d. gases, solids
c. Bird and frog populations would increase.
d. Grasshopper and bird populations would 22. Which has a greater density, a lake full of
increase. water or a cupful of water?
a. The cup full of water
15. Which of the following shows mechanical b. The lake full of water
weathering of rocks? c. Not enough information
a. formation of caverns d. They have the same density
b. acids dissolves rocks
c. freezing water between rock particles 23. A stainless steel spoon feels colder than a
d. iron in rocks combine with oxygen plastic spoon because stainless steel
a. absorbs less heat from the hand than plastic
16. PAGASA announces the approach of the does
seasonal winds. The familiar names used are b. is really colder than plastic
101
c. has a lower temperature than plastic 11A 12A 13D 14C 15C 16A 17D 18B 19B 20D
d. conducts heat away from the hand faster than 21D 22D 23D 24B 25B 26B 27D 28A 29C 30B
plastic does
SOCIAL DIMENSIONS
24. Why is it NOT advisable to repeatedly open
the door of a refrigerator? 1. Which of the following conditions manifests
a. It will loosen the hinges of the refrigerator's trend of globalization?
door a. Establishment of stronger boundaries between
b. Leads to wastage in electrical energy. and among nations.
c. Repeated opening introduces bacteria in to the b. Increased awareness on the importance of
refrigerator. national cultures and traditions.
d. The warm air outside lowers the temperature c. Less and less impact of human activity on the
inside thus making the refrigeration less-efficient. planet earth.
d. The incorporation of local and national
25. What kind of mirror is used in cars to give the economies into a worldwide global economy.
driver a wider area and smaller image of the traffic
behind him/her? 2. According to the Delors report, there are a
a. Double concave number of main tensions central to the problems
b. Convex of the twenty first century that we need to
c. Plane overcome. One of them is the challenge to an
d. Concave individual how he or she can adapt to the
changing world without forgetting or turning
26. Why do we hear thunder some seconds after his/her back from the past. What kind of tension
seeing lightning? or conflict is manifested in this situation?
a. Light appears brighter in the sky. a. Tension between tradition and modernity
b. Light travels faster than sound. b. Tension between the global and the local
c. Sound travels 1.331 m/s c. Tension between the universal and the
d. Sound is released later actually individual
d. Tension between long term and short term
27. Water has a higher specific heat than iron. considerations
What does this mean?
a. Water is hotter than iron 3. Which of the following features represents the
b. Water heats more rapidly than iron new paradigm shift in education?
c. Water is more dense than iron a. Traditional pedagogies
d. Water heats more slowly than iron b. Lifelong education for all
c. Rigid subject matter boundaries
28. If a colorblind man marries a woman who has d. Knowledge as the only learning outcome
normal vision and no history of the disease, it is
most probable that all of their : 4. What is the measure of relevance in education?
a. daughters will be carriers a. Democratization of access
b. daughters will be colorblind b. Functionality and meaningfulness
c. sons will be carriers c. Ability to sustain education through the future
d. sons will be colorblind d. Excellence and effectiveness

29. Albino corn seedlings may grow several 5. What is the concern of Multicultural Education?
inches tall. However, they will eventually die, a. Anticipating the future and imagining possible
primarily because and probable futures.
a. direct sunlight will destroy their cells b. Gender equality and harnessing of the role of
b. they lack adequate root system women in development.
c. they cannot produce their own food c. Promoting care for the environment and
d. they cannot obtain carbon dioxide building a global culture of ecological
responsibility.
30. Mimosa pudica, locally known as Makahiya is d. The exploration of concepts of cultural diversity,
called as such because its leaflets tend to close similarities and prejudices to promote cultural
when touched. This organismal response to a understanding.
given stimuli is called:
a. Hydrotropism 6. Which of the following may be considered an
b.Thigmotropism economic impact of globalization on education?
c. Geotropism a. Increasing commercialization of education and
d. Phototropism the corporate takeover of education
b. Weakening of the notion of the &citizen& as a
Answers: 1C 2D 3A 4D 5D 6D 7D 8C 9D 10C unified and unifying concept.

102
c. New technologies of information and 11. What educational approach/perspective
communication creates new approaches to recognizes the knowledge and experience of
learning women, racial groups and ethnic groups as being
d. Reduction of state and government support and just, as valid and relevant as the knowledge of
subsidy for education dominant groups in mainstream academic
discourse?
7. Which of the following aptly describes Marshall a. Transformative education
Mcluhans' concept of global village? b. Multicultural education
a. The idea that because of rapid globalization c. Inclusive education
and development in technology, the world has d. Global education
become one global village where increased
diversity and difference among people has 12. How does the notion of cultural relativity and
become more pronounced than ever. variability affect the teaching-learning processes
b. Rapid integration of the planet through media in school?
and technology where events in one part of the a. The students' varied cultural background will in
world could be experienced from other parts in now way affect the way they will learn the lessons
real-time, similar to what human experience was in school.
like when we lived in small villages. b. The students can readily adjust to the way the
c. Global Village is the kind of global world we are teacher initiates learning in school because
experiencing, characterized by fundamentalism, children are adaptable beings no matter what
apathy and conflict brought about by clashes of culture they come from.
cultures. c. The child's cultural background influences the
d. People's cultural and religious identities will be children's way of interpreting and viewing the
the primary source of conflict in the post-Cold War world; hence, teachers must consider the
world as evidenced by the conflict between children's world view when teaching.
fundamentalist Muslims and the western world. d. The teacher should be wary of differing cultural
points of view and must make sure that students
will see things the same way.
8. When planning her lessons and units, Mrs.
Jones is careful to include books and resources 13. Which among the following is the focus of
from a variety of cultures and ethnic groups. What Civic Education?
kind of education is this? a. Promote understanding of human rights,
a. Multilingual education concepts and values to enable learners to
b. Transformative education comprehend and transform conditions which give
c. Multicultural education rise to human rights violations.
d. Gender free education b. Learning for effective participation in
democratic and development processes at both
9. Which of the following is NOT a characteristic local and national levels.
of globalization? c. Foster a vision of education for sustainable
a. Stretching a social, political and economic development and care for the environment.
activities across political frontiers, regions and d. Empower people with the skills, attitudes and
continents. knowledge to build a peaceful world based on
b. The growing magnitude of interconnectedness justice and human rights.
and flows of trade, investment and migration.
c. A speeding up of global interactions and 14. Which of the following initiatives would NOT
processes through worldwide systems of help a school address diversity?
transportation and communication. a. Using ability grouping
d. The expansion of economic protectionism and b. Using cooperative learning
isolation of poor countries. c. Working with neighborhood groups
d. Using culturally-relevant teaching methods
10. Which of the following illustrates the major
paradigm shift in education in the 21st century? 15. If the teacher is emphasizing the development
a. Shift from rigid subject matter to a more of the learner's competency to transform
interdisciplinary and multidisciplinary pedagogical knowledge into innovations and job-creation, what
approach. pillar of education does s/he is actually
b. Shift from values education and emotional promoting?
learning to knowledge dominated curriculum a. Learning to Know
c. From contextualized themes generated from b. Learning to Do
global and local realities to pre-organized subject c. Learning to Live Together
matter d. Learning to Be
d. From more flexible learning styles to a
prescribed pedagogy 16.What pillar of education which emphasizes
learning to be human, through acquisition of
103
knowledge, skills and values conducive to Peace and Justice for the 21st Century. What is
personality development? the aim of the Agenda's Global Campaign for
a. Learning to Know Peace Education?
b. Learning to Do a. Helps coordinate local initiatives and unite
c. Learning to Live Together educators in the common practice of educating for
d. Learning to Be a culture of peace.
b. Supports the UN Decade for a Culture of Peace
17. A class is composed of students coming from and Non-violence for the Children of the World
several ethnic communities including Muslims and and to introduce peace and human rights
lumads. They seem to have difficulty education into all educational institutions.
understanding each others' behavior and points of c. Brings together multiple traditions of pedagogy,
view. What should the teacher do? theories of education, and international initiatives
a. Introduce multicuturalism in the class and for the advancement of total human development
provide activities for practice. and care for the environment through learning.
b. Threaten the students that if there are students d. Serves to enhance learning across subjects like
who do not behave and tolerant of their conflict resolution initiatives.
classmates, s/he will be dropped from class.
c. Inform students that they will all be learning 22. The impact of conflict on children whether as
new ways of thinking and behaving in this class, victims of war or child soldiers has been brought
so they might as well leave their cultural to world attention through media, international
idiosyncrasies at home. organizations and eye witness accounts. What is
d. Assign bright students to monitor and control the best thing to do to help children affected by
behavior of poor students. conflict?
a. Employ education to regain parts of a lost
18. Which of the following qualities should be children and to facilitate the experiences that
developed by the pillar, Learning to Live support healthy social, emotional and intellectual
Together? growth and development
a. Strong appreciation of the diversity of the b. Provide employment opportunity for them as
human race well as their parents to attain financial
b. Readiness to take risks and resolve or manage independence
conflicts c. Offer them to migrate in neighboring country as
c. Scientific spirit and an inquiring mind foreign refugees
d. Complete fulfillment of humans, in all the d. Secure their safety by imposing strict curfew
richness of his/her personality hours

19. Which of the following statements about 23. The United Nations is committed to address
Gender is correct? climate through mitigation and adaptation. Which
a. Gender is biologically determined. of the following is the best way of addressing the
b. Gender is socially and culturally-constructed. issue?
c. Gender roles are the same in all societies. a. Deepen strategic and operational collaboration
d. Gender is an ascribed status in society. with international and regional organizations,
including international financial institutions and
20. UNICEF and UNESCO are two key UN regional development banks, and other
agencies which are particularly active advocates stakeholders.
of education for peace. Which of the following is b. Developing a policy framework that identifies
not supported by UNESCO in promoting peace in basic elements needed to prevent human rights
the schools? violations.
a. Uphold children's basic rights as outlined in the c. Facilitate and execute agreements on reducing
Convention on the Rights of the Child (CRC) emissions from deforestation and forest
b. Develop a climate that models peaceful and degradation to protect forests and sustain the
respectful behavior among all members of the livelihoods of the people who depend on them.
learning community d. Enhancing collaboration among humanitarian
c. Demonstrate the principles of equality and non- organizations, particularly from the global South,
discrimination in administrative policies at the local, national and regional levels, to
d. Enable the teachers to stress peace-making in strengthen community resilience and emergency
social studies classroom only when necessary response, and establishing a monitoring system to
assess progress on the implementation of
21. One way to advance peace education is preparedness measures.
through partnerships of various non-governmental
organizations, education institutions, United 24. Why are educational environments very
Nations specialized bodies which link ideals of crucial to peace education?
peace with research and practice. One such a. The social, cultural, economic and political
significant examples is the Hague Agenda for contexts in which educators work shape the
104
specific content and methods they choose for focuses on voc-tech relevant to people-centered
peace education. human development?
b. The variety of different educational settings a. Learning to Know
from rural to urban, school-based to community b. Learning to Do
and within the formal curricula or non-formal c. Learning to Live Together
popular education projects are relevant to peace d. Learning to Be
education.
c. Many teachers infuse peace education into 5. The rapid traversing of ideas, attitudes and
traditional academic subjects such as literature, values across national borders that generally
math, science, history, language, civics and the leads to an interconnectedness and interaction
arts. between peoples of diverse cultures and ways of
d. All of the above life. What is being referred to?
a. Cultural Globalization
25. What is celebrated every December 10? b. Fundamentalism
a. Mother Language day c. Multiculturalism
b. Human Rights Day d. Clash of civilization
c. Earth's Day
d. International Day of Tolerance 6. Which is considered a political impact of
globalization?
Answers: 1D 2B 3B 4B 5D 6A 7A 8C 9D 10A a. Changing role of education in terms of
11C 12D 13B 14A 15B 16D 17A 18A 19B 20D preparing students for the world of work
21B 22A 23C 24D 25B b. The threat to the autonomy of national
educational systems by globalization.
c. Reforms in education as lifelong education
d. Branding, globalization and learning to be
1. What kind of tension is referred to when people consumers
prefer to have quick answers and ready solution
to many problems even if its calls for a patient, 7. What United Nation Decade are we celebrating
concerted, negotiated strategy of reform? for 2005-2014?
a. Tension between modernity and tradition a. Educating for Culture of Peace
b. Tension between long term and short term b. Educating for International Understanding
considerations c. Educating for Sustainable Development
c. Tension between spiritual and material d. Promoting the Rights of the Elderly
d. Tension between individual and universal

2. In what strands of the four pillars of education 8. With the growing competition brought about by
implies a shift from skill to competence, or a mix globalization, what is preferred by most employers
of higher-order skills specific to each individual? in hiring their employees?
a. Learning to Know a. Flexible
b. Learning to Do b. Selective
c. Learning to Live Together c. Quick
c. Learning to Be d. None of the above

3. Which of the following is NOT true about the 9. Which of the following characteristics does
Four Pilalrs of Learning? NOT describe contextualized learning as a major
a. The pillars of learning stress the goal of paradigm shift in education?
contributing to social cohesion, intercultural and a. From limited access to time-bound and space
international understanding, peaceful interchange, limited education, to borderless education, lifelong
and harmony. learning for all in a learning society.
b. The Pillars of Learning imply a shift from b. From traditional pedagogies to more modern
schooling to learning throughout life by &learning strategies of teaching and learning.
how to learn& c. From knowledge limited to the local scene to
c. The pillars of learning stress the importance of the globalized knowledge, values, attitudes, and
closer linkage between education and the world of skills interfaced with local wisdom.
work. d. Pre-organized subject matter to localized
d. The Pillars of Learning adheres to the themes generated from the global realities and
instrumental and purely academic view of the cultural relevant, meaningful and useful to
education that focuses on the achievement of learner.
specific aims of education such as economic
productivity. 10. What current current trend in education
focuses on the study of the basic concepts,
4. What pillar of education of J. Delors (UNESCO) beliefs and values underlying our democratic
political community and constitutional order?
105
a. Civic education 17. What is the kind of education that emphasizes
b. Development education human-earth relationships and fosters a vision of
c. Peace education education for sustainable development to build a
c. Multicultural education global culture of ecological responsibility?
a. Human Rights Education
11. Which of the following is the first target of the b. Development Education
Millennium Development Goals (MDG's) c. Environmental Education
formulated by member states of the UN in d. Global Education
September 2000?
a. Reduce child mortality 18. Which of the following is NOT a benefit of
b. Eradicate extreme poverty and hunger multicultural education?
c. Reduce death due to HIV/AIDS and malaria a. Multicultural education increases positive
d. Achieve universal access to primary education relationships through achievement of common
goals, respect, appreciation and commitment to
12. Which among the following statements about equality among the teachers and students.
Human Rights Education (HRE) is correct? b. Multicultural education decreases stereotyping
a. HRE is more of the responsibilities of the state and prejudice through direct contact and
to implement human rights law rather than the interaction among diverse individuals.
protection of the rights holders c. Multicultural education promotes independence
b. HRE should focus more on rights based on of various ethnic groups in development and
&law in books&, rather than &law in real-life&. supports fragmented view of the world.
c. HRE needs to focus on the values, principles, d. Multicultural education renews vitality of society
and standards and human rights and how they through the richness of the different cultures of its
can be translated into day-to-day actions members and fosters development.
d. Human Rights Standards vary from society to
society and HRE therefore should also vary in 19. Which of the following is NOT one of the
terms of approaches and methods benefits of social media?
a. Mass media decreases prejudice and
13. What is the implication and globalization to the discrimination.
practice and experience of education? b. Mass media enriches the educational
a. Increase of state and government support and programs.
subsidy for education c. Mass media increases student's exposure to
b. Commodification and the corporate takeover of diversity.
education d. Mass media helps provoke discussion of
c. Greater autonomy of national educational current issues.
systems
d. Delocalization of technologies and orientations 20. Which among the following rights manifests
in education rule of law and good governance?
a. Right to education
14. Which of the following skills corresponds to b. Right to environment protection
the Fourth Pillar of Learning, &Learning to live c. Right of participation
together&? d. Right to work
a. Empathy and cooperative social behavior
b. Personal commitment and sense of 21. Which among the following is NOT a core
responsibility principle of human rights?
c. Adaptability to change in the world of work a. Human dignity
d. Reasoning and problem solving skills b. Non-discrimination
c. Universality
15. Which of the following is NOT a characteristic d. Independency
of Multicultural education?
a. Personality empowering 22. How are human rights principles reflected in
b. Socially transformative the activities of national and local governments?
c. Pedagogically humanistic a. Legislating laws to include rights education in
d. Culturally discriminating all levels of schooling
b. Organizing local exhibit or event to highlight the
16. What is the character of education that children's talents and local products
manifests democratization of access and c. Asking the community leaders to volunteer in
inclusivity? the construction of a barangay hall
a. Relevance d. Lobbying to the UN High Commission for
b. Sustainability Human Rights to allocate higher budget for
c. Quality Philippines' Commission on Human Rights.
d. Equity
23. Which of the following could be a reason to
106
justify peace education as a series of &teaching 4. In what instance is the Filipino double-standard
encounters& or teaching-learning process? morality shown?
a. Desire for peace a. A couple brings their sick child to the doctor then
b. Nonviolent alternatives for managing conflict later to the espiritista
c. Skills for critical analysis of structural b. Young parents bring up their children in a manner
arrangements that produce and legitimize different from how they were brought up
injustice and inequality c. Illiterate parents are eager to send their children to
d. All of the above school even if they themselves did not go to school
d. A married man who flirts with someone else other
24. Which of the following is accurate in regard to than his wife seems acceptable but a married woman
working with parents in diverse classrooms?
who flirts with another man is condemned.
a. The parent's culture is important, but should not
influence their children's education.
5. Which part of Asia does the Arabian peninsula
b. Teachers should demonstrate their &expertise&
to parents to show they know best. occupy?
c. Teachers should strive to use a variety of ways a. Northwest
to keep parents informed, including parents who b. Southeast
cannot speak English or Filipino.l c. Southwest
d. The importance of the family's influence on d. Northeast
children's education has diminished over the past 6. The important factors which have contributed to
few years. the weakness in the internalization of desirable
values is the?
25. Which of the following is NOT a guiding a. Lack of models among the very people expected
statement of peace education? to exemplify these values
a. Peace education teaches students what to b. Use of approaches which are mainly cognitive
think rather than how to think. rather than effective
b. Peace education employs holistic and c. Lack of follow up systems from one grade level to
participatory approach. another
c. Peace education aims not to reproduce but d. Minimum recognition and appreciation given to
transform. teachers
d. Peace builds bridges of support among key
participants.
7. To govern is to rule and the government rules by
Answers: 1B 2B 3D 4B 5A 6B 7C 8A 9A 10A 11B laws. Whose main duty is the enforcement of laws?
12C 13B 14A 15D 16D 17C 18C 19A 20C 21D
a. Police department
22A 23D 24C 25A
b. Judiciary department
c. Legislative department
SOCIAL SCIENCE
d. Executive department
8. The term that refers to the class of Filipinos who
1. What factor was a major cause of both World War
were free and independent
I and World War II?
a. Timawa
a. The spread of Marxian ideas into Europe
b. Maharlika
b. The dropping of atomic bombs
c. Aliping namamahay
c. Nationalism and national borders
d. Aliping saguiguilid
d. The rise of totalitarian fascist states
9. The Spanish expedition responsible for naming
the archipelago Filipinas
2. In which organization is the Philippines a member
a. Magellan’s expedition
to fight communist aggression?
b. Loarca expedition
a. APEC
c. Legaspi expedition
b. ASA
d. Villalobos expedition
c. UN
10. The third and last military governor of the
d. SEATO
Philippines was
a. Gen. Wesley Merritt
3. The theory that population increases by
b. Gen. Elwell Otis
geometrical ratio while the means of subsistence
c. Gen. Arthur MacArthur
increases by arithmetical ratio is attributed to?
d. Gen. Douglas MacArthur
a. Karl Marx
11. Which Katipunan member commuted from
b. Robert Malthus
Cavite to Manila to buy materials used to make
c. Emile Durkheim
ammunitions?
d. Aristotle
a. Teresa Magbanua
b. Agueda Esteban

107
c. Teodora Alonso 21. When the Filipino reformists asked for the
d. Trinidad Tecson assimilation of the Philippines by Spain, what did
12.Which economic system is based on free they ask for? For the Philippines to ____
enterprise? a. Become independent from Spain
a. Globalism b. Become a province of Spain
b. Mixed economies c. Be independent from Spain with certain conditions
c. Capitalism d. Be represented in the Spanish Cortes
d. Communism 22. Who among the Presidents changed the date of
13. How is the so-called colonial mentality our celebration of Independence day from July 4 to
manifested? June 12?
a. Cultural relativism a. Ramon Magsaysay
b. Cultural diversity b. Diosdado Macapagal
c. Xenocentrism c. Carlos Garcia
d. Ethnocentrism d. Ferdinand Marcos
14. Which is a safeguard against unfair trade 23. In which country did the Philippines participate
practices like short-weighing? in the world’s peacekeeping operations by sending
a. Total Quality Movement doctors, nurses, soldiers and police?
b. Consumerism a. Israel
c. Consumer vigilance b. East Timor
d. Substandardization c. Iraq
15. If the seven continents were arranged from d. Iran
largest to smallest, in which order does Australia 24. In which poem did Rizal write about offering
fall? one’s life for one’s country?
a. 4th a. A La Juventud Filipina
b. 5th b. Song of Maria Clara
c. 6th c. Sa Aking mga Kabata
8. 7th d. Mi Ultimo Adios
16. In which continent can we find stormy Cape 25. As an effect of our geography, in which of the
Horn which is known as the graveyard of ships and following island/s do people travel mostly by water?
sailors? a. Luzon
a. Africa b. Mindanao
b. Australia c. Visayas
c. South America d. Visayas and Mindanao
d. Asia 26. As an insular country, to which principle does
17. Which led to the creation of Pakistan as a nation the Philippines adhere when it comes to territorial
in 1947? Religious differences between boundary?
a. Hindus and Christians a. Two hundred nautical miles of the country’s coast
b. Christians and Muslims b. Three hundred fifty nautical miles from shore
c. Hindus and Buddhists c. Three-mile territorial limit
d. Hindus and Muslims d. Archipelagic doctrine
18. Which is common to Sun Yat Sen and Mahatma 27. Which part/s of the Visayas has/have
Gandhi? comparatively more excessively moist climate and
a. Promoted a society ruled by religious leaders limited arable lands?
b. Rejected violence as a way to political power a. Western
c. Led a successful nationalistic movement in their b. Eastern
respective countries c. Eastern and Central
d. Supported Marxist philosophy to change existing d. Central and Western
governments 28. Which follows Pres. Garcia’s “Filipino First
19. Nebuchadnezzar was to the Babylonian Empire Policy”? Filipinos
as Asoka was to the ______ Empire. a. Should buy and consume Filipino products only
a. Roman b. Should not contribute to the brain drain problem
b. Gupta c. Should be selective in the entry in the entry of
c. Greek foreign professionals in the country
d. Maurya d. Were to be given first preference in all matters
20. With the opposition of the parity rights in mind, related to the economic development of the country
who does NOT belong to the group? 29. Aside from the Philippines, which countries
a. Claro M. Recto claim part of the Spratly islands in the South China
b. Jose Laurel Sea?
c. Manuel Roxas a. China, Thailand, Brunei, Malaysia
d. Pedro Taruc b. China, Taiwan, Vietnam, Malaysia

108
c. Indonesia, China, Taiwan, Malaysia c. Provisional
d. China, Thailand, Brunei, Malaysia d. Substitute
30. The following are our constitutional rights
EXCEPT to 7. Which of the following rights is intended for
a. Free access to legal assistance which shall not be parents under Education Act of 1982?
denied due to poverty a. The right to academic freedom
b. Have access to all records of the government b. The right to privacy of communication
c. Be presumed innocent until proven guilty c. The right to seek redress of grievance
d. Form association and labor unions d. The right to full access to the evidence of the
case
Answers: 1D 2D 3B 4D 5C 6A 7D 8A 9D 10C
11B 12C 13C 14C 15D 16C 17D 18B 19D 20C
21B 22B 23C 24D 25C 26D 27B 28D 29B 30B 8. What can help achieve relevant quality
education?
TEACHING PROFESSION a. Strong curriculum
b. Competent instruction
c. School-community relations
1. Which of the following emphasizes the right of
d. Competent administrator
citizens to quality education?
a. The basic education level
9. Which of the following provisions under the
b. Tertiary level
Magna Carta for Public School Teachers will most
c. The graduate level
likely promote teachers' welfare and defend their
d. All levels
interests?
a. Be promoted in rank and salary
2. Which educational level/s provide/s for free and
b. Regulate their social involvement
compulsory education as stipulated in Article IV,
c. Undergo and participate in professional
Section 2 of the Philippine Constitution?
development
a. Elementary level
d. Establish, join and maintain professional and
b. Secondary level
self-regulation organizations
c. Elementary and secondary levels
d. Tertiary level
10. What does &teachers are persons in
authority& imply?
3. Who among the following is in the category of
a. Teachers cannot be charged.
non-academic personnel as provided for under
b. No person can assault a teacher.
Education Act of 1982?
c. Teachers have immunity from arrest.
a. Guidance counselors
d. Decisions made by teachers are deemed right.
b. School principal
c. School nurse
11. Who among the following characterizes a
d. School librarian
professional teacher?
a. An education graduate who received honors
4. How is gradual progression of teacher's salary
b. A teacher who has taught for at least six years
from minimum to maximum done?
c. A teacher who has attended national seminars
a. Regular increment every year
on teaching
b. Increment after ten years of service
d. A teacher who qualifies for a permanent
c. Regular increment every 3 years
position under RA 4670
d. Increment after five years
12. Who are covered by RA 4670?
5. Which of the following is NOT recognized by
a. Teachers in all levels
the Magna Carta for Public School Teachers?
b. Teachers in all public elementary schools
a. Quality education depends primarily on the
c. Teachers in both public and private schools
quality of socio-economic status of teachers.
d. Teachers in public elementary and secondary
b. Advancement in education depends on the
schools
teachers' qualifications and ability.
c. Education is an essential factor in the economic
13. Teacher B has been in active service for 10
growth of the nation.
years when he decided to pursue higher studies.
d. Education is development and vice-versa.
Under RA 4670, what kind of leave of absence
can s/he avail of?
6. What appointment can be given to Teacher A
a. Indefinite leave
who possesses the minimum qualifications but
b. Scholarship leave
lacks the appropriate but lacks the appropriate
c. Study leave
civil service eligibility?
d. Vacation leave
a. Contractual basis
b. Permanent
14. When can teachers be required to work on
109
assignment not related to their duties? 21. Teacher J discusses conflicts between
a. When on probation warring groups in Mindanao. Which pillar should
b. When found inefficient he stress more?
c. When lacking in educational qualifications a. Learning to be
d. When compensated under existing laws b. Learning to live together
c. Learning to do
15. Teacher C has been teaching 7 straight years d. Learning to know
and therefore qualities for a study leave with pay
for one year. Should she pursue it, how much pay 22. Teacher K teaches in a public school in her
is she entitled to receive? locality. Due to teacher shortage, her classroom
a. 50% of monthly salary teaching starts from 6 am and ends at 3 pm. Is
b. 60% of monthly salary the assignment given her just?
c. 70% of monthly salary a. Yes, the situation demands that she render
d. 100% monthly salary longer teaching hours.
b. Yes, as long as she signs a conforme letter to
16. Which of the following laws strengthens that effect.
teacher education in the Philippines through the c. No, rendering longer teaching hours would
establishment of centers of excellence? make the teacher tired and exhausted.
a. RA 7722 d. No, Magna Carta for Public School Teachers
b. RA 7784 states that in the exigencies of service, any
c. RA 7796 teacher may be required to render more than six
d. RA 7834 hours and n ot more than eight hours of actual
classroom teaching a day.
17. What does free public secondary educational
under the law mean? 23. Teacher L, a graduate of BSEd with majorship
a. Right of every student to enter public in Mathematics teaches in a national high school
secondary schools in her province. Since she has been rated
b. Free from being screened to enter pubic outstanding in her performance, can she be
secondary schools exempted from taking the LET?
c. Free from payment of school fees identifies and a. Yes, that is a privilege that must be given to
authorized by law teachers whose performance is outstanding.
d. Free from payment of tuition and other fees for b. Yes, if approved by PRC.
students enrolled in public secondary schools c. No, RA 7836 states that no person shall
practice or offer to practice the teaching
18. Teacher D is assigned in a rural area; profession in the Philippines or be appointed as
Teacher E in a depressed community; Teacher F teacher to any position calling for a teaching
in a hazardous area; and Teacher G in a place position without having previously obtained a valid
where standard of living is high. Who is entitled to certificate and a valid license from the
a hardship allowance? Commission.
a. Teacher D d. No, professional license is required of all
b. Teacher E teachers regardless of age and teaching
c. Teacher F performance.
d. Teacher G
24. Which of the following statements is NOT true
19. Teacher H contracted an illness that required about the Code of Ethics for Professional
rest for more than one year. Which leave should Teachers?
she apply for? a. The teacher must select which information to
a. Sick leave keep confidential
b. Personal leave b. The teacher must demonstrate full commitment
c. Vacation leave and devotion to duty
d. Indefinite leave c. The teacher must manifest pride in the nobility
of the teaching profession
20. A school personnel can avail of free legal d. The teacher must make no prejudice or
service under certain circumstances. Principal I discrimination against any learner
was accused of maligning her neighbor. Is
Principal I entitled to the said service? 25. Which of the following could be the reason for
a. Yes, she should defend herself. the teacher's suspension from the practice of the
b. No, if funds are not available. teaching profession?
c. No, it might bring some disagreements in a. Immoral, unprofessional or dishonorable
school conduct
d. No, the case is not related to her professional b. Observing proper procedures in obtaining a
duties. certificate of registration
c. Faithfulness to the code of ethical and
110
professional standards for professional teachers
d. Willingness to attend seminars, workshops, 7. What should a teacher do when he/she falls in
conferences and the like or the continuing love with his/her student?
education program prescribed by the Board and a. Court the student at home.
the Commission. b. Propose and marry the student.
c. Wait till the student is no longer under his/her
Answers: 1D 2C 3C 4C 5D 6C 7C 8B 9D 10B tutelage.
11D 12D 13C 14D 15B 16B 17D 18C 19D 20D d. Act normally as if nothing happens and the
21B 22D 23C 24A 25A student does not exist.

8. When a Principal starts to exercise his/her


1. Ms. Sanchez, a BSE graduate, has not passed powers over making and promoting students, is
the LET yet. On what capacity can she be hired? his/her action acceptable?
a. Permanent status a. Yes, when the teacher cannot make decision
b. Emergency status on time.
c. Provisional for not less six months b. Yes, when there is abuse of judgment on the
d. Provisional for not less than one year part of the teacher.
c. No, teachers are more knowledgeable of their
2. Teacher M suffers from hypertension and student's performance.
experiences difficulty in speech. Which would be d. No, grading and promoting students are
affected if he continues teaching? exclusive functions of teachers.
a. Personality
b. Punctuality 9. Teacher R was asked by her principal to teach
c. Effectiveness pre-school class in addition to her regular grade
d. Devotion to duty one class. What will be the basis for her additional
compensation?
3. Teacher N wants to continue with her study a. Her basic salary
leave for another six months after completing a b. Performance rating
school year. Could she be allowed? c. Providing public information of their policies and
a. Yes, if her grades are excellent. procedures
b. Yes, but without compensation. d. Encouraging appreciation of government
c. No, other teachers should have the chance. agencies
d. No, study leave should not exceed one year.
10. Which of the following shows responsiveness
4. Teacher O tutors her students, who have of public officials and employees?
difficulty coping with Math, after class hours. Is a. Avoiding wastage in public funds
her act ethical? b. Formulating rules and policies regarding work
a. Yes, provided she receives jut compensation. c. Providing public information of their policies and
b. Yes, provided she does not require a fee from procedures
the parent. d. Encouraging appreciation of government
c. No, that is unfair to other students. services
d. No, she should be free after her official time.
11. Teacher S, a Science teacher has been
5. Teacher P, the English coordinator, was accused of sexual harassment by one of her
assisted by Teacher Q throughout the celebration students. What should the school principal do?
of English Week. What could Teacher P do to a. Ask the teacher to surrender to the police.
acknowledge Teacher Q's assistance? b. Tell the teacher to stop reporting to school.
a. Buy her a gift c. Advice the teacher to transfer to other school.
b. Keep quiet about the assistance received. d. Create a committee to investigate the
c. Mention formally to the principal the assistance accusation.
received.
d. Make an announcement giving due recognition 12. Teacher T receives a love letter from one of
of the assistance received. her third year high school students in Eenglish.
What should Mr. Martin do?
6. Is holding a rally to protest the delay of benefits a. Read her letter to the class.
due a person ethically acceptable? b. Let the student express her feelings through
a. Yes, when hold while on official time. letters.
b. Yes, when hold outside the official time. c. Return the letter to the student and tell her not
c. Yes, when hold with approval of the principal. to do it again.
d. Yes, when hold together with parents and d. Surrender the letter to the parent of the student.
students.
13. Mr. Nico, a Social Science teacher is
111
advocating reforms which the principal failed to Philippines?
recognize. What should the principal do? a. Mr. Santos, because of his CSC eligibility
a. Subject Mr. Nico to a disciplinary measure. certificate.
b. Just keep quiet about the behavior of Mr. Nico b. Mr. Cruz, because their credentials are both
c. Call Mr. Nico to the office and clarify things out recognized by law.
with him. c. Both of them, because their credentials are
d. Send Mr. Nico a memo requiring him to explain both recognized by law.
his behavior. d. Neither of the two because they did not take
and pass the LET.
14. Which of the following manifests
&Commitment to democracy& as explained in RA 19. What is RA 6713 also called?
6713? a. Ethical Standards for public Employees
a. Maintaining the principle of accountability. b. Code of Ethical Standards for Government
b. Committing to democratic values and ways of Officials and Employees
life. c. Code of Conduct and Ethical Standards for
c. Manifesting by deeds the supremacy of civilian Public Officials and Employees
authority over the military. d. Code of Ethical Standards and Conduct of
d. All of the above Government Officials and Employees

15. Teacher U was ordered by her principal to 20. Teacher V, a BEED graduate is preparing for
come to school on four consecutive Saturdays for the LET. Which of the following should she focus
the training of students' editorial staff of their her attention more?
school paper. Is this allowed under RA 4670? a. General Education
a. Yes, provided the teacher is compensated. b. Specialization
b. No, because it's not within the regular functions c. Professional Education
of the classroom teacher. d. General Education and Professional Education
c. Yes, because it's part of the teacher's other
duties 21. What norm of conduct is manifested by being
d. No, because it's not clearly indicated in the law loyal to the republic and to the Filipino people?
a. Professionalism
16. Dr. Velasco, a schools' division b. Nationalism and Patriotism
superintendent acted on the complaint filed by a c. Responsiveness to the public
group of parents against the alleged misconduct d. Honesty
of a particular teacher. She issued a
memorandum requiring her to take a leave of 22. Mr. Salazar, a school superintendent, filed his
absence for a week while the complaint is being statement of assets and liabilities upon assuming
heard yet. Was the action of the superintendent to office. Under what ethical standard does this
legal? practice fall?
a. Yes, because she is the superintendent. a. Divestment
b. No, because the complaint has not been heard b. Prohibited Acts and Transactions
yet. c. Statement of Assets and Liabilities
c. Yes, the superintendent has disciplinary d. System of Incentives
authority over teachers.
d. No, the superintendent has no disciplinary 23. Principal B acted on the letter of complaint
authority over teachers. received by his office 30 days after saying he was
preoccupied by more important things the past
17. A school's academic coordinator has been days. Is his reason acceptable?
found to have engaged in gambling which has a. Yes, because he has to prioritize things.
caused him to be absent most of the time. Can his b. No, RA 6713 states that public officials and
certificate of registration as a teacher be revoked? employees must act promptly on letters and
a. No, unless he's proven guilty. requests within 15 working days from receipt
b. No, because he's protected by his rights as a thereof.
teacher. c. Yes, because the letter of complaint can wait
c. Yes, because he's incompetent. and is of no urgency.
d. Yes, because habitual gambling is a d. No, the reason is simple unacceptable.
dishonorable conduct and is against the practice
of teaching. 24. Which of the following is NOT in the norms of
conduct under RA 6713?
18. Mr. Santos is a holder of a valid certificate of a. Professionalism
eligibility as a teacher issued by the Civil Service b. Justness and sincerity
Commissioner and the then DECS, while Mr. Cruz c. Commitment to public interest
is a registered professional. Who is allowed to d. Responsiveness to the private
practice the teaching profession in the
112
25. Which of the following is true about the d. Adherence to Professional Standards
teacher as a person under the Code of Ethics for
Professional Teachers? 5. In line with the principle of professional
a. Live with dignity at all times wherever he/she is competence, it is the responsibility of the
b. Serve as a model worthy of emulation
c. Place premium upon self-respect and self-
counseling team members to.
discipline
d. All of the above.
a. stand up for the basic rights
Answers: 1D 2C 3B 4B 5D 6B 7C 8B 9D 10C
11D 12C 13C 14D 15C 16C 17D 18C 19C 20D b. enhance their counseling skills
21B 22C 23B 24D 25D
c. respect the worth and dignity of the client
PRACTICE TEST FOR PROFESSIONAL EDUCATION
d. observe strictly the professional standards
1. Which stage of the counseling process does
decision making takes place? 6. What should a classroom teacher do if a learner
with a serious emotional problem seeks his/her
a. Establishing rapport assistance?

b. Focusing on concerns a. Personally handle the case

c. Acting on the concerns b. Refer the learner to the school counselor

2. Which type of test would yield significant data for c. Consult the principal to what course of action to
a region wide assessment of school performance? take

a. Aptitude d. Tell the learner’s parents to bring him/her to a


psychologist outside the school
b. Achievement
7. Which of the following best describes the
c. Evaluation individual inventory service of the guidance

d. Placement program?

3. Ms. Tapla is designing a Values Formation a. The administration of psychological test to the
Program for a group of high school students. clients

Which type of test should she use to assess the b. The means of assessing the guidance needs of the
needs of the target participants? target clients

a. Interest Test c. The process of gathering, organizing, and utilizing


pertinent data about the clients
b. Aptitude Test
8. Why should a needs assessment survey be
c. Intelligence Test undertaken before planning the school guidance
program?
d. Personality Test
a. It is integral part of program planning
4. If the school guidance program is highly academic
oriented, the _________ principle is not properly b. It provides for the total development of the
observed. target clientele

a. Holistic Development c. It ensures the relevance and meaningfulness of


the guidance program
b. Confidentiality of information
d. It contributes to the comprehensiveness of the
c. Independence in Decision Making program goals and activities

113
9. Which of the following characteristics of the 14. Which one can help student develop the habit of
classroom teacher is most instrumental in her being critical thinking?
a key member of the school guidance team?
a. Asking low level questions
a. Experience
b. A willingness to suspend judgment until sufficient
b. Personal qualities evidence is represented

c. Educational Background c. Asking convergent questions

d. Close contact with Learners d. Blind obedience to authority

10. Which of the following appraisal techniques 15. Teacher wants his pupils to master the skill in
would be most useful with a big number of desired adding unlike fractions. Which method should he
respondents? use?

a. Interview a. Unit Method

b. Observation b. Drill Method

c. Questionnaire c. Laboratory Method

d. Autobiography d. Discovery Method

11. The use of drills in the classroom is rooted on 16. In the K W L technique K stands for what the
Thorndike’s law of pupil already knows, W for he wants to know and L
for what he
a. Belongingness
a. failed to learn
b. Readiness
b. he likes to learn
c. Exercise
c. needs to learn
d. Effect
d. learned
12. Which is NOT a basic component of a lesson
plan? 17. Teacher Z shows a histogram on Filipino
population growth from 1990 to 2001. Seeing the
a. Evaluation pattern of growth, pupil is asked to tell what will
happen in the next two years. What skill are the
b. Assignment pupils asked to display?

c. Resources a. Inferring

d. Content b. Predicting

13. Teacher V teaches the pupils to find the main c. Intrapolating


idea, sequence events, find the details, and read
creatively and critically. What skills does Teacher V d. Drawing
teach?
18. Which questioning technique would be
a. Utilizing skills appropriate for inductive lessons?

b. Study skills a. Use questions requiring only memory responses

c. Comprehension skills b. Expect participation only among the more


motivated students
d. Word recognition skills
114
c. Involve students actively in the questioning 23. What is the best way to handle a situation when
process a teacher realize that her planned lesson were
inadequate?
d. As a teacher, you ask no questions
a. Send the students to the library to read related
19. Bryan has had difficulty getting the right solution topics
to a problem in algebra. Suddenly he “saw” how
solve to problem. Which of the following explains b. Tell the students to prepare for the next subject
this situation?
c. Give the students an essay for the remaining
a. Revelation period

b. Insight d. Provide more exercises on the major points of the


lesson.
c. Retention
24. After reading “The Horse and the Red Hen,
d. Memory teacher asked the students this question:

20. After a lessons on AIDS, the students are asked “What is the best thing to be according to the
to summarize the lesson in one long sentence by Horse”? Under what level of comprehension does
answering: Who; Does what?; When? Why? This this question fall?
assessment technique evaluates student’s ability to.
a. Interpretation
a. evaluate
b. Literal comprehension
b. analyze
c. Integration
c. synthesize
d. Critical evaluation
d. recall
25. Which of the following is the most important
21. In what level of objective in the affective domain contribution of Gestalt psychology to the theories of
is this objective: To contribute to the discussion by learning?
asking questions.
a. Use of multimedia approaches
a. Organization
c. Importance of the reinforcement in the learning
b. Receiving process

c. Responding b. Cognitive insight

d. Valuing d. Concept of readiness in learning

22. Which activity is best suited to summarize a 26. Which is a popular teaching learning approach
lesson at the end? found in the Philippine schools were classrooms are
generally self contained, teacher is the focal point
a. Ask students to interpret a cartoon related to the and reference, and the pupils are taught the basic
lesson. skills reading, writing, and counting?

b. Present a medical summary a. Situational approach

c. Administer a short Quiz b. Traditional approach

d. Give as homework c. Montessori approach

d. eclectic approach

115
27. Which learning principle marks the highlight of b. Placement Test
Multiple Intelligences?
c. Diagnostic Test
a. Learning is static and permanent
d. Standardized Test
b. Intelligence is not measured in one form
32. Which the following supports the idea that
c. Cognitive theory is stressed in learning teaching is a human activity?

d. People have different I.Q level a. The teacher’s job is quite enormous which
includes the selection of the instructional units,
28. The teacher provides various learning activities lesson plans and methods to be used.
and methods in her teaching so that diverse needs
and interest of learners are met. Which principle of b. Teacher provides students the facilities and
learning did she consider? necessary skills needed to develop their maximum
potential
a. Learning by doing
c. Teaching is just helping students understand how
b. Cognitive development stages they grow change and most of all the how and why
of their behavior.
c. Multiple intelligence
d. The teacher should be well acquainted on the
d. Cooperative learning nature of the learner, subject matter and
instructional material
29. A student has just learned a particular formula
to make math problem solve. She tries it with 33. Which of the following is NOT an example of a
another problem and succeeds at it. So, this student teacher’s non verbal communication?
is even more encouraged to discover easier ways to
solve math problems. Which of Thorndike’s law is a. eye contact
depicted in the situation?
b. gestures
a. Readiness
c. pauses
b. Effect
d. voice
c. Repetition
34. Good teaching is accepting responsibility for
d. Practice helping to make the journey as interesting, as
rewarding and as exciting as possible. This proves
30. Teacher Jen begins her lesson with concrete life that teaching demands?
experiences then leads the students to abstractions.
Which method did she employ? a. rest and relaxation

a. Transductive b. interest in the profession

b. Inductive c. efficiency in service

c. Deductive d. deeper commitment

d. Interactive 35. Which of the following is NOT true about the


professional qualities of a teacher?
31. Teacher B discovered that her pupils are weak in
comprehension. To further determine in which a. Seldom comes on time
particular skill(s) her pupils are weak which test
should Teacher B give? b. Provides quality instructions to the students

a. Aptitude Test

116
c. Knowledgeable of the subject matter he/she 40. A parent regarding the grades of his son
teaches confronted you. If you are the teacher, what will be
the best thing for you to do?
d. Teachers who truly love children can imbue with
the love for learning a. send them to the principal

36. Which of the following lesson plan objectives b. show all the records of the child
needs improvement?
c. compare the results of the child to other students
a. To explain the life cycle of a butterfly.
d. tell the parents that your computation is correct
b. To dramatize situation showing humility and final

c. To define terms in about evolution 41. A generally accepted principle use as a basis for
comprehensively. the whole process of teaching

d. To develop scientific attitudes and apply scientific a. method


processes of thinking.
b. technique
37. To make the lesson meaningful, systematic and
motivating, teachers’ example should be: c. strategy

a. based on higher level skills d. approach

c. interesting aided with illustrations 42. When does effective teaching takes place?

b. easy, simple and understandable a. There is interaction between teacher and


students
d. relevant to students’ experience and knowledge
b. Students perform well in the given tasks
38. What law of learning when a review of the past
lesson is conducted before the discussion of the c. Knowledge has been imparted
new lesson?
d. Desirable changes are evident in the learner’s
a. Law of exercise behavior

b. Law of Effect 43. A vehicle on the street hit a 10 year old pupil
when he requested by his teacher to leave the class
c. Law of Readiness and buy for her snacks at Jollibee. Does the teacher
become liable for the incident?
d. Law of Acceleration
a. Yes, because the incident happen within her
39. Charles dances well. He knows how something official time.
works and he fixes things without asking for help.
Based on Gardner’s theory of multiple intelligence b. Yes, because that is her responsibility
under what intelligence is she strong?
c. No, because the pupil did not watch for his safety
a. Musical
d. No, because that was an incident and it happened
b. Spatial to anyone

c. Kinesthetic 44. You found out that one of your pupils is not
attending your class for almost a month now. What
d. Intrapersonal is the most ideal for you to do?

a. Conduct a home visit

117
b. Drop her in the class d. Listen to the student and make her understand
both situations
c. Ignore the situation
49. What will happen if there is NO Code of Ethics in
d. Write a letter to his/her parents the teaching profession?

45. The parents of your pupil wanted to see the a. Teaching becomes a hit and miss profession
scores of their child in your class record.
b. Refinement of the dignity of the teaching
Can you refuse the parents to see the records of profession
their child?
c. Integrity of the teaching profession will be
a. Yes, because it’s your right elevated to a certain level.

b. Yes, because it’s your personal document d. Respect of the Human Dignity is missing

c. No, because parents have their own right 50. Most of the fresh graduates are deployed to
teach in the rural areas than in the lowland villages
d. No, because class record is a public document of the towns and cities. What is the rationale behind
this practice?
46. A teacher is a model of virtues. How can this be
shown? a. They are young and healthy

a. The teacher is hopping from one relationship to b. They are active and realistic to take risk
another
c. For them to experience trials in teaching
b. The teacher devoted her time for her job with
excellence d. For them to find future partners in life

c. The teacher touches the lives of her students 1. To come closer to the truth we need to “go back
to the things themselves.” This is the advice of the
d. The teacher seriously maintained a structure of A. behaviorists
power and trust B. phenomenologists
C. idealists
47. Which of the following practices is unethical for D. pragmatists
a teacher? 2.Student B claims: &I cannot see perfection but I long
for it. So it must be real.& Under which group can he be
a. The teacher observes mobility of her profession. classified?
A. Idealist
b. The teacher cast her vote during national and B. Empiridst
local elections C. Realist
D. Pragmatist.
c. The teacher can claim money from the services 3.Which of the following prepositions is attributed to
rendered by his colleagues Plato?
A. Truth is relative to a particular time and place.
d. a teacher organizes remedial program for B. Human beings create their own truths.
students with learning difficulty C. Learning is the discovery of truth as Iatent ideas are
brought to consciousness.
48. One of the students was attracted and fell in D. Sense perception is the most accurate guide to
love with the teacher. What is the best option the knowledge.
teachers do in his situation? 4.On whose philosophy was A. S. Neil's Summerhill, one
of the most experimental schools, based?
a. Impose disciplinary action A. Rousseau
B. Pestalozzi
c. Call the attention of the student’s parents C. Montessori
D. John Locke
b. Accept student’s feelings

118
5.As a teacher, you are a rationalist. Which among D. Ego
these will be your guiding principle? 13. Soc exhibits fear response to freely roaming dogs
A. I must teach the child that we can never have real but does not show fear when a dog is on a leash or
knowledge of anything. confined to a pen. Which conditioning process is
B. I must teAch the child to develop his mental powers illustrated
to the full. A. Generalization
C. I must teach the child so he is assured of heaven. B. Extinction
D. I must teach the child every knowledge, skill, and C. Acquisition
value that he needs for a better future. D. Discrimination
6.Teacher U teaches to his pupils that pleasure is not 14 The concepts of trust vs. maturity, autonomy vs.
the highest good. Teacher's teaching is against what self-doubt, and initiative vs. guilt are most closely
philosophy? related with the works of __________.
A. Realism A. Erikson
B. Hedonism B. Piaget
C. Epicureanism C. Freud
D. Empiricism D. Jung
7Who among the following puts more emphasis on 15. Teacher F is convinced that whenever a student
core requirements, longer school day, longer academic performs a desired behavior, provided reinforcement
year and more challenging textbooks? and soon the student will learn to perform the behavior
A. Perennialist on his own. On which principle is Teacher F's conviction
B. Essentialist based?
C. Progressivist A. Cognitivism
D. Existentialist B. Environmentalism
8Which group of philosophers maintain that &truth C. Behaviorism
exists in an objective order that is independent of the D. Constructivism
knower&? 16. In a social studies class, Teacher I presents a morally
A. Idealists ambiguous situation and asks his students what they
B. Pragmatists would do. On whose theory is Teacher I's technique
based?
C. Existentialists
A. Kohlberg
D. Realists
B. Bandura
9. You arrive at knowledge by re-thinking of latent
ideas. From whom does this thought come? C. Piaget
A. Experimentalist D. Bruner
B. Realist 17 Based on Freud's psychoanalytic theory which
component(s) of personality is (are) concerned with a
C. Idealist
sense of right and wrong?
D. Existentialist
A. Super-ego
10. As a teacher, you are a reconstructionist. Which
B. Super-ego and Ego
among these will be your guiding principle?
A. I must teach the child every knowledge, skill, and C. ld
value that he needs for a better future. D. Ego
B. I must teach the child to develop his mental 18. Which does Naom Chomsky, assert about language
learning for children?
powers to the full.
I. Young children learn and apply grammatical rules and
C. I must teach the child so he is assured of heaven.
vocabulary as they are exposed to them.
D. I must teach the child that we can never have real II. Begin formal teaching of grammatical rules to children
knowledge of anything. as early as possible.III. Do not require initial formal
11 Teacher B engages her students with information for language teaching for children.
thorough understanding for meaning and for A. I and III
competent application. Which principle governs B. II only
Teacher B's practice?
C. I only
A. Contructivist
19 Which teaching activity is founded on Bandura's
B. Gestalt Social Learning Theory?
C. Behaviorist A. Lecturing
D. Cognitivist B. Modeling
12. Which is/are the sources of man's intellectual C. Questioning
drives, according to Freud?
D. lnductive Reasoning
A. Id
20. Behavior followed by pleasant consequences will
B. Superego be be strengthened and will be more likely to occur in
C. Id and ego
119
the future. Behavior followed by unpleasant B. substance over &porma&
consequences will be weakened and will be less likely C. art over science
to be repeated in the future. Which one is explained? D.&porma& over substance
A. Freud's Psychoanalytic Theory 27 Student Z does not study at all but when the
B. Thorndike's Law of Effect Licensure Examination for Teachers (LET) comes, before
C. B. F. Skinner's Operant Conditioning Theory he takes the LET, he spends one hour or more praying
D. Bandura's Social Learning Theory for a miracle, i.e. to pass the exam. Which attitude
21. Bruner's theory on intellectual development moves towards religion or God is displayed?
from enactive to iconic and symbolic stages. In which A. Religion as fake
stage(s) are diagrams helpful to accompany verbal B. Religion as magic
information? C. Religion as authentic
A. Enactive and iconic D. Religion as real
B. Symbolic 28 During the Spanish period, what was/were the
C. Symbolic and enactive medium/media of instruction in schools?
D. Iconic A. The Vernacular
22 In a treatment for alcoholism, Ramil was made to B. English
drink an alcoholic beverage and then made to ingest a C. Spanish
drug that produces nausea. Eventually, he was D. Spanish and the Vernacular
nauseated at the sight and smell of alcohol and stopped 29 All subjects in Philippine elementary and secondary
drinking alcohoL Which theory explains this? schools are expected to be taught using the integrated
A. Operant conditioning approach. This came about as a result of the
B. Social Learning Theory implementation of _________.
C. Associative Learning A. Program for Decentralized Education
D. Attribution Theory B. School-Based Management
23 A mother gives his boy his favorite snack everytime C. Basic Education Curriculum
the boy cleans up his room. Afterwards, the boy D. Schools First Initiative
cleaned his room everyday in anticipation of the snack. 30 Under which program were students who were not
Which theory is illustrated? accommodated in public elementary and secondary
A. Associative Learning schools because of lack of classroom, teachers, and
B. Classical Conditioning instructional materials, were enrolled in private schools
C. Operant Conditioning in their respective communities at the government's
D. Pavlonian Conditioning expense?
23 Researches conducted show that teacher's A. Government Assistance Program
expectations of students become. Do not require initial B. Study Now-Pay Later
formal language teaching for children self-fulfilling C. Educational Service Contract System
prophecies. What is this phenomenon called? D. National Scholarship Program
A. Halo effect 31. What was the most prominent educational issue of
B. Pygmalion effect the mid 1980s?
C. Ripple effect A. Bilingual Education
D. Hawthorne effect B. Values Education
24 What does extreme authoritarianism in the home C. Accountability
reinforce in learners? D. Mainstreaming
A. Doing things on their own initiative 32 Availment of the Philippine Education Placement
B. Ability to direct themselves. Test (PEPT) for adults and out-of-school youths is in
support of the goverriment'S educational program
C. Dependence on others for direction.
towards __________.
D. Creativity in work.
A. equitable access
25 Theft of school equipment like tv, computer, etc. by
B. quality
teenagers in the community itself is becoming a
C. quality and relevance
common phenomenon. What does this incident signify?
D. relevance
A. Prevalence of poverty in the community.
32 The main purpose of compulsory study of the
B. Inability of school to hire security guards.
Constitution is to __________
C. Deprivation of Filipino schools.
A. develop students into responsible, thinking citizens
D. Community's lack of sense of co-ownership.
B. acquaint students with the historical
26 A student passes a research report poorly written
development of the Philippine Constitution
but ornately presented in a folder to make up for the
C. make constitutional experts of the students
poor quality of the book report content. Which Filipino
trait does this practice prove? Emphasis on __________. D. prepare students for law-makinig
A. art over academics

120
33. Which one may support equitable access but may 40. A child who gets punished for stealing candy may
sacrifice quality? not steal again immediately. But this does not mean
A. Open admission that the child may not steal again. Based on Thorndike's
B. School accreditation theory on punishment and learning, this shows that
C. Deregulated tuition fee hike __________
D. Selective retention A. punishment strengthens a response
34. With which goals of educational institutions as B. punishment removes a response
provided for by the Constitution is the development of C. punishment does not remove a response
work skills aligned? D. punishment weakens a response
A. To develop moral character 41 It is not wise to laugh at a two-year old child when
B. To teach the duties of citizenship he utters bad word because in his stage he is learning to
C. To inculcate love of country __________.
D. To develop vocational efficiency A. consider other's views
35 Studies in the areas of neurosciences disclosed that B. distinguish sex differences
the human brain has limitless capacity. What does this C. socialize
imply? D. distinguish right from wrong
A. Some pupils are admittedly not capable of 42 John Watson said: &Men are built not born.& What
learning. does this statement point to?
B. Every pupil has his own native ability and his A. The ineffectiveness of training on a person's
learning is limited to this nativeabilty. development.
C. Every child is a potential genius. B. The effect of environmental stimulation on a person's
D. Pupils can possibly reach a point where they have development.
learned everything. C. The absence of genetic influence on a person's
36 Based on Piaget's theory, what should a teacher development.
provide for children in the concrete operational stage? D. The effect of heredity.
A. Activities for hypothesis formulation. 43 Which types of play is most characteristic of a four
B. Learning activities that involve problems of to six-year old child?
classification and ordering. A. Solitary and onlooker plays
C. Games and other physical activities to develop B. Associative and coooperative plays
motor skills. C. Associative and onlooker plays
D. Stimulating environment with ample objects to D. Cooperative and solitary plays
play with. 44 All of the following describe the development of
37 Based on Piaget's theory, what should a teacher children aged eleven to thirteen EXCEPT __________.
provide for children in the sensimotor stage? A. they shift from impulsivity to adaptive ability
A. Games and other physical activities to develop motor B. sex differences in IQ becomes more evident
skill. C. they exhibit increase objectivity in thinking
B. Learning activities that involve problems of D. they show abstract thinking and judgement
classification and ordering. 45 Rodel is very aloof and cold in his relationships with
C. Activities for hypothesis formulation. his classmates. Which basic goal must haye not been
D. Stimulating environment with ample objects to attained by Rodel during his developmental years,
according to Erikson's theory on psychological
play with.
development?
38. Which behavior is exhibited by a student who is
A. Autonomy
strong in interpersonal intelligence?
B. Trust
A. Works on his/her own.
C. Initiative
B. Keeps interest to himself/herself.
D. Generativity
C. Seeks out a classmate for help when problem occurs.
46 Ruben is very attached to his mother and Ruth to
D. Spends time meditating.
her father. In what developmental stage are they
39 A sixth grade twelve-year old boy comes from a according to Freudian psychological theory?
dysfunctional family and has been abused and A. Oedipal stage
neglected. He has been to two orphanages and three
B. Latent stage
different elementary schools. The student can decode
C. Annal stage
on the second grade level, but he can comprehend
orally material at the fourth or fifth grade level. The D. Pre-genital stage
most probable cause/s of this student's reading 47 Which assumption underlies the teacher's use of
problem is/are __________. performance objectives?
A. emotional factors A. Not every form of learning is observable.
B. poor teaching B. Performance objectives assure the learrier of
C. neurological factors learning.

121
C. Learning is defined as a change in the learner's 56 The primary objective of my lesson is: &To add
observable performance. similar fractions correctly.& Before I can do this I must
D. The success of learner is based on teacher first aim at this specific objective: &To distinguish a
performance. numerator from a nominator.& What kind of objective is
48 The principle of individual differences requires the latter?
teachers to __________. A. Major
A. give greater attention to gifted learners B. Terminal
B. provide for a variety of learning activities C. Enabling
C. treat all learners alike while in the classroom D. Primary
D. prepare modules for slow learners in class 57 As a teacher, what do you do when you engage
49 In instructional planning it is necessary that the yourself in major task analysis?
parts of the plan from the first to the last have A. Test if learning reached higher level thinking
__________. skills.
A. clarity B. Breakdown a complex task into sub-skills.
B. symmetry C. Determine the level of thinking involved.
C. coherence D. Revise lesson objectives.
D. conciseness 58 Teacher G's lesson objective has something to do
50. A goal-oriented instruction culminates in with the skill of synthesizing? Which behavioral term is
__________. most appropriate?
A. planning of activities A. Test
B. evaluation B. Assess
C. identification of topics C. Appraise
D. formulation of objectives D. Theorize
51 A teacher's summary of a lesson serves the 59 In Krathwohl's taxonomy of objectives in the
following functions, EXCEPT affective, which is most authentic?
A. it links the parts of the lesson A. Characterization
B. lt brings together the information that has been B. Orgarlization
discussed C. Responding
C. it makes provisions for full participation of D. Valuing
students. 60 &A stitch on time saves nine&, so goes the adage..
D. it clinches the basic ideas or concepts of the Applied to classroom management, this means that we
lesson. __________
52 In Krathwohl's affective domain of objectives, which A. may not occupy ourselves with disruptions which
of the following is the lowest level of affective are worth ignoring because they are minor
behavior? B. must be reactive in our approach to discipline
A. Valuing C. have to Jesolve minor disruptions before they are out
B. Characterization of control
C. Responding D. may apply 9 rules out of 10 consistently
D. Organization 61 How can you exhibit referent power on the first day
of school?
53 The following are used in writing performance A. By making them feel you know what you are
objectives, EXCEPT talking about.
A. delineate B. By telling them the importance of good grades.
B. diagram C. By reminding your students your authority over
C. integrate them again and again.
D. comprehend D. By giving your students a sense of belonging and
54 If a teacher plans a constructivist lesson, what will acceptance.
he most likely do? Plan how he can 62 Teacher B clears his throat to communicate
A. do evaluate his students' work disapproval of a student's behavior. Which specific
B. do reciprocal teaching influence technique is this?
C. lecture to his students A. Signal interference
D. engage his students in convergent thinking B. Direct appeal
55 In mastery learning, the definition of an acceptable C. Interest boosting
standard of performance is called a D. Proximity control
A. SMART
B. criterion measure 63 Teacher H strives to draw participation of every
C. behavior student into her classroom discussion. Which student's
D. condition need is she trying to address? The need __________

122
A. to show their oral abilities to the rest of the class 71 Research tells that teachers ask mostly content
B. to be creative questions. Which of the following terms does NOT refer
C. to feel significant and be part of a group to content question?
D. to get everything out in the open A. Closed
B. Direct
64 Which is a sound classroom management practice? C. Concept
A. Avoid establishing routines; routines make your D. Convergent
student robots.
72 TEACHER: IN WHAT WAYS OTHER THAN THE
B. Establish routines for all daily needs and tasks.
PERIODIC TABLE MIGHT WE PREDICT THE
C. Apply rules and policies on a case to case basis.
UNDISCOVERED ELEMENTS?BOBBY: WE COULD GO
D. Apply reactive approach to discipline.
TOTHE MOON AND SEE IF THERE ARE SOME ELEMENTS
65 An effective classroom manager uses low-profile THERE WE DON'T HAVE.BETTY: WE COULD DIG DOWN
classroom control. Which is a low-profile classroom TO THE CENTER OF THE EARTH AND SEE IF WE FIND ANY
technique? OF THE MISSING ELEMENTS.RICKY: WE COULD STUDY
A. Note to parents DEBRIS FROM THE METEORITES IF WE CAN FIND
B. After-school detention ANY. TEACHER: THOSE ARE ALL GOOD ANSWERS BUT
C. Withdrawal of privileges WHAT IF THOSE, EXCURSIONS TO THE MOON, TO THE
D. Raising the pitch of the voice CENTER OF THE EARTH, OR TO FIND METEORITES WERE
66 Which is one characteristic of an effective TOO COSTLY AND TIME CONSUMING? HOW MIGHT WE
classroom management? USE THE ELEMENTS WE ALREADY HAVE HERE ON EARTH
A. It quickly and unobtrusively redirects misbehavior TO FIND SOME NEW ONES?
once it occurs. Question: Which questioning strategy/ies does/do the
B. It teaches dependence on others for self-control. exchange of thoughts above illustrate?
C. It respects cultural norms of a limited group A. Funneling
students. B. Sowing and reaping
D. Strategies aresimple enough to be used C. Nose-dive
D. Extending and lifting
consistently.
67 How can you exhibit legitimate power on the first 73 Which questioning practice promotes more class
day of school? interaction?
A. By making your students feel they are accepted for A. Asking the question before calling on a student.
who they are. B. Focusing on divergent questions.
B. By informing them you are allowed to act in loco C. Focusing on convergent questions.
parentis. D. Asking rhetorical questions.
C. By making them realize the importance of good 74 Which technique should a teacher use to encourage
grades. response if his students do not respond to his question?
D. By making them feel you have mastery of subject A. Ask a specific student to respond, state the question,
matter. and wait a response.
68 With-it-ness, according to Kounin, is one of the B. Tell the class that it will have detention unless
characterestics of an effective classroom manager. answer are forthcoming.
Which phrase goes with it? C. Ask another question, an easier one.
A. Have hands that write fast. D. Wait for a response.
B. Have eyes on the back of your heads. 75 Teacher P wants to develop the skill of synthesizing
C. Have a mouth ready to speak. in her pupils. Which one will she do?
D. Have minds packed with knowledge. A. Ask her students to formulate a generalization from
69 Which is an appropriate way to manage off-task the data shown in graphs.
behavior? B. Ask her students to answer questions beginning
A. Make eye contact. with &What if ... &
B. Stop your class activity to correct a child who is C. Tell her pupils to state data presented in graphs.
no longer on task. D. Directs her students to ask questions on the parts
C. Move closer to the child. of the lesson not understood.
D. Redirect a child's attention to task and check his 76 The following are sound specific purposes of
progress to make sure he is continuing to work. questions EXCEPT
70 Referring to Teacher S, Nicolle describes her teacher A. to call the attention of an inattentive student
as &fair, caring and someone you can talk to.& Which B. to teach via student answers
power or leadership does Teacher S have? C. to stimulate leamers to ask questions
A. Referent power D. to arouse interestand curiosity
B. Legitimate power 77 For maximum interaction, a teacher ought to avoid
C. Reward power __________ questions.
D. Expert power A. informational

123
B. rhetorical D. Existentialist
C. leading 86 Which is/are the sources of man's intellectual
D. divergent drives, according to Freud?
78 If teacher has to ask more higher-order questions, A. Id
he has to ask more __________ questions. B. Superego
A. closed C. Id and ego
B. fact D. Ego
C. concept 86 Teacher F is convinced that whenever a student
D. convergent performs a desired behavior, provided
reinforcement and soon the student will learn to
79 Which is NOT a sound purpose for asking questions? perform the behavior on his own. On which
A. To probe deeper after an answer is given. principle is Teacher F's conviction based?
B. To discipline a bully in class. A. Cognitivism
C. To remind students of a procedure. B. Environmentalism
D. To encourage self-reflection. C. Behaviorism
80 Principal B tells her teachers that training in the D. Constructivism
humanities is most important. To which educational 87 Which does Naom Chomsky, assert about
philosophy does he adhere? language learning for children?
A. Existentialism I. Young children learn and apply grammatical rules and
B. Perennialism vocabulary as they are exposed to them.
C. Progressivism II. Begin formal teaching of grammatical rules to children
D. Essentialism as early as possible.III. Do not require initial formal
81 Principal C shares this thought with his language teaching for children.
teachers: &Subject matter should help students A. I and III
understand and appreciate themselves as unique B. II only
individuals who accept complete responsibility for their C. I only
thoughts, feelings, and actions.” From which philosophy D. I and II
is this thought based? 88 Which teaching activity is founded on Bandura's
A. Perennialism Social Learning Theory?
B. Essentialism A. Lecturing
C. Existentialism B. Modeling
D. Progressivism C. Questioning
82 To come closer to the truth we need to “go back to D. lnductive Reasoning
the things themselves.” This is the advice of the 89 Behavior followed by pleasant consequences will be
A. behaviorists be strengthened and will be more likely to occur in the
B. phenomenologists future. Behavior followed by unpleasant consequences
C. idealists will be weakened and will be less likely to be repeated
D. pragmatists in the future. Which one is explained?
83 Student B claims: &I cannot see perfection but I long A. Freud's Psychoanalytic Theory
for it. So it must be real.& Under which group can he be B. Thorndike's Law of Effect
classified? C. B. F. Skinner's Operant Conditioning Theory
A. Idealist D. Bandura's Social Learning Theory
B. Empiridst
C. Realist 90 Bruner's theory on intellectual development moves
D. Pragmatist. from enactive to iconic and symbolic stages. In which
stage(s) are diagrams helpful to accompany verbal
`84 Which of the following prepositions is attributed to
information?
Plato?
A. Enactive and iconic
A. Truth is relative to a particular time and place.
B. Symbolic
B. Human beings create their own truths.
C. Learning is the discovery of truth as Iatent ideas are C. Symbolic and enactive
brought to consciousness. D. Iconic
D. Sense perception is the most accurate guide to 91 All of the following describe the development of
children aged eleven to thirteen EXCEPT __________.
knowledge.
A. they shift from impulsivity to adaptive ability
85 You arrive at knowledge by re-thinking of latent
B. sex differences in IQ becomes more evident
ideas. From whom does this thought come?
A. Experimentalist C. they exhibit increase objectivity in thinking
B. Realist D. they show abstract thinking and judgement
C. Idealist

124
92 How can you exhibit expert power on the first day of b. Lesson plan is composed of interrelated
school? parts of a system
A. By making them feel you know what you are talking c. Lesson plan is a part of the total school
about.
system
B. By making them realize the importance of good
d. Lesson planning is a systematic process
grades.
106. Ms. Valenciano is using questions to
C. By reminding them your students your authority
over them again and again.
initiate earning. Which of the following
D. By giving your students a sense of belonging and questions focus learner’s attention or a given
acceptance. topic or issue?
a. Will you read your assignment?
b. Mike is transferring to another school
c. When do energy requirement decrease?
101. Mr. Fernando, a Geography teacher, d. What do you think will happen if I combine
integrates the learning activities and materials this two chemical?
with real life. In line with the principle of 107. In which of the following situations does
expanding environment her context should a teacher formulate low level questions?
start with a. when learners need to analyze a situation
a. projection into the future b. when he wants learner to draw inferences
b. adventures into outer space c. when he wants his students to retrieve facts
c. local situations and community d. when the learners need to establish
d. lands and people beyond Philippine shores relationship
102. When Ms. Narvaez selects activist and 108. During a cooking demonstration lesson.
materials for her Social Studies lessons that Mrs. Cojuangco asked this question: “Why
stimulate curiosity and satisfy the need to don’t you pare the apple?’ What does this
know, which criterion is meet? question express?”
a. Criterion of organization a. A directive without implicit questions
b. Criterion of authenticity b. Expresses command with implicit question
c. criterion of appropriateness c. A command with an interrogative operation
d. Criterion of interest d. Sounds like a question with an implicit
103. The Grade 6 Science teachers are engaged command
in panning the units for Science. Which of the 109. Everytime Ms. Montalban introduces unit
following activities will not be engaged in by in Social Studies she always prepares five or
the group of teachers? six pivotal questions. What is the function of
a. rearranging or adding lesson content to these questions in the whole instructional
provide task-relevant prior knowledge process?
b. translating unit outcomes/objectives into a. Allows for deliberation
specific lesson objective b. Fosters an active audience
c. classifying unit outcome/objectives at a c. Gives the lesson unit and coherence
higher level of behavior d. Encourages students to think about the
d. breaking the lesson contents into chunks question
104. Miss Tantengco is teaching a lesson on 110. Which of the following high level
“sounds”. Assuming that no task-relevant prior questions ask the learner to understand certain
knowledge is required , what should be the stimuli before making any interpretation?
starting point of her lesson? a. What connection does the chart show
a. comprehension of principles between the number of accidents and the
b. knowledge of principle frequency of holidays?
c. comprehension of concepts b. Describe in your words the scene which
d. knowledge of concepts greeted the main character on his arrival
105. What is the primary reason why teacher airport
should take a system perspective? c. What are the three situations in which other
a. Lessons prepared will always be a part of questions would be appropriate?
something greater d. Which of the following statement is most
likely true?

125
111. Ms. Abelardo has been having discipline and “ Can you try as hard tomorrow?”, she is
and management problems in her English rendering:
class. What question tip may sustain the a. Feedback progress c. motivated challenge
students’ attention? b. consequential reward d. praise statement
a. Ask questions that contribute to continuous 117. Mrs. Medina, a Grade Vi teacher, is faced
learning with problem of gender bias in assigning work
b. Ask questioning that are memory testing task. Everytime she would request for
c. Ask questions that are stimulating volunteers she would received remarks such as
d. Ask questions that are sequential “those are boy’s task or girl’s task.” A boy
112. Miss Reyes is using questions for never got to dust the desk because dusting is a
instructional purposes. What characterizes best girl’s task or a girl did not empty the waste can
instructional questioning? because that was reserved for boys. Those
a. generally divergent gender-biased stereotypes simply no longer
b. generally convergent exists even in the curriculum of the elementary
c. designed to elicit information from students schools. Which is the key criterion that should
d. intended to get feedback on students guide Mrs. Medina to avoid gender bias in
learning assigning work task?
113. At the start of the school year, Ms. a. Ability of the individual whether male or
Oriondo attends first to matters pertinent to female to compete the ask
management task. What could have been her b. No criterion is needed both boys and girls
primary goal? can be assigned any task
a. Increase the proportion of classroom time to c. Alternated opposite gender task assignment
constructive and productive activities d. Intellectual and physical demands of task
b. Increase the pupils’ interest in her daily 118. For quite sometime, Liezl habitually says
instructional activities “no” to work assigned to her by the teacher.
c. Increase the effectiveness of her teaching Which of the following is a questionable
strategies technique in handling Liezl who refuses school
d. Increase the quality of her teaching work?
performance a. Focus on situations where the pupil shows
114. Every morning it is routinely for Miss an interest
Canlas to arrange pupils’ chairs and tables b. Treat all pupils who refuse to any work
appropriate for the lesson for today, check the equally
safety of the room and materials and other c. Reduce criteria for the correctness of a task
similar activities that will promote an orderly d. Immediately confront the pupil concerned
and safety environment. She is exemplifying 119. Mrs. Pabalan wanted to establish control
a. content management c. context on the very first day. Which action is
management imperative for her to take/
b. conduct management d. combination of a. Give only compliments and no scolding or
A,B,C correcting pupils
115. How can Mrs. Nolasco best handle the b. Establish yourself immediately as the
routine of attendance taking upon entering the supreme commander
room? c. Give a long assignment top command
a. Ask students to write their names on slips of respect
paper and drop them in an “ Attendance Box” d. Be in room when the pupils arrive
b. ask students to turn their names from a 120. It has been a routine for Ms. De Belen the
pocket chart placed inside the room first fifteen minutes collecting materials from
c. ask a monitor to check and record orally the students. What is the primary reason why this
attendance by rows and tablets practice should not be established?
d. call the roll as fast as possible before a. Students have the habit of packing up their
starting the day’s lesson. things and shutting down their minds at the
116. Whenever Ms. Sanchez gives statements beginning of the class
such as, Do you think you can do that again?”

126
b. Students are likely to remember things that b. Call class attention to the cheating
occur past beyond the middle of a learning committed by pupils
sequence c. Announce disapproval of the pupils’
c. Students are potentially most alert and behavior
receptive to instruction when the new activity d. Take the pleasure of the pupils
is begun 125. Pauline inspite of her I.Q, has been
d. Students are likely least task-oriented at the getting low grades and performing poorly in
first few minutes of a class session almost all the academic subjects. Her father
121. Miss Rivera is busy developing a cluster went to see her teacher, what is the best way to
of concepts on matter. She notices Dave busily manage the situation?
making several paper airplanes. Which of the a. Tell Paulin’s father a solution by which she
following is the best way of handling Dave’s can be helped to improve her performance
misbehavior? b. Tell Pauline’s father that he should not
a. Continue teaching but make your way to complain because you computed her grades
Dave. Without a word place a ready note card objectively
“It seems you are disturbing the class. Please c. Direct Pauline’s father to complain to the
stop right now. return card after class.” principal
b. Stop the less. Tell the class to look at Dave d. Advice Pauline’s father to get a tutor for her
and observe what he is doing. Let them 126. At the end of the periodical examination,
comment on what they observe Mrs. Gonzales administered a summative test
c. Call Dave to bring all the paper airplanes in Filipino. After scoring the test papers she
and ask him to discuss the importance of assigned grades to each test score such as
airplanes in the modern age 95,90,85 etc. What process did Mrs. Gonzales
d. Ask difficult questions and call Dave to use?
answer them in succession a. Ranking c. Computation
122. Mrs. Estrada prefers to use low profile b. Measurement d. Evaluation
classroom control like eye contact and hand 127. Which measure of central tendency is
gesture to high profile from like. “Stop usually affected by extreme scores?
walking around, settle down and get to work.” a. Mode c. Median
Which of the following justifies Mrs. Estrada’s b. Mean d. Quartile
action? 128. The graduating students need information
a. Low profile form of control is simple and regarding future occupation where they will
time saving most likely succeed. What kind of test will the
b. Low profile form of control minimizes guidance counselor administer?
teachers visibility a. Survey test c. Aptitude test
c. Low profile from of control directs attention b. Achievement test d. Attitude test
of the class to target pupils 129. In Sta. Teresita Barangay High School,
d. Low profile form of control has low majority of the students who got very high
probability of its distracting other members of scores in the entrance examination got very
the class low grade point averages at the end of the year.
123. Mrs. Cortez is bothered by many pupils What type of validity does the examination
who are off tasks during her demonstration of lack?
a science process. Which of the following a. content c. construct
intervention is most effective? b. predictive d. concurrent
a. Eye contact c. Asking an answer 130. Which will you least use as measure of
b. Touch/Gesture d. Questioning consistently central tendency?
124. Mrs. Santillan has caught several students a. Median c. Mode
peeking at their notebook during an b. Mean d. Arithmetic Mean
examination. What should be the initial action 131. In terms if its given difficulty and
on her part? discrimination indices, which item should be
a. Confirm the dishonesty and send the pupils rejected?
caught cheating to the principal’s office a. Difficulty index.21 and discrimination index
.24

127
b. Difficulty index.25 and discrimination index d. A study of the effect of the level of
.30 aspiration on achievement of prospective
c. Difficulty index .70 and discrimination teachers
index .65 138. A teacher-researcher wanted to know the
d. Difficulty index .16 and discrimination attitude of college students toward the school
index .15 uniform. She stood by the canteen door at
132. Janice in Grade Vi has a percentile rank lunch tine and asked every fifth student who
of 90 in achievement test in language. This came for lunch. What sample did she used?
mean that ________ a. Biased c. Random sample
a. The scores Janice’s obtained was as high as b. Fixed sample d. Uneven sample
the scores made by 90% of the pupils to whom 139. Why should guidance be an integral party
the test was give of the school program?
b. Eighty-nine pupils obtained scores lower a. Learners and confronted with present day
than what Janice obtained realities and challenges which they themselves
c. 90 percent of the sixth graders obtained cannot handle
scores higher than Janice b. Learners need to be guided so that they may
d. In the class of 100 sixth graders Janice is become useful members of the society
90th from the top c. Learners need to help in their personal and
133. What is the first and perhaps the most academic problems
important step which a teacher should take in d. Learners are still young to be left to
constructing a test? themselves
a. Know the objectives 140. Being part of the total guidance program,
b. Look over the old test questions the first task of the classroom teacher is to
c. Prepare the table of specification a. exemplify a high level of personal
d. Write the preliminary draft of the test adjustment and self-actualization
134. Mr. Villamin graded the essay question of b. foster the development of positive attitudes
his class in Literature. What procedure could c. know and understand the student
he use to reduce the subjectivity of the essay d. be warm to each other
examination? 141. Michelle has been reported buy her
a. Correct the papers of the bright pupils first teacher to be habitual absentee and
to establish the highest score possible consequently a poor achiever in the class.
b. Grade the paper twice and get the average of What initial step should she take?
two grading a. Report her to the discipline officer for
c. Grade all of one paper before going on the appropriate action
next b. Talk with her and encourage her to unload
d. Ask another teacher to grade the paper her problems
135. Miss Nava is constructing a 100 test items c. Recommend her to transfer to another
for the fourth grading period in Mathematics. school
How many items should she have in the d. Call her parents to a case conference
preliminary draft? 142. There are indications that Shaina is an
a. 125 to 150 items autistic child, What should Mrs. Abella her
b. 150 to 200 items teacher, do to help her?
c. 110 to 125 items a. Recommend a tutor for Shaina
d. 200 to 250 items b. Tell her parents to transfer her to another
136. What percent of the cases in a set of school
measure lie before the third quartile of Q3? c. Ask Shaina to stay to stay and give her extra
a. 25% b. 50% c. 755 d. 65% care and attention
137. Which is not an example of research? d. Advise her parents to take her to a specialist
a. Preparation of a project report in Science for diagnosis
b. A survey of the survey habits of BEED 143. Ms. Revilla, a classroom teacher is aware
Freshman students that she is also expected to perform guidance
c. Development and validation of learning function. She has been helping her pupils in
modules on Statistics their personal and academic problems. What

128
must do if there are cases which she finds a. Persuade her to follow your step as a teacher
difficulty to handle? b. Tell her to heed the advise of her parent
a. Refer them to the guidance counselor who c. Refer her to a college guidance counselor
has the professional competence to handle d. Ask her to take an aptitude test
such cases 149. Valerie’s parents often times complains
b. Try to solve them. There are practical about her low grades in the report card.
solutions to every problem Despite the explanations of her teachers, Mr.
c. Bring them to the attention of the school and Mrs. Cariño insist that Valeria deserves
principal higher ratings. What is the proper thing for her
d. make referrals to outside agencies teacher to do?
144. For quite some time, Joseph, a Grade VI a. Change Valerie’s grade to pleased her
pupil has been displaying unusual behaviors parents
which have adversely affected his b. Request the principal to talk with Valerie’s
performance. How may Ms. Ramos, his parents
teacher, best given the necessary help? c. Show to the parents the actual computation
a. seek the advise of child psychologist of Valerie’s grade
b. Refer the pupil to a psychiatrist d. Ignore Valerie’s parents. The would not
c. Arrange for a case conference listen to explanations anyway
d. Conduct a case study 150. Mrs. Serna administered a sociometric
145. Margaret, a Grade V pupil, can hardly test to her class. The sociogram reveals that
expressed her inner feelings to the guidance three students have formed an “island” in the
counselor. What guidance technique may be class. What is the best thing for Mrs. Serna to
utilized so that she may unload herself and be do?
given the necessary help/ a. Ignore the result of the sociogram
a. personality test b. Let the thee students be together all the time
b. projective technique c. Discourage them form being together in
c. sociometric test every class activity
d. autobiography d. Introduce activities where they can join
146. At the beginning of the school year, Ms. other members of the group.
Mina wants to acquaint herself with the 151. Sheryl, a freshman education student, was
personal background of her Grade VI pupils asked by his professor to describe education
particularly their needs, interest and accurately. Which of the following statement
characteristics. what is the most practical thing will Sheryll most likely choose?
that she must do? a. Synonymous to formal schooling
a. Give them a series of psychological test b. Growth resulting from academic study
b. Ask them to fill up personal data sheets c. Acquired basically through the teacher
c. Send a questionnaire to their parents d. A process of individual growth and social
d. Interview them one by one development
147. What is the focus of developmental 152. Mis Diegor has been rated as a very
guidance? effective teacher in disseminating current
a. Developing the varied interest, abilities and information in her Social Studies Class. Which
needs of students, individually and collectively intellectual trait does Miss Diegor manifest?
b. providing students with ample opportunities a. Widely read c. Tolerance
to develop their innate talents b. Intellectual honesty d. Well groomed
c. Identify students with personality and 153. Miss Victoria considers herself a pioneer
behavior problems in the world ideas. What would be her relevant
d. Facilitating the total development of the task?
students a. Establish instructional goal
148. Charlene, a graduating high school b. Establish standards of behavior
student is confused about what course to take c. Lead children to inquire about their
in college. She seems to be torn between environment
teaching and accountancy. How would you d. Articulate social, political and economic
help her? traditions

129
154. Miss Sales is a newly appointed teacher. d. Maintain boundaries of structure between
The principal handed her a copy of the social classes
Professional Code of Ethics. What could have 160. Rachelle brings of her books to school
been the motive of the principal? because she wants to please the teacher and get
a. Give her a set of rules and regulation to good grades. To which of the following levels
observe in school of morality according to Kohlberg does she
b. Familiarize her with the generally accepted belong?
customs of right living in the Philippine a. Conventional
practice of profession b. Preconventional
c. Make her aware of the principles and rules c. Postconventional
prescribed under the authority of the state d. Either preconventional or conventional
d. make her aware of the principles and rules 161. Teacher Cleo observed that the preschool
prescribed under the authority of the state children have generally short attention and
155. As a teacher, which of the following interest span. What kind pf class activities
actions would you judge as unprofessional? should select for them?
a. Engaging in gainful employment a. short, varied and interesting
b. Seeking position known to be vacant b. challenging and interesting
c. Giving due notice in case of absence c. both easy and difficult
d. Taking professional matter directly to the d. long but interesting
highest authority 162. Long before the start of every school
156. Miss Delgado is a newly appointed year, teacher Faith has already started
teacher. The principal advised her to avoid any developing her own comprehensive plan based
conduct which discredits the teaching on the recommended curriculum. Which
profession. Which of the following action is explains best Teacher Faith’s action in relation
permissible in the teaching profession? to the curriculum?
a. Writing anonymous letters a. Planning personalizes the curriculum
b. Assigning underserved grades making it her own
c. Joining social drinking and gambling session b. Planning is entirely dependent on he
d. Revealing confidential information to approved curriculum
authorities concerned c. Planning “screened” possible difference
157. Which statement is NOT regarded as a between the curriculum plan and the
social value of education/ implementation process
a. Education furthers community health 163. Education during this period was
b. Education promotes wholesome family life essentially intellectual discipline based on
c. Education orders and humanizes economic rational arguments. This points to_________
life a. Feudalistic education c. Scholastic
d. Education refines human sentiments and education
feelings b. Saracenic education d. Monastic education
158. In education as agent of modernization 164. Which of the following is NOT provided
the curriculum tends to focus on well defined in the Education Act of 1940?
orientation. In this sociological views, which a. The national support of elementary
describes best the curriculum? education
a. future-oriented c. present-oriented b. The double-single plan in the elementary
b. past oriented d. tradition oriented schools
159. The function of schooling is determined c. The six year elementary course
largely by the generally accepted social d. Vocational education in the public schools
conception of education. What is the function 165. The primary objective in the
of the curriculum in a school that regards regionalization of the educational system is
education as cultural transmission? to__________
a. Remove social status stratification a. encourage the undertaking of language
b. Break boundaries between social classes researches in various regions
c. Unify the elements of various social classes b. take into account local needs and condition
and encourage local development planning

130
c. et assurance that all educational policies and c. Good behavior is based on the physical
implemented nationwide consequences of action
d. promote quality education at all levels and d. Good behavior is that which pleases or helps
in all communities of the country others or is approved by them
166. Dewey’s philosophy stresses the 172. According to the Constitution, the State is
development of an individual capable of required to establish and maintain free public
reflective thinking, specifically that of being and compulsory education in the—
able to solve the problem he faces individually a. elementary level only
or collectively. This is___________ b. secondary level only
a. rationalism c. developmentalism c. secondary and tertiary levels
b. experimentation d. disciplinism d. elementary and secondary levels
167. Identical twins are more alike in 173. The recognition of teachers as persons in
intelligence that are fraternal twins. This facts authority was conferred to them during the
indicates that __________ __________
a. environment affects both fraternal and a. Spanish rule
identical twins b. Japanese regime
b. intelligence is determined partly by prenatal c. American rule
nutrition d. Commonwealth government
c. heredity has a part in determining 174. The relationship between education and
intelligence culture tends to by cyclical. This means
d. intelligence hinges on physical structure that_______
168. Much of today’s classroom practices are a. the school is shaped by culture and culture
influenced by Skinner’s operant conditioning in turn is influenced by the school
which stresses on _________ b. the school function primarily as a
a. involuntary response to a stimulus transmitter of culture and is a change agent
b. progression of subordinate learning c. the school is transmitted by the school
c. connection between stimulus and response system and the classroom is the place where
d. reinforcement of correct response transmission takes place
169. Early childhood is characterized by 175. Teachers should bear in mind that the
educators as a period of morality buy period of greatest mental development is from
constraint. This means that children in this ____
stage consider teachers and parents a. 3 to 6 years c. 6 to 9 years
as___________ b. 9 to 12 years d. 12 to 15 years
a. authorities and models 176. Which patterns of development closely
b. counselors and advisers parallel to the pattern for speech development?
c. peers and playmates a. Emotional and moral c. Intellectual and
d. facilitators and supervisor motor
170. Because learning increases directly in b. Intellectual and moral d. Emotional and
proportion to the extent to which the learner is motor
wholly bound up in his task, the teacher should 177. Democracy from the Christian
give lessons that______ perspective is viewed as________
a. are fictitious to appeal to their imagination a. equal education for men and women
b. have significance and worth to the child b. education controlled by the state
c. are easy to comprehend c. education subsidized by the state
d. portray complex ideas d. education of all human beings without
171. Which of the following corresponds to distinction of sex, age, race, rank, social
Kohlberg’s post-conventional or principal economic or political status
level of moral development? 178. The Athenian ideal of education was the
a. Right action consists of what instrumental formation of a cultural soul in a graceful and
satisfies one’s own needs symmetrical body. This is achieved by_______
b. right action is defined buy the decision of a. Using the seven liberal arts
conscience in accordance with self-chosen b. putting the emphasis on physical education

131
c. a well balanced development of mind, body of the Filipino as a people. How should you
and soul tackle them in the classroom?
d. adopting the philosophy “Know thyself” a. Present them and express your feelings of
179. Computers are now widely used in many shame.
aspects of society including in education. Why b. Present facts and use them as means in
are some school slow in accepting the inspiring your class to learn from them.
accepting the use of computer? c. Present them and blame those people
a. Competently trained teachers are few responsible or those who have contributed.
b. The cost of hardware is high d. Present them as they are presented, and tell
c. Programs needed to run the computer are not the class to accept reality.
available 186. If you agree with Rizal on how you can
d. all of these contribute to our nation's redemption, which
180. A child learns the world FLOWER before should you work for?
he can name Sampaguita, Gumamela and a. Opening our doors to foreign influence
Camia because development_________ b. Upgrading the quality of the Filipino
a. is cephalocaudal in nature through education
b. proceeds from general to specific c. Stabilizing the political situation
c. follows a pattern d. Gaining economic recovery
d. is a continuous process 187. Rights and duties are correlative. This
181. Why is indoctrinating the child by means that __________.
making decisions for him dangerous? a. rights and duties regulate the relationship of
a. It will confuse him men in society
b. It will develop in him negative attitudes b. rights and duties arise from natural law
c. It will thwart his personality c. each right carries with it one or several
d. It will make him dependent corresponding duties
182. The cognitive process refers to the d. rights and duties ultimately come from God
realization that even if things change in 188. A teacher who equates authority with
physical appearance, certain attributes are power does NOT __________.
constant. This is________ a. shame
a. reservation c. integration b. develop self-respect in every pupil
b. construction d. conservation c. retaliate
183. The second basic principle of d. intimidate
development states that the rate of the 189. Which is a true foundation of the social
development is unique to each individual. order?
Educators aptly termed this as the principle of a. Obedient citizenry
a. dynamic change b. The reciprocation of rights and duties
b. variation c. Strong political leadership
c. individual differences d. Equitable distribution of wealth
d. uniqueness 190. In what way can teachers uphold the
184. Teacher A is directed to pass an highest possible standards of quality
undeserving student with a death threat. Which education?
advice will a hedonist give? a. By continually improving themselves
a. Pass the student. Why suffer the threat? personally and professionally
b. Don't pass him. You surely will not like b. By wearing expensive clothes to change
someone to give you a death threat in order to people's poor perception of teachers
pass. c. By working out undeserved promotions
c. Don't pass him. Live by your principle of d. By putting down other professions to lift the
justice. You will get reward, if not in this life, status of teaching
in the next! 191. A teacher/student is held responsible for
d. Pass the student. That will be of use to the his actions because s/he __________.
student, his parents and you. a. has instincts c. has a choice
185. History books used in schools are replete b. is mature d. has reason
with events portraying defeats and weaknesses

132
192. The typical autocratic teacher consistently c. promote obedience to the laws of the state
does the following EXCEPT d. instill allegiance to the Constitution
a. encouraging students. c. ridiculing students. 197. In the Preamble of the Code of Ethics of
b. shaming students. d. intimidating Professional Teachers, which is NOT said of
students. teachers?
193. What should you do if a parent who is a. LET passers
concerned about a grade his child received b. Duly licensed professionals
compared to another student's grade, demands c. Possess dignity and reputation
to see both students' grades? d. With high-moral values as well as technical
a. Refuse to show either record. and professional competence
b. Show both records to him. 198. Teacher H and Teacher I are rivals for
c. Refuse to show any record without promotion. To gain the favor of the
expressing permission from principal. promotional staff, Teacher I offers her beach
d. Show only his child's records. resort for free for members of the promotional
194. Teacher Q does not want Teacher B to be staff before the ranking. As one of the
promoted and so writes an anonymous letter contenders for promotion, is this becoming of
against Teacher B accusing her of fabricated her to do?
lies Teacher Q mails this anonymous letter to a. Yes. This will be professional growth for the
the Schools Division Superintendent. What promotional staff.
should Teacher Q do if she has to act b. No. This may exert undue influence ori the
professionally? members of the promotional staff and so may
a. Submit a signed justifiable criticism against fail to promote someone on the basis of merit.
Teacher B, if there is any. c. Yes. The rare invitation will certainly be
b. Go straight to the Schools Division welcomed by an overworked promotional
Superintendent and gives criticism verbally. staff.
c. Hire a group to distribute poison letters d. Yes. There's nothing wrong with sharing
against Teacher B for information one's blessings.
dissemination. 199. Each teacher is said to be a trustee of the
d. Instigate student activists to read poison cultural and educational heritage of the nation
letters over the microphone. and is, under obligation to transmit to learners
195. Teachers often complain of numerous such heritage. Which practice makes him
non-teaching assignments that adversely, affect fulfill such obligation?
their teaching. Does this mean that teachers a. Use the latest instructional technology.
must be preoccupied only with teaching? b. Observe continuing professional education.
a. Yes, if they are given other assignments, c. Use interactive teaching strategies.
justice demands that they be properly d. Study the life of Filipino heroes.
compensated. 200. Teacher F is newly converted to a
b. Yes, because other community leaders, not religion. Deeply convinced of his new found
teachers, are tasked to leading community religion, he starts Monday classes by attacking
activities one religion and convinces his pupils to attend
c. NO, because every teacher is expected to their religious services on Sundays. Is this in
provide leadership and initiative in activities accordance with the Code of Ethics of
for betterment of communities. ¬Professional Teachers?
d. Yes, because teaching is enough full time a. Yes. What he does is values education.
job. b. No. A teacher should not use his position to
196. In a study conducted, the pupils were proselyte others.
asked which nationality they preferred, if given c. Yes. In the name of academic freedom, a
a choice. Majority of the pupils wanted to be teacher can decide what to teach.
Americans. In this case, in which obligation d. Yes. What he does strengthens values
relative to the state, do schools seem to be education.
failing? In their obligation to __________. 101. c102. d103. a104. d105. d106. D
a. respect for all duly constituted authorities 107. c108. d109. c110. a111. c112. C
b. promote national pride 113. a114. c115. b116. c117. b118. a

133
119. b120. a121. c122. d123. d124. c ______12.To which Family do humans belong? =
125. a126. b127. b128. c129. b130. c a.Felidae b.Canidae c.Hominidae
131. d132. b133. a134. b135. c136. c 13.To which genus do humans belong?= Homo
137. a138. c139. b140. c141. d142. d 14.To which species do humans belong? = sapiens
143. a144. b145. b146. b147. a148. d 15. What is the scientific name of humans?= Homo
149. c150. d151. d152. a153. c154. c sapiens
DOMAIN ARCHAEBACTERIA
155. d156. d157. d158. a159. c160. b
_____16.Which group of bacteria lives in places
161. a162. d163. c164. d165. d166. b which are very extremes in which other organism
167. c168. d169. a170. b171. d172. d cannot survive?
173. d174. d175. b176. a177. d178. c a.Eubactera b.Archaebacteria c.Cyanobacteria
179. b180. b181. d182. c183. c184. b _____17.Which group of archaebacteria lives in
185. b186. b187. c188. b189. b190. a swamps and in the bottom of lakes and are capable
191. c192. a193. b194. a195. c196. b of producing METHANE gas?
197. a198. b199. d200. b a.Halophile b.Methanoids c.Methanogen
18.How is methane being utilized?= as BIOGAS
_____19.Which archaebacteria are adapted in very
GENERAL EDUCATION( General Biology) salty environment?
_____1.What is the term that specifically describe a.Thermophiles b.Halophiles c.Acidophiles
the variety of organisms on earth? ( BIODIVERSITY) _____20.Which group of archaebacteria lives in very
a.biodiversity b. species c.ecosystem hot environment just like volcanic hot springs, small
2.What are the categories of naming organism?( deep sea openings and can turn hydrogen sulfide
Domain, Kingdom, Phylum, Class, Order, Family into essential food for other organisms?
Genus Species) a.Thermophiles b.Halophiles c.Acidophiles
Which level of classification is the most inclusive or _____21.Which group of bacteria are referred to as
has the largest in scope? ( Domain) TRUE BACTERIA and also known as the bacteria
Which level of classification is the most exclusive or group?
has the least in scope? ( Species) a.Archaebacteria b.Eubacteria c.Proteobacteria
3.What is the plural term for Phylum? ( Phyla) _____22.What do you call the complex
4.Which level of classification is composed of carbohydrate in the cell wall of eubacteria?
several genera( singular genus)? ( Family) a.Peptidoglycan b.Murein c.Chitin
5.Which level is a group of similar organism and _____23.What do you call eubacteria which has a
capable of reproducing their own kind, or capable of round shape?= COCCI or coccus if singular
producing fertile offspring? _____24.A bacteria which are rod shape?
a.phyla b.genera c.species a.Bacillus b.coccus c.spirilla
6.What do you call organism whose genetic material _____25.A bacteria which have a spiral shape?
are not enclosed in a membrane bound organelle or a.Bacillus b.coccus c.spirilla
with no NUCLEUS? _____26.Coccus in pair?
a.Prokaryote b.Eukaryote c.Acellular a.Streptococcus b.Diplococcus c.Sarcinae
7.What do you call the organism whose genetic _____27.Coccus in chain ?
materials are enclosed in a membrane bound a.Streptococcus b.Diplococcus c.Sarcinae
organelle or a with a NUCLEUS? (EUKARYOTE) _____28.Coccus in clusters?
a.Prokaryote b.Eukaryote c.Acellular a.Streptococcus b.Diplococcus c.Staphylococcus
8.What do you call the tiny and unicellular _____29.What is the term used for bacteria when
prokaryotes ( MICROORGANISM) they cause disease?
a.germs b.microorganisms c.macroorganisms a.Germs b.dirts c.microorganisms
Multicellualr- organisms with two or more cells _____30.Which disease is caused by LEPTOSPIRA
examples plants, animals, fungi and protest INTERROGANS? Leptospirosis
_____8.To which kingdom do humans belong? a.Tuberculosis b.Liptospirosis c.Hepatitis
a.Chordata b.Invertebrate c.Animalia _____31.Which animal has urine with a greatest
_____9.To which phylum do humans belong? concentration of LEPTOSPIRA bacteria?
a.Chordata b.Invertebrate c.Animalia a.Bats c.Rats c.Snake
_____10.To which class do human belong? _____32.What disease is caused by BACILLUS
a.Aves b.Chordata c.Invertebrata ANTHRACIS? Anthrax
_____11.To which order do humans belong?= a.Cholera b.Anthrax c.Syphillis
Carnivora _____33.What animals is usually infected by
a.Carnivora b.Ungulata c.Primata BACCILUS ANTHRACIS?
134
a.Carabaos and cows b.pigs and chicken c.rabbit and _____52.What is the store foods of brown algae?=
dogs LAMINARIN
_____34.What do you call the substance that kill or _____53.What do you call the leaf like bubble
inhibit disease-causing organisms? structure in brown algae that floats to capture
a.Antibiotic b.biochemical agent c.bio suppressant sunlight?
_____35.Which antibiotic are used to treat a.Floater b.Bladderc.Stalk
tuberculosis and certain types of Pneumonia is _____54.The Red algae EUCHEMA MARICATUM is
made from commonly sold in the market and called as_____?
STREPTOMYCES GRISEUS? a.Lato b.Gozo c.Euchema
a.Streptomycin b.Choramphenicol c.Amoxicillin _____55.What is the group of algae which can
____36.What is the other term for sour milk a change color depending or whether they are
product produced with the help of some bacteria? exposed to sunlight?
a.Cheese b.sauerkraut d.Yogurt a.Red b.Green c.Yellow
_____37.What enzyme in the bacteria can change _____56.To what type of waters do red algae are
the milk sugar into lactic acid? usually found?
a.Lactase b.Lactmase c.Maltase a.Fresh water b.Cold water d.Marine water
_____38.What will happen to protein in milk if it is ______57. What substance is produced by red algae
acted by t an enzyme in the bacteria introduced in this substance are used to manufacture plastics?
it?= it will CURDLE. a.Laminarin b.carageenan c.agar
_____39.Which process is described as breaking 58.A substance derived from red algae that is used
down or removal of pollutants with the use of to manufacture gulaman?
microorganism? a.Laminarin b.carageenan c.agar
a.Bioremediation b.biomagnification c.Biosynthesis _____59.Which phototrophic protist represented by
_____41.The edible TAB-TAB in Luzon is what type PYRODINIUM BAHAMENSE can cause red tide?
of CYANONACTERIA? a.DINOFLAGELLATES b.Kelps c.diatoms
a.Nostoc b.Sporogyra c.Kelp _____60.Which organisms is directly affected by red
_____42.Which group of bacteria established a tide?
SYMBIOSIS OR SYMBIOTIC relationship with root a.CLAMS and MUSSELS b.fishes c.corals
nodules of LEGUMES or LEGUMINOUS plants? ______61.Which organisms is the common example
a.Cynobacteria b.gram positive bacteria c.Rhizobium of euglenoids a microscopic and unicellular protist
_____43.Which kingdom of living things come from that can move through a WHIP LIKE structure
unrelated species?= PROTISTA or PROTIST known as FLAGELLA?
a.Monerans b.Protist c.Fungi a.Amoeba b.Euglena c.Sporozoans
_____44.Which groupings is considered by many _____62. What is the mode of locomotion of
biologist as artificial groupings euglena?
a.Monerans b.Protist c.Fungi a.Cilia b.Pseudopodia c.Flagellas
______45.Which group of protist can produce their _____63. Heterotrophs protist with no structure for
own food? movement like RADIOLARIANS, FORAMINIFERANS
a.Phototrophs b.Chemotrophs c.Heterotrophs and AMOEBA.What structure are used by these
_____46.Which group of protist feed on other protist to move? PSEUDOPODS or PSEUDOPODIA
organism? a.Cilia b.Psedupodia c.Flagela
a.Phototrophs b.Chemotrophs c.Heterotrophs _____64. What does Pseudopods means?
47.Which group of protist are non motile and form a.False body b.false feet c.false hand
spores and are all parasitic? _____65.ENTAMOEBA HISTOLYTICA can cause
a.Phototrophs b.Chemotrophs c.Sporozoans amoebic dysentery or amoebiasis, what protist
48.What are the three examples of phototrophs? causes this disease?
Algae, Dinoflagellates, Euglenoids a.Algae b.amoeba c.paramecium
Types of Algae= Green, Golden, Brown, Red _____66.Which heterotrophic protist can move
_____49.What is the form of food stored by brown through cilia?
algae? LEUCOSIN OIL or CHRYSOLAMINARIN a.paramecium b.algae c.amoeba
_____50.What species of brown algae can grow into _____67.What are hairlike structures distributed in
30 meters in length? a body of a paramecium which they use to get food?
a.Kelps b.Sargasum c.Diatoms CILIA
_____51.What algae are used in ice cream a.flagella b.flagella c.cilia
products, candy and cosmetics? ______68.What species of sporozoans can cause
a.Red Algae b.Brown Algae c.Green Algae malaria?
a.Plasmodium b.amoeba c.dinoflagellates
135
_____68.What type of mosquito carry plasmodium? _____86.What do you call the structure in the tip of
a.Anopheles b.aedes c.both a and b mosses that form spores?
_____69.What group of heterotrophic protist are a. spores b.capsules c.sori
unicellular and has threadlike flagella to move?= _____87.How is the growth of non vascular plants
a.FLAGELLATES b.ciliates c.amoeboids compared?= CARPETLIKE
_____70.Which flagellates lives in the guts of a.carpet like b.needle like c.velvet like
termites destroying digesting woods that the _____88.What is the other term for dried
termites consume?=TRICONYMPHA SPHAGNUM used from wrapping plants and
_____71.Which kingdom do KABUTE OR breakable items?= PEAT MOSS
MUSHROOM BELONG? _____89.Old dead sphagnum is also called= PEAT
a.Fungi b.Protist c.monera _____90.How do ferns reproduce?
_____72.Which group of organism can undergo a.through spores b.throug stem c.through leaves
asexual reproduction by forming buds? _____91.What water fern served as host to
a.Fungi b.Protist c.monera anabaena cyanobacteria?
_____73.Which organisms are capable of forming a.lygodium b.azolla c.whisk fern
many spores used in asexual reproduction? _____92.What fern fiber is used in Philippine
a.Fungi b.Protist c.Monera HANDICRAFT?
_____74.Some fungi feed on decaying organic a.Birds nest b.nito c.fox tail
matters, these organisms that feed on decaying _____93. What groups of plants have seeds which
organic matter are called ______________ ? = are enclosed in a cone not enclosed in a fruit?
SAPHROPHYTES a.angisosperm b. gymnosperm c. mosses
a.Saprophytes b.Detritus c.parasites
_____75.What do you call the structure of fugal _____94.What is the meaning of gymnos?
body with a root like structure called RHIZOIDS and a.wild b.naked c.enclosed
has the ability to absorb nutrients for the fungi by _____95.What is the meaning of sperm?
extending downward into the substrate? a.fertile b.seed c.fruit
a.Hyphae b.rhizoids c.mycelium _____96.What is the shape of gymnosperm’s
76.What structure in fungi is used for sexual leaves?
reproduction?= HYPHAE specifically the male and a.cone b.needle c.liver
female hyphae. _____97.What gymnosperm are short, palm like
_____78.What drug is developed from fungi its an plant growing in the Philippines used as landscape
antibiotic drug? plants?
a. Penicillinb.Cholaramphenicol c.Analgesic a.cycad b.cypress c.redwood
_____79.What toxin is produced by the fungi 98.What gymnosperm plant is represented by
aspergillus flavor which is toxic to animals and WELWITSCHIA which can be found in
humans? NAMBIA,SOUTHWESTERN AFRICA= GNETOPHYTES
a.Aflatoxin b.endotoxin c.penicillium _____99.What are other example of gymnosperm ?(
_____80.What part of plant cells contain Pine trees or the conifers)
chlorophyll? a.mosses and liverworts b.pine trees c.ferns
a.Robosome b.Choroplast c.cell wall _____100.What do you call the group of plants that
_____81.What is the main component of plant cell bear fruits and flowers? ANGIOSPERM
wall? a.angiosperm b.gymnosperm c.non flowering
a.Chitin b. Cellulose c.cholesterol _____101.What is the other term for FLOWERING
_____82.What do you call plants with no tissues to PLANTS?
conduct water and food, they don’t have true a.gymnosperm b.amgiosperm c.seedless vascular
roots,true leaves and true stems? plants
a.Lower vascular b.non vascular c.vascular _____102.What do you call the flowering plants
_____83.What do you call plats with tissues that that live for a year or one growing season and die
conduct water and food?= VASCULAR like rice and corn?
a.Lower vascular b.non vascular c.vascular a.biennials b.perrenials c.annuals
_____84.What do you call the root like structures of _____103.What do you call the flowering plants
NON VASCULAR plants attached to the places that develop roots stems and leaves in the fisrt year,
where they live? produce seeds in the second year then die?
a.rhizoid b.hyphae c.rhizum a.biennials b.perrenials c.annuals
_____85.What are the three common examples of ______104.What do you call the flowering plants
NON VASCULAR PLANTS?= Hornworts, Liverworts that live for many years and develop wood stems?
and MOSSES a.biennials b.perrenials c.annuals
136
_____105.What are the three basic arrangement of _____125.What aquatic organisms represent the fresh
leaves in flowering plants? = opposite just like water Cnidarians?
coffee, madri de cacao ,radially just like garlic and a. hydra b.jelly fish c.corals
onions and alternate just like in guava. _____126.Which example of cnidarians can form
colonies and can build a coral reef?
_____106.What do you call the flowering plants
a.corals b.sea anemone c.jelly fish
with only one SEED cotyledon?= MONOCOT OR
_____127.Platys is a greek word which
_____107.What do you call the flowering plants means_______________?
with two cotyledons? DICOTYLEDONS a.plain b.flat c.ribbon
_____108.What is the leaf venation of monocots? _____128.Helmins is a greek word which
a.Parallel b.variegated c.netted means______________?
_____109.What is the leaf venation of dicots? a.dirt b.worm c.slimy
a.Parallel b.netted c. variegated _____128.To which phylum do flatworm belong?
______110.Which group of plants are source of _____129.Which Phylum consist of flat and ribbon like
staple foods for Filipino example are rice and corn? organism?= Platyhelminthes
a.angiosperm b.gymnosperm c.both and b _____130.Which group of platyhelminthes are free
living which lives in mist surfaces under rocks and
_____111.Which regions in the Philippines supplies
ponds river and even aquariums?
banana to the country?
a.planaria b.flukes c.tapeworm
aDavao b.cotabato c.Nueva ecija _____131.Which example of platyhelminthes are
_____112. Which regions in the Philippines supplies parasites that live in other animals including humans?
mango to the country? a.flukes b.planaria c.tapeworm
a.Batangas b.Guimaras c.Mountain province _____132.Which example of a parasitic flatworm have
_____113. Which regions in the Philippines supplies no digestive system it can have fish, cows and pigs as
pineapple to the country? hosts human can be infected by them through
a.Bukidnon b.South Cotabato c.Davao uncooked fish, beef or pork?
_____114.Which plant an angiosperm contain a.tapeworm b.tapeworm c.planaria
hydrocyanic acid which is a poisonous substance? _____133.To what phylum do roundworms belomg?
a.Annelida b.Nematoda c.Platyhelminthes
a.Cassava b.taro c.potato
______133.What type of worms belong to phylum
Animals are Eukaryotic organisms because their genetic
nematode?
materials are enclosed in a nucleus
a.round worms b.segmented worms c.bristle worms
Animals are multicellular organisms because they are
_____134.What is the other term form round worms as
composed of many cells upto trillions for big ones.
organisms having cylindrical and slender bodies?
_____115.What are the two major group of animal?
a.nematodes b.nematocite c.nematomes
a. Vertebrates and Invertebrates b.Chordates and Non
_____135.Which example of round worms can infect
chordatesc.Aquatic and Terrestial
dogs and cats?=HEARTWORMS
_____116.What is the simplest of all animals?
136.What are other example of roundworms?=
a.corals b.sponges c.worms
hookworms,pinworms and ascaris
_____117.To what Phylum do sponges belong?
_____137.Which phylum in the animal kingdom
a.porifera b.cnidaria platyhelminthes
characterized by a segmented or repeated body parts?
_____118.What do you call the hard skeleton present in
a. Mollusk b.Annelida c.Arthropoda
the bodies of a sponge?
_____138.What is the other term for segmented
a.spines b.spicules c.exoskeleton
worms?
_____119.What are the main component of SPICULES?
_____138. Which example of SEGMENTED worms are
Silica and calcium carbonate
mostly marine living in tubes attached to rocks or sand?
_____12O. What motile structure is used by sponge to
a.Earthworms b.polychaetes c.tube worms
bring foods to its collar?
_____139.What are the common examples of
a.flagella b.cilia c.pseudopodia
POLYCHAETES?= SANDWORMS
______121.What structure of sponge is used as entry
______140.Which type of worms have nervous
point for food and exit of waste materials?
circulatory digestive and excretory systems the internal
a.oscul b.osculum c.mouth
structure are found in each segments?
_____122.What do you call the stinging cells of Phylum
a.Nematods b.annelids c.flatworms
cnidarians?
_____141.What process is involve when gas exchange
a.nematosyst b.nematoblast c.cyst
happens between the body an earthworm and the
_____123.What Phylum in the animal kingdom contain
surrounding environment?
tentacles with stinging cells?
a.Osmosis b.Diffusion c.Facilitated Diffusion
a.Cnidaria b.Molluska c.Porifera
_____142.What do you call the common example
_____124. What is the common example of Cnidarians
segmented worms that lives in the ground and can
which contain nematocyst and can cause fatal attacks?
produced waste called CASTINGS?
a.corals b.sea anemone c.jelly fish
a.leeches b.earthworms c.ascaris

137
_____143.What do you call the waste eliminated by _____163.Which process describes how arthropods
earthworms that can serve as fertilizers? grow by shading the old exoskeleton to give way to
_____144.What do you call the blood sucking annelids new skeleton?
which contain blood suckers and can secrete blood a.regeneration b.molting c.external growth
clotting factor chemicals used in medicines? _____164.A class of arthropods which is represented by
a.pinworm b.leeches c.hookeworm water fleas, crabs shrimps, barnacles?
_____145- Which phylum in the animal king are consist a.crustaceans b.insects arachnids
of soft-bodied invertebrates with most of them _____165.What structure used by crustaceans to bite
covered by a shell? and grind food?
a.Arthropoda b.Mollusca c.Echinodermata a.mandible b.mouth c.antenna
_____146.What are the three body parts of MOLLUSK? _____166.What structure by aquatic crustaceans that
Muscular foot, mantle, visceral mass use for breathing?
_____147.Which body part of mollusks produce the a.lungs b.gills c.trachea
shell? _____167.What are the three major classes of
a.mantle b.muscular foot d.visceral mass Arachnids?
_____148.What are the three classes of MOLLUSKS?= _____168.What is the largest group of arachnids?
GASTROPODS, BIVALVES, CEPHALOPODS a.spiders b.scorpions c.tick ad mites
_____149.What is the other term for gastropods? _____169.To which class of phylum arthropoda do
a.bivalve b.univalve c.cephalopods SCORPIONS ,TICK AND MITES belong?= ARACHNID
_____150.What does the term BIVALVES MEAN?= TWO _____170.Whic member of arachnids has a poisonous
SHELLED sting?= SCORPIONS
_____151.What does the term univalve mean?= ONE _____171.Which arthropods are close relatives of
SHELLED insects have long worm like segmented bodies, have
_____152.To which class do squid octopuses and two pairs of legs per segment and roll up when
cuttlefish belong disturbed and feed on decaying matter?=MILLEPEDES
a.cephalopods b.bivalve c.univalve _____172. Which arthropods are close relatives of
_____152 What class of mollusk are considered as the insects have long worm like segmented bodies, have a
most active and fast moving? pair of legs per segment they have containing claws
a.univalve b.bivalve c.cephalopods and feed on earthworms, even other arthropods and
_____153.in which group would you classify halaan the other insects?= CENTIPEDE
giant African snail and tridacna or taklobo? _____173-What is the largest group among arthropods?
a.gastropod b.bivalve c.univalve a.insects b.arachnids c.crustaceans
_____154.Which phylum in the animal kingdom are _____174.members of arthropods with 3 body rgions,3
consist or organism that are all found in marine(ocean, pars of legs, a pair of antenna and one to two pair of
sea) waters? wings?
a.echinodermata b.mollusk c.arthropoda a.insects b.arachnids c.crustaceans
_____154.To which phylum do star fish brittle stars, sea _____175.Insects are found in almost all environments
cucumber and sea orchins belong? EXCEPT in___________________?
a.Mollusca b.echinodermata c.arthropoda a.cold regions b.marine water c.tropical biomes
_____155.What do you call the extension in the body _____176.An important insect which have a member
of echinoderms which radiate from the center of the which pose danger to people of all ages. MOSQUITO
body and composed of hard calcium? Life Cycle of Mosquito
a.spines b.spikes c.tubules EGG---------LARVA ( PLURAL LARVAE) also called
_____156.What do you call the internal skeleton of WRNGGLER-------PUPA-----ADULT
echinoderms?= ENDOSKELETON _____177- To which phylum in the animal kingdom has
_____157.What do sea stars are capable of doing when four characteristics that are present in any of the stages
an arm is cut?= REGENERATION in their life cycle, these are the notochord, the dorsal
_____158.What do you call the water filled tubes in hollow nerve cord, gill slits and post-anal tail?
echinoderms used for getting food? b.imvertebrata c.Mollusca
a.tube feet b.tube system c.spikes a.chordata
_____159.What do the sea urchins eat in coral reef _____178-In many notochord what structure will arise
ecosystem? from the notochord as the organism matures?
a.algae b.crustacean c.small fish a.Brain b.spinal cord c.backbone
_____160. What do you call the organisms with jointed _____179.What organisms of phylum chordata have
legs? notochord, tail and gill slits only in the embryo stage?
_____161. What phylum in the animal kingdom a.humans b.birds c.reptiles
considered to be the most successful of all the animal _____180.Which member of chordates have only the
phyla? gill slit present but the larval stages have all the four
a.insecta b.arthropoda c. nidaria feautures?
_____162.The hard outer covering of arthropods that a.lancelets b.tunicates c.jawless fish
consist of chitin=EXOSKELETON TUNICATES are filter feeders, taking in food suspended
in water

138
_____181.Which group of chordates forms the large _____201.Which group of mammals bear fully
group? developed young inside the mothers uterus attached to
a.invertebrates b.vertebrates c.marine the placenta and received nourishment from the
_____182.In many vertebrate what will replaced the mother?
notochord? BACKBONE or VERTEBRAL COLUMN a.Placental b.Monotreme c.Marsupial
The protected nerve cord enlarges at the anterio to _____202.Which is known as the most highly developed
develop into the BRAIN. of all animals, they can walk erect and high degree of
FISHES(183.Which vertebrates found in salty, fresh cold intelligence?= PRIMATES
or even hot water? _____203.Which ecosystem has the highest
a.fishes b.reptiles c.mollusk biodiversity?
_____184.To which class in the Phylum chordate do a.Tundra b. Savanna c.Tropical rain forest
jawless fishes belong? D. deciduous forest
a.Chondrichthyes b.Osteichthyes c.Agnatha _____204.What is the acronym to remember the
Examples: LAMPREYS decline of species?= HIPPO
HAGFISH- is wormlike and use tooth like tongue to eat H-habitat destruction
dead organisms I-Invasion of introduced species
_____185.Which group of fish include the rays, skates P-Population increase
and sharks? P-Pollution
a.bony fishes b.cartilaginous fish c.jawless fish O-over collection
_____186.What do you call the soft flexible material _____205.What do you call the producers in aquatic
that constitute the skeleton of cartilaginous environment which is composed of microscopic algae?
fishes?=CARTILAGE _____206.Who eats the phytoplankton?
_____187.Whale shark the largest fish is locally know in a.Zooplanktons b. nektons c.Top consumer
the Philippines as ? _____207.Which organisms are the main producers in
a.dugong b.balyena c.butandeng terrestrial or in land?
_____188.To which class do bony fishes belong?= a.algae b.plankton c.grasses
OSTEICHTHYES _____208.Which component of food web and food
_____189.Animals which has a “double life” live both in chain eats the producers?
water and in land are called? a.Decomposers b.Consumers c.Autotrophs
a.Amphibian b.Reptile c.aves _____209.When organisms died are being decomposed
_____190.Which amphibians can live in water or bury by bacteria and fungi they are termed as____?
themselves in moist soil? a.Ditritivore b.decomposers c.saprophyte
a.frog b.salamande c.toad _____210.What is the total mass of organisms in a food
_____191.Which amphibians live in forest floors under chain or in a food web?
rocks and decaying logs? a.dry weight b.biomass c.bio capacity
a.toad b.caecelians c.salamander _____211.Whic process is the evaporation of water in
_____192.Which class of chordate lay eggs with shells the leaves of plants?
to protect them from drying .The also have smooth or a.Evaporation c.Transpiration c. Dissipation
rough scales for protection and loss of water? _____212.What gas constitute 79 % of all gases in the
a.Aves b.birds c.amphibian atmosphere the most abundant?
_____193.Which group of chordates are adapted to a.Nitrogen b.Carbon c.Oxygen
fly?= Birds ____213.Which gas is used by plants in the process of
_____194.What class of Phylum Chordata ahave hairs photosynthesis?
and can produce milk?= Mammals a.Carbon b.Carbon dioxide c.Oxygen
_____195.Which gland present in mammals is able to _____214. Which gas is released by plants as by product
produce milk to nourish their young? of photosynthesis?
a.Endocrine gland b.sweat gland c.mammary gland a.Carbon b.Carbon dioxide c.Oxygen
_____196.What do you call the group of mammals _____215.In the process when protein is acted upon by
which lay eggs like birds? bacteria it is converted to_______?
a.Ethurian b.Placental c.Monotremes a.Nitrates b.Ammonia c.Nitrite
_____197.What group of mammals are also called as _____216.Which process in the cultivation of a single
the pouched mammals? crop in large areas?
a.Monotremes b.placental c.marsupials a. Hydroponic b.Monoculture c.Horticulture
_____198.Which marsupials mammal live in North _____217.Which chemicals are sprayed to plants that
America? move up the food pyramid and accumulate in the
a.Opossum b.Koala bear c.pand bodies of animals?
_____199. What do you call comprise the largest group a.Insecticide b.fertilizers c.water
of mammals? _____218.Which part of the energy pyramid is the
a.Marsupial b.Monotreme c.Eutherian concentration of chemicals the least?=
_____200.Eutherian mammals are commonly called the _____219.What happens to the amount of chemicals as
____________________? it moves from the base to the top of pyramids?
a. Placental b.Marsupial c.Monotreme a.decreases b.increases c.stable

139
_____220. Which substance found in water will ans: LA INDOLENCIA DE LOS FILIPINOS (the
increased growth of algae and other aquatic plants? indolence of filipinos)
a.Insecticide b.Fertilizer c.Toxins
_____221. Which type of fertilizer is suitable to replace 13. Pen name used by Rizal in his writings in La
artificial fertilizer because it is ecologically friendly? Solidaridad
a.Inorganic b.Organic c.synthetic
ans: LAON LAAN/ LAONG LAAN
JOSE RIZAL
14. spanish governor who SIGNED the death
1. the gifted physician-novelist of the propaganda? sentence of Dr. Rizal

ans: JOSE RIZAL ans: GEN. CAMILIO G. DE POLAVIEJA

2. the number of languages which rizal could speak? 15. a translation of Rizal's farewell poem written by
Andres Bonifacio in tagalog
ans: MORE OR LESS 22
ans: PAHIMAKAS
3. first written novel of rizal
16. date of founding of the fortnightly newpaper "La
ans: NOLI ME TANGERE (Touch me Not) Solidaridad"

4. place where the novel Noli Me Tangere was ans: FEB 15, 1889
published
17. date of last issue of La Solidaridad
ans: BERLIN (1887)
ans: NOV. 15, 1895
5. city where the novel the El Filibusterismo was
published 18. the first spanish friar to attack " Noli" and " Fili"

ans: GHENT (1891) ans: FRAY JOSE RODRIGUEZ

6. the spanish priest who denounced Dr. Rizal's 19. Place where Dr. Rizal conceived the idea of
novels as enemies of the catholic region? establishing Liga Filipina, a civic association
composed of filipinos
ans: FR. JOSE RODRIGUEZ
ans: HONGKONG
7. a supporter of the propaganda movement and
rizal's best friend 20. helped Rd. Rizal in preparing the constitution of
La liga Filipina
ans: DR. FERDINAND BLUMENTRITT
ans: JOSE MARIA BASA
8. the first poem written by rizal
21. date when rizal was exiled to Dapitan
ans: SA AKING MGA KABATA
ans: JULY 14, 1892
9. the best poem ever written by Dr. Jose Rizal
22. Dr. Rizal dedicated his famous poem " A LA
ans: MI ULTIMO ADIOS (ang huling paalam) JUVENTUD FILIPINA" (to the filipino youth)

10. the most cultured of the reformist ans: THE LIBERAL MINDED STUDENTS AT
THE UST
ans: DR. JOSE RIZAL
23. to whom dr. Rizal send his letter with this line: "I
11. know as "Dimasalang" and " Laon Laan" AM INNOCENT OF THE CRIME OF
REBELLION. I AM GOING TO DIE WITH A
ans: Dr. Jose Rizal TRANQUIL CONSCIENCE"

12. the book written by Rizal in defence of alleged ans: FERDINAND BLUMENTRITT
laziness of the Filipinos
24. The priest who baptized Rizal
140
ans. FR. RUFINO COLLANETS ans: LEONOR RIVERA

25. Philippine president who made December 30 as 37. Jose Rizal died at the age of ____
Day of National Mourning in Rizal honor
ans: 35
ans: EMILIO AGUINALDO
38. The mother of Jose Rizal was imprisoned for
26. The Rizal family had this many siblings how many years due to allegedly poisoning her
cousin-in-law?
ans: 11
ans: 2 YEARS AND 6 MONTHS
27. In 1868, Don Kiko brought Rizal to this
pilgrimage 39. The godfather of Jose Rizal.

ans: PENAFRANCIA ans: REV. PEDRO CASANAS

28. ) Left an impression to Rizal about the sacrifice 40. The ninth child of Francisco and Teodora who
on one's life was an epileptic and died a spinster.

ans: GOMBURZA ans: JOSEFA

29. Rizal started his formal schooling in 41. Jose Rizal's first teacher.

ans: BINAN ans: TEODORA ALONZO REALONDA

30. Our national hero was born on ____ 42. The title "El Filibusterismo" means

ans: JUNE 19, 1861 ans: SUBVERSIVE

31. The complete name of our national hero. 43. The main character of Noli Me Tangere is

ans: JOSE PROTACIO RIZAL MERCADO Y ans: CRISOSTOMO IBARRA


ALONSO REALONDA
44. The main character of El Filibusterismo is
32. The law which provides that "courses on the life,
works and writings of Jose Rizal, particularly his ans: ISAGANI
novels Noli Me Tangere and El Filibusterismo, shall
be included in the curricula of all schools, colleges 45. The novel, Noli Me Tangere is about
and universities, public or private."
ans: RELIGION, LOVE, SOCIETY
ans: RA 1425 / RIZAL LAW
46. The novel, El Filibusterismo is about
33. Jose Rizal was fondly called ____by his family
and friends. ans: POLITICS & REVOLUTION

ans: PEPE 47. The novel, Noli Me Tangere is dedicated to

34. What does filibustero mean? ans: MARIA CLARA

ans: ONE WHO IS AGAINST THE 48. Noli Me Tangere was inspired by
GOVERNMENT
ans: UNCLE TOM'S CABIN
35. In what school did Jose Rizal finish his medical
course? 49. Simoun is a wealthy jeweller who came back to
the Philippines after how many years?
ans: UNIVERSIDAD CENTRAL DE MADRID
ans: 13
36. Jose Rizal's true love who personified Maria
Clara in his novel Noli Me Tangere. 50. Crisostomo Ibarra was a student abroad who had
his homecoming after how many years?
141
ans: 7 A. Uncle Antonio Rivera

51. Pilosopo Tasyo is a character in Noli Me B. Paciano


Tangere personified by whom in reality?
C. Soledad
ans: PACIANO
D. Both Uncle Antonio Rivera and Paciano
52. The heaven-sent financer of Noli Me Tangere.
2) When did Rizal left for Spain to complete his
ans: MAXIMO VIOLA university studies and improved his knowledge of
arts & sciences and his God-given talents?
53. The novel, El Filibusterismo came off the press
with the financial assistance of A. May 3, 1880

ans: VALENTIN VENTURA B. May 3, 1881

54. The girlfriend of Isagani who dumped him for C. May 3, 1882
another man, believing that she has no future if she
marries him. D. May 3, 1883

ans: PAULITA GOMEZ 3) Rizal's departure for Spain was kept secret to
avoid detection by the colonial officers and friars,
55. The famous Rizal monument in Luneta was not but the following below knew that Jose was leaving
the work of a Filipino but a Swiss sculptor. What EXCEPT
was the sculptor's name?
A. His parents
ans: RICHARD KISSLING
B. His sisters
56. Rizal discovered 3 species
C. Uncle Antonio Rivera
ans: DRACO RIZALI (Wandolleck), a specie of
flying dragon, D. Paciano

RACHPHOROUS RIZALI(Boetger), a hitherto 4) In Singapore, Rizal boarded this French steamer


unknown specie of toad named __________, which was sailing to Europe.

APOGONIA RIZALI (Heller), a small beetle, which A. Belvic


were later named after him.
B. Salvadora
57. At what age could Jose Rizal read and write?
C. Hayfong
ans: 2
D. Djemnah
58. The surname Mercado when translated to english
means? 5) In the steamer bound to Europe, Jose tried to
speak French with some passengers but to his
ans: MERCHANT surprise the bookish French which he learned from
Ateneo couldn't be understood. Jose tried to do the
59. Rizal's dog name is following EXCEPT

ans: UZMAN A. Speak it with Latin and Spanish words

60. name of Dr. Jose Rizal's son B. Aid it with much gesticulation

ans: FRANCISCO (he name his son after his father C. Aid it with frequent sketching on paper
Francisco Mercado)
D. Speak it with German and Greek
1) On his last night in Manila, Rizal rode about the
city and its suburbs. He was accompanied by _____. 6) Where did Rizal Rizal hear mass at the sunrise of
his embarkation date to Madrid, Spain?
142
A. Sto. Domingo Church 11) The article that Rizal wrote in Diariong Tagalog
urging his compatriots to love their fatherland just
B. San Agustin church like his prize-winning A La Juventud Filipina

C. Manila Cathedral A. Los Viajes

D. San Francisco Church B. Felicitacion

7) On June 2, 1882, Rizal arrived at the city of C. Amor Patrio


___________ ( Red Sea terminal of the Suez Canal),
whose beautiful moonlight of this picturesque city D. Revista de Madrid
reminded him of his family and Calamba.
12) The article that Rizal wrote in Madrid on
A. Colombo November 29, 1882 which was returned to him
because the Diariong Tagalog ceased publication.
B. Singapore
A. Los Viajes
C. Aden
B. Felicitacion
D. Suez
C. Amor Patrio
8) Rizal landed this place to see the interesting spots
of ____________, the Mediterranean terminal of D. Revista de Madrid
Suez Canal, where he heard the multi-racial
residents speaking a babel of tongues - Arabic, 13) Rizal visited _____ (in Marseilles) where
French, Italian, Spanish, Greek, Egyptian, etc. Dantes (hero of the Count of Monte Cristo) was
imprisoned.
A. Point de Galle
A. Tower of babel
B. Port Said
B. Chateau d' If
C. Colombo
C. Hanging Garden of Babylon
D. Aden
D. Point de Galle
9) In his two-day sojourn in the English colony of
Singapore, what did Rizal particularly admire? 14) Naples: Italy; Marseilles: France; Port-
Bou:_____
A. Their languages and culture they spoke
A. Spain
B. Their customs and traditions that were somewhat
alike to the Filipinos B. Germany

C. The confidence which natives of Singapore C. Japan


seemed to have in their government
D. Singapore
D. Their discipline and strict compliance of
imposed laws 15) Filipinos in Barcelona, some of whom were
Rizal's school mate in Ateneo gave him party at their
10) The editor of Diariong Tagalog who favorite cafe in _____.
congratulated Rizal for the article and requested him
for some articles A. Hotel de Paz

A. Marcelo H. del Pilar B. Plaza de Cataluna

B. Juan Luna C. Las Ramblas

C. Francisco Calvo D. Port-Bou

D. Antonio Luna
143
16) While sojourning in Barcelona, Rizal received a every Saturday evening because the hero was far
sad news about the ______ that was ravaging Manila away and lonely in Madrid. He and other Filipino
and provinces causing Calamba folks to have students played parlor games with his 2 daughters.
afternoon novenas to San Roque and nocturnal
processions and prayers to stop dreadful epidemic, A. Guillermo Puatu
which Spanish authorities were impotent to check.
What was the disease? B. Francisco Calvo Muñoz

A. Dengue C. Ramon Blanco

B. Malaria D. Don Pablo Ortega y Rey

C. Typhoid fever 21) On August 22, 1883, Rizal wrote a lovely poem
entitled A la Senorita C.O. y R. Who was he
D. Cholera dedicating the poem to?

17) On November 3, 1882, Rizal enrolled in A. Leonor Rivera


Universidad Central de Madrid (Central University
of Madrid) in two courses aside from his studies in B. Pilar
painting and sculpture in Academy of San Carlos,
lessons in French, German and and English under a C. Consuelo
private instructor, and practice in fencing and
shooting in the Hall of Arms of Sanz y Carbonell. D. Concepcion
What was the 2 courses that he matriculated in
Universidad Central de Madrid? E. None of the above

A. Philosophy and Letters & Medicine 22) Shortly after Rizal's arrival in Madrid, he joined
the Society of Spaniards and Filipinos, in which the
B. Philosophy and Latin & Medicine members of the society requested him so he wrote a
poem entitled Me Piden Versos(They Asked Me for
C. Philosophy and Latin & Diploma in Surveying Verses). What's the name of the organization?

D. Ophthalmology and Philosophy A. Circulo Hispano-Filipino

18) Rizal invested his three pesetas (peseta is the B. El Consuejo de los Dioses
currency of Spain between 1869 and 2002) on _____
C. Liceo Artistico-Literario
A. A lottery ticket
D. Marian Congregation
B. Wine
23) Rizal as lover of books was able to build up a
C. Food fair-sized private library. His collection of books, to
name a few, were The Bible, Hebrew Grammar,
D. Books Lives of Presidents of the United States from
Washington to Johnson, History of the French
19) The historic waterway Suez Canal, which was Revolution, Ancient Poetry, The Byzantine Empire,
traversed by Rizal for 5 days, was constructed by the The Characters of La Bruyere, The Renaissance,
famous Frenchman named __________. Uncle Tom's Cabby Harriet Beecher Stowe, Works
of Alexander Dumas, and so on. Whos is the owner
A. Ferdinand Magellan of the second-hand book store where Rizal
purchased books from?
B. Ferdinand de Lesseps
A. Señor Roses
C. Miguel Lopez de Legazpi
B. Francisco Calvo Muñoz
D. Juan de Salcedo
C. Juan de Salcedo
20) A liberal-minded Spaniard and Civil Governor of
Manila (during General Carlos Ma. de la Torre's D. Jose M. Cecicilio
administration) whom Rizal used to visit the home of
144
24) In 1883, Rizal joined the Masonic Lodge Acacia student and a member of a rich family of San
and his Masonic name is Dimasalang. His reasons Miguel, Bulacan.
why he became a Mason: (1) the abusive bad friars
in the Philippines and he needed the help of the A. Don Miguel Morayta
Masons as shield to use in his fight against evil
forces of tyranny. On November 15, 1890, he B. Señor Eusebio Corominas
became a Master Mason in ______.
C. Paz Pardo de Tavera
A. La Liga Filipina
D. Maximo Viola
B. Lodge Solidaridad
29) During his week-stay in Paris, Rizal befriended
C. Circulo Hispano-Filipino _________ the editor of La Publicidad.

D. Propaganda Movement A. Don Miguel Morayta

25) At the National Exposition of Fine Arts in B. Trinidad Pardo de Tavera


Madrid in 1884, joined by many painters in Europe
in that art competition, Luna won first prize (gold C. Paz Pardo de Tavera
medal) for his _____.
D. Señor Eusebio Corominas
A. Christian Virgins Exposed to the Populace
30) Owner of La Publicidad and statesman whom
B. Spolarium Rizal made a crayon sketch when Rizal went to Paris
visiting Maximo Viola.
C. Felicitacion
A. Don Miguel Morayta
D. Uncle Tom's Cabin
B. Trinidad Pardo de Tavera
26) Rizal graduated a degree of Licentiate in
Medicine in June 1884 at Universidad Central de C. Dr. Louis de Weckert
Madrid as well as a degree of Licentiate in
Philosophy and Letters in such school with the rating D. Señor Eusebio Corominas
of __________.
31) In October 1885, Rizal living in Paris, where he
A. Aprobado (passing grade) sojourned for about 4 months, worked as an assistant
to ___, leading French ophthalmogist.
B. Bueno (good)
A. Felix Pardo de Tavera
C. Notable (very good)
B. Dr. Javier Galezowsky
D. Sobrasaliente (excellent)
C. Dr. Louis de Weckert
27) After Rizal completed his studies in Madrid,
Rizal went to Paris and Germany in order to D. Trinidad Pardo de Tavera
specialize in ___________.
32) Rizal discussed with Luna, the great master of
A. Optometry brush, by posing as model in several painting. In
Luna's canvas "The Death of Cleopatra posing as
B. Philosophy and Letters a/an _____.. Rizal discussed with Luna, the great
master of brush, by posing as model in several
C. Ophthalmology painting. In Luna's canvas "The Death of Cleopatra"
posing as a/an _____.
D. Philosophy and Latin
A. Ferdinand Magellan
28) After completing his studies at Universidad
Central de Madrid, on his way to Paris for further B. Lapu-Lapu
study in Ophthalmology, he stopped at Barcelona to
visit his friend named ___________, a medical C. Dagohoy

145
D. Egyptian priest B. Travels in the Philippines

33) In another Luna's great painting "The Blood C. William Tell


Compact" Rizal posed as ______ while Dr. Trinidad
Pardo de Tavera posed as Legazpi D. The Count of Monte Cristo

A. Spanish priest 38) In Berlin Rizal was enchanted because of the


scientifice atmosphere and the absence of racial
B. Lapu-Lapu prejudice. And he came in contact with the German
scientist-traveler named ___________, who was the
C. Dagohoy author of Travelers in the Philippines which Rizal
read and admired for such author foretell the
D. Sikatuna downfall of the Spanish rule and the coming of
Americans to the Phillippine shore.
E. Egyptian priest
A. Dr. Feodor Jagor
34) In Heidelberg, Rizal worked in the clinic of
__________, famous Polish ophthalmologist B. Alexander Dumas

A. Felix Pardo de Tavera C. Hans Andersen

B. Dr. Javier Galezowsky D. Beecher Stowe

C. Dr. Louis de Weckert 39) Rizal lived in Berlin not just as mere student or
a curious tourist, but for the following reasons
D. Trinidad Pardo de Tavera EXCEPT

35) The Austrian professor, Director of Ateneo of i.) to gain further knowledge of opthalmology and
Leitmeritz, whom Rizal had had long and frequent sciences & languages
correspondence as their beautiful friendship lasted
all their lives. ii.) to observe the conditions of the German nation
and associate with the German scientists and
A. Dr. Javier Galezowsky scholars

B. Dr. Louis de Weckert iii) to finish his novel El Filibusterismo

C. Jose Alejandrino A. I only

D. Professor Ferdinand Blumentritt B. Ii only

36) Rizal sent two books entitled "Arithmetica" C. Iii only


written in Spanish and Tagalog by University of
Santo Tomas Press in 1868. The author was D. I and ii
__________, a native of Santa Cruz, Laguna.
40) The winter of 1886 was his darkest winter in
A. Felix Pardo de Tavera Berlin for the following reasons EXCEPT:

B. Rufino Baltazar Hernandez i.) He ate only one meal a day consisted of bread
and water or some cheap vegetable soup
C. Jose Alejandrino
ii.) The diamond ring from Saturnina got lost
D. Marcelo H. del Pilar
iii.) He had no money arrived from Calamba and
37) Rizal stayed in Leipzig from August 14, 1886- could not pay for his landlord
October 29, 1886, he translated Schiller's
___________ from German to Tagalog so that A. I only
Filipinos might know the story of Swiss
independence. B. Ii only

A. Fairy Tales C. Iii only


146
D. I and ii and armed with letter of recommendation from
Blumentritt, years later He spoke highly of Rizal,
41) How much did Rizal owed to Maximo Viola so "whose genius HE so much admired."
that the novel Noli Me Tangere could be printed?
A. Alexander Dumas
A. P100.00
B. Eugene Sue
B. P150.00
C. Beecher Stowe
C. P200.00
D. Norfenfals
D. P300.00

42) In Rizal's Europe tour, he (together with Viola)


visited such because of the following reasons: 46) After the publication of Noli Me Tangere, Rizal
was warned not to return home by the following
i) to further study opthalmology and sciences & people EXCEPT:
anguages
i.) Silvestre Ubaldo
ii) Rizal was interested in botany to study numerous
varieties of extraordinary beauty and size ii) Josephine Bracken

iii) to visit Dr. Adolph B. Meyer iii) Paciano and Jose M. Cecilio

A. i only A. i only

B. ii only B. ii only

C. iii only C. iii only

D. ii and iii D. i and ii

43) In Leitmeritz, when Rizal and Viola was visiting 47) Rizal, deciding to return home, left Rome by
Professor Ferdinand Blumentritt who was amazed by train for Marseilles, a French port. And on July 3,
Rizal's fluency in German language that Rizal was 1887 he boarded WHAT STEAMER?
asked how long did he study German. Then Rizal
answered eleven months. A. Tabo

A. Alcalde mayor B. Djemnah

B. Governor general C. Hayfong

C. Gobernadorcillo D. Salvadora

D. Burgomaster 48) In Saigon, on July 30, 1887, he transferred to


ANOTHER STEAMER, which was Manila-bound.
44) In Prague, as part of Europe tour, Viola and What was the steamer he was riding?
Rizal visited the famous cave where the Catholic
saint was imprisoned. Who was that saint? A. Tabo

A. San Antonio Padua B. Djemnah

B. Saint Francisco de Assisi C. Hayfong

C. Saint Pascual Bailon D. Salvadora

D. San Juan Nepomuceno 49) In his hometown (Calamba), patients from


Manila and provinces flocked to such town, which
45) One of the greatest novelists in Europe (Austrian Rizal came to be called ___ because he came from
novelist) whom Rizal and Viola visited in Vienna
147
Germany, busy attending his lucrative medical C. I and ii
practice.
D. All of the above
A. Doctor Laon
54) When Rizal was exiled in Dapitan, the different
B. Doctor Dimasalang perspectives of Rizal's "many-splendored genius"
were enhanced such as the following EXCEPT:
C. Doctor Uliman
A. Rizal as Educator
D. Doctor Soliman
B. Rizal as a Traveler*
50) While the storm over the Noli was raging in fury,
Rizal was not molested in Calamba because of this C. Rizal as a Farmer
Governor-General that assigned a bodyguard for
Rizal. Who was that Governor-General? D. Rizal as Inventor and a Scientist

A. Terrero 55) What is Jose Rizal's height?

B. Blanco A. 4' 9"

C. Miguel Lopez de Legazpi B. 4' 10"

D. Juan Salcedo C. 4' 11"

51) The lieutenant who defended Rizal in the court D. 5' 0"
whom the later chose because the former's name was
familiar to him, for the former was the brother of 56) When was Rizal arrested and deported to
Rizal's bodyguard in 1887 Dapitan (without benefit of trial)?

A. Juan Salcedo A. June 21, 1892

B. Blanco B. June 26, 1892

C. Pio Valenzuela C. July 3, 1892

D. Luis Taviel de Andrade D. July 7, 1892

52) The lottery prize that Rizal won. 57) Rizal was accused of this/these crime or crimes.

A. P10,000 i. rebellion

B. P20,000 ii. sedition

C. P30,000 iii. illegal association

D. P40,000 A. I only

53) In his aspiration to reconcile Rizal with the B. I and ii


church, Fr. Pastells sent one or the following:
C. I and iii
i. Father Obach
D. All of the above
ii. Father Jose Vilaclara
58) On July 31, 1896, Rizal's four year exile in
iii. Father Paula de Sanchez Dapitan came to an end. At midnight of that date he
left, together with Josephine, Narcisa, Angelica
A. I only (Narcisa's daughter), his three nephews, and 6 of his
loyal pupils, on the steamer named ___.
B. Ii only
A. España
148
B. Castilla a. Amenhotep

C. Isla de Luzon b. Hatsheput

D. Isla de Panay c. Tutankhamen

59) one of the priests who visited Rizal before the d. Cleopatra
latter was executed whom Rizal made an image of
the Sacred Heart of Jesus during the latter's Ateneo 4 What group of people created the first alphabet?
days
a. Chaldeans
A. Fr. Antonio Rosell
b. Assyrians
B. Father Luis Viza
c. Israelites
C. Fr. Federico Faura
d. Phoenicians
D. Father Jose Vilaclara
5Which king united Upper and Lower Egypt?
60) The exact time Rizal died when he was shot at
Bagumbayan Field by firing squad a. David

A. 6:00 am b. Menes

B. 6:15 am c. Ramesses the Great

C. 6:30 am d. Hatshepsut

D. 7:03 am 6 Who was the Egyptian good of wisdom and


writing?
ANS KEY:
CCADDADBCCCDBABDAABDCAABBDCDDA a. Amon-Re
CDDBDBCACBDDDDDBBCCADBBBCDDAB
b. Thoth

c. Isis
FINAL COACHING SOC SCI PART1 (1-200)
d. Osiris
1.What is largest lake in Africa?
7 Who was the king of Babylon from about 1792 to
a. Lake Kilimanjaro 1750 BCE.?

b. Lake Atlas a. Shalmaneser

c. Lake Victoria b. Sargon

d. Lake Sahel c. Hammurabi

2 Who was the first female pharaoh? d. Nebuchadnezzar

a. Hatshepsut 8 Which of the following is NOT a continent?

b. Thutmose II a. Africa

c. Ramesses the Great b. North America

d. King Tut c. Europe

3 Which Pharaoh tried to make Egypt monotheistic? d. Cental America

149
9 What was added to Victoria's list of titles in 1876? 15Who built the Tower of London?

a. Empress of India a. William the Conqueror

b. Queen of Ireland b. Maria Stuart

c. Monarch of Canada c. Henry VIII

d. Duchess of Australia d. Oliver Cromwell

10 Which people group minted the world's first 16Who was the god of the sun?
coins?
a. Anubis
a. Lydians
b. Ma'at
b. Hittites
c. Ra
c. Canaanites
d. Horus
d. Phoenicians
17Gladiators were typically _____and _____ .
11 Who was the first ruler of Akkadia?
a. trainers and emperors
a. Ashurbanipul
b. patricians and plebeians
b. Nebopolassar
c. brother and sister
c. Sargon
d. slaves and criminals
d. Gudea
18The largest pyramid in Egypt, known as The Great
12 In 539 BCE., which Persian ruler defeated the Pyramid was built for which ruler?
Neo-Babylonian Empire?
a. Menes
a. Cyrus
b. Tut
b. Hammurabi
c. Khufu
c. Xerxes
d. Tutmose II
d. Darius
19Tradition says that early Rome was ruled by how
13 Tyre was a city in Phoenicia that was famous for many kings?
a purple dye that came from what source?
a. One
a. Beetles
b. Two
b. Quartz
c. Seven
c. Ostrich eggs
d. Four
d. Sea snails
20 How many symbols were in the Phoenician
14How many senators are in the U.S. Senate? alphabet?

a. 50 a. 12

b. 435 b. 22

c. 100 c. 26
150
d. 32 27 Which Amendment abolished slavery?

21.The Middle Ages began with the collapse of what a. 14th


empire?
b. 13th
a. Byzantine Empire
c. 15th
b. Roman Empire
d. 16th
c. Greek Empire
28 I am indicated by an arrow pointing upwards with
d. Phoenician Empire the letter 'N' on top. I show the directional
orientation of a map. What am I?
22.Victoria much preferred Benjamin Disraeli to
William Gladstone. a. The Title

a. True b. The Compass

b. False c. The Key

23Victoria was the last monarch in what house? d. The Border

a. Windsor e. The Scale

b. Hanover 29 After what event would America gain control


over Puerto Rico and the Philippines?
c. Tudor
a. Civil War
d. Stuart
b. Mexican - American War
24What is the name of the desert that stretches
across most of North Africa? c. World War I

a. Savanna d. Spanish - American War

b. Sahara 30What country was the birthplace of Protestant


Reformation?
c. Sahel
a. Italy
d. Kalahari
b. France
25 _____ is the idea that the national government
shares power with the state governments. c. Ireland

a. Popular Sovereignty d. Germany

b. Consent of the Governed 31Clearing land for cultivation can sometimes


contribute to desertification.
c. Federalism
a. True
d. Republicanism
b. False
26What is the term for members of the House of
Representatives? 32 What is the name of the religion that Muhammad
created?
a. 2 years
a. Christianity
b. 4 years
b. Islam
c. 6 years
151
c. Muslim c. Germany

33 _____helped slaves escape slavery through the d. United States


Underground Railroad.
39 Victoria ruled from 1837 until what year?
a. Fredrick Douglass
a. 1883
b. Nat Turner
b. 1895
c. Harriet Tubman
c. 1901
d. Sojourner Truth
d. 1912
34Who was the god of the dead?
40 A caliph is _____
a. Anubis/Osiris
a. a tool used to write
b. Ma'at
b. a Muslim holy book
c. Ra
c. a Muslim leader of a geographical area
d. Horus
d. one of the Five Pillars of Islam
35 In the tundra, winters are...
41 Who was the father of Alexander the Great?
a. mild
a. Xerxes
b. cool
b. King Darius III
c. bitterly cold
c. Constantine
d. average
d. Philip of Macedon
36What is the longest river in the United Kingdom?
42Which two civilizations fought in the Punic Wars?
a. Great Ouse
a. The Greeks and the Romans
b. Thames
b. The Byzantines and the Scythians
c. Trent
c. The Carthaginians and the Romans
d. Severn
d. The Hittites and the Persians
37 Which group conquered the Western Roman
Empire? 43 In which country does the White Nile originate?

a. Visigoths a. Ethiopia

b. Huns b. Burundi

c. Mongols c. Sudan

d. Abbasids d. Egypt

38Which nation colonized both India and South 44 Spanish conquistadors came to Mexico to look
Africa for gold from which ancient civilizations: (choose
two)
a. Great Britain
a. Incas
b. France
152
b. Aztecs a. He required conquered people to convert.

c. Vikings b. He had edicts carved into pillars and walls.

d. Mayans c. He had peasants taught how to read and write.

45 Ignatius Loyola began a religious organization d. He made speeches in many places around the
called the Society of Jesus or the . empire.

a. Mormons 51How many times a day do Muslims pray?

b. Moravians a. 2

c. Jesuits b. 7

d. Loyalist c. 5

46 Cardiff is the capital where? d. 1

a. England 52Which battle contributed to the single deadliest


day in US History on the battlefield when General
b. Scotland McClellan lost over 20,000 men in rural Maryland in
Sept. 1862?
c. Northern Ireland
a. Antietam
d. Wales
b. Sherman's March
47 Muslims face_____when praying
c. Appomattox
a. Heaven
d. Bull Run
b. the hospital
53 Sacagawea was a teenage Shoshone Indian who
c. Mecca helped the Lewis and Clark expedition as what?

d. Home a. A cook

48People follow the faith of Islam are known as . b. A house maid

a. Christians c. A guide and translator

b. Muslims d. A hunter

c. Godly 54In which country does the Blue Nile originate?

d. Saints a. Ethiopia

49 Who began to build the Mauryan Empire? b. Lbya

a. King Ashoka Maurya c. America

b. Chandragupta Maurya d. Egypt

c. Prince Ceylon Maurya 55Temperatures in the Sahara can reach as high as


how many degrees Fahrenheit?
d. Mahinda Chakra Maurya
a. 120 degrees
50 In what important way did Ashoka spread
Buddhist teachings? b. 100 degrees

153
c. 135 degrees 61 What was the capital of the Eastern Roman
Empire?
d. 115 degrees
a. Italy
56 Who do Muslims believe was the last messenger
of God? b. Rome

a. Jews c. Constantinople

b. Hebrews d. None of the above

c. Allah 62 In what year did the American, British, and


Canadian troops liberate France from Germany
d. Muhammad during WWII?

57. Another name for people who gathered wild a. 1941


plants and actively pursued animals for food would
be . b. 1942

a. hunter-gatherers c. 1943

b. warriors d. 1944

c. nomads 63 The Iliad and Odyssey are long story-telling


poems called what?
d. Paleo-Indians
a. epics
58What signaled the end of Reconstruction?
b. tragedies
a. Compromise of 1877
c. comedies
b. Amnesty Act
d. books
c. Civil Rights Act of 1875
64Who was the first Secretary of the Treasury?
d. 15th Amendment
a. Thomas Jefferson
59 What are the floating gardens built by the Aztecs
known as? b. Henry Knoc

a. Chinampas c. George Washington

b. Quipu d. Alexander Hamilton

c. Potlatch 65 North Korea's 4 wireless companies are own by


the North Korean government. This is an example
d. Wigwam of:

60 Opportunity cost is? a. mixed economy

a. the loan you choose not to take out b. monoploy

b. the cost for starting a business c. traditional economy

c. the benefit you give up by choosing to do one d. command economy


thing instead of another
e.
d. none of the above
66 What document was written by the Pilgrims that
later influenced the framers of the Constitution?
154
a. Magna Carta b. Second Continental Congress

b. Mayflower Compact c. Constitutional Convention

c. Bill of Rights d. Stamp Act Congress

d. Virginia Declaration of Rights 72 Roman builders used rows of arches in their


buildings to form a curved ceiling called what?
67 The United States has which type of economy?
a. aqueduct
a. market
b. colonnade
b. command
c. dome
c. traditional
d. vault
d. mixed
73 What lines of latitude determine the "tropics"?
68 Which of the following was NOT invented by
Ben Franklin? a. Prime Meridian and Equator

a. Bifocals b. Prime Meridian and International Dateline

b. Poor Richards Almanac c. Tropic of Cancer and Tropic of Capricorn

c. American Flag d. Arctic Circle and Antarctic Circle

d. Lightning Rod 74 When supply and demand are equal, economists


call this condition..
69 Which of the following is true of the Mauryan
Empire under King Ashoka’s rule? a. deficit

a. The Mauryan Empire reached its height during his b. supply and demand curve
reign.
c. loan
b. The Mauryan Empire collapsed due to his laws
and edicts. d. market equilibrium

c. The Mauryan Empire shrank due to him giving 75 Europe can best be described as
back land.
a. a country north of Africa
d. The Mauryan Empire lost financial power under
his rule. b. a continent north of Africa

70 The White Nile and Blue Nile meet where in c. a country west of Asia
Sudan?
d. the place where the Crusades were fought
a. Shedi
76 ____was a Spanish explorer who defeated the
b. Giza Aztec

c. Cartouma a. de las Casas

d. Khartoum b. Columbus

71 The Declaration of Independence was written c. PIzzaro


after which major event?
d. Cortes
a. First Continental Congress

155
77 What religion did the Roman Empire help b. old woman
spread?
c. prophet
a. Islam
d. teacher
b. Judiasm
83 What were the essays written to defend the
c. Christianity Constitution called?

d. Paganism a. Federalist Papers

78 Why is King Tut's tomb so important to b. Treaty of Paris


archaeologists?
c. Bill of Rights
a. It was very large
d. Declaration of Independence
b. He had 30 wives
84 Which problems DID threaten the Roman
c. It was the only tomb found totally intact Empire?

d. King Tut ruled for 50 years a. Arguing military

79 According to the Missouri Compromise, which b. Weak, greedy emperors


state would enter the Union as a free state?
c. Arguing over who was the emperor
a. Florida
d. Each answer is correct
b. Maine
85 Which situation does NOT involve a right
c. California protected by the Bill of Rights?

d. Texas a. The ability to own a gun, as long as the laws are


followed
80 The ____were Spanish soldiers sent to the New
World to claim land and resources b. The guarantee of a well paying job

a. Encomiendas c. Freedom to publish opinions in the local


newspaper
b. de la Casas
d. Freedom to join the church of choice
c. Santa Marias
86 _____is important to Muslims because he came
d. Conquistadors down to Muhammed and told him to spread the
teachings of Islam.
81 This Native American Empire conquered most of
what we today call South America a. Gabriel

a. Inca b. Messiah

b. Aztec c. Shariah

c. Olmec d. Sunni

d. Maya 87 A globe is a model of the Earth in the shape of a


sphere.
82 The Qur'an was written by a _____called
Muhammad. a. True

a. child b. False

156
88 What is the main religion of the countries in the 93 Which hemisphere, the Eastern or Western, has
region of North Africa? the most land area?

a. Islam a. Northern

b. Christianity b. Eastern

c. Hinduism c. Western

d. Judaism d. Southern

89 The second largest sea in the world is located 94 A tax on imports.


south of Cuba. What is its name?
a. Quota
a. Caribbean Sea
b. Embargo
b. Red Sea
c. Tariff
c. Dead Sea
d. Import Tax
d. Mediterranean Sea
95 An imaginary circle around the earth, equally
90 Why was the annual flood of the Nile River distant from the North Pole and the South Pole
important to Egyptian farmers?
a. cartographer
a. It gave them time to rest after harvesting their
crops. b. location

b. It swept away weeds and other trash left on the c. equator


fields after the harvest.
d. longitude
c. It left behind silt that helped fertilized the soil.
95 Which feature of the landscape connects
d. It enabled them to travel by boat to sell their Morocco, Algeria, Tunisia, Libya, and Egypt?
crops.
a. the Atlas mountains
91 At first, Rome had a king. Then, it became a
republic where people voted for their leaders. Rome b. the Sahara
really grew to be large and successful when it was an
. c. the Nile River

a. Village d. the Atlantic ocean

b. Government 96 A measure of distance east or west of a line called


the prime meridian.
c. Kingdom
a. meridian
d. Empire
b. map
92 What was the name of most famous church that
Justinian built? c. longitude

a. The Parish d. location

b. The Church of Christianity 97 A _____occurs when a country spends more on


its imports than it makes from exports
c. The Church of Hagia Sophia
a. trade deficit
d. The Church of Constantine
b. import tariff
157
c. free market d. Amendment 19

d. trade surplus 103 What does the phrase "insure domestic


tranquility" in the Preamble of the Constitution
98 A measure of distance north or south of the mean?
equator.
a. Give land to the homeless
a. latitude
b. Fight wars on foreign soi
b. artifact
c. Keep the homeland at peace
c. Prime Meridian
d. Provide citizens with insurance
d. Equator
104 Which plan for rebuilding the South required
99 During the Tang Dynasty, who ran the schools, 50% of the white males of a state to swear loyalty to
public baths, hospitals and inns? the Union before readmission?

a. The aristocrats a. Ten Percent Plan

b. The Mongols b. Reconstruction Act of 1866

c. Confucius c. Freedmen's Bureau

d. Buddhist monks and nuns d. Wade-Davis Bill

100 A combination of market and command 105 Coins were first produced, in China, under
economy. which dynasty?

a. Mixed Economy a. Han

b. Traditional Economy b. Saing

c. Market Economy c. Silla

d. Command Economy d. Zhou

101 What is one way King Ashoka spread Buddhism 106 When describing how people adapt to their
beyond India? environment and change their environment what
theme of Geography are you describing?
a. He sent his son to Ceylon, who then converted its
King. a. Location

b. He became ascetic, and traveled the country with b. Place


monks.
c. Movement
c. He published many famous texts promoting
Buddhism. d. Region

d. He invited other leaders to India to learn about e. Interaction


Buddhism.
107 Who was the Greek Physician who was known
102 The amendment that allows the President and as the Father of Medicine?
Vice president to be on the same ballot
a. Hippocrates
a. Amendment 14
b. Euclid
b. Amendment 10
c. Eratosthenes
c. Amendment 12
158
108 The branch of government that interprets laws is 113 Locating or being located; position; place
the
a. equator
a. legislative
b. location
b. judicial
c. longitude
c. executive
d. latitude
d. political
114 The Emancipation Proclamation was made to .
109 The Aztec and Inca Civilizations
a. free slaves
a. had advanced societies before the Europeans
arrived b. end the war

b. traded heavily with Asia before the arrival of c. make friends in the South
Europeans
d. help Southern landowners
c. were greatly influenced by their contact with
Africa and Asia 115 Europeans discovered silks and spices while

d. were large but not well organized a. exploring for ways to get to Asia

110 Define the term Culture b. fighting in The Crusades

a. A people’s way of life, including beliefs, customs, c. discovering the New World
food, dwellings, and clothing
d. exploring was to get to Africa
b. A vast, treeless plain in the Artic regions with
very cold winters, cold summers, and little rain or 116 Three important world religions that began in
snow. the region of Southwest Asia include Islam, Judaism,
and
c. Useful material found in nature, including water,
vegetation, animals, and minerals. a. Hinduism

d. Movement by people from one place to another b. Christianity


with intentions of settling in new location.
c. Daoism
111 The Code of Hammurabi was based on the
concept of _____________. d. Buddhism

a. Thou shall not lie 117 This economic system is generally limited to
farming and herding.
b. An eye for an eye
a. Command
c. be nice to others
b. Mixed
d. cruel and unusual punishment
c. Market
112 _____was a Spanish explorer who defeated the
Inca d. Traditional

a. Pizzaro 118 Powers that are shared both by the states and
federal government
b. Cortes
a. Expressed Powers
c. Columbus
b. Concurent Powers
d. de las Casas
159
c. Reserved Powers d. Scales are not accurate

d. Government Powers 124 Powers that the constitution does not give to the
national government that are kept by the states.
119 These gunboats protected Grant's troops when
they crossed the Mississippi and were ships heavily a. Expressed Powers
armored with iron.
b. Concurent Powers
a. tugboats
c. Reserved Powers
b. battleships
d. Government Powers
c. battlecruisers
125 What ended the Texas Revolution?
d. ironclads
a. Treaty of Velasco
120 _____was the first Roman Emperor.
b. Treaty of Guadalupe Hidalgo
a. Constantine
c. Treaty of San Juan
b. Diocletian
d. Adams-Onis Treaty
c. Augustus
126 To move things from one place to another,
d. Pontius Sumerians built

121 The beliefs, customs, laws, art, and ways of a. ziggurats and mosques
living that a group of people share.
b. canals and dikes
a. interdependence
c. wheeled carts and sailboats
b. culture
d. qanata and wadis
c. dominant culture
127 When describing a location using specific
d. region location and relative location what theme of
geography are you using
122 How did the yearly flooding of the Nile River
affect the people of ancient a. Place

Egypt? b. Location

a. It washed away homes and forced the people to c. Region


move to a new location.
d. Movement
b. It left a strip of fertile land on both sides of the
river, and this land was used for e. Interaction

c. It washed fish up on the shore, and the people 128 Define 'sectionalism':
gathered and used the fish for food.
a. Group of several families
123 What is the main problem with using a map to
represent the earth? b. Line of descent through the mother

a. It is too small c. Allegiance to local interests rather than national


interests
b. Distortion occurs
d. Tax imposed on manufactured goods IMPORTED
c. You can't tell what type of map it is into America

160
129 Individuals can not own or start their own a. Buddhism
business in this type of economic system
b. Shintoism
a. Traditional
c. Taoism
b. Command
d. Confucianism
c. Mixed
135 What is the capital of Egypt?
d. Market
a. Cairo
130 Powers specifically granted to the national
government. b. Algiers

a. Expressed Powers c. Tripoli

b. Concurent Powers d. Tunis

c. Reserved Powers 136 Where did the first attack occur to start the Civil
War?
d. Government Powers
a. Fort Sumter
131 The Branch of government that makes the laws
is the: b. Virginia

a. Authoritarian c. Gettysburg

b. Despotic d. Bull run

c. Legislative 137 Who served as vice-president during Franklin


Roosevelt's first two terms as president?
d. Judicial
a. Eisenhower
132 The largest trading partner of the United States
is b. Garner

a. Mexico c. Johnson

b. China d. Rayburn

c. Japan 138 The President is the head of the_____Branch.

d. Canada a. Judicial

133 What are the five categories of the Shariah? b. Legislative

a. Discouraged, Required, Forbidden, c. Military


Recommended, and Approved
d. Executive
b. Allowed, Required, Forbidden, Approved, and
Discouraged 139 Define the term Natural Resources

c. Obligatory, Forbidden, Approved, Discouraged, a. Useful material found in nature, including water,
and Recommended vegetation, animals, and minerals.

d. Forbidden, Allowed, Obligatory, Recommended, b. All of the physical surroundings in a place,


and Discouraged including land, water, animals, plants, and climate

134 What religion's teaching includes Four Noble c. Movement by people from one place to another
Truths and the Eightfold Path? with intentions of settling in new location.
161
d. A people’s way of life, including beliefs, customs, In 1895, which country rose in rebellion against
food, dwellings, and clothing Spain?

140 How long were the Franks in hiding? a. America

a. 3 months b. Russia

b. 1 year c. Cuba

c. 2 years d. Canada

d. 4 years 146 To elect a leader means to...

141 Where did the Franks move after escaping a. impeach


Germany?
b. choose a leader by voting
a. Paris
c. remove a leader from office
b. Warsaw
d. nominate a leader for an office
c. Rotterdam
147 John Locke and Jean Jacques Rousseau would
d. Amsterdam be most likely to support .

142 What is the national capital of Mexico? a. A return to feudalism in Europe

a. Mexico Meridian b. A society ruled by the Catholic Church

b. City of Mexico c. A society in which the people chose the ruler

c. Mexico City d. A government ruled by a divine right monarchy

d. Mexico Court 148 How does rule of law affect U.S. government
officials and institutions?
143 What is the name of the strip of land that
became the site of the Sumerian and Babylonian a. It requires government officials and institutions to
civilizations? create new laws.

a. Nile River Valley b. It prevents government officials and institutions


from making their work public.
b. Asia Minor
c. It ensures government officials and institutions are
c. Indus Valley exempt from laws.

d. Fertile Crescent d. It holds government officials and institutions


accountable to the law
144 Large surface area;place, division or part.
149 The protection of the natural environment from
a. region destruction can be called .

b. place a. erosion

c. artifact b. conservation

d. Resource c. deforestation

145 This question is a part of a group with common 150 The code of honor that knights were expected to
instructions. View group » follow is called

a. feudalism
162
b. serfdom b. Stonewall Jackson

c. chivalry c. Jefferson Davis

d. Magna Carta d. Abraham Lincoln

151 All of the following are human characteristics 156 A bill cannot become a law until it is
EXCEPT
a. approved by Congress and judged constitutional
a. language
b. proposed by the president and approved by
b. religion Congress

c. politics c. approved by Congress and signed by the president

d. climate d. proposed by Congress and ratified by the states

152 A/An ____is the watering hole that most cities 157 The Old World refers to
surround to supply the people with water.
a. North and South America
a. oasis
b. Europe
b. laut
c. North America
c. oacis
d. South America
d. well
158 This question is a part of a group with common
153 Why are most modern economies referred to as instructions. View group »
"mixed" economies?
This economic system is based on customs and
a. Poverty is always highest in countries with market habits.
economies
a. Mixed
b. Most countries have aspects of market and
command economies b. Traditional

c. Government planners do not know how to handle c. Market


economic problems
d. Command
d. Products made by traditional economies have no
markets in the modern world. 159 Which warrior King, called the "Lion Price",
ruled Mali from 1217 to 1255 AD?
154 This group settled on the entire west coast of
South America. They kept records on a system of a. Sundiata Keita
knotted strings called "quipu". The villages were
connected by rope bridges and roads. b. Sundiata Gaines

a. Inca c. Mansa Musa

b. Aztec d. Muhammad Ali

c. Olmec 160 Who has the power to ratify or approve a treaty


made by the president?
d. Maya
a. The House of Representatives
155 Who was elected as President of the
Confederacy? b. The Senate

a. Thomas Jefferson c. The Supreme Court


163
d. The vice-president b. Abu al Malik

161 Which body has the power to impeach the c. Muhammad


president?
d. Augustus Caesar
a. The Senate
167 The education and skills of the workers.
b. The House of Representatives
a. Post-Secondary Education
c. The Supreme Court
b. Training
d. The vice-president
c. Human Capital
162 How was it ensured that the Roman Emperor
treated everyone fairly d. Capital Goods

a. Everyone voted to be nice to each other 168 Which of the following is a system of writing
created by the Sumerians?
b. The laws were posted for all to see
a. ziggurat
c. They paid taxes
b. cuneiform
d. Anyone who didn't pay taxes was punished
c. Code of Hammurabi
163 Tropical wet and dry climate zones are
characterized by... d. iron

a. wet summers and dry winters 169 The Progressive Era ran from

b. wet winters and dry summers a. 1890-1908

c. wet summers and wet winters b. 1895-1915

d. dry summers and dry winters c. 1900-1917

164 Who were the first inhabitants of Mesopotamia? d. 1910-1920

a. Assyrians 170 What were black codes?

b. Babylonians a. Laws limiting the rights of African Americans.

c. Sumerians b. Law freeing African Americans.

d. Egyptians c. Laws offering unlimited African American rights.

165 Which of the following is not an idea from the d. Laws enslaving African Americans again.
Enlightenment?
171 The Sumerians believed that gods descended to
a. People have natural rights the earth using the ziggurat as a ladder. Based on this
sentence, which of the following is true?
b. Problems can be solved by using reason
a. The Sumerians were monotheistic.
c. Kings ruled by divine right
b. The Sumerians were polytheistic
d. Government should serve the people
c. The Sumerians were dicots.
166 Who was the emperor of Rome at the beginning
of the Pax Romana (the time of Roman peace)? d. The Sumerians were papyrus.

a. Julius Caesar 172 What is Europe's fastest growing religion?


164
a. Islam 177 Define a Tundra

b. Judaism a. All of the physical surroundings in a place,


including land, water, animals, plants, and climate
c. Christianity
b. To change in order to adjust to a new condition or
d. Hinduism environment.

173 Define the word Environment c. A vast, treeless plain in the Artic regions with very
cold winters, cold summers, and little rain or snow.
a. A people’s way of life, including beliefs, customs,
food, dwellings, and clothing d. A people’s way of life, including beliefs, customs,
food, dwellings, and clothing
b. A vast, treeless plain in the Artic regions with
very cold winters, cold summers, and little rain or 178 A Sumerian _____was made of terraces,
snow. because it was gods' ladder.

c. Useful material found in nature, including water, a. civilization


vegetation, animals, and minerals.
b. city-state
d. All of the physical surroundings in a place,
including land, water, animals, plants, and climate c. ziggurat

174 Which two Union generals attacked Vicksburg d. nomad


and Jackson, MS along the Mississippi river to seize
it as a great asset for the North in a battle in which 179 The pope is the leader of
Abraham Lincoln dubbed "Vicksburg is the key!" (to
the Civil War)? a. Roman Catholic Church

a. General Lee/General Jackson(Stonewall) b. Constantinople

b. General Ulysses S. Grant/Admiral Farragut c. Eastern Orthodox Church

c. General Scott/General McClellan d. Western Orthodox Church

d. General Sherman/General Stewart 180 Governments have the power to make and
enforce laws
175 How many lives were lost during the American
Civil War? a. so the police can arrest you

a. 60,000 b. to find people who are not good citizens

b. 100,000 c. to keep people safe and prevent conflicts

c. 620,000 d. to run peoples' lives

d. 600,020 181 Where did the concept of citizenship begin?

176 In what country today do you also have the right a. Ancient China
to vote, but the responsibility to pay taxes and
register for the army? b. Ancient Rome and Greece

a. America c. medieval Euorpe

b. Cuba d. the United States

c. China 182 The strip of land that was once located many
thousands of years ago that linked northeast Asia and
d. North Korea present day Alaska was called .

165
a. mesoamerica b. Prime Minister, Courts, Congress

b. quipu c. Senate, King, Supreme Court

c. Bering Land bridge d. Legislative, Governor, Mayor

d. Tenochtitlan 188 Which of the following is a job of the


government?
183 This group was a powerful group of fierce
warriors. They controlled a huge trade network a. limit travel
which made them very wealthy.
b. make laws
a. Inca
c. manage information
b. Aztec
d. provide merchandise
c. Olmec
189 Who invaded India and settled around the
d. Maya Ganges river around 2000 B.C.?

184 The Virgin Queen was . a. Uphanishads

a. Mary, Queen of Scots b. Aryans

b. Joan of Arc c. Vedas

c. Elizabeth I d. Sidartha Gautama and Buddha

d. Bloody Mary 190 General Sherman's "March to the Sea" destroyed


anything they found useful to the Confederates,
185 This gave Byzantine emperors a great amount of including plantations, railways, and even cities. This
power. type of campaign is called...

a. Justinian's Code a. Salted Ground

b. Nicene Creed b. Burn it down!

c. Roman Constitution c. Scorched Earth Campaign

d. Bible d. Overland Campaign

186 How did Cabeza de Vaca contribute to the 191 All of the following were a result of Sherman's
history of Texas? March to the Sea EXCEPT...

a. He was the first person to map the Gulf of Mexico a. Lee's men grew inspired by the great Southern
and the Texas coastline. victory.

b. He conquered the Aztecs and contributed to b. Many cities in the South would be burnt.
Tenochtitlan becoming Mexico City.
c. Once freed, many slaves joined General Sherman's
c. He was the first person to report on Texas, and army.
inspired the Spanish to continue search for gold.
d. Planation owners in the deep South grew tired of
d. He discovered the New World which led to the war.
exploration by several European countries.
192 If you lived in Egypt, the desert climate would
187 What are the 3 Branches of US government? be similar to which American

a. Legislative, Judicial, Executive city?

166
a. Albuquerque, New Mexico 200 Which dynasty had the earliest written records?
a. Shang
b. New York, New York b. Zhou
c. Hanu
c. Chicago, Illinois d. Xia
KEY TO CORRECTION:
d. Los Angeles, California 1. C2. A3. A4. D5. B6. B7. C8. D9. A10. A
11. C12. A13. D14. C15. A16. C17. D18. C
193 _____divided the Roman Empire into the 19. C20. B21. B22. A23. B24. B25. C26. A
Eastern and Western Roman Empires 27. B28. B29. D30. D31. A32. B33. C34. A
35. C36. D37. A38. A39. C40. C41. D42. C
a. Alaric 43. C44. B45. C46. D47. C48. B49. B50. B
51. C52. A53. C54. A55. C56. D57. A58. A
b. Odoacer 59. A60. C61. C62. D63. A64. D65. D66. B
67. D68. C69. A70. D71. B72. D73. C74. D
c. Theodosius 75. B76. D77. C78. C79. B80. D81. A82. C
83. A84. D85. B86. A87. A88. A89. A90. C
d. Constantine 91. D92. C93. B94. C95. C,B96. C97. A
98. A99. D100. A101. A102. C103. C104. D
194 Who defeated the last remaining Roman army in 105. D106. E107. A108. B109. A110. A111. B
Europe? 112. A113. B114. A115. B116. B117. D118. B
119. D120. C121. B122. B123. B124. C125. A
a. Charlemagne 126. C127. B128. C129. B130. A131. C132. D
133. D134. A135. A136. A137. B138. D139. A
b. Charles Martel 140. C141. D142. C143. D144. A145. C146. B
147. C148. D149. B150. C151. D152. A153. B
c. Clovis I 154. A155. C156. C157. B158. B159. A160. B
161. B162. B163. A164. C165. C166. D167. C
d. Constantine 168. B169. C170. A171. B172. A173. D174. B
175. C176. A177. C178. C179. A180. C181. B
195 Who was the first Secretary of War? 182. C183. B184. C185. A186. C187. A188. B
189. B190. C191. A192. A193. C194. C195. A
a. Henry Knox 196. D197. D198. B199. A200. A

b. George Washington

c. Alexander Hamilton

d. Thomas Jefferson

196 Who said "A house divided against itself cannot


stand"?
a. Stephen A. Douglas
b. George Washington
c. Dred Scott
d. Abraham Lincoln
197 Who were Darius I and Xerxes?
a. Spartan Kings
b. Leaders of Athens
c. Generals of Persian Army
d. Emperors of Persia
198 Who came up with the Anaconda Plan (North's
strategy)?
a. Francis Scott Key
b. Winfield Scott
c. Frederick Douglas
d. Robert E. Lee
199 Tundra is a treeless plains of the Arctic regions.
a. True
b. False
167
168

Potrebbero piacerti anche